-and their ans
by the
creators
of the test. THE OFFICIAL GUIDE FOR GMAT® VERBAL REVIEW 2ND
EDITION
rJ!h e
rJ
off\G\' Guid
• Actual questions from past GMAT tests, including more than 75 questions new to this edition • 300 past questions, answers, and explanations in Reading Comprehension, Critical Reasoning, and Sentence Correction • Questions organized in order of difficulty to save study time
From the Graduate Management Admission Council
Table of Contents
1.0
1.1
1.2 1.3 1.4 1.5 1.6 1.7
1.8 1.9 1.10 2.0
2.1 2.2 2.3 2.4 3.0
3.1 3.2 3.3 3.4 3.5 3.6
What Is the GMAT"'? Why Take the GMA~ Test? GMAT" Test Format What Is the Content of the Test Like? Quantitative Section Verbal Section What Computer Skills Willi Need? What Are the Test Centers Like? How Are Scores Calculated? Analytical Writing Assessment Scores Test Development Process
4 5 6 8 8 8 9 9 9
10 10
How to Prepare How Can I Best Prepare to Take the Test? What About Practice Tests? Where Can I Get Additional Practice? General Test-Taking Suggestions
12 13 13
Reading Comprehension What Is Measured Test-Taking Strategies The Directions Sample Questions Answer Key Answer Explanations
16 18
14 14
20 21
2.0 How to Prepare
2.0 How to Prepare 2.1 How Can I Best Prepare to Take the Test? \ Ve at the Graduate Management Admission Council'" (GMAC"') firmly believe that the test-taking skills you can develop by using this guide-and The Official Guide for GMAr Review, 12th Edition, and The Official Guide for GMAr Quantitative Review, 2nd Edition, if you want additional practice-are all you need to perform your best when you take the GMAT' test. By answering questions that have appeared on the GMAT test before, you will gain experience with the types of questions you may see on the test when you take it. As you practice with this guide, you will develop confidence in your ability to reason through the test questions. No additional techniques or strategies are needed to do well on the standardized test if you develop a practical familiarity with the abilities it requires. Simply by practicing and understanding the concepts that are assessed on the test, you will learn what you need to know to answer the questions correctly.
2.2 What About Practice Tests? Because a computer-adaptive test cannot be presented in paper form, we have created GMATPrep software to help you prepare for the test. The software is available for download at no charge for those who have created a user profile on www.mba.com.Itisalsoprovidedonadisk.byrequest.to anyone who has registered for the GMAT test. The software includes two practice GMAT tests plus additional practice questions, information about the test, and tutorials to help you become familiar with how the GMAT test will appear on the compllteUCIRB at the tat em•
•~ d1a.t you download the s~ftwa.re.as n;st:. T~ oPe RllCli
Get Additional Practice? ,de and thank lOU would like additional praelice, you ma~
MAr R
, 12tb Ed,tlon, or The OjJiClal Cudrjor <;,\1/1r
If you do not know the correct .n',"" ... que lion 1 too t1m -co u.m.i.n,g n t:o elillun..te chOiCes you know are ron~ sele . t
d Edlll n, t IVIVW mba om. me rI p L_ V(;twec:n th I bj )(,"h. ..'nd the re..iew seelions of the G:\I \TPrep
...r..lll' Test-Taking Suggestions trat gJ
tor indi",,!ual que tIC'" tvpe are presenred later flO to help IlU pc.:rtorm your hc~[ on the test.
In
n
lly f.uuli,,, wirh ,til qllestion types, use the sample questions In thiS book a1t t. II 10,')' he u dullo time yourself as you answer the practice questions hllw 10111: ·'H' will h,1\"e fi" c,llh questioll durill!!; thc artual Gl\IAT test .1' well as h th r 1111 ,m .1Il well Ill: 'lUll kl) ellough to complete the te,t in the time allotted.
,tl) dl,lt
I
fl
'1'"11'd
10
,Ill wer e,ll h question type. If rou re.ld h.1Stily, you
tru tHin. lull(lwc.:r ,our lOf[S. 1'0 rC:\'ll'\\' dircdions dllrin~ the tcst.
B thwart: tli t
t
6. Confirm your answers ONLY when you are ready to move on.
AlIhol.gh th G I T tes, '" c .,<"wran more than speed, it b importanr to usc your time wisely. u w,ll h,nc .hollt I minures ti,r e,,,h verhal question and about 2 minutes for each $lJlntiraltlve qu lion 0111 \ ou r.lI' the te t, all OIlSITeen clock will conrilluously count the time t You call hod, rh" disp!.') if rou want, but it is a good idea to check the clock J.:~:~J1 to mlllutor lOur progrrs.. '/he clork will automatically alert you when 5 minutes •nth 1I0tted rm1< tor rhe 'Cllion you arc working on.
111
rematnmg- ,answer Ch01C~ and m ~ quest'on I n not to worn' about the Imp your l:ore-gue mg rna\ 100'IoeT \'JUT re tini,hlll~ the section IV,lll wer \ ill Bear in mind that if ,ou do not tim h. e n the allotted time, you Will still recene d. re
thIS book The
'a,1_JOU,t1me wisely.
r Jt
5. Do not spend too much time on any one question.
It
the time
\IHI
"'p Ild n.·\'I('·WIO~ tHn'ctions will count
dick
.lgllin t
the
[IlUIU' fhl fe, t
IIlut IS bein '.1 k"d, then elilllllutc 1 '1m tlollllr the po...... ihlc .in Wlr; .\llit
(\1I
lport
kllll
lilt 1Il10IID,ltll)n til Illl,UlllS
Once you ha..e selected your an:wer to a mullplechoice question, ),ou will be a"ked to confirm It. Once ~'ou confirm Your response. )OU cann t ~ back and change it. You may not skip que !Ion , because the compUler ,elects each que'tion on rhe basis of your responses to preceding que.t. >0< .
7. Plan your essay answers before you begin to write. The best way to approach the rwo wcitlO2; ta that compll'e the Anah-rical \\'riting :\_ men ,- to read the directions c-arefulh, take. re" mlllUles to think about the qu"tion, and plan a re 'pOlbC before ,'ou be~in writing. Take \.~re to org.mize lour idea' and dewlap them lWh. b t leave time to reread lour re"'r n.. and make m\ renSlon, thatmu thlllk would imprO\ Ir.
F -n... .1Ii
3.0 Reading Comprehensio Reading comprehension questions appear In the Vabal __••f1Il111 section use multiple-choice questions to measure your ability _ltilliitii material, to reason and evaluate arguments, and to corra:t wri.... a.....~. written English. Becau e the Verbal section include content from a generally familiar with some of the material; however, neither the ..1IIi. . . . . knowledge of the topics discussed. Reading comprehen Ion qulestiollIJ reasomng and sentence correction questions throughout the Verballle(:tioll
l.
-_:fl'.
You will have 75 minutes to complete the Verbal ection, or an average answer each question. Keep in mind, however, that )OU will need time to passages-and that time is not factored into the 1 • mmute avetagl: You shlllllld t1:==I~ proceed more quickly through the reading comprehen ion quesnon In orda l!l time to read the passages thoroughly. Reading comprehension questions begin with written pa age up to 350 words discuss topics from the social sciences, humanities, the phy ical or biological sdall~••" ' _ . business-related fields as marketing, economics, and human re ource manageme are accompanied by questions that will ask you to interpret the pa age, apply the iJdmMIiDIll. gather from the reading, and make inference (or informed a umption based these questions, you will see a split computer screen. The written pa age will rel'naln~l1Iibla left side as each question associated with that pas age appears in tum on the right "'......,;01 see only one que tion at a time, however. The number of question a SOC1ated with may vary. As you move through the reading comprehen ion ample que tion , try to determine that works best for you. ¥ou might begin by reading a pas_age carefully and thOrolugl~......" some test takers prefer to skim the pa sages the fir"t time through, or even to read .. before reading the passage. Vou may want to reread any sentence- that present colIlJdic.d iI introduce terms that are new to you. Read each question and ene of answers you under tand exactly what the question i a king and what the answer eho
1IIIl.
C811t:f.,.
ading Comprehension
If you need to, you may go back to the passage and read an) parts that an: re1I~~ question. pecific portions of the pa sages may be hi hli hted in the related qwcsai... The following pages describe what reading comprehen Ion qu non present the directions that will pre-cdc que.tion of thi npe. nd dellCIiillc types. This chapter al"o provide. test-taking strateglc-, ample qu 11II11 ..of all the questions. The explanation> further illu trate thc In whichftlll'" questions eYaluate basic reading skills.
.
t
IOn
'l!ilSure prehension questions measure your ability to understand, analyze, and .apPly 1Iltortb. concepts presented in written fOrm. All questions are to be answered on the. ba~lS of what pJied in the reading material, and no specific prior knowledge of the matenalls required VlVl/,,T
reading comprehension questions evaluate your ability to do the followmg:
Understand words and statements. Ithough the questions do not test your vocabulary (they will not ask you to define terms' they do test your ability to interpret special meanings of terms as the~ are used in the ~ passages. The questions will also test your understanding of the Enghsh language. These questions may ask about the overall meaning of a passage. Understand Ioaical relationships between points and concepts. This type ofquestion may ask you to determine the strong and weak points of an argument evaluate the relative importance of arguments and ideas in a passage.
Draw Inferences from facts and statements. The inference questions will ask you to consider factual statements or information preseNtd a reading passage and, on the basis of that information, reach conclusions. • Understand and follow the development of quantitative concepts as they are P....._
written material. This may involve the interpretation of numerical data or the use of simple arithmetic to reach conclusions about material in a passage.
There are six kinds of reading comprehension questions, each of which tests a different skill. 1he mlding comprehension questions ask about the following areas:
la . a unified whole-that is, the individual sentences and paragraphs suppon and .n idea or central point. Sometimes you will be told the central point in the pasIIf sometimes it will be necessary for you to determine the central point from the ovmIl ~ OC'~nr of the oua~. You may be asked in this kind of question to -ParaPhrasing, ofthe main idea
~VJ:inwriti
'nte,ences These questions ask about ideas that are not explicitly stated in a pasla; author. Unlike questions about supporting details, which ask about UlmliDlltion m. stated in a passage, inference questions ask about ideas or meanings that must be iDfcrred from information that is directly stated. Authors can make their points in indirect wayt without actually stating them. Inference questions measure your ability to undenrmd mtended meaning in parts of a passage where the meaning is only suggested. 'These quatinns ask about meanings or implications that are remote from the passage; rather, they meanings that are developed indirectly or implications that are specifically suggested by To answer these questions, you may have to • logically take statements made by the author one step beyond their lite • recognize an alternative interpretation of a statement made by the author • identify the intended meaning of a word used figuratively in a pusa
If a passage explicitly states an effect, for example, you may be asked compares two phenomena, you may be asked to infer the basis for asked to infer the characteristics of an old policy from an explicit description you read a passage, therefore, you should concentrate not only on the ~ mnnin author's words, but also on the more subtle meaning implied by those words.
information to a context outside the These questions measure your ability to discern the re1a.tionsbips bel weeD -,., presented by the author and other situations or ideas that miPt nari In this kind of question, you may be asked to hypothetical situation that is comparable to a situatiQo
Sltuatioo
1-' , ' . i
-.
pic.'"
These questl
n
Ie V\.lU • how a pa$sage Idea. or rcflltl • how the author r •
•
~h ..
t,
-ason behind th
ASS~SS
th
i wealCnc
nile Rpp
The n answe
e and tone
s
,,,-rt'pt h
Questions 7-11 refer to the passag
,..1li01oIfCI1Iy ItlittJlSYSft.ms on Earth.
The paSSage is prlmanly concerned With
7.
oJ file fasCinating paradoxes
(A)
cID clear, and thus nutrientsuch prolific and productiVe
(8)
of the answer Ires within the orals themselveS. SymbiotiC cells of 'JiI-1IIIown as zooxanthellae carry out ".Id!IISIS using the metabolic wastes of the m-.lIIereby producing food for themselves, for ani hosts, and even for other members of tile reel community. This symbiotIC process allows """'SIllS ,n the reef community to use sparse fIlIrieIlt resources efficiently. Unfortunately for coral reefs, however, avanety of human activ,tles are causing worldWide "eeradation of shallow marine habitats by adding nutrients to the water. Agriculture, slash·and·burn IIIKI cleanng, sewage disposal, and manufactunng that creates waste by-products all increase nutnent loads In these waters. Typical symptoms of reef dIcIme are destabilized herbivore populations and _increasing abundance of algae and filter·feeding • nlmals. Declines in reef communities are nllstent With observations that nutrient input is mcreeSlllll In direct proportion to growing human ons, thereby threatening reef communitieS Iitive to subtle changes in nutrient input to their
(C)
8.
9.
discussing the process by which . Coral r detenorate In nutnent-poor waters
(C)
descnblng the abundance of algae and feeding anrmals In coral reef areas Nt.
The passage suggests which of the follo wm coral reef communities? g Coral reef communities may actually be likely to thnve In waters that are relative: nutnents. The nutrients on which coral reef commu thrive are only found In shallow waters. Human population growth has led to changq ocean temperatures, which threatens cor~ communities.
(D)
The growth of coral reef communities tends destabilize underwater herbivore populatioll
(E)
Coral reef communities are more compleui diverse than most ecosystems located on ar land.
The author refers to "filter-feeding animals' (/ines 23-24) In order to (A)
prOVide an example of a charactenstic Sign reef deterioration
(B)
explain how reef commUnities acquire sustenance for survival
(e)
(0)
fE)
(A) (8)
(E)
(C)
According to the passage, which of the follOWing IS a factor that IS threatening the surVival of coral reef communities?
explaining how human actiVities are threat to coral reef communities
explainrng how coral reefs prodUce fo themselves Cd
(B)
~"'j,pfl.S
describing the effects of human ac algae in coral reefs
(D)
(A)
10.
Identify a factor that helps herbivore P thrive indicate a cause of decreaSing nutrient I waters that reefs inhabit identify members of coral reef commun rely on Coral reefs for nutnents
(0)
(E)
11.
The waters they mhabit contam few nutnent resources. A decline In nutnent Input IS disrupting thell symbiotiC relationship With looxanthellae. The degraded waters of thell manne habitats have reduced the II ability to carry out photosynthesis. They are too biologically complex to survive 10 habitats With minimal nutnent mput. Waste by-products result In an mcrease 10 nutrient input to reef communities.
It can be inferred from the passage that the author describes coral reef communities as paradoxical most likely for which of the follOWing reasons? (A)
They are thnvlng even though human actiVities have depleted the nutrients in therr environment.
(8)
They are able to survive in spite of an overabundance of algae inhabiting therr waters. They are able to survive in an environment With limited food resources. Therr metabolic wastes contribute to the degradation of the waters that they inhabit. They are decltnrng even when the water surrounding them remains clear.
(C) (D) (E)
. 18-23 refer to the passage a QuestiOns
1994 a team of SClenhslS led by Davld McK~h d be"n rudy,"g the meleonte ALH84001, WhlC a been dl overed," Antarcllca In 1984 Two yea7 later the McKay leam announced that ALH8400 , 5) whICh SClenl, I generally agree onglnated on Mars, conlllned compelling eVIdence that hfe once eXI led on Mars This eVidence Includes the dl covery of orgamc molecules In ALH84001, the fir I ever found In Martian rock. OrganIC (1O) molecules-complex, carbon·based compoundsform the baSIS for terrestnal hfe. The orgamc molecules found In AlH84001 are polycychc aromatic hydrocarbons, or PAHs. When microbes dIe, the" orgamc matenal otten decays Into PAHs. (15) Skepticism about the McKay team's claim remain , however For example, AlHB4001 has been on Earth for 13,000 years, suggesting to orne SCientists that lIs PAHs might have resulted from terrestnal contamination. However, McKay's (20) learn has demonstrated that the concentration of PAH Increases as one looks deeper Into AlHB400J, contrary to what one would expect from lerre t"al contamination The skeptiCS' strongest ar umen!, however, IS that processes unrelated to (2) organic hfe can eaSily produce all the eVidence found by McKay's ledm, including PAHs. For .ample, till forrnatlon produces PAHs. Moreover, PAH IreQul'fllly dppear In other meteorites, and no on a""bule IIle" pre ence to life processes. Yet (0) McK y' ham nol that Ihe parllcular combination 01 PAH '" AI H84001 I more Irl1llar to the ombln t,un produc d by decaYing organisms than lolho 0" In 1m!: frolllllOllblologlccJl processes.
urpose of the passage IS to The pnmary p lB. neW ways of studying the pass, (A) des c"·fbeonce existed on Mars that II e . a theory regarding the eXistence of revise . (8) ·ght of new eVidence Mars In II (e) reconCile conflicting viewpOints regarding pOSSibility that life once eXisted on Mars (D) (E)
9 1.
20.
The passage suggests that a meteorite ecwltli_ PAHs a were e resu of et'res efl one Nould £'leec the fole_1I!.
21.
(AI
d have been
tBl
om a
(el
e eeo e odhave
evaluate a recently proposed argument concerning the ongln of ALH84001
(01
describe a controversy concerning the sigfllficance of evidence from ALH84001
(E)
The passage asserts which of the follOWing about claim that ALH84001 originated on Mars? (A)
It was initially proposed by the McKay team of scientists.
(B)
Earth
22.
Which of the followmg bes descn es e the last sentence of the first paragrap J
c on
(A)
It identifies a possible orga c so rce PAHs found mALH8400
It is not a matter of Widespread scientific dispute.
(8)
It deScribes a feature 0 P s a IS no s ared by other types a orgafliC olecu es.
(C)
It has been Questioned by some skeptics of McKay team's work.
(C)
It explains how a charactens c co mon meteontes onglnates
(D)
It has been undermined by recent work on P
(0)
(El
It is Incompatible with the fact that ALH84001 has been on Earth for 13,000 years.
It suggests how the terres a ntam na 0 ALH84001 might ha e a en la e
(E)
It presents eVlden e ha that hfe once e Isted (I
(A)
These PAHs are n tie mete nte t at ha een ear or m re
The passage suggests that the fact that ALH84001 has been on Earth for 13,000 years has been used some SCientists to support which of the foliowlfIg claims about ALH8400l? ALH84001 may not have originated on Mars.
(8)
ALH84001 contains PAHs that are the result nonblologlcal processes.
(B)
ALH84001 may not have contained PAHs wh landed on Earth.
The e P H re n t el meteonte that on mated
()
The orgamc molecules found In ALH84001 not PAHs.
The e P Hare n tie f rmatl n
(D)
(C) (D)
(El
The organtc molecules found 10 ALH84001 not be the result of terrestrial contaminatIon.
(E
0
most
earn
23
(A)
r the
to
d Ed ""
. 24- 26 refer to the passage ill QuestiOns
an animal's maintenance ot certain Inb!mal varlabtes within an acceptable range. partiCularly ,n extreme phySical environments, has long ,nterested biOlogIsts. The desert rat and the (5) camel In the most water·deprlved environments. and marine vertebrates in an all·water environment. encounter the same regulatory problem: maintaining adequate Internal nUid balance. for desert rats and camels. the problem is (101 conservation ot water In an enVIronment where standing water IS nonexistent, temperature is high. and humidity IS low. Despite these handicaps, desert rats are able to maintain the osmotic pressure of their blood. as we/l as their total body· 1151 water content. at approximately the same levels as other rats. One countermeasure is behaVioral: these rats stay In burrows dUring the hot part of the day, thus avoiding loss of nUid through panting or sweating. which are regulatory mechanisms for 1201 maintaining Internal body temperature by evaporative cooling. Also, desert rats' kidneys can excrete a urine haVing tWice as high a salt content a eawater. Camels. on the other hand, rely more on simple (25) endurance They cannot store water, and their reliance on an entirely unexceptIOnal kidney results In a rat at water loss through renal function Igmh ant/y higher than that of desert rats. As a r ull, camel must tal rate losses In body water of (3D) up 10 30 percent of th Ir body weight. Nev rlh Ie ,camel~ do r lyon a speCial m cham m 10 keep wilter los~ wllh,n a tolerable ran . by weallng and panting only when their body I mp ralure exceeds that which would kill a (35) human. Ih y can erv Internal water
t /lowing most accurately states the
HomeostaSIS.
Marin v rlebrales experl nce dlthculty With th "wat r balanc becau~e though ther is no hortaa of awal ( 10 drink, th y must drlf1k a lot ot It to malntam th "ml rnal flUid balan e. But the 5
om how,
r
It
tram th
wat r musl be dl charged nd th kldn y of rna t marin
to x r tUrin In wh, h mor one ntr t d th n In waf r. '.... nlm , h v P I I s It eer ling r
un bl
kidn y th t
n bl th m to
24
I
I
Which of the a ? at the passage. purpose two different approaches to th_ pa rem A) To c o . u~ ( study of homeostasIs mmarlze the findings of several stUd Ie (B) To su .•. t S regarding organisms main enance of Interllal . bles In extreme environments varia (C) To argue for a particular hypothesis regarding various organisms' conservation of water In desert environments (D)
To cite examples of how homeostasis is achieved by various organisms
(E)
To defend a new theory regarding the maintenance of adequate fluid balance
25. It can be inferred from the passage that some mechanisms that regulate internal body temperature, like sweating and panting, can lead to which of the following?
I I
I I
(A)
A nse in the external body temperature
(B)
A drop in the body's internal fluid level
(e)
Adecrease in the osmotic pressure of the bl
(D)
A decrease in the amount of renal water loss
(E)
A decrease in the urine's salt content
26.
IA)
IB)
Ie)
(D) (E)
lunc ons much as the Iudney does no aid the came m toplI1g excep onal a er loss resulting tmln " . . extreme cond ons of s e ron does no enab e e came 0 excrete as salt as do e Idneys of manne vertebrates IS Similar In Srueture 0 the dneys of mammals liVing n ater-deprlved en r me reqUIres the help 0 other organs n e m excess sal
g
Questions 27-32 refer to the passage hoo of politi al history that emerged m . .lIti1DsIIId 19 Os sought to go beyond the sof political hlstonans on leaders lI'JVI!'IIIII8I nstiMiOns by examining directly ilicIl practices of ordinary citizens. like the Illproach however, thiS new approach excluded WllIlllIII The very techniques these hlstonans used to uncover mass political behaVIor mthe nmeteenth ntury United States-quantitative analyses of lIIeclion returns, for example-were useless m analyZIng the political actiVIties of women, who were denied the vote until 1920. By redefining 'politlcal activity; historian Paula Saker has developed a political history that mcludes women She concludes that among ordinary Citizens, political actiVIsm by women in the nineteenth century prefigured trends in twentieth century politiCS. Defining 'politlcs' as 'any action taken to affect the course of behavior of , government or of the community; Baker concludes that while voting and holding office were restricted to men, women In the nineteenth century organized themselves Into SOCieties committed to SOCial sues such as temperance and poverty. In other words Saker contends, women activists were early practitioners of nonpartisan, Issue-oriented politiCS and thus were more Interested in enlisting awmakers, regardless of their party affiliation behalf of certain Issues than In ensurrng that o~~n party or another won an election. In the twentieth .IIIJt~ more men drew closer to women's Ideas ~tII",~ and took up modes of issue-oriented _1IIII1I'ed. taker sees women as haVing
27
The primary pur pose of the passage IS to (A)
29
enumerate reasons why both tradlt lanai scholarly methods and newer schOlarl have limitations Y
(B)
Identify a shortcoming In a scholarly app! and deSCribe an alternative approach
(C)
prOVide emprrical data to SUpport aIon scholarly assumptIOn g
(0)
compare two scholarly publications on th of therr authors' backgrounds e
(E)
atlemptto proVide a partial answer to aI standing scholarly dilemma 0I't
28. The passage suggests which of the following concerning the techniques used by the new poll~ hlstorrans described In the first paragraph of the passage?
It can be Inferred that the author of the passage Quotes Baker directly In the second paragraph primarily In order to clarify a POSition before providing an alternative to that POSition (Bl differentiate between a novel definition and traditional definitions (Cl prOVide an example of a POint agreed on by different generations of scholars provide an example of the prose style of an (0) Important historian IE) amplify a definition given In the first paragraph
(A)
30.
According to the passage, Paula Baker and the new political historians of the 1960s and 1970s shared which of the following? (A) (B)
(Cl
rA)
(B)
(C)
pa:c:
(0)
(E)
They involved the extensive use of the biographies of polittcal party leaders and political theoreticians. They were conceived by polittcal hlstonans were reacting against the political climates 01 the 1960s and 1970s. They were of more use In analyZing the pas of United States political parties In the nineteenth century than In analyzing the positions of those in the twentieth century. They were of more use in analyzing the pol behavior of nrneteenth-century voters than In analYZing the political actiVities of those whO could not vote dUring that period. They were devised as a means of tracing the Influence of nineteenth-century politrcaltrend on twen t'leth-century political trends.
(0)
(E)
A commitment to Interest group politics A disregard for political theory and ideology An Interest in the ways in which nineteenthcentury politiCS prefigured contemporary politiCS A reliance on such Quantitative techniques as the analySIS of election returns An emphasis on the political involvement of ordinary citizens
31
\
IAI
\
fBl
\
IC) (0)
IE)
32.
s:~:===
The information In the pa g 1960s political histOrian w uld hi to undertake which of the followm (A) (B) (C)
An analYSIS of voting tTen voters of the 1920
Astudy of male voter g,ra:.::::~ from party politiCS to Issuule-Olrienfld Abiography of an mflu nt ni~I!tIl""..ei .... minister of foreign affa r
(0)
An analySIS of narrative unrecognized women ~rtiui~lt~
(E)
A study of voting e d Immigrant laborer 10ggIOg camp
al::::==
III
ftlllItl~ns 33-38 refer to the passag
• ......AmfriCIII til beI/II recordi"I
~~AI~mtII:'W:1IIS.
iJ!t'.",
EtfmO/oIISts
I'llr wantillg til hear lI1e _ _ after finglllstic or anthropologICal • •. . , .."pIemeIIt lI1elr own field ...1ItI.IIIS,1IIllI they believed that the personal . . . . . . ..,. of a siJlgle mdMdual, could Increase understandmg of the cultures that they had observing from without. In addlllOn many ethnologISts at the tum of the century believed that Native Amencan manners and customs were rapidly dlSlllP"Rng, and that It was Important to preserve for posterity as much information as could be adequately recorded before the cultures dISappeared forever. There were, however, arguments against this (20) method as a way of acquiring accurate and omplete mformatlon. Franz Boas, for example, desoRbed autobiographies as bemg "of limited value, and useful chiefly for the study of the perversion of truth by memory; while Paul Radin 6) contended that mvestrgators rarely spent enough time with the trrbes they were observing, and Inevitably derrved results too tinged by the Investiptor's own emotional tone to be reliable. Even more ,mportantly, as these life stories moved from the traditional oral mode to recorded written form, much was IneVitably lost. Editors often decided what elements were significant to the field ,..Irch on a given trrbe. Natrve Amerrcans fICOII!lZed that the essence of their lives could not be communrcated In English and that events that tIIey thought slgnrficant were often deemed Important by their mtervlewers. Indeed, the very III telling their storres could force Native ~. 4lRiriclIR narrators to distort their cultures, as had to be broken to speak the names of ftIIIIives crUCIal to their family stories. aU 01 thiS, autobiography remains a for Ithnologlcal research: such personal nd ImpreSSions. Incomplete as they to throw more light on the IIId emotions than any amount IJIU ~IJJ Ithnologlst or ethnological
IIIIture
33
follOWing best descrrbes the 0 WhtCh of th e e1 of the passag
fAl
The hlstorrcal backgrounds of two Cur used research methods are chroniCled.
(B)
The validity of the data collected by USing different research methods IS compared.
(e)
The usefulness of a research method IS Questioned and then a new method IS pr
(D)
The use of a research method IS descnbe\! the limitations of the results obtained ale
(E)
discussed. Aresearch method IS evaluated and the c necessary for ItS adaptation to other Sub'ett areaS are discussed. 36.
34. Which of the follOWing IS most Similar to the actlo~ mneteenth·century ethnologists in their editing of life stories of Native Americans? (AI
Awitness In a Jury tnallnvokes the Fifth Amendment In order to avoid relating perso incriminating eVidence.
(B)
Astockbroker refuses to divulge the Source ~ her information on the possible future increa In a stock's value.
(C)
Asports announcer describes the action In a team sport With which he IS unfamiliar.
(D)
Achef purposely excludes the special ingred from the reGIpe of hiS prrzewinnlng dessert.
(E)
Apolitician falls to mention In a campaign speech the slmilarrties in the POSitions held by her opponent for political office and by herself
According to the passage, collecting hfe storres can be a uselul methodology because life stones proVide deeper inSights Into a culture than the hypothesIzing 01 academiCS who are not members of that culture (B) 1I1e storres can be collected eaSily and they are not subject to Invalid Interpretations (C) ethnologists have a limited number 01 research methods from which to choose (D) life stones make It easy to distinguish between the Important and unimportant leatures 01 a culture (E) the collection of life stories does not require a culturally knowledgeable investigator
lB
(A)
I1 101 lEI 38
Investigator flmlillll' wi..... study (Bl Alanguage attIer thin recording life ston lC) Ufe stones as the ethillOlt_ of Information (Dl Complete transCrIption deSCrIptions of tribal be lEl Stringent gUideline lor the . ._ cultural data .-.",."
(A)
Information In the passage suggests that which of the follOWing may be a possible way to eliminate bias In the editing of Iile stories? BaSing all inlerences made about the culture on an ethnological theory Eliminating all 01 the emotion laden Information (B) reported by the inlormant (Cl Translating the inlormant's words into the researcher's language Reducing the number 01 questions and carefully (0) specifying the content of the questions that the Investigator can ask the informant (El Reporting all of the Information that the Informant provides regardless of the investigator's personal opinion about ItS IntrinSIC value
(AI
\ \
fIIlIIIlIfDlII
19-" refer to the passage a 42.
J9 1IIe priIllIr}'
1iIitIII CIII
1!::::==.. .
(0)
lor the popularity of a practice acco unt . he utility 01 a practice t t evalua e trate how to institute a practice demons weigh the ethics 01 uSing a strategy
IE)
explain the reasons for pursuing a strategy
IAl
fIiICllIIlIIIiI
(BI
S
1IId1lllllt IIIe firm will do if ftW sjleCify ujllCtItioIIs. PartiCularly with
-::==i.:,":uncondilional guarantee can be OWIDlllill1 tool if the client,s very lees are high, the negabve
III
t.m's
;;1::::: ~
of bad $eMce are grave, or difficult to obtain through relerrals and _lkJfonouIh NDMlier en uncondmonal guarantee can
lDlliilllJih!S hinder marketing efforts. With Its lRelwllchlthat fallure IS possible, the guarantee lilly. paradoxically, cause clients to doubt the IIIVfce firm's ability to deliver the promised level 01 servIc. It may conflict with afirm's deSire to _ r sophisticated, or may even suggest that a fimI begging lor bUSiness. In legal and health care servIc.s, /I may mislead Clients by suggesting that 'fIwId/ls or medical procedures will have Il/Iranteed outcomes. Indeed, professional service finns with outstanding reputations and performance to matl:h have little to gain Irom offering unconditional guarantees. And any firm that IIIIIlItm.nts an uncondItional guarantee without undertlking a commensurate commitment to QUI/jly of servICe IS merely emplOYing a potentially stIy martceting gimmIck.
function of the passage as a Whole
(C)
The passage's deSCription of the ,ssue raised by unconditional guarantees for health care or legal services most clearly ,mplles that which of the following IS true? (A)
(B)
are mentioned In the passage as I Ilowmgo 40 AIIolthe . . . Clrcums tanc es mwhich professl.o.nal service firms benefit Irom offering an unconditional guarantee
(C)
EXCEPT: (A) IB)
41.
The firm IS havmg dllficulty retaining ItS clients 01 long standing. The firm is having dilficulty getting bUSiness through client recommendations.
IC)
The firm charges substantrallees lor ItS services.
(D)
The adverse effects of poor performance by firm are significant for the client.
(E)
The client is reluctant to Incur risk.
Which of the following is Cited in the passage as ag of some professional service firms In offering unconditional guarantees of satisfaction? IA)
Alimit on the firm's liability
(B)
Successful competition against other firms
IC)
Ability to Justify fee Increases
(D)
Attainment of an outstanding reputation in afi
IE)
Improvement In the quality of the firm's service
43.
The legal and medical profeSSions have standards of practice that would be Violated by attempts to fulfill such unconditional guarantees. The result of a laWSUit or medical procedure cannot necessarily be determined in advance by the profeSSionals handling a client's case. The dignity of the legal and medical profeSSions IS undermined by any attempts at marketmg of professional serVices, Including unconditional guarantees.
(D)
Clients whose lawsuits or medical procedures have unsatisfactory outcomes cannot be adequately compensated by financial settlements alone.
(E)
Predicting the monetary cost of legal or health care services is more difficult than predicting the monetary cost of other types of profeSSional services.
Which of the following hypothetical situations best exemplifies the potential problem noted in the second sentence of the second paragraph (lines 15-191? (A)
A physiCian's unconditional guarantee of satisfaction encourages patients to sue for malpractice If they are unhappy With the treatment they recerve.
(B)
A lawyer's unconditional guarantee of satisfaction makes clients suspect that the lawyer needs to find new chents qUickly to Increase the firm's income.
(e)
A business consultant's unconditional guarantee of satisfaction IS undermined when the consultant fails to proVide all of the services that are promised.
(D)
An architect's unconditional guarantee of salisfactlon makes chents wonder how often the architect's bUildings fail to please clients
(E)
An accountant's unconditional guarantee of satisfaction leads clients to believe that ta returns prepared by the accountant are certa,n to be accurate.
IA (81
They com
unc (CI (D)
IE)
They outcomes are The,r fees are charged by olher The,r cijents are the quality of
Questions 45-49 refer to the passage a • • •1tllrfl1l.W
47.
B OIl Bois adVISed African
:::::.llIatinll for equality and to idarily with White Americans for of the f; r t World War The editorial : upr/llldmany African Americans who vllwed an uncompromiSIng African American and a chief opponent of the accommodabonlsl faclles urged by Booker T Washington. In fact, IIowlMI: OIl 801 often sMted poslbons along the wm between Washington and confrontatIOnist such a William Trotter. In 1895, when Wa hmgton called on African Americans to OOllC8ntrlate on Improving thllr communities Instead oppo Inl dl Crimination and agltaling for political RJht$, Du 8015 pr8lsed Washington's speech. In 190 however, Du BOIS aligned himself with TrOller, Wa hlnglon' mrlitant opponent, less for Ideological rea n than because Troller had deScribed to him Wa hnglon' ettort to silence those In the African Amen an pre who opposed Washington's po n OIl 801 ' wartime poslllOn thus reflecled not a ge In hi long term goals but rather a pr gmab re pan e In the face of social pressures: rnment of~cl8l had threatened African A:;':'."n, urnall I with censor hip If they cc eel I VOl grievance Furthermore, lived Ihal African Americans' CCIIImbullOln t p I war ettorl had broughllhem nd polill al advance Ou BOIS'
45.
The passage IS primarily concerned with fA)
Identifying historical circumstances that led Ou Bois to alter his long-term goals
(A)
(B)
de~ning "accommodaltonism" and shOWing Ou Bois used this strategy to achieve certa n
(B)
goals accounting for a particular position adopted Ou BOIS dUring the First World War
(G)
(C)
The contributions were made largely In response to Du BOIS'S 1918 editOrial. The contributions had much the same effec as African Americans' contributions to preVIous wars. The contrlbulions did not end d,scrlmlnalion agamst African Americans mthe military.
(0)
contesting the view that Ou BOIS was slgnific influenced by either Washington or Trotter
(D)
The contrlbulions were made in protest agalns Trotter's confrontalionlst tactics.
(E)
assessing the effectiveness of a stralegy that Ou BOIS urged African Americans to adopt
(E)
The contrlbulions were made primarily by cMI rights actiVists who returned to aclivism after the war.
46. The passage indicates which of the following about Ou Bois's attitude toward Washington? (A)
It underwent a shift dUring the First World War Ou Bois became more sympathetic with Trott views.
(B)
I! underwent a shift In 1903 for reasons other than Du BOIS'S disagreement with Washington's accommodationist views.
(C)
I! underwent a shift as Ou Bois made a long·t commitment to the strategy of accommodatl
(D)
It remained consistently positive even though Du Bois disagreed with Washington's efforts to control the African American press.
::~:':::
did nolla t, however Upon lemab dl rlm,nat,on e~perlenced Dr '","CIII Am"'''··n In th mllllary, he ailed on ft h from Ihe war
The passage suggests which of the followmg abo the contributions of African Americans to the Un ed States war effort dUring the First World War?
(E)
I! was shaped primarily by Ou BOIS'S appreda of Washington's pragmatic approach to the advancement of the mterests of African Amencans.
48.
The author of the passage refers to Washmgton's call to African Americans in 1895 primarily in order to IA)
(B) (G) (0)
(E)
Idenlify Du Bois's characterislic position on the contmuum between accommodalionism and confrontatiOnlsm e~plam why Du Bois was sympathetic with Washmgton's views In 1895 c1anfy how Trotter's views differed from those of Washmgton In 1895 support an assertion about Du BOIS'S tendency to shift hiS political posllions diSmiSS the claim that Du BOIS'S pOSllJOn In hiS 1918 editorial was consistent vlth hiS pre us VieWS
49 true Air an Am.",.,.....
a straIlI!IN-'" B
D
E
1IIIll1ll.1Ide fur GMA~ Verba' Review 2nd Edition
.
QuestIons 56
Line In an attempt to Improve the overall performance of c/errcal workers, many companies have Introduced computerrzed performance mOnltonng and contro,1 systems (CPMCS) that record and report a worker s (5) computer-dnven actiVities. However, at least one study has shown that such mOnltonng may not be haVIng the deSired effect. In the study, researchers asked mOnitored c1encal workers and their supervisors how assessments of productiVity (10) affected supervisors' ratings of workers' performance. In contrast to unmonltored workers dOing the same work, who Without exception Identified the most Important element In their Jobs as customer serVice, the mOnitored workers and (5) theIr supervisors all responded that productiVity was the cntlcal factor In assigning ratings. This finding suggested that there should have been a strong correlation between a monitored worker's productiVity and the overall rating the worker (20) received. However, measures of the relationship between overall rating and indiVidual elements of performance clearly supported the conclusion that supervisors gave considerable weight to criteria such as attendance, accuracy, and indications of (25) customer satisfaction. It IS pOSSible that productiVity may be a "hygIene factor"; that IS, If It IS too low, It will hurt the overall rating. But the eVidence suggests that beyond the POint at which productiVity becomes (30) "good enough:' higher productivity per se IS unlikely to Improve a ratrng.
I I
I
-60 refer to the passage abo lle• 58.
I
(A)
That wor kers With the highest productiVity wo also be the most accurate
I
(8)
That workers who initially achieved hIgh productivity ratings would continue to do so consistently
I (e)
57.
Which of the followrng, If true, would most clearly have Supported the conclusion referred to In line 22
. the passage, before the final results of 56. According to k own which of the follOWing seemed the study were n , likely?
(A) (B)
(C) (O)
That the highest performance ratings would be achieved by workers With the highest productiVity
{DJ
That the most productive workers would be those whose supervisors claimed to value productivity
(E)
That supervisors who claimed to value productivity would place equal value on customer satisfaction
(E)
59.
(8)
(C)
Most supervIsors based overall ratings of performance on measures of ProdUCtIVity alone Overall ratings of performance correlated more highly With measures of productiVity than the researchers expected. Overall ratings of performance correlated more highly With measures of accuracy than With measures of productivity.
According to the passage, a "hygiene factor" (Irne 27) is an aspect of a worker's performance that has no effect on the rating of a worker's performance
(8)
IS so baSIC to performance that It is assumed to be adequate for all workers
(C)
compare the ratings of these workers with the ratings of monitored workers
IS given less importance than It deserves In rating a worker's performance
(D)
is not likely to affect a worker's rating unless It IS judged to be inadequate
provide an example of a case monitoring might be effective
(E)
IS Important pnmanly because of the effect It has on a worker's rating
In
which
provide evidence of an inappropriate use of
CPMCS (D) (E)
Electronic monltonng greatly Increased productiVity.
(A)
It can be inferred that the author of the passage discusses "unmon!tored workers" (line 11) primarily In order to (A)
Ratings of productiVity correlated hIghly WIth ratings of both accuracy and attendance
60.
The pnmary purpose of the passage IS to
emphasize the effect that CPMCS may have on workers' perceptions of their jobs
(A)
illustrate the effect that CPMCS may have on Workers' ratings
explain the need for the mtroduct/on of mnovatlve strategy
(8)
discuss a study of the use of a method
(Cl
recommend a course of
(D)
resolve a difference of
(E)
suggest an alternati
61-
QuestlO"S
63
refer to t e passage a
63. se of the passage
·.II'COCOlIISIIIIIIIIIJII/IIRM! forests are usually
::~:~ mangr0Y8 5P8c1es found
psJu,*1Il1tIY in the seaward portion of the habit~t oItIIr mlflll'0Y8 5P8c1es on the more landWar
portions at the coast. The earliest research on mIflII'0Y8 forests produced descripliOns of species dIstribution from shore to land, without exploring the causes of the distributions. The Idea that zonatIon IS caused by plant (l0) succession was first expressed by J. H. DaVIS In a study of FlOrida mangrove forestS. According to Oms' scheme, the shoreline IS being extended In a seaward direction because of the "land-building' role at mangroves, WhiCh, by trapping sediments over (l5) time, extend the shore. As a habitat gradually becomes more mland as the shore extends, the .'and-buddlng' species are replaced. ThiS continuous process of accretion and succession would be Interrupted only by hUrricanes or storm flushlngs. (20) Recently the universal application of Davis' succession paradigm has been challenged. It appears that mareas where weak currents and weak tidal energies allow the accumulation of sediments, mangroves will follow land formation (25) and accelerate the rate of soil accretion; succession will proceed according to DaVIS' scheme. But on stable coastlines, the distrrbutlon of mangrove species results In other patterns of zonation; "land budding' does not occur. (30) To find a prrnClple that explains the varrous distribution patterns, several researchers have looked to alinity and ItS effects on mangroves. Whde mangroves can develop mfresh water, they can also thrive msalimtles as high as 2.5 times that (35) at awater However, those mangrove species found In freshwat r habitats do well only In the III nc of compet,llon, thus suggestmg that nlty tol rance IS a critical factor," compet,llve .mona mangrove species. Research tIlIt manarov 5 WIll normally dominate no r alon5, .Ithough not b c use they r, th y r m t bohc lIy efficient ....nltyW I) In portion of n nVironment '. Iud plln d pt d to dIff r ntd.r 50f
CftIrlCtlr!
IS
to
The primary purpo 61. dea that the zonation exhibited in efute the I (Al r forests is caused by adaptation to mangrove salinity . 'b the pattern of lonatlon tYPically fa descrr'da e mangrove forests (B) In FI orr . ' at Davis' succession paradigm cannot th (el argue successfully applied to Flon'd a mangrove fore (D)
(E)
discuSS hypotheses that attempt to explain the zonation of coastal mangrove forests tablish that plants that do well in saline fore es environments require sa It to ac h'leve maximum metabolic efficiency
According to the passage. the earliest research on
62. mangrove forests produced which 0f the following) (Al
Data that implied random patterns of mangrove
(8)
species distribution Descriptions of species distributions suggesting
(e)
zonation Descriptions of the development of mangrove
(D)
forests over time Reclassification of species formerly thought to be identical
(El
Data that confirmed the "land-building" role of mangroves
It can be inferred from the ...... t1. . . . .o-.~ paradigm does NOT apply to which at The shoreline of Flonda mangrove for studied by DaviS (8) A shoreline In an area with weak cooents (e) A shoreline," an area with weak tidal energy (D) A shoreline extended by ·'and-bulldmg" specIeS of mangroves (E) A shoreline," which few sediments can accumulate
(A)
.....I1111'. ...ilI-"'" refer to the passage ab
primIIY purpose
of the passage IS to
66.
demonstrating the benefits _""'" researc h . . or ."".... gers and aCQuIsitions and vv rate mer clJlPO e of the drawbacks that aCQuls examine sam behaVIor entails ntrast the effects of corporate mergers and r8) co on aCQuIring firms and on firms aCQUIsItIOnS are aCQUired • dngs that raise Quesllons abou. t a re) report lin Icorporate mergers. and aCQulSl!lons reason for and suggeSt possible alternative reasons ,.,
IIlItpostfilmS were not lJIIid to ob!8ln
1fImOIIstrIled t/1af, fo/kIWJrIg prospec:lfwe rnergeJ: the IC'QlIIrInc firm tends to IIIUl:h IUs thin does that of the YIt mergers and acquisitions IIIlI bidders continUe to assert
MIl hIw the deSirable effect of IIIIfon's llsources efficiently from less IIClDrs of Its economy. but the executives arranging these fIIIm. .dvanclng either their own f)riwIte economic Interests. It having little to do WIth corporate mIIruts expl./n .cqUls,!lons. These lllIl~ fII,clad. the Incenlive compensation of of monitoring by boards of
.
. . .rilmn. tIrI.1Id for acqUIsition. Alternatively,
may derIVe from :::=:~:me""erofdoebIdders what other managers do.
The findings cited in the passage suggest which of t 65. f t following about the outcomes a corpora e mergers and aCQuisilions with respect to acqUlnng firms? (A)
They include a decrease in value of many acqUiring firms' stocks.
(8)
They tend to be more beneficial for small firms than for large firms.
(e)
They do not fulfill the professed goals of most acqUiring firms.
(D)
(E)
(A)
Their known benefits to national economies explain therr appeal to Individual firms dunng the 1970s and 1980s.
(B)
Despite therr adverse Impact on some firms, they are the best way to channel resources from less to more productive sectors of a nallon's economy.
(C)
anges In atlltude on the part of (D) expIaln ch aCQulring fi rms toward corporate mergers and aCQulsllions (E) account for a recent decline In the rate of corporate mergers and aCQUIsitions
.11 economIC ones.
It can be Inferred from the passage that the author would be most likely to agree With whIch of the follOWing statements about corporate acqulsillons'
67.
68
lA B
They are as likely to occur because of poor mOnitoring by boards of drrectors as to be caused by Incentive compensation for managers.
(D)
They will be less prevalent in the future. since therr actual effects will gain Wider recognition.
(E)
Factors other than economic benefit to the aCQuIring firm help to explain the frequency with which they occur.
69
(B)
were modeled
10
anll Dated gr prospeQNe
IE)
expene ed when apr
According to the pa age true of corporClle ac 1970s and 1980s IA)
Few of the acqullSilions Nt till.... subsequently d ested
(B)
Most such acquISition Increases ,n 8CQlJlred
tC)
provide an explanation for the mergers and acquiSitions of the 1970s and 1980s overlooked by the findings discussed in the passage
Most such aCQu siti n overesbmabon of the valueofll1n".
tD)
suggest that national economic interests played an Important role In the mergers and acquisitions of the 1970s and 1980s
The gains real zed b m not equal the amounts eixpt!l1dlllll target firms
tE)
About half of such alCQlUSlliIllllS ,ncreases In the al e
The author of the passage mentions the effect of aCQuIsitions on national economies most probably In order to
(Al
C)
They tend to be beneficial to such firms In the long term even though apparently detnmentall the short term.
IC)
They discourage many such firms from attempting to make subsequent bids and acqUisitions.
support a noneconomic explanation for the mergers and aCQuisitions of the 1970s and 1980s that was cited earlier in the passage
(Dl
cite and pOint out the inadequacy of one possible explanation for the prevalence of mergers and acquiSitions dunng the 1970s and 19 Os
(A)
explain how modeling affected the deciSions made by managers Involved In mergers and acqUisitions dunng the 1970s and 1980s
(B)
(E)
70.
leI
manaaer (D
(E
l
on
IiItIllvI reetlIlIY
• •_ be/IIViOr by file IIIlInIn brain of a _."'d1I1lIllNllmlcal caIted adenOsine depn!sses neuron finng III many apparently does thiS by
~!!:::~ Of neurotransmitters, .:::nerve Impulses from one ~
tMIIY other agents that affect neuron ~~~:=::::mu5t first bind to specific
::.
on nauronal membranes. There are at
two classes of these receptors, which have A and Az' Snyder et al. propose fblltGilffeine which 15 structurally similar to Menasine 15 able to bind to both types of tIOIPlors, which prevents adenosine from attaching nllld allows the neurons to fire more readily tIIln they otherwise would. For many years. caffeine's effects have been ettrIbuted to its Inhibition of the production of D1I01phodl8slerase, an enzyme that breaks down IIJe memlcal called cyclic AMP. A number of -.rotransmillers exert their effects by first tJerulsil/li cyclic AMP concentrations in target Iliumns Therefore, prolonged pellods at the I/Iveled concentrations. as might be brought about IlIIosphod,esterase Inhibitor, could lead to a I/UlII' Imount of neuron filing and, consequently, bIhIvioralstlmulatlon. But Snyder et al. pOint out ,c:·"flllttllll effe,ne concentrations needed to inhibit lIrOduc:tion of phosphodllsterase In the brain AmItl11 than those that produce Moreover, other compounds that block "'''1 es ICtivity are not stimulants. their cise that caffeine acts
some apparent exceptions to the - rrel were observed between adenOSinel co aliOn and stimulatton. One of these wit receJllof bln~:~ed 3.,sobutyl·!·methylxanthlne acompound d very well but actually "VI which boun d (55) 6BlftM locomotion Sny er et al. Sugge e . sedmous dePres aJor stumbling block to their that this IS nToht ap~oblem IS that the compound has hypotheSIS. .e the brain a not unusua I oc.currence effects In ' miXed ychoac tIVe drugs . Even caffeine, which IS (601 With ps only for ItS stimulatory effects rally known . ' gene s thiS property, depreSSing mouse display t ery low concentrations and locomotion a v stimulating It at higher ones.
Questions 71-76 refer to the passage above
aenera
71.
The pnmary purpose of the passage IS to (A) (B) (C)
(E)
challenge the validity of a theory by exposing the Inconsistencies and contradictions In It
:
ry effects 01 a selles of
wItIllhtir Ibltity to dislodge
III-'::::::.tlIt:brIln:101
mice
Ufthe ..elllu ".-correlltel
(B)
Ie)
75
According to Snyder et aI., caffeine differs from adenOSine In that caffeine stimulates behaVior In the mouse and In humans, whereas adenosine stimulates behavior in humans only
(B)
has mixed effects in the brain, whereas adenOSine has only a stimulatory effect
(C)
Increases cyclic AMP concentrations In target neurons, whereas adenosine decreases such concentrations
(D)
permits release of neurotransmitters when It is bound to adenosine receptors, whereas adenosine inhibits such release
(E)
""her
:::ed~e:noslne bInding. Snyder et
present two explanations of a phenomenon and reconcile the differences between them summanze two theories and suggest a third theory that overcomes the problems encountered In the first two descnbe an alternative hypothesis and provide eVidence and arguments that support It
(A)
73.
tAl
diSCUSS a plan for investigation of a phenomenon that IS not yet fully understood
(0)
_""ated
n.
74
inhibits both neuron filing and the production of phosphodiesterase when there is a suffiCient concentralton in the brain, whereas adenOSine Inhibits only neuron filing
In response to experimental results concerntng IBMX, Snyder et al. contended that It IS not uncommon for psychoactive drugs to have (A)
mixed effects In the brain
(B)
inhibitory effects on enzymes In the brain
(C)
close structural relationships With caffeine
(D)
depressive effects on mouse locomotion
(E)
the ability to dislodge caffeine from receptors In the brain
76.
(01
!heophyIn1
lEI
phosp
Snyder et al suu AI and A receptors to which of the to (A)
The chemICal r phosphodiesterBe
(B)
The structural r adenOSine
(C)
The structural Similarity neurotransmitters
(0)
The ability of caffeine t
(E)
The natural occurrence of adenOSine in the brlln
The author quotes Snyder et aI. probably In order to (A)
reveal some of the as theory
IBI
summarize a maIO'
IC)
POint out that their the mouse
(0)
IndICate that ltIIir general COtleleli_
(E)
refute the older theory
-84 refer to the passage abo"..
Questions 77
80.
--=~'=~IInllId~S at .. of fema'e seMCe -::::~:: wages In occupations III domeStiC servant, and office
ssage, job segregation by sex '"
According to the pa the United States was
7Z
A) (
IIJstonans tocused ,nstead on - - - primariJy because it seemed so lrlditional, unpaId "women's work' ,n lII'fIlIme. IIId because the underlying economIc Industriafism were presumed to be I ~:::""":rd~~end hence emanclpatory In eftect b emancIpatIon has been less profound _.expected tor not even Industnal wage labor escaped continued sex segregatIOn mthe
(B)
(C)
one mean s by which women achieved greater
(DJ
Job secunty reluctantly challenged by employers except when the economic advantages were ObVIOUS
WlIItlpIace (E)
IC)
They argued that women were Inherently sUited to do well In particular kinds of factory work. They thought that factory work bettered the condition of women by emancipating them from dependence on Income earned by men.
(A)
(8)
required skill mdetailed tasks
(8)
(C)
was assumed to be less characterized by sex segregation
(E)
was more readily accepted by women than by men
histOrians of women's labor In the United States paid little attention to women's employment In the service sector of the economy because IAJ
(B)
IC)
(0)
lEI
(el
(0)
(E)
They felt gUilty about disturbing the traditional diVISion of labor ,n the family.
entry of women Into the Industrial labor market recognition that work done by women as homemakers should be compensated at rates comparable to those prevailing in the service sector of the economy
(C)
development of a new definition of femininity unrelated to the economic forces of industrialism
(0)
introduction of equal pay for equal work In all professions
(E)
emancipation of women wage earners from gender-determined job allocation
fit the economic dynamiC of industrialism better
79. It can be mferred from the passage that early
IBI
It can be inferred from the passage that the "unfinished revolution" the author mentions in line 15 refers to the
involved the payment of higher wages
(D)
(AI
83. 81.
(A)
""'IlJllllItll beckoned And dunng
They sought to Increase the size of the aVlllable labor force as a means to keep men's wages low.
(E)
82
They hoped that by creating relatively unattractive "female" jobs they would discourage women from lOSing Interest In marriage and family life.
(B)
(0)
aconstant source of labor unrest In the young textile mdustry
78. According to the passage, h,storlans of w.omen's labor focused on factory work as a more prom'Slng area of research than service-sector work because factory work
"nco,
_ _NIl
(A)
labor
_it...
To explaIn th,s unfinished revoluoon mthe ItIlus ot women, hlstonans have recently begun to Mlphlsue the way aprevllllng definition of flImlnlnlty otten determmes the kmds of work led to women, even when such allocallon 's pproprlate to new condItIons. For mstance, early textrle-mill entrepreneurs, mJustIfying women's mployment In wage labor, made much of the umption that women were by nature skillful at detailed tasks and patient In carrying out repet,tlve hare the mill owners thus imported into the new U al order hoary stereotypes associated With tile homemaking activIties they presumed to have been the purvieW of women. Because women pltd the more unattractive new mdustnal tasks reid Iy than dId men, such Jobs came to be ·-'ftilfl/illed I temale Jobs. And employers, who I~'~::: thl! women's ·real' aspIrations were tor ~ nd flmily hfe, declined to pay women ::::lUrate with those of men. Thus : /Hed lower-pajd. less Secure Jobs ~ peUl/td a ·female' IIfllllJllrkable than the orll,nal has been the ex segregation In twentieth. IIJUIJ· In upation came to be 1IIIIIi•."1II1ll/Oye howed . . .un_Nthll
shed by labor mobilization dunna greatlY d,ml nl • th Second World War e t d by those textile·mlll owners Who perpetua e 'I argue d In favor of women s emp oyment In Wage
The passage supports which of the followmg statements about the early mill owners mentioned In the second paragraph?
Even war Indus1l~es:::~:::::: War were relucta work The service secto more nearly gende 'b than has the manufaet1~rilllll"fi'iI
Which of the followmg word be opinion of the author at the pa notion that women are more ski carrying out detliled tasks
(B)
"patient' (line 24) "repetItive· (Ime 24
IC)
"hoary" (Ime 26)
(0)
"homemaking ( ne 2
(E)
"purview" (line 28
(A)
84
Ind women homemake Post-Se women 101 employment apoorllunilhl.
Which of the following bes of the final paragraph 10 (AI
The centra Idea eVidence d awn
(Bl
The central forma
IC)
The
tewer women found employment In the service sector than in 'actory work
(0)
A
the wages paid to workers In the service sector were much lower than those paid In the Industr sector
I
the extreme vanety of these occupations made It very difficult to assemble meaningful stalislic about them
women's employment In the serVice sector tended to be much more short-term than in actory Work ' :iIOyment In the Service sector seemed to ~~ In common With the unpaid work "'I8U WIth homemaking
- : : : : : : : have been used OIl " l!ftIiIR:ipatiO nwestern
~;~Arpments mwhatmaintain could bethe called thene f8miIIist tradition docln
~
III diffeA!nce,· or equity as dlsbnct from
dlvldualrst framework, With ItS cl Line women·lfn~~~onomY, could be harmonized With for WOm e t d concerns of relational feminists, f mly.Qnen e the a I . f I model for contemporary feminist amore frUit u politicS COUld emerge. (55)
TIrey posit that blologrcal dlsbncbons 11I11..11I tile sexes result In anecessary sexual division of labor In the family and throughout sOCiety IIICIlIIIt women's procreative labor IS currently
undetvalued by SOCiety, to the disadvantage of women By contrast, the IndlVldualrst femlmst lnJditiDrl emphaSizes mdlVldual human nghts and elebrlles women's quest for personal autonomy, while downplaymg the Importance of gender roles iii and mlRlmlzmg diSCUSSion of childbeanng and ItS lIttendant responslbllibes. Before the late mneteenth century, these views coexisted Within the feminist movement, often wltllln the writings of the same indivIdual. Between 201 1890 and 1920, however, relallonal feminism, which had been the dominant strain In femlmst thought, and which stili predominates among European and non Western femlmsts, lost ground in England and the Umted States. Because the concept of Individual rilhts was already well establrshed in the Anglo· Saxon lelal and polrtlcal traditIon, IndlVldualrst femlRlsm came to predominate In Englrsh·speaklng countrieS At the same time, the goals of the two approaches began to seem Increasingly rreconCllable Indlvldualrst femlmsts began to advocate a totally gender·bllnd system With equal IfIIIlI for all Relational femlmsts, while agreeing that equal educational and economiC opportumtles outllldethe home should be available for all women ClGIIIlnuId to emphaSize women's special ' n to society as homemakers and
86.
(A)
(B)
. 85-90 refer to the passage abOve Questions . 85.
h f the passage alludes to the wellThe aut or a . .. bl h d nature of the concept of IndiVidual nghts esta IS e .. . the Anglo-Saxon legal and polrtlcal tradition In order to (A)
(B)
(0)
English history argue that feminism was already a part of the larger Anglo-Saxon Intellectual tradition, even though this has often gone unnoticed by cntics of women's emancipation explain the decline in individualist thinking amo feminists in non-English-speak 109 countries
(0)
help account for an increaSing shift toward individualist femlRlsm among femmlsts in English-speaking countries
(E)
(C)
illustrate the influence of Individualist feminist thought on more general intellectual trends In
(C)
account for the philosophICal differences between individualist and relational femiRlsts In English-speaking countries
The passage suggests that the author of the passage belreves which of the follOWing'
(E)
87.
The predominance of Indlvldualrst feminism In Englrsh-speaklng countnes is a hlstoncal phenomenon, the causes of which have not yet been investigated
(AI
The Indlvldualrst and relational femlmst views are Irreconcilable, given therr theoretICal differences concerning the foundations of society.
(el
IBI
A consensus concerning the direction of future feminist POlrt,CS will probably soon emerge, given the awareness among femimsts of the need for cooperation among women. POlrtical adversaries of feminism often misuse arguments predicated on differences between the sexes to argue that the eXisting SOCial system should be maintained.
(A)
89.
Relational feminism provides the best theoretical framework for contemporary femlmst pOhtlCS, but Individualist femlRlsm could contribute much toward refiRIng and strengthening modern femiRlst thought.
A diVISion of labor 10 a social group can result in Increased effiCiency With regard to the performance of group tasks.
(B)
A diVISion of labor In a SOCial group causes Inequities in the distribution of opportuRltres and benefits among group members_
(C)
A diVision of labor on the basis of gender 10 a SOCial group IS necessitated by the eXistence of sex-linked biological differences between male and female members of the group.
(D)
(E)
Culturally determined distinctions based on gender In a SOCial group foster the e Istence of differing attitudes and opinions among group members. Educational programs aimed at redUCing Inequalities based on gender among members of a social group can result in a sense of greater well-being for all members of the group
the shoukI be laws guar Clbzen
It can be inferred from the passage that the rndivldualrst femlRlst tradition denies the validity of which of the following causal statements?
III Protective 'ellslatlon for women sored maternity benefits, and forhou work n hIvt a major pl"all: WOIIItII physiological and they Ire often IdvIrurI and uled 10 Indivldual I _NO~II. dlllyfnilha
88
90.
101
a greater dejpee con emmgthe would be benefic
IE)
the same eGJc opportumties
Accord ng to the autho wh of femlRlst thought In we (AI
lndllnduahst fernm st ill In the thought or wrltil1ll feminists
(B)
IndNlduahst feminISm thought. but another predominated
(C)
Relabonal and 100 equally prevalent In fe
(0)
The predominant VIew the welfare of women Important than the
(E)
The predominant VII!W the sexes shoo d ree the law
The author Implies that of most !emIR st Stales after 192O?
IAl 181
Questions
94.
-:~~ ~ un/VeI'S! ~ UIiIII recenIIY went
the passage, conventional SPiral According ta , ,g 91. f 10w_surface-brightness galaXies In differ rom the follOWing ways?
~~::~ PoSSIbly as numerous
bI
these galaXies have the
!~=~shape evenas thea same number and of stars common type of
~.1Ii:
I galaX}( the spiral. but tend to be much 8eeause these galaXies' mass IS spread out 1l!I;"" ..... lilli' areas. they have far fewer stars per Unit voIumI than do conventional galaXies. Apparently kJw.surface-bnghtness galaXies. as they are CIIted take much longer than conventional galaXies to concIense their pnmordl8l gas and convert It to ~atls. they evolve much more slowly. 1 These galaXies may constitute an answer to the lo""standlng puzzle of the missing baryonlC mass mthe unIVerse. Baryons-subatomic particles that are generally protons or neutrons-are the source of stellar. and therefore galactiC, luminosity, and so their numbers can be estimated based on how luminous galaXies are. However, the amount of helium ,n the unrverse. as measured by peclroscopy. suggests that there are far more baryons In the unrverse than estimates based on 5) galactic luminOSity Indicate. Astronomers have long peculated that the missing baryonic mass might eventually be discovered In Intergalacllc space or as ome large population of galaXies that are difficult to detect.
91-97 refer to the passage a
(AI (BI (C)
They contain less helium than do low-surface. brightness galaxies.
(E)
They are larger than low-surface-brightness galaxies.
(A)
they, like conventional galaXies that contain many baryons, have evolved from massive. primordial gas clouds
(C)
they may contain relatively more helium, and hence more baryons, than do galaXies whose helium content has been studied uSing spectroscopy
(EI
They are large spiral galaXies containing fewer stars than do conventional galaxies.
(B)
They are compact but very dim spiral galaxies
(CI
They are diffuse spiral galaxies that occupy a large volume of space.
(D)
They are small, young spiral galaxies that contain a high proportion of primordial gas.
lE)
They are large, dense spirals with low lumino
95.
(A)
It can be Inferred from the passage that the "longstandmg PUzzle" refers to which of the follOWing?
(C)
(A)
The difference between the rate at which conventional galaXies evolve and the rate at which low-surface-brlghtness galaXies evolve
(0)
aryonlc mass derived from measuring helium and estimates b ased on measuring galactiC I " umlnoslty
(E)
(81
Ie) (0)
IE)
~he discrepancy between estimates of total
The mcons t of h I' .IS ency between the observed am elum In the u in t ' nlverse and the number of s yplcallow-su f . U ' r ace-brightness galaxies nCertlinties r . baryonic egardlng what proportion ,of mass IS Cont . . and what pro ' . alned In IntergalactiC s O,ffi I . Portion In conventional galaXies cu ties Involved' aalaxles ad' In detecting very distant n 10 mvestlgallng their luminOSity
they have recently been discovered to contain more baryonic mass than scientists had thought when low-surface-brightness galaXies were first observed they contain stars that are sigOificantly more luminous than would have been predicted on the baSIS of 100tiai studies of luminosity in lowsurface-brightness galaxies
The author mentions the fact that baryons are the source of stars' luminosity primarily In order to explain
(B) 93.
they contam baryonic mass that was not taken Into account by researchers uSing galactiC luminOSity to estimate the number of baryons 10 the umverse
(B)
(0)
92. It can be Inferred from the passage that which of following IS an accurate phYSical description of typ low-surface-brightness galaxies? (AI
96 (A
They have fewer stars than do low-surface. brightness galaXies. They evolve more qUickly than low-surface. brightness galaXies. They are more diffuse than low-surface_ brightness galaxies.
lDI
The author Imphes that low-surface-brightness galaXies could constitute an answer to the puzzle discussed In the second paragraph primarily because
how astronomers determine that some galaXies contam fewer stars per UOit volume than do others how astronomers are able to calculate the total luminOSity of a galaxy why astronomers can use galactiC luminOSity to estimate baryonlC mass why astronomers' estimates of baryonic mass based on galactiC luminOSity are more reliable than those based on spectroscopic studies of helium how astronomers know bright galaxies contain more baryons than do dim galaxies
IB)
l In
IC
A abo wiler found
101
Astro me esbmabng the unrverse Astronomer amount of bary
(E)
97 (AI (B)
(C)
descrrbe a phenome sClenbfic slgmli an I contrast two phenome difference between Idenbfy a newly dl explalO Its onglOs
(D)
compare two c1asse phYSical properbe
(El
diSCUSS a dlscove and Inconsistency w th e
_ _lIdlnNn an.rIicIe publiShed In 1992.
no bluepnnt for transforming a largely IIIIlIk:OIlltrollled economy Into afree one. the , : :of the United Kingdom since 1979 • one .pproach that workS: privItiution IrI whICh state-owned Induslnes are ...dtD1l'tv81l1 companies By 1979. the total tloInMilIlS and losses of state-owned Industnes WIle rumlng at about £3 billion a year. By seiling tMIIY of these Industries. the government has decre8sed these borrowings and losses, gained CIIIIr £34 bIllion from the sales. and now receives II revenues from the newly pnvatrzed companies. Along with a dramatically Improved overall economy. the government has been able to repay 1 5 percenl of the nel nallonal debt over aIwo· yurpertod In facl. prtVallzallon has not only rescued individual mdustrles and a whole economy headed for dl$8ster. bul has also raised Ihe level of performance ,n every area. At Bntrsh Airways and British Gas, for example, productrvlIy per employee IIa n en by 20 percent. At Associated Bnlrsh P IIbor dlsruplrons common In the 1970s and urIy 1980 have now vlrlually disappeared. AI IIritish Telecom. there IS no longer a wailing list-as there Iwsy was before pnvallzalron-Io have a lIIephone nslBlled Part of Ihl Improved produClrvlty has come IIbout be au e the employees of pnvatlzed trie were Illven the opportunity to buy shares llleit own ompanles They responded 1IlII.lskally to the olfer of shares: at Bntlsh
~
=:~~8~~9 per enl ollhe elillible work force
••1As oClaled Bnllsh Ports. 90 IIId 81 Br/llsh Telecom. 92 percent. When PI1S0111I like In somelhlnll, Ihey .boull!. work 10 make II NItIonIl Fr Ishl Con orllum, Ihe IFIW con rned .boul
durtnlwlIl thllr union 10
as Paine's pOint that 'what we obtain toa line miSS Thom t mtoo lightly." In order lor the lar. we es ee cheaP benefits 01 Individual ownership to be rangl~:d by owners. companres, and countnes, achle and other indiViduals must make their (50! employeeS ..ons to buy and they mu.st commit same ' own decis l ources to the chOice. res n of their ow
Questions 98-104 refer to the passage above. 98.
According to the passage, all of the follOWing were benefits 01 pnvatlzlng state·owned industnes rn the Unrted Kingdom EXCEPT: (A)
99.
Pnvatized Industnes paid taxes to the government.
101
W
IBI
(B)
The government gained revenue lrom selling state·owned Industnes.
(C)
The government repaid some of Its national debt.
IC)
(D)
Profits from Industnes that were stili stateowned rncreased.
(D)
(E)
Total borrowrngs and losses of state·owned Industries decreased.
According to the passage, which of the follOWing resulted in increased productiVity in companies that have been privatized? (A)
A large number of employees chose to purchase shares in their companies.
(B)
Free shares were Widely distributed to indiVidual shareholders.
(C)
The government ceased to regulate major Industries.
(D)
Unions conducted wage negotiations for employees.
(E)
Employee·owners agreed to have their wages lowered.
(E)
(B)
a positive sign of employee concern about a company
(C)
a predictor of employee reaction to a company's offer to sell shares to them
(D)
a phenomenon found more often In tate-owned Industries than rn private companres
(E)
a deterrence to high performance level In an rndustry
Compan es that ertlOl15lr.1IId ........,.;. productivity were employees the 00"'1\I1"n,"" Ehglb tty to buy share employees agree g
With the pnnclple descnbed (A)
A democratic govemm".ntlhalt decillllS rnappropr ate to a a ppa~'~c:':r:.:::~:: no ay abd cated s eSI of he public nte est
(B)
(C)
(DI
an ineVitable problem In a weak national economy
share ApproJUmal at three differe shares the. COITIjIiII1I8S. The opportun by at least so e
102. Which of the followrng s ateme
100. It can be Inferred from the passage that the author considers labor disruptrons to be (A)
difh!re~::~=
At outthree of ten
(E
The Ideal way lor a govemnlelliit t:::~.. emp oyee Interests C1 ma nta n the r sha e thout gave me!nt !iUb!iicIiIIlS. The fa ure t ha ne IS an mporta t re rndustr es pe poclrly. Governments pr abza reSlsta e
following can be Inferred frOIl1:eUnited ssage about the pnvalizaliOn process In Kingdom? degree on It depends to a potenlJally dangerouS ndMdual ownership of shares. r to IBI It conforms In its most general out Ines Thomas Paine's prescnptlon for business ownership. Ie) It was onglnally conceived to include some gIVIng away of free shares. 10) It has been successful, even though pnvatizatlon has faded in other countnes. (E) It IS taking place more slowly than some economists suggest is necessary.
l.
e
27.
B
53
0
2.
D
28.
D
54
E
3.
B
29.
B
55
B
4.
D
30.
E
56
5.
e
B
3l.
A
57
D
6.
E
32.
e
58.
E
7.
B
33.
D
59.
0
8.
34.
e
60.
B
9.
A A
35.
A
6l.
D
10.
E
36.
E
62.
B
1l.
37.
e
63
E
12.
e e
38.
B
64.
13.
B
39.
B
65.
e e
state a solution to aproblem described 10 the prevIous sentence
14.
E
40.
A
66.
E
15.
e
4l.
B
67.
0
show how opponents of the viewpoint of the author of the passage have supported their arguments
16.
D
42.
B
68.
E
17.
A
43.
D
69.
18.
E
44.
E
70
D B
19.
B
45.
C
71.
0
20.
e
46.
B
72.
D
2l.
E
47.
C
73
A
22.
A
48.
D
74.
E
23.
49.
E
75
24.
D D
50.
B
76
B B
25.
B
5l.
e
77
B
26.
B
52.
A
78
(AI
104. The Quotation In lines 46-47 is most probably used to (A) (B)
(C)
(01
IE)
3.5 Answer Key
counter a posilion that the author 01 the passage believes is Incorrect
POint out a paradox contained In acontroversial viewpoint present a hlstoncal maxim to challenge the principle Introduced in the third paragraph
Evaluation
.' . e yOU with the most l!ffl f ml hariZ . h I 'ntended to ~ comprehension. T e particular mpreftenSlOn Is n to reading d' g comprehension qUl!sti =:=::=:of rNdmg co comma . ~ til /lIm/s of problemS f the kmds af rea . gInstrategy that .IS Important to e sentaliVe bre salvin , ....alii dlIpt8r are generally repre I it IS Ihe pro m on tile GMAT. Remember tha a particular quesllon.
~
Answering this question requires under tandln~ how a particular part of the pa sa~e function In the passage as a whole. The econd sentence describes the GDP as being solely concerned with the prices of goods and services produced in the United States. aside from any other kind of value The passage then goes on to imply that by ignoring value other than price, the GDP may actually mask problems present in the nation's overall economy.
°
==::~::ils~of -fj refw to
Main idea .. . quires determmmg the main Th'IS ques tlon re h I Th . h passage as a woe. e pa sage purpos~\~~:o:dl)' defining GD~ and describing IOs beg .. d s an indicator of the economic h W ItlS use a Th o . of the United tates. e passaget well-beIOg h' d. 'b . more detail w at I. an IS not taken descn es 10 d h d .IOto account b\'. the GDP. an. t en raws a ca f . between the ItmltatlOns 0 what the connection . . . CDP measures and disturbmg trends WithIn t!Jd U.S. in recent decades.
the passage on page 22.
pnmery pulJ)Ose of the passage IS to entity ways In whICh the GOP could be modified so that .t would serve as a more accurate Indicator of the econom,c well·belng of the
United States I uuest that the GOP, In spite of certain hortcomlngs, IS stili the most reliable Indicator of the economic well·belng of the United States I exlmlne crUCial shortcomings of the GOP as an indicator of the econom,c well·belng of the United States (0) Iriue that the growth of the United States economy In recent decades has diminished the effectiveness of the GOP as an indicator of the nltion s economic well·being l£l d' u s how the GOP came to be used as the prlmlry Indicator of the economic well·belng of the Umted States
A
B
C
D
A
There is no discussion in the passage about modifying how the GDP is calculated. B The passage makes no judgment about the merits of using the GDP in relation to ot economic indicators. C Correct. The passage portrays the GDP as having limitations that make it a problem indicator of real economic well-being. D The passage does not portray the GDP as being any less useful as an economic indicator than it ever was. E There is no discussion in the passage of the history of how the GDP came to be usedll an economic indicator. The correct answer is C.
E
3.
It can be Inferred that the author of the passage would agree With which of the follOWing about the "economic significance" of those goods and services that are Included in the GDP? (A)
Which of the follOWing best describes the fundon 01 the second sentence of the passage In the context the passage as a whole?
(C)
(A)
(D)
(B) (e)
It descnbes an assumption about the GDP that IS defended In the Course of the passage. It contnbutes to a diSCUSSion of the origins of the GOP. It clanfies a common misconception about th use of the GOP.
(0)
IE)
It Identifies a major flaw In the GDP. It Suggests a revIsion to the method of calCUlating the GOP.
The passage is concerned with calling IOta question the use of the GDP, not defending it. The passage does not mention how the GDP came to be used as a primary economic indicator. The passage does not describe the function of the GDP as being commonly misunderstood. Correct. The limitations of the GDP as described in the second sentence are then, in the rest of the passage, tied to problems in the United States. The passage makes no explicit recommendations about revising how the GDP is measured.
A
B
C D
E
The correct answer is D.
(8)
2.
Int.,.
(E)
It is a comprehenSive Indicator of a nation's economic well·belng. It IS not accurately captured by the pnce of those goods and services. It IS usually less than the intrinSIC value of those goods and services. It IS more difficult to calculate than the economic Significance of those goods and services that are not Included In the GDP It IS calculated differently In capitalist countnes than In noncapltalist countnes.
In de cribm th G mea ure • the ut:hor wlllJlII the GDP i a corDprm~~-:,:;==:. Correct. The aut it the GDP i~nore th ea_lIIiIr__ certain thmgs. It economic wcll-bem The author make n the economIc valu good and sCrvtcc . The author make n difficulty of measur and service not mC'uund The author doe not IndiClllIl noncapitali t countn
The correct answer i B. 4.
The companson of the GOP t a calt~.lm.",< serves to do which of the foUowirrt/ (A)
(8)
(Cl
(0)
IE
Refute an asserbon th c:;:::~ Involved ,n the GOP a e eilmlillllly nature Indicate that the certain types others
0lIIOI_".....
IIIIIlrtIl.'.
uggest that It IS suppla ted econom indic_~
_,*,oon requires understandi .ad' iJJIplies about the main iSSue 13-14, the passage states that "lIlllDlIIlt/ltf(Onomic utili~" of things su ~"j",-fl"""nment and social cohesiveness. I s that in counmes that are de indi .c indicators such as the GDp eeonom • tnt and the sOCIa . I ' structure hao•• tIIfII'IOnm (I' v< trDdtd in recent decades me! 19-20).
Inference
env.:re
,ee:;1he :a - •
The passage does not m~ntion how GDP affects the capItalist nature of policies. B Correct. In indicating that the GOP ignores envir.onmenral and Social passage im~lies that poliCIes depen the GDP wIll also Ignore these issuer. C The passage indicates that the GDP into account the commercial activity generated by envIronmental disasten does not suggest that the amOUnt of activity is overestimated. D The passage indicates that the GDP the value of social cohesion. E The passage indicates that the GDP that the economic significance ofgoods a services lies solely in theirprice (lines 5-
A
Miltd. .nIdll bot1llltaubb'let atuladDB the GDP.
uojudp",ubout ~e
af.the GDPin reconling rIt/M1IJfb.ry 1I8JI1ICtI0/15. no mentlOD of other ~==: tIIdll:aton aide fiom the GDP.
".!I&maka
By~theGDPasa iiVj_~L'IIIidlltm,~ that cannot subbllet, ~"'.plII.G1I",iIluatr&te why IOmethmg an ail ap1ll tnIltepl1llented by the
E
flfbMIlIDa' aU• • JIIIIIF does mention that the
,qi.itGE.
commercial activity . ._lid ~by an oil apill, It does not suggest
~ GlIlUIbOlWJ.P lllIlIgpIteI the amount of
ty.
'The correct answer is
. . . . . . .lIIIWIir.. D.
B.
This question requIres understandmg what the author implies about information m the passage The. passage states that the GOP is th~ chief indIcator ofthe economic well-being ofth~ Unit~d States (lines 4-5). It also states that the GDP ignores th~ economic uti/ity (lines 13-16) of thing such as a clean environment and social cohesiveness. Therefore, the GOP doe not take into account the economic utility of certain environmental and social conditions.
A
A
B
B
C
D
E
The passage makes no comparison among different nations' economic indicators. The passage does not describe the GOP a being inaccurate in its estimates of the prices of goods and services. The passage does not describe the GOP as overestimating amounts of commercial activity. The passage does not describe the GDP as confUSing different types of economic activity. Correct. The passage states that the GDP ignores the economic utility of a clean environment and social cohesiveness.
The correct answer is E.
a detail suppiIIHIII
C
o E ThecorRCt_ 8.
The passage suggests .....rIl •••1Ii coral reef communitie~ (A)
Questions 7-11 refer to the passage on page 24 . (8)
6. . . . . ." !hit nllianal polICIes that rely =~.~an nomic ndlc.tors such as the GOP IlIoIlllllIllcrtl
It can be Inferred that the author of the passage agree With which of the following assessments GOP as an indicator of the economic well.beIfW United States?
lIllY ClpillHsticIII nature
The passage IS primarily concerned With (A)
(8)
(A)
ii===ImJDIfInaI:;Gf ~
7.
do not
It masks social and environmental era fully than the chief economic Indicators nations,
It is based on inaccurate estimations of prices of many goods and services. Ie) It overestimates the amount of com activity that is generated in the United (D) It IS conducive to error because it con distinct types of economic activity. (B)
It does not take into account the ecO of Certain environmental and social
(C) (0)
(E)
deSCribing the effects of human activities on algae in coral reefs explaining how human actiVities are posing a threat to coral reef communities discussing the process by which coral reefs deteriorate in nutrlent·poor waters explaining how coral reefs produce food for themselves describing the abundance of algae and filter· feeding animals In coral reef areas
The Ina'l!lllllil
(C)
Coral reef cOIT1lnullillil!!li.'"
likely to thnve III WlIII!fS"'lIlJll nutnents. The nutnents on thnve are only fauIId itSl_ _ Human populatian.,.,
I."
ocean temperabns.
dIII
(OJ
comlTllllities. The growth of ClnlJ_i destabilize undilllWl_11lII
(E)
CoraIIllIf CtIIIlI"""!fIIl
diverse"' ,. land
. /ilter-feeding animals I crease In .. d 111 ted with dntablltze , not thriVillg, ~ re opulation. herbiVO P'n nutrients, rather than a • _. crease 1 IV' In ses reef decline, when the deetel e, cau fi d' . I . of /ilter- ee 109 antma s then population
D
n
......110 ed only In the !III IUIg manne hahllllS (I"In 1OUI'l:e of nutnenlS, the passage Indlcale that the nutnenlS are lhaUow waren. . Ocao temperatures are not mentioned In
'=:==::~.lIieiJ
decline. not reefgrowth,leads ~o dllillblilized herbIVore populations (hnes
e
E
The cortect answer is A. the passage, which of the follOWing IS a 10. According t.0 threatemng the survlva I af Coral reef factor that IS commumties? (A)
mparl
are made between llClIlI)'lItenllliJ warer and on land. Ii
(B)
Adecline In nutnent input IS disrupting their symbiotIC relationship With looxanthellae.
(G)
The degraded waters of their manne habitats have reduced their ability to carry out photosynthesis.
(D)
They are too biologically complex to survive In habitats with minimal nutrient input.
order 10
DnMlle .n example of a eharaetenstle sign of deteflor.tlon
.....in how reef commumtles acquire for urv,v.1
_.1IICe
identify. faclor that helps herbivore populations
of decreesing nutnent Inpulln 1IlIdI_11IIIt IIIfI inIlIbit . . . .. ,IIlIIIIINIrI of corel reef eommunrtres that for nutrlenb
The waters they Inhabit contain few nutnent resources.
lbt IUIIIor refer 10 'filledeedlng ammals' 23-~lln
grows. h .ncludes /ilter-feeding animals The aut or 1 f I t of the decline 0 cora reefs n.. the contex f i e ' '" · tic process 0 cora reels. the sym b10
(E)
Waste by-products result In an Increase In nutnent Input to reef communities.
Supporting ideas The phrase according to the passage indicates that the necessary information is explicitly stated in the passage. Look at the threats to coral reefs listed in lines 18-21 and match them against the possible answers. Waste by-product increase nuttients in the water, and reefs decline as nutrients grow more plentiful (lines 21-24).
II.
It can be mferred from the passage that the author descnbes coral reef commumbes as paradOXical most likely for which of the follOWing reasons'
12 (A) (B)
(C) (D)
(El
Their metabolic wastes contribute to the degradation of the waters that they mhabrt
(0)
They are declinmg even when the water surrounding them remams clear
A paradox is a puzzling statement that seems to contradict itself. To answer this question, look for information that appear puzzling. The author calls coral reefs one ofthejilscinatingparadoxes of the biosphere because the reefs are prolific and productive despite inhabiting clear waters with few nutrients. The paradox is that the reefs seem to flourish with little food.
B C D
E
B
Coral refs thrive in nutrient-poor waters, the first paragraph explains. utrient input is increasing, not decrea . (Une 20-21).
1h
pa age does not say that the Jegrad inhibit photosynthesis. mpl cosystem of coml reefs th I 0 poor waters. ~lI'Illlt. Wi Ie by-products contribu A~iClllllldoulri ot input, which cau
PISSl._.
sanrtation . .,.~ likely to OCClrr?
Inference
A
The
They are thnvlng even though human actIVIties have depleted the nutnents In their enVIronment They are able to survive m spite of an overabundance of algae mhabltmg their waters. They are able to survive In an enVIronment WI h limited food resources.
Human activities have harmed coral reefs by increasing nutrient input (lines 24-29). An increase in algae is a sign of reef decline• not reef survival (lines 21-23). Correct. Coral reef thrive in waters that prm'ide little food. Algae cell, use the metabolic w-astes of the coral. to carry out phorosynthe.is; the result is sustenance tor the reef communin', not a degradation oh\ .lters (lines 9-12). Coral reets thrive in clear. nmrient-poor "'.ltcr ,lnd decline in nutrient-rich water.
The correct a nswer is C.
~.II(,IjIIC COIlIl:Xt
QUi. . . . . . . .
an.......
(AI IB
An ouIbr_
(C
An epidt!ll1lC of ~.ir An epidern of naralvk
IE
An outbreak
An epKlem of pantJtic: adolescents and 8lIIJIlS
Inference
ince the que non Its lack of modem sanrtan n, bc"III.J~. what the pa age a modem amtanon. Lin modern amtanon m dellayiing Line 14-15 tate that t pm.'ent typhOId tpid. Plralytlc poho tpidem It I from thi 'ratement that a sanitation could lead to a tyJIlhoiid elli1l
A
B
c D
E
ned from tepa
e fila . Yllle
.iI"llt nfI revalent in parts of the United S
1JICOIl1~ eh of the following?
beClUse of (AI (8)
(e) (01
(EI
I
dYertent introduction of Lyme dise The ,n8 to the United States ase baCtena The Inability of modern samtatlon methOds to te Lyme disease bactena eradlea . . ·c mutation In Lyme disease bactena AgenetI . makes them more virulent The spread of Lyme disease bactena frorn Infected humans to nomnfected humans An Increase In the number of humans who encounter deer ticks
Inference To make an inference about Lyme disease, examine the discussion of Lyme disea e in lines 16-24. The disease is caused by bacteria carried deer ticks. It has become prevalent in parts of thl United States as the deer population has grown. This population growth has occurred at the sa time that the suburbs have expanded and outd activities in the deer's habitat have increased. What can be inferred about the growing prevalence of Lyme disease? It is logical to infer that more people are encountering the deer ticks that carry the disease. A
No inadvertent introduction of the bactem is mentioned in the passage. B Modern sanitation plays a role in typhoid and polio epidemics, but it is not linked to Lyme disease. C a genetic mutation of bacteria is discus in the COntext of Lyme disease. D Transmission by deer ticks is discussed, bur not human-to-human transmission. E Correct. As the deer population grow and as humans encroach on the deer habitat, more people encounter deer ticks and beco . r: me inJected with the disease. The eorrect answer is E.
15.
Which of the followlOg can most reasonably be concluded about the mOSQUIto Aedes a/bop/ctus on the basis of IOformatlon given In the passage? (AI (8) (e) (0) (E)
It IS native to the UOited States. It can proliferate only In Asia. It transmits the dengue virus It caused an epidemiC of dengue hemorrhagiC fever In the 1950s. It replaced Aedes aegyptllO ASia when ecological changes altered Aedes aegyptl's habitat.
Inference An inference is drawn from stated information rather than explicitly stated. Begin by finding the information given about this mosquito. The mosquito Aedes a/hopictus is mentioned in the final sentence of the passage, which says that this mosquito was inadvertently introduced into the United States and has spread widely. Because of this, the author states that the stage is now set in the United StatesfOr a dengue epidemic. It is reasonable to infer that this mosquito can transmit the dengue virus. A B C
D E
It has been inadvertently introduced into the United States. The mosquito that proliferated in Asia during the 1950s was Aedes aegypti. Correct. The mosquito Aedes a/hopichLS transmits the dengue virus. The mosquito Aedes aegypti caused the epidemic in the 1950 , not Aedes a/hopichiS. 0 information in the passage upports thi statement.
16 GIllie. . . .
(AI 181
Two _ _
leI
arlUld, and A tIleory descnptions
JIll.
the theory (0)
(El
Ageneralization three Instances An argument by three counterexaflllllll argument
Logical structure
Analyze the structure of the plII ii answer a question abou how.t _ organized. The passage be some epidemics are not ca mutations in bacteria and viJllIIIIS, . . . . . . social and ecological ~ III Three specific examples polio, If-... and dengue hemorrhagic ~ • • upport this general statement.
A
B C
D
The correct answer is C.
E
sho,..
The pa age opens by epidemics may be ","'IiI. thi i not a paradox. Only one explananon. - . .r*_ upporting examples. Thethreeexam ail....... experiments they are doo_.... _ . . Conut. htlw dis ",.
COI.*I.,.
=s==:::.-::=__...
19
• """",'RfS of studymg the IlOS tIIIlllle tI/d eJi5ted on Mars , lJIeOfY regarding the ex stence Of 1W!; .. 1glIl ot new eYIdence _!VI!CiedeconfloCll"g vlewpo,n s regard fig
;:"dJily tIIilllrfe once eXIsted on
a O'Iililter
ars
~ it recently proposed argumen
III concerJWli the ong,n of ALH84001 deSCriIJe a controversy concerning the lEI scnificanc e of eYldence from ALH84001 MI/II1dH I\IIlWtnng th. que tion require determining """"ofthepa age a a whole. In the lien pu.,--. d' h ragraph. the pi!' age on Kate r at a re l:m found that a 1artian metelJrtte in nlarCflca contalO' compelling eVIdence that exi ted on Mar . The re t of the pa a e then de rlbe arguments by 'keptiL .lg.lIn t the re ar h team', condu ion together with the re rch team' rebuttal- to the kcplil '
Suppor1Jng Ideas
j
II
r~ment
I
Ih
riB
foIOWI"g best desOl bes the funetior, ~~e of the first paragraph? lIIIla5f ...'_.jdtlrtifies a possible organic source for !he /AI ~AHs fOllnd In AlH8400 1. ~ deSC nbes a feature of PAHs that IS not shar lSI by otller types of organic molecules.
lDlletrue?
_It._.-1Ieen on EIrlII tor
1t...
qlnlllld from nonbioIoIJtal proct 5t lIlIlilontf would cont.1lll fewer PAH l/lan mth!Ofllt cont.1" PAHs II) tht rntltonlt would be COIICIIIlrllted toward tht rntltot~e urface
AppIUtion
AJIIIftri,ng lh
The passage sugges that I i - ' WIth whICh f ._, s team WOI*l agree a e follOWlng regarding tlle p produced by nonorga c processes' (A)
how a charactenslic common to meleo n1es Onglnates, (0) It sUggests how the terrestnal contamlnalion AlH84001 might have taken place, 1£) It presents eVidence that undermines the cia thaI life once eXisted on Mars. (e)
IIIe meteontt would have
23
n explains
Evaluation This que,tion requires understandIng how a pall of the pa, age functions within the passage as a whole. The lirst paragraph begins by establishing that McKay', team believes that the PAHs fo u in ALI 184001 provide compelling evidence thaI lite exi,ted on Mars. To explain this, the passa indicates that PAHs are a type of organic molecules, which form the basis for life. Furthermore, to connect PAHs to possible life 1\13"" the final sentence of the lirst paragraph indicates that one source ofPAHs i> the decal' dead microbes. .
A
Correct. Without evidence of an organic ource lor the PAJ Is in LH8-1001· the .te.lIn's argument would not m.lke e~se. B Ihe passage indicates that PAl Is can be 10rl1led hI' the decal' of nrganic material f nu, robe, bUI dn" not imply th.lt other ~llK,\fllC molecules cannot be 1'1fI11ed thl " c Ih" 'Cnlcncc rclCrred to expl.\lns one . ot . .1 molecule f'ltlnd in 1'0 Ibl c ongln I IIR4ll01 Ililt, I. oes not Imph th.llmo,t meteonle ' I I mllk·UIlc. 1 wn .lIn llat () h elllen,',' rcte'r rn,I to l Ioc:s not 111\''') ' I\ ~Rl\1 ultnrc'tn.1i ""nt.lIl1in.ltion. ' Ito·... lnt"lfllli . nc Illen,c rekrr{l. I n t pl\1nl I uy IUlh,' Ic.III1' .rrgul\I,·nl th.lt Ii Ii i l'd on I.rr.
(B)
Ie) (D)
(E)
These PAHs are 0 e a be meteorite a as bee 0 Ea years or more These PAHs are 0 0 oe a meteorite tha or g a ed from These PAHs are no o be p od ed by sta formation. These PAHs are I ely 0 be fo nd comblnalions tha diS ngu sh em "0,,", e PAHs produced by organ c processes. These PAHs are likely to be found n 'e er meteorites than the PAHs produced by 0 gao processes.
Inference Th.!S qu~s~ion involves unders(andmg a pam ular pomt of view presented in the passage. The passage indicate in line 11-13 that-the organi.: molecules found in ALH -1001 are P_ills. In hnes 23-27, skeptics of:\ IeKa} 's (earn's rindin point out that ~roces es unrelated to OrganiC lil- , mclu,dlng tar lormation, can produ.:e PH·, In the hna! entence of the pa -age, _ IcKa\ '. =m llote. that the (\'pe ofP_ Hs lound m ALH 0 1 are more similar to tho. e produ.:ed b\ or,alll proc" '" than to those produced b\ nono , III proce e,.
B
:.\ Ie K \' te.1m d,-.e, not ne th pr nonor anlC P-\,H 111 m ( 10(( knl:th 01 nm< th m'( I 3rth. ;\ I. Ka\' t< nhl I ht [hat ul ,I t lm< nd lfl \1 rtI n Ill·t On(!lJ'llfll In t. f1
III
l
D
IllIhlClanllNer i
Ib
tans_
24
E
am Idea
It
_A Irom the passage that the a lie II1lerr... • kd .. 5 the camels I ney as entirely
~ I' (line 26) primarily to emphaSlle
Questions 27-32 refer to the passage on page 32
IAJ tunc hans much as the kidney 01 a rat fun
27.
unexcephOna
t aid the camel In coping with the (8) does no excepho nal water loss resultmg Irom the extreme co nditIOns 01 ItS environment does not enable the camel to excrete as rn (e) salt as do the kidneys of marine vertebrates
'*"'"
lIP:'Il~.R'" IJody Iempetature A4'lIII in the ~ inlemailluid level ••_ _ lIlthe O$II1Obe pressure 01 the bloOd . ..,.., ..._ _ III the llIIOtJIIt 01 renal water loss A_ _ lIlthe unne's salt content
infI:rence I drawn from SllIn.d IIllOnnation. To 1DI""lI-thiI quemon, look at the inlOnnation about .-ung and panllllg in line 18-21 and 33-35. fllI:s that desert rats avoid Ion ofjfuid • •&~'1lJD1f1JTSWNhng, which 11" rrgull1tory .-";"ruj&rtnllmll1mmg inln"nl1lbody tlm/'fflltu" b~f/It»rllilW ttJDImg. These mechanisms reduce body lmIpcraturcs. Additionally, camels 'I/#mIlI fIJII/rr (line 35) when they avoid IIld pantmg. except at very high body !/pture:a. ThcrclOrc, they must lose internal when th do 8WCat and pant.
Identify a shortcoming in a scholarly approach and descClbe an alternallve approach proVide emplCIcal data to support a long-held scholarly assumption
(E)
requires the help of other organs in ellmmall
(D) (E)
To find the primary purpose, look at what the author is doing in the entire passage. In the first paragraph, the author examines two approaches to political history, both of which suffer from the same flaw, the exclusion ofwomen. In the second paragraph, the author reviews an alternative, more inclusive way to understand political history.
A
B
C
special to help the camel cope with its difficult environment.
Marine verteb h help 1" rates ave other organs that e ImInate ext I 1h ra sa t; camels do not. e COrrect answer is B.
compare two scholarly publications on the baSIS 01 their authors' backgrounds attempt to proVide a partial answer to a longstanding scholarly dilemma
Main idea
Since a contrast is drawn between the kidneys of camels and those of desert rats the two must function differently; the ' passage makes no reference to the kidnel~ other rats. .
o comparison between the kidneys of camels and the kidneys of marine Vertebrates is made. D There is no info . . kid ormation given about the ?ey structure of most mammals in de envlConment h' justified. s so t IS conclusion is not
E
(B) (e)
B Correct. The camel's kidney does nothing C
enumerate reasons why both traditional scholarly methods and newer scholarly methods have limitations
IS similar In structure to the kidneys of mosl mammals living ,n water-deprived enVlronrn
To answer this question, look at the phrase tntird,. untxuptional in the context of the pass Dese;t rats and camels share the problem of conserving water in an environment where wara is lacking, temperature is high, and hl/midit)' is j (lines 10-12). Desert rats have as part of their coping mechanisms exceptional kidneys that produce urine with a high salt content. The au compares camels' kidneys to those of desert rau and shows that the camels have ordinary kidn that do not help the camels conserve water.
A
(A)
(0)
Inference
.r:w
The primary purpose 01 the passage IS to
D E
The first paragraph identifies only one reason that the two approaches are flawed; an alternative approach is discussed in the second paragraph. Correct. The author points to the flaw in earlier approaches to history and shows an alternative way of thinking about political history. No data are offered to support an assumption. Only one historian is mentioned by name; her background is not mentioned. o long-standing dilemma is discussed.
The correct answer is B. 28.
The passage suggests which 01 the lollowing concerning the techniques used by the new political histOrians descnbed In the first paragraph 01 the passage? (A)
They Involved the extensive use of the biographies of political party leaders and political theoretiCians.
Ie)
thel They of United nneteenth po ition
(0)
(E)
They were 01
behiMOr ofn analyzing the could not vote They were deY eel Influen e of nm on twentleth-century
Inference The question's usc of the indication that an I Examine the first paragraph. '!lpoliti{al biltary I
used techmques uch elution rtluT1lS that the in anl1~vzing /hl po/llt 'Ill wITe denied /he 'lJf)k tnl
however, be a sumed that did prove useful in u poli/ital behl1V1or of voters A
The fi rst enten hnorian s ugb jams . .. on II msltlulttnU.
B
C
The pa age does historians were dimate of their The new hi to pnutu.l
0
E
1h
po ition of poll Correct. ne his analyzing the allowed to the polt
30. ~inB
_.INt~. novel definitiOll and
clllfillilions ~~=: IIlIIl¥lIe 01. JlOInl agreed on by .; ......tions 01 scholars : :.. ~ IllImp/e 01 the prose style of an hlStonan : • lJIily • delinitiOll ,Mln HI the first paragraph &illig! ItrVcture UIl~~why the
author uses a direct quotation, _" ar die Iogi I tructure of the passage in the quotatIon. The hi torians discussed ...,Ill. pangraph define political activity as PIII1a Baker, however, has a new ,::::~"~of:po1ltJl:a1 activity, one that includes 'l of thote who were not allowed to Dable to mfer that the author draw attention to this new ~:=: whIch provIde an innovative, of thinking about political history.
,-IIiIII.
.
r1 I the a1t rnative position alternahve to hen; i discussed. Ca.... B ker quoted to emphasize that new and that it di ffers • DifiCUldy from the tradltwnal definition
;
a5~5~~~~
Ihow that different
p ed angrethed first _not lXIlIIilIICDI on Baker'
to the passage, Paula Baker and the new
ns of the 1960s and 1970s shared politiCal hlstorla which of the following? mentto Interest group politics (AI Acom mlt (81 Adisregard for political theory and Ideology In the ways In which nineteenth· (e) An In terest century politiCS prefigured contemporary POhbc (D) Areliance on such Quantitative techniques as l!'t analySIS of election returns (E) An emphaSIS on the political Involvement of ordinary Citizens
31.
Which of the follOWing best descnbes the structure of the first paragraph of the passage' (Al (Bl (C) (D)
IE)
supporting ideas Since the question uses the phrase according to thl passage, the answer is explicitly tated in the passage. Look for a point on which the new political historians and Baker agree. The first sentence of the passage says that these new historians were interested in the political activitiel of ordinary citizens (line 5). Paula Baker is similarly interested in the political activities of ordinary citizens (lines 15-16), especially of female citizens, who were not allowed to vote.
A
B
C
o E
mention at all is made of interest group politics, neither in relation to Baker nor in relation to the new historians. The passage does not show that they disregarded political theory and ideology. The passage only discusses Baker's interest in the way women's political activitie in the nineteenth century prefigured twentiethcentury trends (lines 16-18). The passage explains that new historians relied on such techniques but that Baker did nol. • are. said to have studied the political actIVIties of ordinary citiznls.
The correct an wer is
E:
The~E~:!! 1960s
to uncIeftake
An """"'. at. . . . .
(A)
voter Astudy 01-· _ . . . . . from party poi!ics tD"'~ A btography min ster 01 forelll/llltcln
(Bl IC)
An analys s 01 narJ :::"'~. unrecognIZed women
(0)
Astudy of voting lr mmlgrant laborers logging camp
IE)
Logical structure To answer this question, analyze the structure of the first paragraph. It compares the old approach of studying political history through emphasis on leaders and government institutions with the new school ofpolitical history. which turned instead to the political practices of ordinary citizens. Both approaches suffered from the same drawback: the failure to include women in their analyse .
A
0
Correct. Both the new historians and Baket
Two scholarly approaches are compared. and a shortcoming common to both IS Idenbfied Two rival schools of thought are contrasted, and a third IS alluded to. An outmoded scholarly approach 15 descnbed, and a corrective approach IS called for. An argument 15 outlined, and counterarguments are mentioned. Ahistorical era is deSCrIbed in terms of Its political trends .
32
B
C
D E
Correct. Two approaches to history are discussed, and a flaw shared by both, the exclusion of women, is identified. The first paragraph does not allude to a third school of thought. A corrective approach is not discussed in the first paragraph. The fir t paragraph doe present an argument. but no counterargument are made. The political trend of an historical era are not detailed in the first paragraph.
The correct answer is
, -.::::=~
a III
Inference In using suggest. thi question apply information stated 111 paI. .' . . . . . . .1 an inference about the meth before the 1960 . These m'etbodal&. . . .,._ the first paragraph. Lines 3-4 traditlonaljocus ofpo/II a/ltisIlrilu (1. . ._ - : ad\ ent of the new school ofllistl"'_. '_I' 1960, and 1970s was InStJfUtIOru. It i reasonable 1960, hi torian Iikdy go\'Cmment in titunon
B
c o E
34
glWO
by
wruch of the following IS most similar to the ~ nneteenth-century ethnologistsJ In their editing 01 Itle stones of Na lYe Amencans.
propo ed
Awitness In a Jury tnallnvokes the Fifth Amendment In order to aVOid relating Per Incnmlnatmg eVidence.
IBJ
A stockbroker refuses to divulge the SOurce of her InformaMn on the possible future Increase In a stock's value, A sports announcer describes the acbon In a team sport With whICh he IS unfamihar,
nbed and
obta
at
and th change other soblect
Ie)
e
rA
lA)
d
35
e
ar o
(0)
A chef purposely excludes the speCiallngre from the reCipe of hiS prizewinning dessert
IE)
ApohtlCian fails to mention In a campaign speech the Similarities In the POSitions held by her opponent for pohtlCal office and by hers
Bl
leI
r~~-:
(OJ
a I:
IE) CUI
es
ura
Application
'//) an wer the question, determine what the ,Illtlmr say' about the ethnolo~i,t, ,lnd apply the illforrnatlOn to the example' ~ivcn in the an \\Cl ,holle . Lthnolo~"ts believed th.lt rc(ording pcr oll.d torre' (olild ill(fe(/I, Iht'lt IIlId,'n/'lIldm 11,( (/I1111f'" Ihallhe)' had IWII ObUT 'I!lgfrolll 11.',lholll
di";UIl~lllll
\
I
B
Ih.1I th,
1\
I
'-""',",,1 1lO1'1VCI'
nJ
a'
Supporting ,deas t
(line 10 12). BUI the\' were crilldzed 6 II/)t l'el1lllll~ enough time In the e Llilture (Ii J~ II) ,'l1llthu 1I0t under t.llldin ' the culmr \\ 11 rll",~h 'Ihell 1'0 ltlOIl " ItIO t IIkc th.1l of Ihe pOll ,lItItOlll1ll'r," ho 1 .11 0 .In oh erver, who I 1I0t t.lllllll.lr" lth th ',lltIe (or, 11\ thc olth ,thltolo '\ I • the ,'uhur h Ing oh n l'l
ea
the
liifetence
f ed from the passage that a 38 It can be ,n err f the ethnological research on Nat;,_ haractenstlC a . "lit! C nducted dUring the nineteenth centul) ns Amenca coof which of the follOWing? was the use (AI (B)
(C) (D)
(EI
Investigators familiar with the culture under study A language other than the Informant's for recording life stories Life stories as the ethnologiSt's pnmary SOurce of ,nformation Complete transcriptions of Informants' deSCriptions of tribal beliefs Stringent gUidelines for the preservation of cultural data
Inference not an explanation; ...iOd IIIP Iliin und nding of a
An inference is drawn from stated information. To answer this question, find what the passage says about the ethnological research on Native American cultures in the nineteenth century. The third panlgraph tells us that much was inevitably 10J/ (line 31) when investigators wrote the oral stories down. Na/ive Americans recogmud tha//IN men",' of/heir lives could not be comllJllniCllted in Engli,h (lines 33-35). From these two statements, it is reasonable to infer that the stories were written down in a language different from that of the stor}'teller.
A
B
the investigators were criticized for not bein~ suitahly familiar with the culture (lines 25-20). arrect. ativc AmenGlns hclil'\'l:d that En~lish LOuld not express their culture; .It least some inve ti~.llors. therelllCe, must hav wriuen the stories down In English. thno(oKists w.lnteo the stnries to • upp" !II III their lidownrk (lines 'i-9). nol to pia it a th ir primary me.lI\s of Inv ttg II n. .
1
Questions 39-44 refer to the passage on page 36 39.
The primary func Ion of the passage as a (A) (B) (e) (D) (E)
0
e s
0
account for the popular yof a prac ce evaluate the utlhty of a pract ce demonstrate how to InstItute a prac ce weigh the ethiCS of uSing a strategy explain the reasons for pursu ng a Slra egy
EXCEPT
B
es.
Main idea
Thi question explicitly requires looking at the pa sage as a whole in order to determine he author's purpo e. The first paragraph e:\-plains the practice of offering guarantee and list circum tances in which an unconditional guarantee may be an appropriate marketing tool. The second paragraph begins with Hrr,::R:'" implying that a contradiction i about to tollow. The erious drawbacks to guarantees are e 'amined, and the passage closes with a warning.
A B
C
D E
The pas age doc. not di cuss the popularin of guarantees. . Correct. The passage examine- and }ud~s the advantage: and diad\':tntages ot a bu-iness practice. The passage does nor show how to put guarantees into place. . The passage doe not dL ·us_ ethl The fir. t p.lC.lgraph dlle, e pIJIIl rh rea n tor offering gu.u·.lI1tee,. but that I nh J portion of the p.l· .1 e, not the p '.1 " wholc.
The correct .lIlswer i' 8.
adverse efIel::t5 III
are SIlP'''1IIlliar tllUlIlI!!I E
. . . ._~ of the Issue raised sntees tor health care or lehl ~ ImPlies that which of the
tIIlJSl
trU8J7
d medical professions have Tbe reg~sa;f practice that would be VIOlated stJIIdar fulfill such unconditional guara ..-mplsto •• ~... It of a laWSUit or medical procedure u sarrly be determined In advan~ (Il) The res cannot necesnals handling a client's caSe the profes slO . f the legal and medICal profess ICI The dlgnrtYeod by any attempts at marketlngl~OI ln IS underm . al services, Indu d'Ing unconditional professIon guarantees. . se laWSUits or medical procedures Clients Wh0 ID) t sfactory outcomes cannot be have unsa I fi' ompensated by nanClal C I adeQua te y settlements alone. the monetary cost of legal or heal'" lEI Pre dIC tIng . . '" es is more difficult than predicting rVlc care se the monetary cost of other types of professio~ (1J
....... lind an ~r ri/ttl m wfIJ be expbdtly rared lOIII
operu With an /irmJ want to o/fer SNmng II Mmpth/tflf
~_
elfectJVely agams t oIIir SUarantee ' not menoon liability
servICes.
. . . . .Id PUlfllI'IJ!lh uggut the r verse:
~
:i~~§r:;ntee may hurt alirm',
quality of ervlcc IS not IUIOR to /fCr guarantees.
Inference The question's use of the wo~d impl!es means that the answer depends on makmg an mference. 'This uestion refers to one sentence in the passage q . I . h (lines 21-24), so it is essentla to review w at that sentence says in order to understand what it implies. An unconditional guarantee of satisfaction rna)' have a particular disadvantage in the case of health care and legal service because clients rna)' be misled into believing that lawsuits or medical procedures have guaranteed outcomes when they do not. ince an inference may be drawn only from explicitly stated information, the correct response must be about the problem of guarantees and outcomes.
A
B
o
Although thi . statement may be true, it cannot be derived from the cited reference. orrect. Legal and medical professionals cannot guarantee the outcomes of their work. This statement cannot be drawn from the description of the issue. ompensation is not discussed in the ref! renee. Pr di ting costs is not discussed in the
fer n e. conec:t n w riB,
43.
WhIch 01 the IoIIownllll'WlOlllIItieII _ _• exemplifies the ootentiaIlJli' I h nabld lie .....111 sentence of the seconcl llaJ'agraph Clines 15-19P.
(A)
AphysICian's ul1COnditional guarantee of satlsfacbon encourages Pa!Jents to sue for malpractice" they are unhappy with the treatment they receIve. (B) Alawyer's uncondtlJonal guarantee of satlsfaclton makes chents suspecl that the lawyer needs to find new chents qUIckly 10 Increase the firm's Income. (e) AbUSiness consultant's uncond,bonal guarantee of satlsfacllon IS undermined when the consultant falls to provide all of the servIces that are promised. (D) An architect's unconditional guaranlee of sallsfactlon makes chents wonder how often the architect's buildings fall 10 please chents. (El An accountant's unconditional guaranlee of satisfaction leads clrents to believe Ihallax returns prepared by the accounlant are cerlaln 10 be accurate.
IBI
c ID
thalged Their'e:~-:::==
Their C:~:::::
IE
thecplity
Inference
Application This question involves taking the problem identified in lines 15-19 and applying it to the hypothetical situation that best fits it. Offering an unconditional guarantee may not work as a marketing strategy because potential clients may doubt the firm's abi/it), to deli~'er the promised lewl of service. This strategy may actually introduce doubts or reservations on the part of potential clients and in fact discourage them from e,'er hiring the tirm or the individual providing the en·ice. A B
l
D
F
In this ca -e, the problem occurs after, not before, the sen·ice is rendered. Thi situation e.'emplitie, another problem of unconditional guar,1Otee , the -ue ge tion that 'ljirm IS b ggingjor bwinrss lline 21 , The problem occur- after, not betl)re, the -en' ice \,' rendered. Correct, The ,uchiteer" apparent need to oller ,10 unconditional guarant make potential client que tion th outcoffi ot th archite 't\ work b,'u tin th Ii lh of their diss.lti'[l ,tion ith the archl tural sen i(' ~\i. Ihis silll.lti"n "<)ntradi 'IS th prohl
The corr ct llns eri D.
A
B
c D E
about TIle pas age Ind/Ca tes whIch of the following , Du 80i 5 attitUde toward WashIngton.
46
fAI
It underwent a sh,ft dunng the First World h TWar as Du 80'S became more sympathetIc WIt rOllef
It un derwe nt a shIft ,n 1903 for reasons other than Ou BOIS'S disagreement wIth Washlngton's accommodatlon,st vIews. eCl It underwent a shift as Du BOIS made a long-terrn commitment to the strategy of accommodabon. (0) It remained consistently posItIve ev~n though Du Bo,s disagreed with Washington s efforts to control the African Amencan press. (E) It was shaped pnmarily by Du BOls's appreclabon of Washington's pragmatic approach to the advancement of the Interests of Afncan Americans. (B)
hOW/nghow BY to a h,eve cerla,n
..his
1""~'
l.'
j~~:::: particular poSlbon adopled by I the World War the vfew thaI Du 8015 was s,gnrficantly tllllII/Icea by ither WashIngton or Trotter mre.slngthe etfectlVeness of a slrategy that urged Atncan Amencans 10 adopt
' '51
M.tnldH
Supporting ideas
I
I I
I
B
C
I)
t that there W.I f th trat KI
The passage indicates that the shift described occurred in 1903. not during the First World War. Correct. The passage indicates Du Bois's shift was not due to differences he had with Washington's ideas or vIews. The passage indicates that Du BOIS accommodationist stance did notl,/st (lines 30-31) and therel,)(e wa, not a 'Ille passage indic.ltes in lines 16-17 that Du Bois ,It one point aligned himself with 'Va,liin~ton's militant opponent. 'Ihl' passa~e indit.lte, that Du BOI' e\"entuilllr rejected the ,Iccommodationi,t vil'\\" of 'Vn,hinAton.
'Ihe correct answer is
as
wars
eC) (01
IE)
The contnbutlons d d not end dscr'IITlll>a!i
Inference
information given in the passage. The pas>age begins by indicating that Du Bois caJ.led on African Americans to suspend their fight for equality and to help with the war effon during the First World War. The final sentence of the passage, however, indicates that Du Bois learned
n.
wnot<:. The paSSll!." bc:giDI
that African Americans were experiencing
systematIC discrimination in the military during
this time.
A
C
The passage does not indicate ho\\ African Americans responded to Du Bois's edirorial other than that mall)' African American, were surprised b), it. The passage indicates that African '\mericans' participation in pre,;olli ,.",. brought legal ,,,,d political advance' but that Afric.", mericans experienced discrimin.uion in the First World \\ ar. Correct. The p,lSS.lge indicate, that Atncan
D
di'crimin.uion in the military dunn~ th FiN \\'orld \\'.If 'lhe p,b,.I~e doe not describ h~\\:\1n n
B
long:·term commitment.
E hi IIcl\\'
(8)
The contnbutlons were made Iar to Du Bo,s's 1918 edllonal The contnbut/ons had much the same elf Afncan Ameneans eontnbuboos 0 PI'
ThIS question requires making an inference from
Answering this question involves recognizing what the passage indicates about a parncular point of view it describes. In line 15, rhe pa~sage indicates that Du Bois praised Washmgton s ideas, but that in 1903 Du Bois aligned himself with Wasbington's militant opponent (lines 16-17), a shift the passage describes as being due less to ideological reasons (lines 17-21) than to political reasons. A
The passage gge the contrIbutions of A States war effort durmg the FiI'S1t w........._, (AI
views. ~~~mPrrand
47
\meriLJnS experient.;ed 'Wjtcnutit.:'
\nleri('.lIl: re~pondt"J
to TJ\)t[ I
tJ"f
dunng the hr,t \ \'l,rld \\.lr
E
l1H~
p.b....lgl doc.. not indkj,t "h th
\fri'''1Il \meri, Ill' lIl",h dill th , dh'n" 'r . pnnurih ." il n hIS 1I The correct
l.lOS\\ er
i!i
,
-\
B
D
so-ss refer to the passage on page 4Q QuWIfO/IS TIle primary purJlO
50
!:::::=~~1JIlWl1e1lDu 80f
w.::::::..
~rn
::~::::
recII!finflJ()I1 at the
to the war
reSllOn e to h dIsclrwmnltlO1l , ed by
the war I •••IIMIJeIIINI by Du 80f III P rt be au e of f ga n At" an dur ng pa I war
51
se of the passage IS to
bJ outcomes of a type of contrast pass' e lA) buSinesS ,nvestment careful evaluallon of a type of (8) SUggest more bUSIness Investment s ways In whIch a type of (C) ,IIustrate vano~ent could fa,lto enhance bUSiness Inves revenues to a (OJ trace th e general problems of a company t certain type of bUSIness Investmen (E) CrItICIze th e way In which managers tend . to analyze th e co sts and benefits of bUSiness Investments Main idea
Look at the passage as a whole to find the , urpose example, pnmary p . This passage uses h an 11 described in the second parag~ap ,to I. ustmte ' 'pie of business practIce explamed m the the pnncl tirst paragraph, The author begins by saying t~at etlorts to Improve service do not always result In a (otllpt'titivr advantage for a companr Thus, an inveslment in service must be carefully evaluated 10 delermine if it will reduce costs or mcrease ,,'venues (lines 4-8),
A wher/" r If tr '\
p(J1ll
tlndl I tflJl I)" 1111, ;;;~""Iy oDm'pn>mr d Wllh I '"lIe r'
It,_.lndlatet th t hi
I'll
f"
'V:IIIUI1l
t
til
tI
/I
Accord ng to the pas"l • comparable to lIl\esbueills dlstnblJlJon n erms of the
I)
'II. eurn'ct n w~'r is n.
01 the benefrts
tang b
(B)
creased rEvefl\les lII>,ttt_ ... maltely bas s on e '!y need 0 be _ghE':(jprOCllIICE
IC) 10J
nsuf c e a a
s at managers
them (£)
degree 01 eOffiPe I~ely
0 prov de
e a a age U'Q' u." are
Supporting ideas
The phrase according t. tlH paw 'india the que tion cover material tha is eo plicitI) tated in the pa sage. The answer 0 demands a careful reading of the _ cond sen (line 4-8). lnve tment in sen;ce are like investments in production and distribution becau e they must b, bala1lud agairut .thn I) Inv,slmmts on Ih, basis ofdirm. ta1lgibl'
=
ifi
Thu , the e inve tments hould be '-ei<>hed n same basis, ~
A
B
Only one outcome, fadure to gain a l'llmpetitive advantage. is eX'lmined. Correct. Investments ,n service l11U t be carefully evaluated for the return the)' \ViU Onl) onc' \V.lY, an unnece".I!') Ill' e tment In 11ll1'",vt'd sen'jec·. is cI.seu sed. '111(' c .lInl'le of the h.lIlk is used onl\' to diu t"ItC·.1 gener.II husine" prinCiple; the h.lllk It cit i not the locus of thc' 1'.1 s.lge. 'lhc 1'.1 s.lge t ,it "'lies the ,Ihsc'nce of such an .Ul.lh I • not the \V,l' It I mndueted,
.'.*At••
lA)
hrin~.
l
II
D
E
The author i not equating the tan 'bl benefits the different kinds ofin,'C- ment reap, but rather the ba is on whi -h d 1 to make inve tment' are made. Re"enues generated from im _tJn~ in service are not -:tid to be comparabl t revenues generated !tom im""snn" m production and di. tribution, orrect. n e,-aluanon of, 'hether r no t n1,lke these m,'estments must be mad n the _.lme ba is, How managers anal\"7 m -tm nt m pr, ductit'n and di -tribun n i t fhe compe tit;, e ad nr c'n icc I .1 - nowl d~ d. b tnt it, It Is nN III nn n d t 11m t "t prc Ju, tl , lnd d. tr n n
The corr et an lIer i
Inference
!::=.....
,=::::: ~
e regional bank In the second The discuSsIon of th h of the foliowlOg functIons h serves whlC paragrap a whole? WIthIn the passage as an exceplJOnal case In whIch IAI It descnbes rVlce actually faIled to produce Investment In se a compebbve advantage. th pItfalls of chOOSIng to IOvestln (B) Illllustrates t ee when Investment IS needed servIce at a 1m more urgently in another area. the klOd of analysIs that Il demons Irates (e) managers app Iy when they choose one kInd of service Investment over another (0) It suppor ts the argument that IOvestments 10 certalO aspects of servIce are more advantageous than Investments In other aspects of service. (E) It provides an example of the pOint about Investment In servIce made 10 the first paragraph.
cfeICribed to
d • • i1l11e expenences
bInIl'I offices
C:::::::~tllllUIIber
of customers
...II1II_111"'" competmg banks could 1lJl1IIUI1II1D brlnIlIIe bank s generallevel ~"l(>'."""'DI """ "'" WIS comparable wIth
~::=:::;,::,:ofrhe word UXf(111l mean. rllJt II on m.klng .n Inferen e, To
......lr11b11cw:~~~.~ Ioo:k .llhe entire enle",l' ~ (line 20 23).
dunk head. Would Ihe rtrael new u lomer would find II dlffilult III Ift'vict Imprnvemen! hI c.I'ily
~::::~:~::~~ The m.naKer hlluld I re before IIlV llOK III
Logical structure 1hi. question requires thinking about what the second paragraph contribute to the whole passa/(c. The first paragraph makes a . /(enerali'l.ation about investing in improvements 10 savile; in certain conditions, such improvements dll nllt result in the compl'/ilivt' tldt'tll/lilge a romp,lIl)' hopes for. The second paragr.lph ofrers the hank a' an example of th" generalization.
II
d
l
III1K
D l'
'[he tirs, 'entence nf the pass.lge e.'plains th,lt improvlIlg sen'lce does not necessarily hrlng.1 'OIl/Pt'I/IIl't' tldt'i1lllilge. s" the bank i' nllt l'. cepILlln,d. '11l<' h,lllk illusrr,ltc' thc pitt:lll of not l·v,duat,ng.1 • n'lcl' impm\'ement on the h.1 i ot t,lngible bendit<; "ther ,Ir.'as of the h.\IIk .Ir.' not Ill.'ntlonnl. Ih pa .1/(.' dll.'s lIot .Ii ,'Us ho" m.IIl,lgeC$ .\11,11 'l.l' .lI1d dlllo e ,hlll'r.'1ll ,.'n·lce ,1I\'l' IIl1ents, Inv IIll nls in dill.'r.'111 ,I pc, t 01 Sl't \'ICc oIre nol l'valll,lled III 11\l' 1',1 s,lg', o t, Ih h.\IIk Is .111.' ,lIl1pll' Ilt Ih,' po l!lllll lal d 10 th lir I p,lr.1 'r,lph th.11 10 !lng 10 IIl1pmv tI l'rvi ,',.111 Ill' .1 "a II th in 1m nt I nlll \ alu.1! . I ,.If ·tull
55
The author uses the word "rrit( lib l7 non.......... ,n order to
(AI (B)
(e)
(D)
(E)
high Igh the oddity of the serYlCe 0\Il!lIlent emphaSIze e rela ty low aIue of IOvestment n ser ce lIl1llI'oveme distingUIsh e pr arya eo e Improvemen from seeo <1ary attr slOgle out a cer a mer .0 e ser lICe tmprovemen from 0 er me POlO out the I m, ed dura 0 0 e ac a servIce Improvemen
56
logical structure The que tion a ks you to consider he Io::lC 0 • the author's word choice. The previous '() sen eoce discuss why the en'ice improvement was a wasted investment. In contrast, the final sentence turns to the sale adyantage of the service improvement. which i triyial b,' comparison. The author u e on~v to modify rrun/ in order to emphasize the minimal nature of this ad -anta e
E
Suppcwan,
A
The pa sage doe not indicate that the service improvement i. somehow stran e or pecuhar. B Correct. On/I' empha ize' the low ,-alu att,lChed to the 'ingle benefit. C , '0 attributes of the service impro' ement are mentioned. DOn/v :ignifies that there was one I m rit of the sen'ice imprO\·ement. E The duration of the benefit Is n t dl ru in the pas, age, The correct an wer is B,
B
of the follOWIng, if true, would most clearly h WInch d th conclusion referred to mlines 22-25' supporte e /AI Ratings 0 f pro ductivity correlated highly with . 0 f both accuracy and attendance. ratings
According to the passage a ~ factDr" IS an aspect of a worker's Pet10nnBnee that (A)
has no effect on the rating of a worller' performance s
(B)
IS so baSIC to performance that .. " IS assumed to be adequate for all workers IS given less Importance than It de ' ra t Ing a wor ker s performance serves m
(Bl
=:"
Electronic momtOrlng greatly Increased productiVity. re) Most supervisors based overall ratmgs of performance on measures of productiVity alone (0) Overall ratings of performance correlated more highly with measures of productivity than the researchers expected. rE) Overall ralings of performance correlated more highly with measures of accuracy than with measures of productiVity.
59.
which
..., .. NppnlIlriIte use of . . . .11lA lhItCPMCS may have
rM_1Icrns of lIIeir lobs
on
=:::~lhIt CPMCS may have on
Logical structure
drawn fiom stared information.
To answer this question, look at the actual conclusion of the study: in rating workers' performances, supervisors gave considerable weight /0 ... attendance, accuracy, and indica/ions of customer sa/iifac/ion. What additional piece of information would support this conclusion? If one of these three criteria mattered as much as or more than productivity in assessing workers' performances, then the conclusion would be strengthened. Thus, the supervisors' overall performance ratings should correlate with measures of attendance, accuracy, or customer ervice to at least the same extent that they correlate with measures of productivity.
"'1Id~ line reterence in the question to t the author included the detail
BlalClnd workers. Unmonitored 0CIII1pIRld With monitored workers on !tP:dm: what they believe to be the most IIU of their job (and thus of their unmonltored workers believe it is ~~~ mOnitored workers point to JogIca1l1lkrence from the that the author is using this _fIIlmw that PMCS affect how workers
- . - their Joill. liP.llIIJlIOJ~ workers' ratings are not
......_. the pusage. ,..... does not IlIlk unmonitored a polentla1ly ffective use of not connect unmonitored •,~"",,,,,,plOprlale u es ofCPMCS. COJIbUt III the workers'
~::::
..
that PMCS may hlnk about their
nn t unmonitored -,.,. of PM on
I
A
I
I I
B
I
I D E
Th
The conclusion is about the value supervisors place on criteria other than productivity, so a finding about productivity is irrelevant to the conclusion. I ncreased productivity is not relevant to the conclusion, which concerns other criteria in performance assessment. This tatement contradicts the conclusion that supervisors value other criteria. 1his statement contradicts the conclusion stated in lines 22-25. Correct. 1he conclusion is strengthened when measures of accuracy, one of the three criteria supervisors consider other than productivity, correlate with overall pertiltlnance ratings more highly than measures of productivity do. correct an weris E.
(e)
(0)
IS not likely to affect a worker's rallng unless It IS Judged to be Inadequate
(E)
IS Important primarily because of the effect It has on a worker's rating
Supporting ideas The phrase according /0 the passage indicates that the answer is explicitly tated in the pa sage. By puttlOg hygzenefiu/or in quotation marks, the author is calling attention to the expression, which is explained in lines 27-31. A "hygiene jac/or" is a criterion in assessing job performance that hurts the overall rating if it is too low, but that does not improve the rating beyond a certain point.
A B
C
D
E
If too low, a "hygienefiu/or" affect the rating negatively (lines 27-28). Because the"hygieneftc/or" can affect the rating, it is not assumed to be uniformly adequate. The passage does not provide enough information to make this determination. Correct. \"'hen low, the"hygienejiMor" ma~ affect workers' overall rating' negati\'e!y, but when high, it does not affect the ratin beyond a certain point. l1;e passage does not assert that the prima~ importance of the"hygiencjaaor' i its effect on ratings.
The correct answer is D.
Main . . .
~-~:== exp,1aini1Dl tlla cootpanJa.
emp ciDB~ unexpcctal resuIb discusses the A
The
does not B
COiiecto 1hc
feWldiug thmtqlO C
0
method. The does The
conflicting E the 1heCOiiOtt
IlIe pa
ge e earliest re ear prod ed whICh of the fol!ol~
can be
63
pa,a<j
IfI1IlIJed random pa terr>s of ma soeo~s dl
lJOII peoes d Sir bUllOn Stlgge
~
De
crip
B 0
e developmen of mangr
e (Ner tJme R a n of species formerly thought
lEi
be ldentJca Data lha confirmpd lhe "land-building" ro e 'If rro~
C D
E Application
r E:
Supporting Ideas
, ,mh trJ b) rhe phra c aaordtnlf 10 Ih, pal thl 11"( rum I ba ·d on !actual intormarion lat 1m the pJ aKe Lone 5-8 de nbc the arl" I , UtJrrh on manx,Off'jorllil. 'rh, re reh pro 11/ J d"",pl/om ojlltCU/ dilirtb/lilonJrom I I /,,,,,, 1111 hnre Io-Iand pJ!lern I de erilled 1/11/ on th fir t 'enrence.
\
'Ihe 1'" JKe uKKe't> rh.lI rhe Ji Irihutlr," wa nor r,,"doru.
II
Il
a
Ih \
Cllfr' I. "hc dc'nipl,oll of rhe pellC d, t"bUflnn goc frolll horc 10 I Illl, II' ,Ioc .I, hlliflollol "ZOIlIlI" 11\ rh, h r fit II( II" 1'.1 JlI:c dol' lIor 1I111".,t, Ih'l thc carl, I Ie "lid, looknl .1I 111111 rt \ for d \dllpmcllt U\' 'r tnnl' II. r , "" \ 111"",, rlut th, rli r r u 10 Oil "I. d I 1.\ I " 1I10n 'I. IIhc I h h plodll I, nil I "pto,". rio 10111 bud loll Ih I rr
B
\l
0"
II,
"""I ""nl
II.
lite, rr
t
II
rt
F
the author II can be mferred from Ihe passage Ihat h' h f h be mosl hkely 10 agree w,th w IC ate would followmg sta tements about corporate acqUlslt,ons>
66
fAl be more beneficial for small firms (8)
a ge firm do I fulfill the prole sed goals at mosl acqwnng firm They IndIo be beneficial 10 such firms 10 Ihe glerm en though apparenlly delnmenlalln the orllerm They dl OIlrage many such firms lrom all mpt ng 10 make ubsequenl bids and u Ilion I
67
The author of the passage menbons the e1tect of
acquisitions on natIOnal economies most prObably order to
Theil known benefits to national economies explam thell appeal 10 mdlvldual firms dunng the
(AI
1970s and 1980s. Despite thell adverse Impact on some firms, they are the best way to channel resOurces fro rn less to more product,ve sectors of a nation's
(8)
economy,
IC)
They are as hkely to occur because 01 POor
(C)
momlonng by boards of directors as to be caused by rncenllve compensation for managers. (D)
Inf./ence
fE)
(D)
They will be less prevalent In the future, since
thell actual effects will garn wider recogmtlon, Faclors other than economic benefit to the acquiling firm help to explain the frequency w'th which they occur,
(E)
II}(/(' 1
IIIoif ,Ill'
rll,
k IIf
"dl 10111ou/f, in '1/11(/ 11). IIh or I, rh,lIl rill lirm or firm
I
I I I
A
I /I C
I>
E
The passage indicates rhatwhile mergers and acquisitions mal benefit the national emnom)'. the appeai of mergers and a"julS,'ions must be tied to companies' p'n'l/I,' <'(Ol1om;c IIIla"ls (lines 19-22). /lIe p.lssage makes no judgment as to the oe't IV I,· 1'0 fl' I I h . '. r rms to le p c anne! resources frorn Ie,s to 'narc' e·ffi .,' > t ' " len cronoml( scctor~. l1"'I',I5''1''e 'n'll. 0ctween e" ...... • K , no comparison
Ih" InfluenlC o(poor monitoring b, boards ,ll1d th,u of t' ccurivl.' illernt",,, ' II e" "·I'·'ss""e' "'Ik', I I' ,.... • t.: I n prl'llction .ts to fUll"" trend, In II, k . lc: milr "l't tor Illergers ,lI1d ,lullIl. ItUlII .... Cor r ct. '111"1 11 I ' SS,I,l{l' st.lfes r lar t'tl'tllrs or IIl'r rh.ln l'l'tlllulll' , . ' . Il IIHl'rl'Srs drirl' llll'r~t'r I ,llIl .Iulul ItIUIl .
The <:orftCI _ _riliia. 58
A
8
c modee
o E
Evaluation
Inference
This question requires understanding what view the author has about a particular issue. 1he three studies cited by the passage all suggest that mergers and acquisitions do not necessari!v bring economic benefit to the acquiring firms, The author concludes therefore thatJOcto's h"~'mg lilli, to do with corporate economic inltres!s explain "'glluiliollS (lines 23-24) and then goes on to 'peculate as to what the reasons may actually be,
prOVIde an explanation for the mergers and acqUisitions of the 1970s and 1980s over OO~ed by the findlllgs dISCUssed 'n the passage suggest that natIOnal econom c In erests p aJed an Important role III the mergers and acqUlsllions of the 197Ds and 1980s Support a noneconomic explanation for the mergers and acquisitions of the 1970s and 198Ds that was Cited earher III the passage cite and point out the Inadequacy of one pass ble explanation for the prevalence of mergers and acquiSitions dunng the 1970s and 1980s explain how modehng aHected the deCISions made by managers IIlvolved In mergers and acquislt,ons dunng the 1970s and 1980s
This question requires understanding why a piece of information is included in the passage, After the passage cites the reswts of the thsee slUdie' on mergers and acquisitions. which call into question the economic benefits of acquisitions, it indicates that firms nonetheless claim that their objectives are economic, The passage then srates that while acquisitions may wellha,'( a desirable effect on national economie (lines 17-19), the results of the tudies soggest that faerors orher than economic interest must drh·e e..xecun\'e' to arran~e mergers and acqui itions,
A
B
The passage doc< not mention nanon:li economICs as part of an explanation tor the occurrence of mewrs and acqui 'ition'. The pass.u:e sugge~~ rhat the efut of i.lcquisitions on national ~\xmoml~' l' .n?t tied to any explanations tor wh, ",qUl'monoc("ur.
D
Supporting ideas
B
D
The efleet of .1Cqui,mlm, on nar nal el:onomle~ i, not mc:ntIOnM In th I J J~ as ,In e p1.ln.ltion t~)r \\ h\ Jl.'lui Itl" ~ ur Correct. Th . pa' .Igt' u e th m ntllm ll,llion,,1 e,'l)llomie • part of r :;t qUl>,til",ill~ th' t.ll "I m ti at. h hrm 'efl"r! til ,I IU1I'C th r lmn
.1.
an pa ed g ea e eco,non'lic a_ _1IS1_ p ospec emerge e pe e ed ess en a prospe
70
r,;:::f/IIIl
IA)
frl\'ls lIlIIde were
~1=:::;::prodUCed small ~ ICl/llIftd firms levels only of profilablllty.
Managers ml scalculated the value of firms that were to be acqUired, (C) Lack 0 f cons ensus Within board.s of directors resulte d 10 the rr ImpOSIng conflicting goals On managers. (0) Tota I com pe nsatlon packages for managers Increased dunng that penod, (E) The value of bidding firms' stock Increased Significantly when prospective mergers were announced,
"'-fIrm
AbDul ha" of such aCQuIsitIons led 10 long·lerm
se In the value of aCQUlnng firms' slocks, IuJ!porlfnll 'dRS
qu bon requIre recognizing information
Interence
GINIII.iDed an the pusage. The passage reports on ItUdi ofmetgers and acquisitions In the
This question requires recognizing what can be inferred from the informatIon In the passage, After providing the results of the studies of mergers and acquisitions, the author concludes. that even though acquiring firms state that thelf objectives are economic, factors having little to do with corporate economic interests explatn acquisitions (lines 22-24), Among alternative explanations, the author points to managerial error in estimating the value of firms targeted for acquisition (lines 27-28) as possibly contributing to acquisition behavior in the 1970s and 1980s,
and 1980s. In lines 7-10. the passagc ::~~ thar rhe second srudy found rhat thc .• U /bon gains ro most of rhe acquiring did nor olfiet, or at least equal. thc pricc 10 cqUlte th firms.
S
~:=~~doc
nor mdicare whal " acqwnng firm , ifanI', .1IICII1lI1on lCrm galD m rheir SIOl k
WIShed 10 Imllale olher managers Managers e they saw how financ18l1y manly becaus pn . 0 th er firms' acquISitions were, benefiCial
(8)
lICI/uislIions were based on an : : : ; ; of the value of largel firms. ,1ft..... NO rea zed by most aCQurrmg firms dId lIQIIalthe amounts expended 10 aCQUlnng
11Ie pa age docs nor discuss postUl ilIOn d,vesring. 'lbe pa age mdlCatcs that on avcrage, the profirabtliry ofacquired firms fell after being wred (lines 5-7), The p age docs nor indicate whether most wnng firms overestimated the v.• lue of linn th ya quired. ~lI8let, 'lbe pa age .tates rhal ti'r most wnng firm rhe ost. of buying the . .lIinld lirm were greater rhan the /(ain, m cquJrlng if.
0'
flJe otthe passage ,mphes Ihal which f The author ,ble parMI explanallon Or IoI/OWlng ,s a POss durmg Ihe )970s and) 980 ? s aCQUisitIon behaVior
A
I I
8
I c D
E
While the passage indicates that managers may have modeled their behavior on other managers, it does not provide a reason for why this would be so. Correct. The author states that one explanation for acquiSition behavior may be that managers erred when they estimated the value of firms being acquired, The author discusse a lack of monitoring by boards of directors but makes no mention of COnsensus within these boards. The author does not discuss compensation packa,ges for managers. The passa,ge dnes not state how significantly the value of the biddinl( firm's stock increased upon announcing a me'Ker but onlv that it increased less in value than did the' stock of the prospective firm bein/( acquired, OJTect an w riB.
Questions 71-76 refer to the passage on page 41 71.
The pnmary purpose of the passage IS to (A)
(8) (C)
(D)
(E)
dISCUSS a plan for inVestIgation of a Ilhenomenon that IS not yet fUlly understood present two explanations of a phenomenon and reconcile the differences between them summanze two theones and suggest a third theory that overcomes the problems encountered In the first two descnbe an alternallve hypothesIS and provide eVidence and arguments that suPPOrt it challenge the validity of a theory by expoSIng the InconsIStencies and contradlcllons In rt
Main idea
B
CJ
ID
bound adenosme inhibits uc~::, tE)
Examining the structure of the whole passage helps to identify the passage's primary purpose or main idea. The first two paragraph introduce a recent hypothesis about how caffeine affects behavior. The third paragraph looks at an earlier, widely accepted theory and then presents the objections to it made by the scientists proposing the more recent hypothesis, The fourth and fifth paragraphs provide evidence to suppa,: the newer hypothesis. Since most of the passage IS devoted to the recent hypothesis, dearly the primary purpose must be to present that hypothesis to readers. A
B
C
D
E
The passage discusses a current investigation. not one planned for the future. The passage examines two explanation. but the earlier theory is discussed only to e""pose its weakness and the differences betwee~ the explanations are not reconciled. ~Iost ot the passage is devoted to the more recent hypothesis. , Only two theories are presented 10 the passage. , Correct. The recent hypothe i ~roVlde ~ alternative to .m eulier one and 1 uPPO by evidence and arguments, th Lines 32-37 do pose uch a challenge ~ earlier theotl'; however, the challe~larlyl hlpaage.lffil,m small part at"h t e woe the the fiml par.lgmph. an ptlon toonly to .. trodu more recent theory 1 an . nnrtlOot dismissed a, an umm,. _ con
The correct answer i D.
perm
B
D
-:a
Inh btts both neuron phosphodoester'ase, Whllllll concentration mh bits on
. d caffeine bi nd to receptoll denOSlne an [lOU' a The third paragraph attests to the Ihe btlll n. .ne to bind to the receptors. The laat L~L
oftheophyl!id 'bes IBl\IX as a compound th ph escn h at p~ra gra deno sine receptors. Thus, t e excepti bonds to a 'me phosphodiesterase. 011 has to be the enZ)
_
. . . .fllltIrIIn 011 enzymes In the brain .~Il:fiIIIII8IIlIons/rips with caffe,ne IMlIfue effect on mouse locomobOn ' ..~ ..NIl", to dislodge caffeine trom receptors In
_.,tWlN:lll
. 5' -55 state that IE IX bmds to Lones"-receptors. a . b' . 17 19 state that callelne Inds to B Lones receptors. . . CLines 13-14 state that adenOSine bmds to receptors. . o Lines 016-50 state that theophylline binds 10 receptors. E Correct. The passage includes no eVidence that phosphodiesterase binds to receptors.
I
_lJIOoind bmd to the adenosine receptors, iIIIlUd of acnng as a stimulant as other ne derMlt.ves do, it was found to depress monon m mIce nyder et al. explain that 1BIw( IHu mlUd1ftm in Ih( brain, a l/olll11UJual M1Jm1",a Wllh /'JythOa(liv( druK' (line 58-60).
I I 75.
Snyder et al. suggest that caffeine's ability to bind to A, and A2 receptors can be at least partially attributed to which of the following?
be pJained by mix'"1ft(IS il/lh,. brail/, which nyd ret al. say may occur with p hoaellve drugs. B Thl re ponse refers back to the earlier theory, not to Snyder et al.'s response n mmg IBMX xperimenl results. aHeine I only included within the broad tqjury of p yehoadive dru/(s. D 1JIiI eRi II atlnbUled to one compound, I nullo all psychoa"'ive dru/(s. Th. bJllty I nol d,S(ussed in Ihe passa/(e.
~
I
bind 10 spec,fic receplors ,n the brain
I
I
I
The chemical relationship between caffeine and phosphodiesterase
(6)
The structural relationship between caffeine and adenOSine
(C)
The structural Similarity between caffeine and neurotransmitters
(0)
The ability of caffeine to stimulate behavior
(E)
The natural occurrence of caffeine and adenOSine in the brain
Supporting ideas This question asks the reader to find information that is suggested but not directly stated in the passage. The A, and A 2 receptors are mentioned i:1 line 16, so look at the surrounding material. Snyder et a1. propose that caffeine, whit'h is I//'/I(/lirl/I(I' limilur/o at/mosi",., binds to hath types of receptors, just as adenosine does. afTeine's ahilit}, to hind to tllese ' receptors may he due to , th,s structural "'I' ml'J' . aflty to ad enoSlIle .
B
aJJth
".lvID.a lhe ap1Iw th
t
E
results frnm, rather than h cause ,t IS pro<: Th e passage does not di.~· h '"'-u s t e natural occurrence of these compoun ds .
The correct answer is B.
.
Phosphodiesterase is discussed in an entirely different l'Ontext in lines 22-25. orr ct. Lines 17-19 suggest that catidne' structural . .( . SIlHl anty to adenosine may he responsible for its ahility to hind to AI and A receptors. .
77
(A
greaItf
the (B
76
perpetualed
argued
The author Quotes Snyder et al. In lines 43-48 most probably In order to
labor C1
one means
(A)
reveal some of the assumptIOns underlYing the r theory
(6)
summanze a major finding of their expenments pOint out that their experiments were limited to the mouse
E
(0)
Indicate that their experiments resulted only In general correlalions
Supporting ideas
(E)
refute the objections made by supporters of the older theory
(C)
Logical structure (A)
I
~~:::ID~e:SnylItr el aI, all of the follOWing
C a/feme acts on neurotnns tall' st ruc ur Y 'milano them.II'IIt1len: It not a· , C alleme ability tn 1m I L-hav U ate UIIC lOr #
The correct answer is E.
Correa. The results ofone experiment can
111I' llDIINeta/ll1wer i
O
A
........1llI1dNs
'Il Ifll queSfJOn, look at the Ia
C
To find the reason that the author quotes nvder et aI., examine the fourth paragraph, where ;he quotation appears. The paragraph starts with evidence supporting the new hypothesis on the basis of experiments with mice. The quotation then begins with the phrase in general, which implies a summary of the results of the work with mice. The quoted material explains that the more the compound were able to bind to the recepton;. the greater the stimulatory effect. This major finding supports the hypothesis.
JOb sec 0
reluc antly challellll!d when the ec a constant source te Ieondustry
The que tion use the p Ie~:~C pu= (, mdicanng that th a information tated In the PU88(l~ paragraph explain that" prr.rJIIiJ.'.... ftmininity often dictates wba women. For example, textilc:-mIill_lI!IIiI. hoary slmotypes am'CiaJ~d ,ruiIJ~ • acti< itits in order to justify tht:ir 'CIII. .,.llIiIll worn n. claiming that _1IC1I skillful dI ti. fa d t ..".jrptdiarta...a.,iIIli r~(tlm ch
,\
1
Ju r rhe reverse I~~~==t men were permill1l!d dunng the 5eclood WClliM ,how rhat t
A
B C
The quotation explains results at' an e, 'periment, not assumptions about a theory. Correct. The quotation 'ummcuizes the e "periment with mice and reports a major finding in support ot'the hypathe:i s. . The quotation generalizes on the ~asis ot th e. "periment; it does not limit the hndm to Inlce.
D
E
Specific. nllt gener,u. correlation were made between the ability to bind to re(eptors and to stimu1.ue lcKomN,on. l1le p,1S ,u~e includes no su hob) non thl're'lllte no refutation are needed.
The correct 1\IlSWer is B.
B
D
CofftCt. "f,atiile-IDiII __
ed from the passage that early 79. It can be Inferr men's labor ,n the United States p hlStonans of wto women's employment In the se~ little attenllon a ."" he economy because f t sector a
_.,...em
the extreme variety of these occupations rnaile (A) It very difficult to assemble meaningful stalisbc about them (8) fewer women found employment In the serVice sector than In factory work (el the wages paid to workers In the service sectOr were much lower than those paid In the indus sector (D) women's employment In the service sector tended to be much more short-term than In factory work (E) employment in the service sector seemed to have much In common With the unpaid Work associated With homemaking
of /lilller wages
detIiIed taslcs ~~=:::~ tv be less characterized by sex mote readIly accepted by women than by
fit the economIC dynamic of industrialism better IUflJlOlrtlnll rd.s
I
1JJ phrue tltttmlmg 101M ptnlOgt indicates that the an r I rated In the passage. Look at the tim paragraph, which discusses hislOrians of women labor. These historians disregarded terVlce work In favor of faclOry work not only becau e factory work differed from traditional m' work,' but also because the forteS of 'ntJus/n(lliJ1/l fNrt prtJumrd 10 br gmder-b/illd (lines 8-10). The passage does not indicate that historians stud,ed factory workers because of higher wage . B The passage gil'es no evidence that hlstonan chose this research area for this n:ason. Correct. The passage indicates that the hUtonan ho e this research area because they umed that sex segregation was less Jl"VRlenl In fa lor}' work than in SCtI'lcetor work. Although the pas ag slales lhat women 1Cl:.~ factory work more readily than did thi diffi:n:nce j nOI dIed in' lhe JIIlIIIJ~ the n: on hlSlorians focused on
ric may have lit the dynamic ltl betlcr, bUllhis is nOI lhe - ... il've for the hislorians
I
Inference
I I
I
ince this question asks for an inference, the answer is not directly stated in lhe passage; it must instead be derived from the information given. To answer this question, look at what the first paragraph says about the historians' focus on factory work. The historians disregarded service work and focused illslead onftelory work in part because il seemed so differenlfrom traditional, I/Ilpaid "womm's work" in the home (lines 5-8). Since the two kinds of work are explicitly contra ted, it is reasonable to infer that what is not true of factory work is true of service work; setl'ice work is similar to traditional, unpaid "womm's work" in the home.
80.
The passage supports which of the fOllOWing statements about the early mill Owners mentioned In lhe second paragraph? (AI
(B)
They sought to Increase the size of the available labor force as a means to keep men's wages low.
(e)
They argued that women were Inherently SUited to do well In particular kinds of factory work. They thought that factory work bettered the condition of women by emancipating them from dependence on Income earned by men. They felt guilty about disturbing the traditional diviSion of labor In the family.
(D)
(E)
supporting ideas The second paragraph discusses the assumptions and actions of the mill owners. To answer this question, look for a perspective or action that the paragraph explicitly supports. The mill owners accepted and perpetuated the stereotypes of women, including their supposed greater attention to detail and patience with repetitive tasks, and thus argued that women were inherently (by lIature) suited to the work in a textile mill.
A
B A
B C D
E
No mention of statistics is made in the passage and service-sector jobs are not characterized as extremely various. The numbers of women in each kind of work are not compared. Wage earned in the two kinds of work are not compared. Duration of employment in the two kinds of work is not compared. Correct. Historians disregarded service ~ork because it was similar to unpaid womm$ work" i7llhe home.
The correct answer is E.
They hoped that by creating relallVely unattractive "female" Jobs they would discourage women from 10sIOg Interest In marriage and family life.
C
D
E
The passage states that the mill owner assumed thaI womel1's "real" aspiraliollS wn-e for marriage alldftmiry' lift, which they used as an excuse to pay women less. The passage does not say that mill owners tried to keep men's wages low. Correct. The mill owners contended that certain facton' work was suitable to a woman's alleged patient, detail-oriented nature. The passage does not credit mill owners with trying to emancipate women_ There is no indication in the passage that the mill owners teIt any guilt.
The correct answer is C.
81
lAI
enlryolwo-.._. .
IB)
recogn
homema comparable to-::~~::: sector of the econclllly leI development of a new dIlIinili••IHI. . unrelated to the ecol1OlllC ll:raI.II~" ID) Introduction of eQUal profeSSions IEl emancipation of warne gender-determined JOb a111ocatM:1ft Inference An inference require going be:~td explicitly Slated 10 the pa age t aulMr, ideas that underlie that matenal T. undttllf.llllll this reference, it i, fir t necessary to 'lDIily:Ire~. context. The first paragraph explai histonans focused on factory w rk assumption that it was gender-h n emandptltory In eff~cf. However, th pa:rag,.,ib
concludes, mJ,me'fa/,oll ha exped"i, for nol t"< en mdustr (Seaptd .-onlmued sex s~ an
(lines 11-14). The phra e tim un/iJTiJIJ.ti ,...tWi.. occurs 10 the ne.·t enten,e, and ron/mued lex re~ga/, n m
passage tmplie, that the iluth r belieYl:s revolution is unfini hed beca allocated to women on the b.
B
c D
ng words besl expresses the
83 WI1iCh oIlhe fa::,r of the passage concernIng the QllinlOll of the a re more skillful than men In l/lal women a nolJOO I deta ed taskS? carryll1g au
IA) IBI
'paMn" (hne 24) 'repelillVe' (hne 24)
(e)
'hoary (line 26)
(0)
'homemaking' (hne 27)
""...._l1li WOrIcl Wit hlrfIW pokle CIU ed at the warllme III n ,n
(E)
'purVIew' (hne 28)
. .~'"' aoDOflurlfly
Inference
~::~:~"'6Iy prefer 10 llIfe DjletIItICe 8
;
durini the and World "'" I IIlr women for facrory
lI1CIllll1lc.
, ks the reader to make an Thi quesliOn as .' , ,n(erence abo ut the author s arlllUde, hWordh choict I • as it doe here w ben t e may revea I at(tude aut hor dcscn'bes the hoar. J Jttreotyp" a out womer that mill owners perpetuated, Hoary means old_ Itera IIy wh'te I with age-and so the stereotypes are " being dismissed by the author as old(ashlOned. even obsolete,
I r at the e anomy ha prOl/ed nearly pncIer bI nd ,n,t h rln pohCle
manuf MIIlIl
10'
'lI..,al1llltlldN. R.rjilw c-e,h _,.,
A
11,c mill owners stereotyped women as patltnt; this does not express the author's
B
C
I) , UI filum ludu rll.&1 Jill
n wlrh
I,
"pinion, [ill owners claimed that women were suiteu to r(petitive work; this doe not express the author's opinton, orreet. Iloa,,! carries with it a Judgment. sUAAe'tinl( th,;t the mill owner' tereotvp" .lfe impossibly antiquated. Ilom,mtlki"K uescribe, activities but does nOl r 'vc.tlthe author', op.nton, 1'1/1 ,/ •• mpl\' mean, .1 r.lO~e or.1 cope; it Illlt r"veal .111 Opln"ln.
ch 0 he fo g bes descnbes rr~:~1ll of the na paragrap 0 e passage as a ~ (A)
T e centra dea s re "'ur~ed e"dence drawn from
(B)
The centra Idea s res a'ed - s _ a form a trans n to a CeN Ou
Ie)
The central Idea s resta ed a" ;a- Sed ellldence ha rrugh aPPea':o co "'a-:
(0)
A partla' excep n 0 e ge-era za'"S -e central Idea IS d'sllllssed as r.acRecent history .s CI ed 0 s gg e central Idea's vahd, IS grad a
IE)
01
..
~'lti~.fts-.,._-.'.~
85
B
logical structure Consider the final paragraph in the comext r M whole passage to evaluate 1(; relation hip <) ( whole. The fi"t two paragraphs e.umin, job segre~ation in an unspecified but earlier rune, The final paragraph brings the reader into the twentieth century. when, as the e.xample drawn (rom the econd ''\Torld 'Var hO\, ,job segrel(atlon per i~ted. Thu , the fi~al p~guph updates and reinlorces the author s thC51> about the per istence ofjob segregation.
A
B
tI",·
·'I.e correcl n weri, C.
D
Correct. The cemral idea that -e:' e re"',uion continued in the l\S nti th,., entury workplace is reinlorced \S,th an e 'ample imm the ewnd World \\"a~ The <emral idea is the peNsten - 'J egre ,.,.llI',", whi -h "the nl~ t pic 10 t I'.uagr.lph. 111 la t p.lr.l!:raph ,UpP'-)r[' th P ,.,. no app.lrenrh
Ii Ill. "I.
I,,, IIllethth I
'ntun hi t Illral. I . n tt h
dllllllll
llw COtT ct
lun
11'''
ri
I l\\
I
th t
I
E E
g COUl1tt1l!S
E
liboi' In a social group can re lfIiciency with regard to the Of group tasks, IOn of labor In a social group causes /\I uities In the dlstnbutlon of opportunities lienefits among group members,
leI AdIVIsion of labor on the basis of gender 10 a social group IS necessitated by the eXistence 01 sex·hnked biological differences between male and female members of the group,
(OJ
IE)
Culturally determined distincltons based on gender in a social group foster the existence of differing altitudes and opinions among group members, Educaltonal programs aimed at redUCing Inequalities based on gender among members of a social group can result In a sense of greatet well· being for all members of the group,
Inference
'Ii. m,lkl' .111 infcrt'ncc ahout thc individu"list "'mini I Ir,lditinn. hC1;1I1 h}' rcrcading thc first p.tr.lgr,lph, which scts t,mh thc Jittering mod<,scf '''K'IIII,lIlal;oll. ReI.ltion.11 tenllnisrn hold th.lt the 11IOI",.. . , 0 I test', h " "tI d itll'r"ll ... us ·CS nUlst n:sult in a d,,', Ion oilihor 111 I h , ,SCl on t OsC J'IttcrCI1l'CS (hne " II), 11"'01111,"1, indi\'ldu.tlisl lerlllnisrn
. . . . lmplillllh I ~. plltllflle. m kl/1
downl'!.I\ Ih l 'nnport,tncc 01 . gl'ndcr ",Ics ,tnd ,111,,,1,, hldl' nnp0rl.tnl'l' 1<, thc hiologi"tI 11""11011 0' ""Idllc,uing (hnc 11 Ill), 'Ihus, il l III I" nlftorrl'lllil .1 I'IIll I'I\'ll Ill.' I 1st ' knlinlS(s \\'\.\\11 J I I I .Igrt'l wnh Ih'l I,l'nlt'nl II I It.I1 hll,logll',tI I II I I I, Il,rmllll' ,I gl'ndl'l h,lsl'd I rt nl<' h,,"11I Il l 1\ I 101101 I.ahul 1I1 slIrit,t\.
I"
1'.1 ,I' n'lIhll dl '" , illli ,I ",I th"l ", \ 110'" 1 I It.Il In, I"Idu.tli I I IIIUIl I "onld I je, I II
"I'''
The correct answer is C.
88. According to the passage. relational femnsts and Individualist feminists agree that (Al (B)
(C)
mdividual human fights take precedence Oller most other social claIms the gender-based diviSion of labor In SOCiety should be eliminated laws guaranteeing equal treatment for aft citizens regardless of gender should be passed
(D)
a greater degree of social awareness concerning the Importance of motherhood would be benefiCial to society
(EI
the same educattonal and economic opportunlttes should be available to both se es
Supporting ideas Thl' question asks for intormation that is e. 'plicith stated in the passa~ in sli!!;hrly dilf rent langu,lge. \ "hile the 1'.1. ,a > is lan:el d \ ted t the differcnce' betwcen the n\ lemini,t traditions. lines 30-34 indicate a point 01 convcrgencc, Indi\iduali,t Jemini,t, It III (qll,I1 righl.< {or .,11. Rebtional f mLfiisf'> It that (qu
-\
B
b 11 \ th
t t
t
B
. ns 91-97 refer to the passage On
ouest.o
92.
91
1:::mrrllCl
According to the passage, conventional sPiral galaxies differ from low-surface-brlghtness 1 In the following ways.
willi po/iIiCS than with (A)
IlIpn to ruch • broadB audience and i*lIIJIIIIS began to be adoPted by mlln· sIrIIm political parties. lbey called repeatedly for Intemabonal _penllton III10IIg women's groups to achieve
(B)
fe)
_pis
lbey moderated their Inillal CrItiCism ot the ICOnOIIIIC systems that characterIZed their
-
(D)
IE)
lbey did not altemptto unrte the two different feminist'llllroaches In their thought.
They have fewer stars than do low.surf~ bnghtness galaxies.
opJlJDIlng priontle ofthe relational feminists
lndMduai femml .It i rcasonable to inter both mln..t traditiOns pursued their own and did not try to ~ oneile the two different ~::: m their work The linal paragmph
fI ~~;::b~:the~twoalthough
mmi ttradilJon continue to the author offcrs a r harmomzatlon.
~
a
~§;~;~;§ad;voe sYJ/rm relatIOnalted mmll It
Ie r both
the uthor ;:A~::~1C~=11ei more
(AI (B)
They are large spiral gala~les contammg fewer stars than do conventional gala~les. They are compact but very dIm spiral galalUes They are diffuse spllal galaXles that occuPY a large volume of space.
They evolve more qUickly than 10w,surtiQ. bnghtness galaxies.
Ie)
They are more diffuse than low-surtacebrightness galaXies.
(0)
They contain less helium than do 10w.sUrf~ bnghtness galaXies.
(El
They are larger than low.surface-brightness galaXies.
This question requires drawing an inference from information given in the passage. The first paragraph compares dim galaxie and conventional galaxies. Dim galaxies are described as having the same general shape (lines 4-5) as a common type of conventional gala.'\" the spiral galaxy, suggesting that dim gala.'Xies are, themselves, spiral shaped. The pas age also indicates that, although borh types of gala.xies tend to have approximately the same number of stars, dim galaxies tend to be much larger and spread out over larger areas of space (lines 4-10) than conventional gala.'Xies.
Supporting ideas
Inferwnc.
A1111Wl:ring th qu tlon involves making an wim~. Befo~ 1890, the two feminisltraditions _rim"- Afler 1920, Ihe goo" of/he fWD approl/chtJ m ",mIlJ"'gly Im(()ntilahle (Jines 28-30). 30-40 provide detail on the differing and
It can be mferred from the passage that wIIch of 1tle followmg IS an accurate phYSical descr'llbon of !ypIcaI low-surface·brlghtness gala~les)
Thi question requires recognizing informatioo that i provided in the passage. The first pa describes and compares two types ofgalaxiel: conventional galaxies and dim, or low-surfaetbrightness, galaxies. It states that dim galaxies have the same approXimate number of stars as common type of conventional galaxy but tend be larger and more diffuse because their rna I spread over wider area (lines 4-10). The pas also indicates that dim galaxies take longertlu: conventional gala.xies to convert their primonli gases into stars, meaning that dim galaxies C\ ~lIch more slowly than conventional galaxies (lines 10-14), which entails that conventiorul galaxies evolve more quickly than dim galaxJCS. The passagc states that dim gala.xies halt approximately thc same numbers of tall " common type of com'entional g-al~ B Correct. The passa~e indicatcs that dim galaxies evolve much more slowl\' than conventional galaXies, which ent~i1 that onventional galaxies evolve more q The passage statcs that dim galaX! morc spread out, and therd'lle more than onv ntinn,d g,llaxies. 1h pa sag does not mention th Unt orh Hum in th tWO fVl-~ I'Illdes und r dl u ion. t th t dim gaIIt~
r hn
n nt
They are small. young spllal galaxies tha contain a high proportion of primordial gas. They are large. dense sPllals With low lumlnos
The cisCl1!CIiIIq
B
baryonc
and estimates IlaMd. '_ _ IuI11lllOSlty
C
The 1IIC000000;fe1ei,...:yy bet_i.ttl" of helium 1tle lliwtrse . . . . typIcallow-surfat:e-brW1-.. . . . .idIj
D
baryonc mass UncertaR1tJes
Inference
A
B C
D
E
Th
The passage states that the twO type- of gala.xies have approximately the same number of star. The passage indicates that dim gala.xie: a~ relatively large and 'pread out. Correct. Th~ pa.sage indicate that dim g.ua.xies h,we the, arne gen~ral hare asre; d spiral gal,t:.es and th~tlhel/ rna 1 sp out lwer large arc.l5 at ·pa,'C. The pas-age andicates that dim galill 1 IlK rel.lti,·ch 1.lrge and 'p~ad out. 111e n.ls.1 e ,tate' that dim galm~ h r '''Ill!:csnng that fel' st.U' per unit llt, 1um,'U thel' are not den e but di/fusc
""'at
regaI~~ciI:C::E=5!~
and wlIat .. prmOllOlmrrtiDllimn
in~::::::=
IE
D1ffic s onvoIved galaxM!s and IIM!sl
Inferenc:e
an IDem
than mcaSUJel of~......,'_
211hedi8i:_. . . . .
correct. The passage st~tes that the
t~e Universe maybe dis~overed in the dim ~alaxies that only recently been noticed by astro, The passage does not suggest that dint conventional galaxIes both originating rimordial gas clouds help solve the ~ ~tanding puzzle of the missing baryO!ll: mass in the universe. The passage does not suggest that dim galaxies might con~ain more helium tlu: conventional galaxIes, or that measurfl baryonic mass using spectroscopy do Ik:( take some dim galaxies into aCCOunt. The passage does not suggest that dim galaxies contain more baryonic mass tIu: scientists originally believed upon discovering these galaxies. The passage suggests that scientists measured the luminosity of galaxies, IlO( individual stars.
barvo nic mas
B
..
~SS
~~=::::
1II5IlIl' to the PIlule
IICGIIll per"""" prin8riIY beCause
ill
c
CGlIfIIIiIIlIr)'lllIIC IlIISS Ihal was notlaken
-:=~ byatrnIle resurcller U5lIII galaclJe the number of baryons m
"
.r.•11 CClIMIIlJOnI/ plaIJes!hat contam b1rJona.1IIYe evolved from maSSIVe, IIrimlIlRlalJlS cloud lIllY corQlII relltJve/jl more helium, and more baryon •than do galaxies whose conl8nl ha been studied uSing 11M recently been discovered to contain bar)'llIIIC mal than sctenbsts had thought ...........ce~l'Ilhln. 5 plaxl.s were ~rst abIIrillld
1hIl .re Iiniflc.nlly more lIIIillDUl tIIIn would 11M been predICted on the tudII ollumlnoStly In low
o E
che galaxy (line 17-19, a tronomers can em~ the baryonic rna s of a galalC)' b) measuring I lumino icy.
lfi
"-t
A
The passage disC\! ion ofbanons does no addre. the number of tar I~ individual galaxies. B The pas age disC\! ses how the luminos 0 galaxie can be u.ed to e timate baryomc mass, but doe not addre show 0 al luminosity i measured. C Correct. The pa age indicates chat because baryon are the ource of e;ala c luminosity, mea uring luminosity can be u ed to e timate baryonic mas oi galac .es. D The pa age sugge ts chat estimates based on luminosity may have been less accurate. not more accurate, than those based on pectroscopy. E The pa:sage does not indicate that bri~ ht galaxies contain more baryon' than do dim galaxies.
The correct answer is A.
95. The author mentions the fact that baryons are the source of stars' luminosity primarily In order to (A)
how astronomers determine that some ga contain fewer stars per Unit volume than do others
(B)
how astronomers are able to calculate the luminosity of a galaxy
--iHrfllhlnnl plllI
Ie) why astronomers can use galactIC lumina estimate baryonlC mass
IOJ
Why astronomers' estimates of baryomC nlI\I based on galactic luminosity are more re than those based on spectroscoPiC studIeS helium
(I
how astronomers know bright gala 115 more baryons than do dim gala les
v lu tlon
I qu t
n r qui
11,,._.
The correct answer is C.
96.
The author of the passage would be most leo disagree With which of the followmg statements' Low.surface.bllghtness gala les are more difficult to detect than are con entiOllal ga a es Low.surface.bllghtness gala les are 0 en s ra (8) mshape. Astronomers ha e ad anced plaus e leas (e) about where missing bar Ole rn3SS found. (0) Astronomers ha e de est,malln the tol um er e IE) Astronomer mount ot
IAl
B
. 98-104 refer to the passage on page 56. Questions
supporting ideas
According to the passage, all of the follOWing were 98. benefits of privatiZing state-owned Industries In the Umted Kingdom EXCEPT:
This question is based on information explicitly state~ in the passage. The second paragraph descnbes the Increased productivity, and the third paragraph begins by stating one reason for it:
,"1IIIII'islD
"'lIIIliIlf consider its
hMlrilllllena and discusS a puzzling
Infe_
Ilherween them ~ r.:4~!P
i.::: .lUII.
(A)
. .discovered phenomenon and cqins
(B)
two classes of objects and discuss the
~rtiesofeach
(C)
ctiscoIIery and point out Its ~ with eXisting theory
(D)
quaaon n=qwml understanding, in broad
(EJ
~_..dJepurpote ofthe
passage as a whole. The JIUIII1Iph ducribes a phenomenon: the cIiIlDVeryof dim galaxie and some of their pncnIattributes. The second paragraph describes diIcovery may help astronomers to solve "A'_r-eallldmg puzzle about the baryonic mass of
employees ofprivatized induItries 'Were glten the opportunity to buy shares in their OWn companies
(lines 28-31). The paragraph also cites the high percentage of employees buying shares in three privatized companies, supporting the idea that many employees bought shares. A
B
supporting ideas This question begins with the phrase according to the passage, indicating that it can be answered using facts stated in the passage. The fir t paragraph lists the benefits of privatization. Use the process of elimination and check the five possible answer choices against the benefits described in lines 8-16. The point that is OT discussed in the passage is the correct answer.
Co_=t. The passage describes the
=
Privatized industries paid taxes to the government. The government gained revenue from selling state-owned Industries. The government repaid some of its national debt. Profits from industries that were stili stateowned Increased. Total borrOWings and losses of state-owned industries decreased.
phenomenon ofdim galaxies and describes IIjpUlicance in solving the long.nellng puzzle of the missing baryonic In the umver e. MlboiiIP the pas agc discusses the puzzling
A Lines 11-12 discuss tax revenues. B Lines 10-11 discuss revenue from the sales. CLines 14-16 discuss debt repayment. D Correct. Profits from state-owned industries are not discussed. E Lines 9-10 discuss decreased borrowings and losses.
=~=:~mbetween the two estimates of • thi option does not accou nt broader topiC of dim galaxies. . . . . nge identific the newly . . .1lI phenomenon of dim galaxies, it Ilpllicant explanation for
........'0Jil!na.
C
D
E
The correct answer is A.
100. It can be Inferred from the passage that the author considers labor disruptions to be (A)
Thc correct answer is D. (B)
99.
According to the passage, which of the follOWing resulted In Increased Productivity in companies that have been privatized?
(e)
(D) (A)
Alarge number of employees chose to purchase shares In their companies.
(8)
Free shares were Widely distributed to indiVidual shareholders.
Ie)
The government ceased to regulate major Industries.
(DJ
Unions conducted wage negotiations for employees,
l£)
mploy e-own rs greed to h ve th Ir w ges
low red
Correct. Productivity increased after employees became shareholders in their companies. The theoretical advantages and disadvantages of free shares are di cussed (lines 43-52), but the passage does not say that any were given away. The passage does not examine governmental regulation. Although wages are discussed in lines 38-42, the passage does not analyze the relation between wages and productivity. Lines 38-42 cite one (,'{ample of employeeowner willingness to accept lower wage, but this is not said to have resulted in increased productivity.
(E)
an ineVitable problem In a weak nallonal economy a posilive sign of employee concern about a company a predictor of employee reactions to a company's offer to sell shares to them a phenomenon found more often In stare-owned Industnes than In pnvate compames a deterrence to high performance levels In an Industry
1hU quanDO ._11..... this mfiermcc thc second par1111J1l1ph.
_:: •
1lIiJ_.•"",
pnvat.zabon a, a the author 19-27. Onc cnmpIc labor dl rupbon labor d. rupbOns then the author must beliicge daft d. labor di rupnon unpcdcs performance level A
B
C
D E
The author does 1101 link "'iboI~ "11111~• .; with a ~k naMnla! '=1110I111. The aurhor doc not pn_1t di ruprions In a posl The author does IIOt identilfy disruptions as a predi r:Jllmplap" re ponse to opportum company's share Labor disruption in
stallHJ_".".1_.
private Industries an: ':~:::::i.lli~ Correct. The author iJ di ruption IJ'IlCi fae with ",,.I_IIr.•• pertormance in industry.
Thc correct answu' E.
101 The passage supports whM: statements about emplovees bul~ sIle_.~ own campame At three different ClII'II. . . . out of ten shares
(8 at three diIfI. . . .
4.
..
Application answe r this question. first identify the . . Ie ,'nvolved , and then . find the. statement . pnnclp that is most compatible WIth that pnnclple. Line, 35-38 argue that having a personal stake in a business makes employees work to make itprosp,,W'hen there is no personal stake. or self-interest, involved. employees do not have the same incentive to work hard to make their industry prosper. Thus, the poor performan~e of stateowned industries can be ascnbed In part to employees' lack of motivation when they have no personal stake in the business. or JO
• •4_lIIIlmt by comparing it to the ........ p_ted an the passage. Only one 1lIf'JlOfted. The thiro paragrnph pen:enlllge5 of the eligible. employees ".,,·,.11 pan:hued hlRS in their compames: 89 "Nlll!Ift at one compan}. 90 percent at a second,. patent at a thiro (line 32-35). Thus, It IS that IUlIghly 90 percent of the eligible work t three different companies bought shares an their mpame5 once the}' were given the opponunny to do so.
:=::
a~ cite the percentage, of the e1lgtble employee who bought shares, not the percentages of the total workforce that were eligtble. B orree:t. The passal(e shuws that roughly 90 percent of the e1il(ihle employees at three dJiferent companies boul(ht shares in their Companies. The pas8al(e docs not address the attitude of labor unions tuward employee share buying. The pa alte ulfers no evidence that mpanie with hil(h productivity were the fint to olfer hares to their employees. The p ag doc nut ,how eligibility to be d pendent on inucased workload.
The p
A
B
C
o
ri R.
Whlch I the 10110wlng lalements IS most conSistent Ifle IlrJ/IClpie d cnbed In hne 35 38?
The principle involves a personal, rather than governmental, relationship. According to the principle. self-interest rna)' inspire people to do more; government coercion is not consistent with this principle. Correct. State-owned industries perform poorly in part because employees do not have the powerful motivation of selfinterest.
D
The principle has to do with the motivation of individuals, not governments; eliminating all resistance to the free-market system is not discussed.
E
Lines 35-38 describe the principle of selt: interest, not self-sacrifice.
103. Which of the foliowlOg can be IOferred from the passage about the pnvabzatlOn process In the UOiled Kingdom?
fA (A)
IB
It conforms In Its most general outlines to Thomas Paine's prescnpbon for busIOess ownership.
C
It was ongIOally conceived to Include some giVing away 01 free shares.
(0)
(Dl
It has been successful, even though pnvabzabon has failed In other countnes.
IE)
(El
It is taking place more slowly than some economists suggest is necessary.
Logical structure
(B)
(el
Inference Answering thi question requires looking at each possible inference to see if it is supported somewhere in the passage. SuPPOrt for the inference about the pace of privatization is provided by the suggestion of some economists that giving awayfree shares wouldprlJ'l)ide a . needed acceleration ofthe privatizotion process (hnes 43-45). If some economi ts think privatization needs to be accelerated, then it mu t be going too slowly, at least according to these economist . A
The correct answer is C.
B
to
-JIll P1111lC InW I
i
EE5::I:to~ lor compames to oil compebllv market lid
It depends to a potentially dangerous degree on
counter a pa sage state a preYlOUS show howop author of the PI sage argumen s po nt out a paradox vewpa n present a h sto a pr nClple IIltroduced
indiVidual ownership of shares.
ClInlOCtlbc IDVernmentlhal deCides It IS opr Ie own a partIcular Industry has 10 IIIdicIted It te ponSlbJlllles as guardian
- , - - IIIIUIIr llO\Itrnmenllo protec I
104
C D
The passage does not allude to any danger in individual ownership of hares. Paine i quoted only in reterence to emplovees' receiving free shares a. opposed . to buying hares; aIso, the process ot privatization had occurred betoreemployees bought shares in the newly pn\'atlzed companie . . " he distribution at t a evidence supports ,.'. free shares as part 0 t"t he United Kmgdl'ffi NI r.ts plan to privatize. h A phr'lse in line -I, Ot/( "tP-.- t h • "there were ot er suggests th.\[ perh aps · t t work' ho that approaches that dII no . d'. h nowhere does tepa aeo""- m .ll-ate oth . . h vorked m er pri\".ltlz.ltlOn a~ not' countrles. C ornct. The e.:ooomi ts sug;ges
,r.
. -• •UIlIlOWIt 01
IIlnlere I ~"lIIIlloOWln d
l
E
from wh.lt th pmce s that 1
1b
th
rakiDI
toll, . . .
Lookinll; at the qu tatl n understanding ofwhy the quloc.ltiCln _... Pame's quotation olfers a 0 honored counterargument to the precedIng sentence. The eCl_lIIIi...,_ . _ giving away free share • but the autbur these economists are furgcmng Paine. people do oat alue what cheaply. The author use the qw:J!alioa ••__ the ba 'ic error In the ecoo,omisls' rbi_Ill-
"'.".1.
su
B
c D E
'Ihecomm _ _.iaA,
4.0 Critical Reasoning Critical reasoning questions ap ear in th Vi uses mulhple-ch " . p e erba1 sectxm of the GJilAtIlJI• • Olce questIons to measure your ability to read and "'IIl1llp. . . . . reason an d to evaluate ar d W h B guments, an to correct written material ~;:=:' E ng1·IS. ecause the Verbal . . I d (T "h eChon InC u es content from a van..... ami lar Wit some of the material· howeYer . h h -'I knowledge of the topics discussed' C 't" al , nelt .er t e pas ages nor the queetlli h . . rI IC reasoning questIon are JDltelminallld ... compre ensIon and sentence correction question throughout the Verbal section You will ' . have 75 minutes . . to complete the Y,e rb aIeChon, or about 1 IIlJIIlIla question. Although crItical reasoning que tions are ba ed on written pallH@:a. shorter than reading comprehen ion passages. They tend to be less than Wl:~~~~.:~I~:. generally are f~llowed by one or two que tions. Fo~ the e que tions, you will ~creen. The wfltten passage Will remain visible a each question as ociated with In turn on the screen. You will see only one question at a time.
100
Critical reasoning questions are designed to te t the reasoning skills mvolved m 1 "::~~= (2) evaluating arguments, and (3) formulating or evaluating a plan of action The mal questions are based are drawn from a variety of sources. The GMAT test does not SIIIlPGfIIllg'familiarity with the subject matter of those materials.
bu.
In these questions, you are to analyze the situation on which each question I _II t11.-1_1lI the answer choice that most appropriately answer the question. Begin by reading ":::~ carefully, then reading the five answer choices. If the correct answer i not unmlcdi_ly~ you, see whether you can eliminate some of the wrong answers. Reading the . . . . .1 may be helpful in illuminating subtletie that were not immediately evtcknt
4.0 Critical Reasoning
Answering critical reasoning questions require no 'pecialized knowledge of any S-niI:a11... you don't have to have knowledge of the terminology and ~nventions offilnMl critical reasoning que tions in this chapter ilI~ trate the vanetyoftop ._~lilill'_ _ kinds of que tions it may ask, and the level ot anal} It reqwres. The following pages describe what critical ,:a~ning question aare:::::~. the directions that will precede que tlon ot thl type. ample q correct answers follow.
4.3 The Directions
Measured . designed to provide one m ~(MticlII reasoning que tlons are the fiJl/Dwing area :
easure 0 f your abilitv ./ to reason effectively .
truction , h thin s as the ba ic tructure of an • category may ask you to well-supported explanatory ~ d oncluslOns , un aJgUment, properly rawn c tructurall)' similar arguments, hypothe es, and parallels between
~::~t ~:~~iS
dr:~~)~~glz:s:~~ptiOn~,
• Arpment evaluation , r ument and to recognize uch things a The e questions may ask you to analyze a glve~ a g gument. reasoning errors committed in d ' h veaken the gIven a r , factors that would strengt en or , h db' which the argument procee s. making that argument; and aspects of rhe met 0 ) 'ng a plan of action , • Formulating and eva Iua t I , h h' s as the relative appropnateness, k t recogmze sue t lllg k This type of question may as you 0 f" f: tors that would strengthen or wea en ' ffi . f different plans 0 actIOn, ac d effecti,'eness, or e C1ency 0 d Ian of action; and assumptions underlying a propose the prospects of success of a propose p plan of action,
4.2 Test-Taking Strategies 1.
Read very carefully the set of statements on which a question is based. Pay close attention to • what is put forward as factual information • what is not said but necessarily follows from what is said • what is claimed to follow from fact that have been put forward • how well substantiated are any claims that a particular conclusion follow from the facts that have been put forward In reading the arguments, it is important to pay attention to the logical reasoning used; the actual truth of statements portrayed as fact is not important.
2.
Idllltlty the conclusion. The conclu ion does not necessarily come at the end of the text; it may come somewhere in the middle or even at the beginning. Be alert to clue in the text that an argument follows logically from another statement or statements in the text. IJIIlIIIlln••uetly whit elch question asks. mlibt find it helpful to read the question first, before reading the material on which it is don't sumc that you know what you will be asked about an argument. An argument ObvlOU ftaw I and onc question may ask you to detect them. But another question u lD elect th onc answer choice that does NOT describe a flaw in the argument.
"«_I' cholc.. clr.fully. p¥ n answer is the best without first reading all the choices.
These are the directions you will see for critical rcasonmg qwlltiOlll you read them carefully and understand them clearly bd'ore ani_ lID ... th. til. to spend too much time reviewing them when you are at the test,...,,For these questions, select the best of the an wer choice gIven
nt a
r
pi
I)
of
n
: : : : : required to rna e aI garette by eqlJlllPlng them Ih safel : u ely to resun In a the number of fires cau ed b
."''''1WinII
with filhters because ch Idren given . . . .nlllily CIII figure out how 10 rk Ihe safety
Iddition of the safely levers has made
/illllets more expensive than they were before
A I·effecll e solullOn 10 the problerl at IIpoil can esllOn IS to provide hi h peed roulld IransporlallOn between malar 'tie I III "00 to 500 miles apart. The succes fullmplenenlallon of thl plan I'oOl,ld cost tar less Ihan e pandlng I tlng allports and would also reduce the numbel of allplanes cia glng bolh alrpor t nd 111 W ys Which of the follOWing, If true, could proponents of the plan above most appropriately cite as piece at eVidence for the soundness of their plan (A)
reQUlfeITIent was Inslrtuled
IlIuIts are more likely to leave chlld-reslslanl Wtters than non-cMd-reslslanl IIghlers in places that are accessIble to children many of the fires started by young children are quickly detected and eXbngUished by Ihelr parents
(8)
not chlld-reslstant can be operated by children )'IlUng as two years old IPPRlXlIIlately 5000 fires per year have been Itlributed to children plaYing with IIghlers before fety levers were reqUired
(Al
Whether the proposed method of momtaring viewer hip can distingUish between people and pets
(8)
Whether radar speed momtors also operate on the prinCiple of analYZing reflected waves of elec tromagnetlc radiation
(C)
One-half of all departing flights In the nation's bUSiest airport head for a destmatlon In a majOr city 225 miles away.
Whether the proposed system has been tried out In various areas of the country or In a Single area only
(D)
What uses are foreseen for the Viewership data
(E)
Whether the average dose that the proponent deSCribes IS a short-term dose or a lifetime cumulative dose
The majority of travelers departmg from rural airports are flymg to destinations IP cities over 600 miles away
(D)
Many new airports are bemg built In areas that are presently served by high-speed groundtransportation systems.
(El
Which of the follOWing ISsues would it be most Import,lOt to re olv In evaluating the dispute concermng the health etfects of the proposed system?
An effective high-speed ground·transportat,on system would reqUire major repairs to many highways and mass-transit Improvements.
(C)
unIiIle child-reslstant lighters. IIghlers Ihal are
People' teleVISion VieWing h bil could be monitored by h,IVlIlg teleVISion el ,when on, end outlaw level electronM Iletlc wave Ihat are reflected back to the set The lefl ted wave auld then be analyzed 10 detel/lIlll how mallY per on are Within the viewing alea of lhe et Crilic fear adver e health effect of such I momtorlllg ystem, but a proponent re pond, 'The average dose of radiation IS less than one ehe t xray A they watch, Viewers won't feel a Ihlng •
A large proportion of air travelers are vacationers who are taking long-distance flights.
(A)
(B) (e) (D)
(El
5.
Since the mayor' pull,""II. dill bus serVIce began me traffic Into the midtllwn .......
-::==
7 percent Durlnll the eqUivalent n51 In the IIU
Into the Il'IidtlIvmrm~:~:;; campaign ha C4 cars at home .1Id 11IflI"'1.~
.,...!
Which of the follcMil& n . doubt on the COII• •11!l
W F_far. . . . . (81
1
archer d,scovered that people who have low A rese. ne.s stem activity tend to Score mUch 'evels of Immu f ~ental health than do people With lower a mune.system activity.h The researcher ormalonortesths hlg 1m from this experiment that t e Immune n concluded system protec tS agalnst mental Illness as well as against physical disease.
IIauIt In tile
M1ckMthe at end of !~?3l1l11fL lIlItwwll TIle weekly 1/1 hourly
IIIUIIy lIdeQlIIIe but not
=:':'PIFfic*Iy: HI the WImer, when
=
..
and be unpredictably
The researc he r '5 conclusion depends on which of the . ? following assumptions.
Ifllllftlllfll'DlkJwlllJlrfllq plans, if put,nto effect, has the =;:tI~lIIIIlinI:; Patrick's weekly ,ncome both
~~~.flIIICII"C4~ with aneighbor who provides
care under mllar arrangements, 50 that ~*,,'tilf~~ of ltIem cooperate In caring for twice as II IIIk1ren as Patrick currently does.
"lICe JNlYIII8nt ,~ WIth
by actual hours of child care a fixed weekly fee based upon the .fI)r./l' !II of hours of child care that Patrick would be Illpected to prDVIde. fuJl.tfme helper and ,nvest ,n facilitIes for ,,",,1IlI1"" chtld cere to sick chIldren.
=~;;~ lite hourly rate to a 'eve/lhat would
~
_uatllncome even In a week when ~""'4lllJIdr.n Patrick usually cares tor are
~.ft IIl/IIIber of hours made avaIlable for
cIt>I SO that parents can leave
~:::::: ~PatrIck~ 'care for a 'onller penod rrent hourly rate
:.
B.
(A)
High Immune-system activity protects against mental illness better than normal Immunesystem activity does.
(B)
Mental illness IS similar to physical disease in Its effects on body systems.
(C)
People with high Immune-system activity cannol develop mental illness.
(0)
Mental illness does not cause people's Immunesystem activity to decrease.
tE)
Psychological treatment of mental illness is not as effective as is medical treatment.
Extinction 15 a process that can depend on a variety of ecological, geographical, and physiological vanables. These variables affect different species of organisms In different ways, and should, therefore, yield a random pattern of extinctions. However, the fossil record shows that extinction OCcurs In a surprisingly definite pattern, with many species vanishing at the same time. Which of the following, If true, forms the best baSIS for at least a partial explanation of the patterned extinctions revealed by the fossil record? (A)
Major episodes of extinction can result from widespread environmental disturbances that affect numerous different species.
(B)
Certain extinction episodes selectively affect orllanlsms With particular sets of charactenstlcs unrQue to their species.
(e)
Some specIes become exllnct because of accumulated liradual chanlles In their local environments.
(0)
In lleOIOlllcally recent times, for which there IS no foss,' record, human Intervention has chanlled the pattern of extinctions. Sp cl Slh t are widely dispersed are the least IlJlely to become extinct
9
In parts of South Amenca, Vltamln-A deficll!llCy a serious health problem, especially among children k1 one region, agriculturists are attemPting to IIl1IJrove nutrition by encouraging farmers to plant a new vanety of sweet potato called SPK004lhat IS nch HI betacarotene, which the body converts Into Vltamm A The plan has good chances of success, since sweet DOlala IS a slaple of the region's d,el and agrlcu~ure. and the varieties currently grown contain Irttle betiH:arolene Which of the follOWing, If true, most strongly suPPOrts the prediction that the plan WIll SUcceed? (A)
(B)
(C)
The growing conditions reqUIred by the vanetles of sweet potato currently cultivated In the region are conditions In which SPK004 can flounsh. The flesh of SPK004 differs from that of the currently cultivated sweet potatoes In color and texture, 50 traditional foods would look somewhat different when prepared from SPK004. There are no other vanetles of sweet potato thaI are Significantly richer In beta-carotene than SPK004 is.
(D)
The vaneties of sweet potato currently cU~lvated in the region contain some important nutnents that are lacking in SPK004.
tEl
There are other vegetables currently grown In the region that contain more beta-earotene than the currently cultivated varieties of sweet potato do.
fA) (8)
c:::::::=
iI'IY UIiIomI. W to be""" any SlAisIatte tan act n'y
(CJ
the CfU1lnes tau. those II -41
6ents
~::::;
ID)
aM of any gM!II
IE)
erMrOI1IIIl!I1IaI ddli:I.... North Sea IS l1!IIel5illlti
ae:tuaIy reaches . .
::~~~~==
Iarae
I"IIIhIIUI8 trade tile largest budget ~_st1rade deficitS. In Ifjusted so that different iii'III,-Mlle to each other, there
14.
From the passage above, It can be properly Inferred that the effectiveness 01 the sustained masSive use of pesticides can be extended by dOing which 01 the folloWing, assuming that each IS a realistiC POSSibility?
Fllf'!'JlfM. OllIlIl1ries with large Mbonal budget deficits tInd "striet foreign trade. Reliable compansons of the deficit figures of one eauntry with those of another are Impossible. RedUCing a country's national budget deficit will not necessanly resun In a lowering of any trade deficit that country may have. lDI When countries are ordered from largest to smallest In terms of populatIOn, the smallest ountrles generally have the smallest budget and trade deficits. lEI Countnes with the largest trade deficits never have similarly large national budget deficits. Which of the fOlloWing most logically completes the
IrlUmant7 The II t members of a now·extlnct species of a European wild deer called the giant deer lived In Ireland about 16,000 years ago. Prehistoric cave IIIillinIs In France depict thiS animal as haVing a large hump on it back FOSSils of thiS animal, however, do not show any hump. Nevertheless, there IS no reason conclulle that the cave paintings are therefore InIccurete nthl regard, since
The sustalne d massive use of pesticides In farming ff t hastwoe ec s that are especially perniCIOUS. , .First ' rt often kills aff th e pests' natural enemies . In the area. Secon d,I't aften unintentionallY gives rise to Insec tICI de- resl'stant pests, ,since.those Insects that survive a pa rtl'cular InsectiCide will be the ones most resls tan t to I't, and they are the ones left to breed.
(A)
USing only chemically stable Insecticides
(B)
(0)
Periodically SWitching the type of Insecticide used Gradually Increasing the quantities 01 pesticides used LeaVing a few fields fallow every year
(E)
Breeding higher-yielding varieties of crop plants
(C)
15.
In an attempt to promote the Widespread use of paper rather than plastic, and thus reduce nonbiodegradable waste, the council of a small town plans to ban the sale of disposable plastic goods for which substitutes made of paper exist. The council argues that since most paper is entirely biodegradable, paper goods are environmentally preferable. Which of the following, If true, indicates that the plan to ban the sale of disposable plastic goods IS ill SUited to the town council's environmental goals? (A)
Although biodegradable plastiC goods are now available, members of the town council believe biodegradable paper goods to be safer for the environment.
(B)
The paper factory at which most of the townspeople are employed plans to increase production of biodegradable paper goods.
(C)
After other towns enacted Similar bans on the sale of plastiC goods, the environmental benefits were not discernible lor several years.
(0)
Since most townspeople prefer plastiC goods to paper goods In many Instances, they are likely to purchase them in neighbOring towns where plastiC goods are available for sale.
(E)
PrOducts other than those derived from wood pulp are often used In the manufacture of paper goods that are enhrely biodegradable.
prahl tonc cave parntlngs In France also
.lictother animals as haVing a hump of the giant deer are much more common . . .I_lICIlhan In France
:
: : : : :ar
compOsed of fatly hssue,
fo IlIze
".•_11 01 000
the IlInt deer were painted year ala
nl plCles 01 deer has .lIIure that lVen r motely
16. Since the deregulation of airlines, delay at the nation's Increasingly busy arrports have Increased by 25 percent. To combat thiS problem, more of the takeoff and landing slots at the bUSiest arrparts must be allocated to commerCial airlines. Which of the follOWing, If true, casts the most doubt on the effectiveness of the solution proposed above) (A)
The major causes of delays at the nation's bUSiest airports are bad weather and overtaxed air traffic control equipment.
(B)
Since airline deregulation began, the number of airplanes In operahon has increased by 25 percent.
(C)
Over 60 percent of the takeoff and landing slots at the nation's busiest airports are reserved lor commercial airlines.
(0)
(E)
After a small Midwestern airport doubled ItS allocation of takeoff and landing slots, the number of delays that were reported decreased by 50 percent.
(AI
~::~=~:
(8)
Phys c a
IC)
The rev ew P l!III-. .IlllII.... always resu procedure The company's rev I!W orOl:esS does..... With the prerogabve
(0)
Since deregulation the average length of delay at the nation's busiest airports has doubled.
ft that are he pfu bul the heanh of e patn
efl~~~~:==tJ
(E)
where more!ha one avatlable to select !he a prefer. The number of mem appomted reVl1!W panel procedure depends the COlItaI procedure
" '••"1IIIlI1II. the wholesale IMliIIIn ClDIlSiderIbIY nthe last - : : : : : pIKlI! of cotton clothmg
not yet fallen, it will
•
~!:::::~' if true most seriously weakens I of processmg raw colton for cloth has Increased dunng the last year. The wholesale price of raw wool IS typically igtJer than that of the same volume of raw cotton The operallng costs of the average retail cIoIh,ng store have remllned constant dUring the lasl year. OJ Changes mretlll prices always lag behmd hanges ,n wholesale prices. lEI The cost of harvestmg raw colton has increased ,n the last year.
20.
Start-up co mpames financed by venture capitalists have amuch lower flliure rate than companies financed by other means. Source of finanCing, therefore, must be a more Important causative factor In the success of a start-up company than are SUch factors as the personal characteristics of the entrepreneur, the quality of strategic planning, or the management structure of the company. Which of the following, If true, most seriously weakens the argument above? (A)
(B)
lC) (D)
9
A rnputer eqUipped with slgnature·recognitlon software, which restricts access to a computer to tho epeople whose signatures are on file, identrfies a PI n ignature by analyzing not only the form of the ""alure but also such characteristiCS as pen re and slllmng speed. Even the most adept fortflfS .nnot duplicate all of the characteristics the am analyzes. the followmg can be logically concluded from 1111 Pllisag,e above' me it takes to record and analyze a Qllture makes the software Impractical for
rydeyu e equipped with the software will soon
~11n1t11Jed nrno t banks
In .cce s to a computer the Oftware solely by VIrtue of
-110 '''''''eture
~l/llIlon
ftware has laken many If.et
IIltldhclrlzed users are demed -IlUlUterl .quiPPed wllh
(E)
Venture capitalists tend to be more responsive than other sources of finanCing to changes In a start-up company's financial needs. The strategic planning of a start-up company IS a less Important factor In the long-term success of the company than are the personal characteristics of the entrepreneur.
21
Aphasia, an Impairment of the capaclly to u e language, often OCcurs when a stroke da mages the left half of the brain. Many people With str k I . oereated aphasia recover at least some capacity to use language Within a year. One proposed explanation for such recoveries IS that the nght side of the brain, which IS not usually the major language center develops ItS latent language capabilities to ' compensate for the damage to the left side. Which of the followmg, If true, most strongly supports the explanation? (A)
(B)
More than half of all new companies fail Within five years. The management structures of start-up companies are generally less formal than the management structures of ongoing bUSinesses. Venture capitalists base their deCisions to fund start-up companies on such factors as the charactenstics of the entrepreneur and quality of strategic planning of the company.
wate th etr Imga
(C)
In a study of local brlln aclivlty In people performing a language task, people With strokerelated aphasia showed higher activity levels In the right half of the bram than people who did not have aphasia.
fAI
IB)
A blow to the head Injuring the left half of the brain can result In Impairment of the capacity to use language Indistinguishable from that produced by a stroke.
Ie)
Among people With stroke-related aphaSia, recovering lost capacity to use language does not lead to any Impairment of those capacities normally controlled by the right half of the bratn.
(D)
(D)
A stroke that damages the left half of the brlln often causes physical Impairments of the nght Side of the body that lessen over time.
(E)
Studies of numerous people with aphaSia have indicated that the functions that govern language production and those that govern language comprehension are located in separate areas of the braln_
IE)
The tree a the u and amou t Owners If undertake along the ban greater aloca Many of the Ire the nver's ban growing In pia e constantly wet The slnp of land whe nver s banks we Id crops if the tree The d,slnbution IS .ntended to PI from using water downstream
24. Automobile Dealer's Advertisement: fDr tIDor IPICl!
The Highway Traffic Safety Institute reports that the PI 1000 has the fewest Injuries per accident of any car In Its class. This shows that the PZ 1000 IS one of the safest cars available today.
pndJce III h gIr, howeVer, about four times as 1IeId sprI8Ch, the spinach
Which of the following, If true, most seriously weakens . t1 the argument in the advertisemen .
true best suPpOrts a "IN! the spinlch-growing facility near
fA)
jlIuftlllblf' (B)
I'w.tllllGlll1lll operators of the lacility are ~......rilnCld, they Will be able to cut operating
.,·.dI_1IIII by about 25 percent
fC)
virtually no scope lor any further
~.i>':::~B1~the co t per pOund lor California (D)
1.<4• • Wlllred field spinach, the hydroponically mlCh I untainted by any peshcldes or ....·'4lIrl11oidls .1Id thus Will sell at exceptionally high to such customers as health food • • •llIClh I I crop thlt ships relatively well, lhIlIlIdlltfor the hydroponically grown sPinach ,jtllOl mar. JlIIlited to the Chicago area than the .'dorOla field spinach IS to • • hydrOpOnic 'acility IS being bUilt In IUn,ldvlntage of inexpensive "':trId1ly Ind hl,h v getable prices
(E)
25.
Which of the follOWing, If true, most seriously weakens the editorial's argument'
In recent years many more PZ 1000s have been sold than have any other kind of car In ItS class.
(A)
Cars In the class to which the PZ 1000 belongs are more likely to be Involved in accidents than are other types of cars.
(B)
The difference between the number of injUries per accident for the PZ 1000 and that for other cars In ItS class is quite pronounced.
IC)
The fire department gets less information from an alarm box than it does from a telephone call
(D)
The City'S fire department is located much closer to the residential areas than to the commercial distriCt.
Ie) (D)
(I
there are more fossilized mastodon remains from the period before mastodon populations began to dechne than from after that period the average age at which mastodons from a given period reached reproductive maturity can b e labhshed from therr fosslhzed remains It an be accur I Iyestlm ted from fosslhzed remains when mastodons bec me e tinct IllS nol known when hum n first began huntln' m lodon 11m I ch nil m y h ve Ilradu lIy r duced Iht lood v II bl 10 m lodons
•
mpl'OIIeIl'N!I1ts
The Highway Traffic Safety Institute issues reports only once a year.
Either food scarcity or excessive hunting can threaten a population of animals. If the group faces food scarcity, individuals In the group will reach reproductive maturity later than otherwise. If the group faces excessive hunting, IndiViduals that reach reproductive maturity earlier will come to predominate. Therefore, It should be Possible to determine whether prehistoric mastodons became extinct because of food scarcity or human hunting, Since there are fossilized mastodon remains from both before and after mastodon populations dechned, and .
(B)
EditOrial: The mayor plans to deactivate the Dnlrc .._ alarm boxes, because most calls recetved from them are false alarms. The mayor claims that the alarm boxes are no longer necessary, Since most P80IlIe now have access to either pubhc or Private telephones. But the CIty'S commerclli d'stnet, Where there IS the greatest risk of fire, has few residents and few pubhc telephones, so some alarm boxes are s necessary.
The Highway Traffic Safety Institute report listed many cars In other classes that had more Injuries per accident than did the PZ 1000.
Which of the following most logically completes the reasoning?
(A)
26.
(E)
over the past
Malntarmng the fire alarm boxes costs the city more than five million dollars annually
B
CommerCial bUildings have automatic fire alarm systems that are hnked directly 10 the fire deparlment.
other states. AdeQuacy
lD~:::::::
less fnloortant dlltllg business ,
C
10
On average, almost 25 percent of the pubhc telephones In the city are out of order.
E
QUalIfied l!IIlI*I~ Over the past hIIIl! left the 5POIII!SIII'.n s-.~ many businesses In general the runllef ol'IIiII• •~1l sates road system dell_I....... and the llOIIlIIabDn Only states systems need
on road lInt:irOllll!llll!llts.. 2
Company Alpha are awarded the COllpOlliS bf g eq
8rI.,.
tile taetors that
c/etIriCJrate c:hIrJgeS have
l1Ie restorers therefore llllleria cafted gesso, ClUsesthe deterioration.
If the airspace around centrally located airports were 34. restrlcled to commerctal airliners and only those private planes equipped with radar, most of the prlvate.plane traffic would be forced to use outlYing airfields. Such a reduction In the amount of Private· plane traffic would reduce the risk of midair colliSion around the centrally located airports. The conclUSion drawn In the first sentence depends on which of the follOWing assumptions?
-:::~
i
==::drrllllc
na
with a thin layer of
(A)
tiIleIy to how deterioration 10 changes than those with a
InIInp are often parnled on 'Wh hswell when humidity
(8) (C)
ntrIctwhen It declines.
end ontracts readily 10
(0)
II ntemperature, but II (E)
•' : : : : : nd nonab orbenttype of •
matenal for moldrngs on the ""'_IIlCl I pllntrnllS
•
i
:=:::bv:Rena, Ince parnters oar ba e layer onto finellralned layer
OutlYing airfields would be as convenient as centrally located airports for most pilots of private planes.
35.
Two decades after the Emerald RIVer Dam was buit, none of the eight fish species natIve to the Emerald River was stili reproduCing adequately ,n the rIVer below the dam. Since the dam reduced the annua range of water temperature In the nver below the dam from 50 degrees to 6 degrees, SCientists have hypothesized that sharply riSing water temperatures must be Involved In signaling the native species a begin the reproductive cycle. Which of the following statements, If true. would most strengthen the scientists' hypothesis?
(AI
Most oullying airfields are not eqUipped to handle commerclal·alrline traffic. Most private planes that use centrally located airports are not equipped With radar.
(8)
Commercial airliners are at greater risk of becoming Involved in midair colliSions than are private planes. A reduction in the risk of midair collision would eventually lead to Increases In commercial airline traffic .
(e)
(0)
(El
The native fish species were still able to reproduce only In Side streams of the rrver below the dam where the annual temperature range remains approximately 50 degrees.
36
aa::.~:.::::=
58 would praeillal
any messages sent by ll1le . ' ._ referred aIJulIe? W
asthma attacks.
Before the dam was built, the Emerald River annually overflowed ItS banks, creating backwaters that were critical breeding areas for the native species of fish.
18
The lowest recorded temperature of the Emerald River before the dam was built was 34 degrees. whereas the lowest recorded temperature of the river after the dam was built has been 43 degrees.
(C)
Nonnative species of fish, introduced rnto the Emerald River after the dam was bUilt, have begun competing with the decllnrng native fish species for food and space. Five of the fish species nabve to the Emerald River are not native to any other river ,n North America.
do;::::::= do 1I1l!S1". arnedir::lIIioA:::=
Resean:hers produces the n Rese.dlers
IIlIl,,, • •_ ..
person s
III. . . . . . . . .
activatlld U. anaIlIII'!t.
tor Suchsewraj ....,.,..,
both dlMIlIPIIIIIllt
ID
tEl
;..:::.w:==
..
38
~,eenlwidely used by
lllrJl9fiIils. eIfiUlation 10 eof fruit on which lMecled That regulation IS - I I I hu ....e1Itat allows sale of frUit on re5fclue are del1!cted. In IMIIlIIlQl8wi1l not allow more TDX on . _ " e 19605. because _ _.
..wmx
1If::::
nlques for del1!cting TDX residue only when it was present on frUit '!he nee amounts allowed by the
II
lII\Y''!IIllII~ple today than In the
19605
--::::hlse Ind eat fruit without making ill een ..sidues off the fruit
~
E5::~o'l1~ot ,ndlvldually consume any fnJlt on everage, than did the
Which of the followmg best completes the passage below? The more warned Investors are about lOSing their money, the more they will demand a high potential return on their Investment; great risks must be offsel by the chance of great rewards. This prinCiple is lhe fundamental one In determining interest rates, and It is illustrated by the fact that _ _ (A)
(8)
(C)
(D)
5
.Whellon of the fruit sold each
:
==:rIy~1960s
has had on It greater lIellllation allows ~M1~ on fruit 10 greater than been shown to cause any • 1d1lll1who ell large amounts of
."":ftOt
(E)
successful Investors are distingUished by an ability to make very risky Investments wlthoul worrYing about their money lenders receive higher interest rates on unsecured loans than on loans backed by collateral in times of high Inflation, the interest paid to depositors by banks can actually be below the rate of Inflation at anyone time, a commercial bank Will have a Single rate of interest that it will expect all of Its IndiVidual borrowers to pay the potential return on Investment In a new company is tYPically lower than the potential return on Investment In a well-established company
39
Acerlaln mayor has proposed a fee of five dollar per day on private vehicles entering the City, claiming that the fee Will alleviate the city'S traffic congeslion The mayor reasons that, Since the fee Will exceed the cost of round-trip bus fare from many nearby pOints, many people Will SWitch from uSing their cars to uSing the bus.
4
Which of the follOWing statements, If true, proVides the best eVidence that the mayor's reasoning IS flawed? (A)
(B)
Projected increases in the price of gasoline will Increase the cost of taking a private vehicle Into the city. The cost of parking fees already makes It conSiderably more expensive for most people to take a private vehicle Into the city than to take a bus.
The answer to wh be most u eful In (A)
Has anybody el disclosure require election reported Bergeron?
(B)
Is submittilll fulfill the ele tiD disclosure reqw O,d the mfo lournalist comml55
(C)
Most of the people currently riding the bus do not own private vehicles.
(D)
Many commuters oppOSing the mayor's plan have Indicated that they would rather endure traffic congestion than pay a five·dollar·per-day fee.
(C)
(E)
During the average workday, private vehicles owned and operated by people living Within the city account for 20 percent of the city's traffic congestion.
(D)
Have Bergeron value m recent
(E)
HadBergIJ eli
an
_ _ _lIMl'WI 2nd Hilton
-:::'~ICii=1Ity diSCO'lered that tooth·
strike against Coalco Le aders of a miners' umon on I t are contemplating additlona measures a preSSure the company to accept the umon's contract proposal. The umon leaders are consldenng as their pnncipal new tactic a consumer boycott against Gasca gas stations, which are owned by Energy Incorporated, the same corporation that owns Coalco.
42.
•
contaminated with bactena that streP throat They found that UlUlIIy occurs after toothbrushes have RIlf4l1r four weeks For that reason, people their toothbrushes at least once a
.._an
_lIP-
__'VI the following, it true, would most weaken the _llIIIion
.bove7
The dental researchers could not discover why toothbrush contamlnabon usually occurred only 8fIIr toothbrushes had been used for four WNks "'. d.ntlIl researchers filled to investigate contImlnation of toothbrushes by viruses, )'lilts, and other pathogenic microorganisms. lba dental researchers found that among people who used toothbrushes contaminated with bIcIIrII thlt cause pneumonia and strep throat, "'~lIIlnc1dence of these diseases was no higher IlIen among people who used uncontaminated
:fDothbruahe
dental r.searchers found that people who d1.lhe r toothbrushes thoroughly In hot water eech use were as likely to have CIDIIIflmIinltId toothbrushes as were people who lhllr toothbrushes humedly In cold IfIIr eech use re earchers found that, alter SIX ~"'DI UN ".ater length of use of a ~lDIu. dkI not correlate With ahigher .j-lIllCtliril beln. present
The answer to which of the follOWing questions IS LEAST directly relevant to the union leaders' consideration of whether attempting a boycott of Gasca will lead to acceptance of therr contract proposal? (A)
Would revenue losses by Gasca seriously affect Energy Incorporated?
(8)
Can current Gasca customers easily obtain gasoline elsewhere?
(C) (D) (E)
43.
Laws reqUlnng the use of headlights dunng daylight hours can prevent automobile colliSions. However since daylight Visibility IS Worse In countnes farth~r from the equator, any such laws would ObViOusly be more effective In preventing colliSions In those countnes. In fact, the only countnes that actually have such laws are farther from the equator than IS the continental Umted States. Which of the follOWing conclusions could be most properly drawn from the Information given above? (A)
(8)
Have other miners' unions won contracts similar to the one proposed by this umon? Have other unions that have employed a similar tactic achieved their goals with it?
Drivers In the continental Umted States who used their headlights dunng the day would be Just as likely to become involved In a collision as would dnvers who did not use therr headlights. In many countnes that are farther from the equator than is the continental Umted States, poor daylight visibility is the single most Important factor in automobile collisions.
(C)
The proportion of automobile collisions that occur in the daytime is greater In the continental United States than in the countries that have daytime headlight laws.
(D)
Fewer automobile colliSions probably occur each year In countnes that have daytime headlight laws than occur Within the continental Umted States.
(E)
Daytime headlight laws would probably do less to prevent automobile colliSions In the continental United States than they do In the countnes that have the laws.
Do other corporations that own coal companies also own gas stations?
44
_1lSi::~i1~:~:::::
Bank protected ill Insure aIll1Clllllllli_ argues that th IISlIl'II1lce iI",,,"~ hgh rate of ban ffailures, it'• •_ depOsrtors any fina the bank that holds fa lure If depO would need to be s depOsrtors mo ey
.:e
The economist s argument malll1S . ." " follOWing assumptions (A) (B) (C) (D)
(E)
Bank fl/lures are BUSied_.. I"I. ., default on loan rep'ayn1ell1ls, A Significant propO,.Ik~'I~~:::~.~ accounts at severa d The more a depOsitor careful he or she tend The difference In the inllInsrr• •I,. depOsrtors by diffeI1!Ilt IIInIIIs 1I1_ttl1!' factor In bank R!S. Potentia deposito;sare,.ltll_ _ banks are secure aaam: taI_
I
fnH:lures that are stable, and
~~~~~~
Ireconcerned grven follow-up Ire aboutHays the stability of the fracture. rolkJw.up x-rays were reVIewed, .~_frIctum that had InrtJaily been fudged "'''flllllCllo 11M healed correctly. Therefore, . . . . G1 money 10 order follow-up x·rays of ~1lIIII1I1nIIIaIy judged stable.
A study of mantal relationships In which one partner's 46. sleeping and waking cycles differ from those of the other partner reveals that such couples share fewer activitieS With each other and have more Violent arguments than do couples In a relationship In which both partners follow the same sleeping and waking patterns. Thus, mismatched sleeping and waking cycles can senously jeopardize a marnage. Which of the following, If true, most seriously weakens the argument above?
_'''u.. toIIowing if true, most strengthens the
(A)
_!ltDriwho are genera' practItioners rather ~1lIlIl."roped'sts are less lIkely than
~:::~:to JUdge the stabJilly of an ankle
..
-,,1IIlII
Ilft~:::
,
fly
njurles for whIch an ImllIl x·ray is revealed by the x'ray not to Involve of theankle
(B)
-'."'X-rlY
The individuals who have sleeping and waking cycles that differ significantly from those of therr spouses tend to argue little with colleagues at work.
(0)
People In unhappy marriages have been found to express hostility by adopting a different sleeping and waking cycle from that of their spouses.
(E)
According to a recent stUdy, most people's sleeping and waking cycles can be controlled and modified easily.
~ ~;:::;~ routinely order follow·up x·rays for
.. :;:.
ne other than ankle bones.
The sleeping and waking cycles of indiViduals tend to vary from season to season.
(e)
lUVl~vlllltien" of many dillerent orthopedists r,' hosPitals were reviewed. ~'~mllJot ankle fractures that have been Ired I always checked by means
Mamed couples In which both spouses fallow the same sleeping and waking patterns also occasionally have arguments that can jeopardize the couple's mamage.
c
Crops can be traded on the futures market before th~ are harvested. If apoor corn harvest IS predicted, pnces of corn futures rise; If a bountiful corn harvest IS predlCled, pnces of corn futures fall. This morning meteorologists are predicting much-needed rain for the corn·growlng region starting tomorrow. Therefore, since adequate moisture IS essential for the current crop's survival, prices of corn futures Will fall sharply today.
50
tpellc'lintless to Ira,n new
_MIHIllt.good enough
-.t.....1lIIIY people who already ._MbtIlDll computer.
I
Which of the follOWing, If true, most weakens the argument above?
=:~lIIIIII&er's true mostobjection senously to the
(A)
(lOIllJlUlers with Vllechs?
(B)
(C)
(0)
I of replacement parts make
lir!llUtln more expensIVe to maintain :IIIIllft C11lmputer
I
I
I
(E)
Corn that does not receive adequate mOiSture dunng ItS cntlcal pollinalion stage Will not produce a bounllful harvest. futures prices for corn have been fluctuating more dramatically this season than last season. The rain that meteorologists predicted for tomorrow is expected to extend well beyond the corn·growing region. Agriculture experts announced today that a disease that has devastated some of the corn crop will spread widely before the end of the growing season. Most people who trade In corn futures rarely take physical possession of the corn they trade.
51.
Acompany plans to develop a prototype weeding machine that uses culling blades WIth oPtical senSOr and microprocessors that dlslingUish weeds from crop plants by differences In shade of color. The 'nventor of the machine claims that It Will reduce labor costs by VIrtually eliminating the need for manual weeding.
The
Which of the follOWing IS a conslderalion In favor of the company's Implementing Its plan to develop the prototype? (A)
There IS a considerable degree of vanallon In shade of color between weeds of different species.
B
(B)
The shade of color of some plants tends to change appreciably over the course of their growing season.
IC
(C)
When crops are weeded manually, overall size and leaf shape are taken Into account In distinguishing crop plants from weeds.
(0)
Selection and genetic manipulation allow plants of Virtually any species to be economically bred to have a distlnclive shade of color WithOUt altenng their other charactenstlcs.
(E)
farm laborers who are responsible for the manual weeding of crops carry out other agncultural duties at times In the growing season when extensive weeding IS not necessary.
(DJ
(E)
53. Useful prate n drugs .....,..'" ~~;:== admln stered by the cunllbet'SOllllflle Injettion under the skin
c::::
they are dgested and Certa n nonprotein drugs bonds that are t broken ........ system They c -""
._.~er,
uses
large quanlilJes _ihlJIIIIllrW I1lImal hides New ~ hive Slgnificanlly Increased "NlIIlIIinI 01 salt water that resulls from consequence, Tanco IS consldenng a 1&11. ~1CIII_m chloride In place of common ....un~h8 shown that Tanco could reprocess 1IIe~.,.,._d 01 pota urn chlonde use 10 YIeld a '-~-'-lelving arelabvely small volume of cbposal
There IS a great deal of geographical variation In the 55, frequency of many surgical procedures-up to tenfold varia IIon per hu ndred thousand people betwee.n different areas mthe numbers of hysterectomies, prostatectomies, and tonsillectomies. To support a conclusion that much of the variation IS due to unnecessary surgical procedures, It would be most Important to establish which of the followmg? (A)
exammes the records of every operation to determine whether the surgical procedure was necessary.
fih ".... tile mpact on company profits of usmg pallllilmchloride In place of common salt, It would :=~1IIl for Taneo to research all of the follOWIng (B)
WhIt dlrterenee, /f any, IS there between the 01 tile common salt needed to preserve a n QUintIty 01 ammal hides and the cost of pota urn chlOride needed to preserve the quantity of hides'
(C)
wh8I extent I the eqUipment involved in IlIItenllnJ ,mmal hides uSing common salt (0)
IIItIble lor pre ervmi ammal hides usmg IIOIIlUiI/lII Clhlorlde' ronmenlal regulalJons, If any, constrain _10111 01 tile te generaled 10 "PrDCI.lllng lIIe by produCI of potassium
w.
HoI. c41l111J, dollle.ther th.t results when u lei to pr erve hides resemble ,... . . . .III wIwI pota lum chlOride IS
0
'
A local board of review at each hospital
(E)
The variation is unrelated to factors lather than the surgical procedures themselves) that influence the incidence of diseases for which surgery might be considered. There are several categories of surgical procedure (other than hysterectomies, prostatectomies, and tonsillectomies) that are often performed unnecessarily. For certain surgical procedures, It IS difficult to determine after the operation whether the procedures were necessary or whether alternative treatment would have succeeded. With respect to how often they are performed unnecessarily, hysterectomies, prostatectomies, and tonSillectomies are representative of surgical procedures in general.
Gartland has long been narrowly self·suffic ent In both 56. H grain and meat. owever, as per caPita mcome III Gartland has risen toward the world average Per capita consumplion of meat has also nsen toward the world average, and It takes several pounds of gra n a produce one pound of meat. Therefore, snce per capita income conlinues to rise, whereas domeSlJc gram production will not Increase, Gortland WI I soon have to Import either gram or meat or both. Which of the followmg is an assumplion on which the argument depends?
IAl (B)
The total acreage devoted to grain production 10 Gartland will not decrease substantially. The population of Gartland has remained relatively constant dUring the country's years of growmg prosperity.
57
'''''''''':::~:::
The re5POflSe nt by complex COIl1IllJter II'OIIrlIIII CIrcumstance
::1l:::~::~5~
be drawn the a ef g Iy a orna ed
ear'mi!iSIIe
cOl1troDed by a complex ClllllClull!r ...._
ith ace sys em would react 'natilll"lllrilw, acc den a start a nudlear
IB
IC)
The per capita consumption of meat In Gartland is roughly the same across all Income levels.
(0)
In Gartland, neither meat nor grain IS subject to government price controls.
(C
(E)
People in Gortland who increase their consumption of meat will not radically decrease their consumption of gram.
o
The system wou be de5itra:wed of a arge meteontl! occ:um!d atmosphere wou d be ompo d s ngwsh the ex:pIol5Kll1 from the explosion a nud_ , _ _ ether the SYS:tenll would napprlJl)riately
meteontl!
s E
not ertaIII ex deSIgnerS
I.-
bleCO Industry IS
stili profitable and prOleCtions t it WIll remain so. In the Umted States this ~ ::: total amount of tobacco sold by tObacco farmers has Increased, even though the number of adults \liIio smol<e has decreased.
TIIe:
60. . .rise ImlIDrt.. or IIllIIl! of
Each of the follOWing, If true, could explain the simultaneous Increase In tobacco sales and decrease In the number of adults who smoke EXCEPT:
-~ ..... CJiJ.tuPpIy _,.11I of Ihe toIkIwine poIlCH!S ,n an most IiIre/y to reduce the long~: 0II1het IIIIion of sharp and it .lItiilational 01' pnces'
(A)
(B)
(C)
(D)
iIIn_OVIl' • much larger area than do r IZll The reason for this behavior 1/1II11 m.mmals on which owls .rce in the forests where boreal relltlve erclty of prey reqUires the Illten IYtIy to find suffiCient food.
::=:~ "tru .most helps to confirm . . . . .IIMII rang over an area elghl thin lilt er aover which any other
rop • • • ~.. CIVIr larger area In region •ten by mall mammals region whir uch
(E)
61.
Dunng this year, the number of WOmen who h~ begun to smoke IS greater than the number of men who have quit smoking. The number of teenage children who have begun to smoke this year is greater than the number of adults who have qUit smoking dUring the same penod. Dunng this year, the number of nonsmokers who have begun to use chewing tobacco or snuff IS greater than the number of people who have qurt smoking.
62.
In response to vlrallnfeClJon, the Immune 5YSlI!IiiS mice tYPically produce antibodies that destr~ tile VIIUS by binding to proteins on its surface Mice Infected With a herpesVIrus generally develop keratitis a degeneralive disease affectmg part 01 the eye s..ce proteins on the surface 01 cells In thiS part of the eye closely resemble those on the herpesVIrus surface SCIentists hypotheSize that these cases of keratitis are caused by antibodies to herpesVIrus. Which of the follOWing, II true. gives the greates additional support to the SCientists' hypothes s' (A)
Other types of VIrUS have surface prote ns that closely resemble proteins found mvanous organs of mice.
(B)
There are mice that are unable to foom antibodies In response to herpes Infecnons, and these mice contract herpes at roughly the same rate as other mice.
(C)
The people who have continued to smoke consume more tobacco per person than they did In the past.
Mice that are Infected With a herpeSVirus but do not develop keratitis produce as many antibodies as infected mice that do develop keratitis.
More of the cigarettes made In the United States this year were exported to other countnes than was the case last year.
(0)
There are mice that are unable to form antibodies in response to herpes Inlecltons. and these mice survive these infections WithOut ever developing keratitiS.
Amilepost on the towpath read "21" on the Side facmg the hiker as she approached It and "23" on ItS back. She reasoned that the next milepost forward on the path would indicate that she was halfway between one end of the path and the other. However, the milepost one mile further on read "20" faCing her and "24" behind.
(E)
Mice that have never been Infected With a herpeSVirus can sometimes develop keranbs
Which of the follOWing, If true, would explain the discrepancy described above?
(AI (B)
(e)
(0)
()
The numbers on the next milepost had been reversed. The numbers on the mileposts indicate kilometers, not miles. The faCing numbers Indicate miles to the end of th p th, not miles tram the begll1mng. Amilepost wa ml Ing between the two the hlk r ncount red. Th mllepo t h d Originally been put In pi for th u of mount In blk r ,not for hlk r
The
B
'Ii
C
re,iISOlWtg.
-=:,
o
Top miillll_1!I!r.I the majority otf tflIiri' I
IE
1i
pesticides needed when _ yrelds of the same crop water supplIes Experts to dIVersify their crops and to yearly I pnce support benefits for a must have produced that same crop for years
.bow!, if true, best support which of conclusions?
IWe5 for governmental support of farm
--
work .garnst efforts to reduce water
only solutIon to the problem of water pollution from ferttlrzers and peslicides IS to take flnnlend out of production. r.rm.r can continue to make aprofit by fllIItInI dIVerse crops, thus redUCing costs for chenuc.'s, but not by planting the same crop
lIChyear
65. Which of the following most logically completes the argument? Utranra was formerly a major petroleum exporter, but In recent decades economic stagnation and restncbve regulations inhibited Investment In new 011 fields. In consequence, Utranian 011 exports dropped steadily as old fields became depleted. Utranla's currently Improving economic situation, together With less· restnctlve regulations, will undoubtedly result In the rapid development of new fields. However, It would be premature to conclude that the rapid development of new fields will result In higher oil exports, because
(A)
the pnce of oil is expected to remain relatively stable over the next several years
(B)
the Improvement in the economic Situation In Utranra IS expected to result in a dramatic increase in the proportion of UtraOians who own automobiles
(C)
most of the investment in new oil fields In Utrania is expected to come from foreign sources
tion of fertrlizers and pestiCides.
(D)
new technology IS available to recover all from old oil fields formerly regarded as depleted
mmentel pnce Supports for farm products II levels that are not high enough to fermer to iet out of debt.
(E)
many of the new oil fields In Utrania are likely to be as productive as those that were developed dunng the period when Utrania was a major oil exporter
Hardin argued that grazing land held In common Ithat IS, open to any user) would always be used less carefully than pnvate grazing land. Each rancher would be tempted to overuse common land because the benefits would accrue to the Individual, while the costs of reduced land quality that results from overuse would be spread among all users. But a stUdy companng 217 million acres of common grazing land With 433 million acres of pnvate grazing land showed that the common land was in better conditIOn. The answer to which of the follOWing questIOns would be most useful In evaluating the significance, in relabon to Hardin's claim, of the study descnbed above7
--
New f.mulli technrques Will be developed to ke it possible for farmers to reduce the
66.
(A)
(B)
Did the ranchers whose land was studied tend to prefer using common land over uSing pnvate land for grazing?
(C)
Was the private land that was studied of comparable quality to the common land before either was used for grazing?
(D)
Were the users of the common land that was studied at least as prosperous as the users of the pnvate land?
(E)
67.
Did any of the ranchers whose land was studied use both common and pnvate land?
velocity and size makes objects appear to be moving more slowly the larger the objects are. Therefore, a motorist's estimate of the time available for crossing a highway With a small car approaching IS bound to be lower than It would be With a large truck approaching The conclusion above would be more properly drawn If It were made clear that the (A)
truck's speed IS assumed to be lower than the
(B)
car~ ~ truck's speed IS assumed to be the same as
(C)
car's thin tilt truck's speed IS assumed to be higher
(E)
often product
Newrtheless. more profrt by not holdinlli1l1l1rn. WhICh of the
car's in_motOriSt's estimate of time .va.. ilalflble ..... nllclil to be more accurate With cars With trucks apprOlchlna motorist's estimate of 111I1 _ _ to be more ur th n WIth
1IlIII.-
11*II'"
the claim above abo IA)
(8)
IC)
The amount of discounl ~:=:: manufacturer n to represerrt the mlllllllllllll _ _tt.,. . . consumers attention
prO~::::;:::~f.~
For manyngconsume adverns discounted about a week not suffic consumers to become For products that are
such
purpose of PIr:7::::':::~' products In the mmds attract consumers competmg produc ID)
Dunng such aprQInobon Illtailers til.. accumulate theilrllllllRmase, iI_ _ boughtatdis later at!helr egular
lEI
If amanufacturer but ts compe Win tend to attrac:t cc_ _rs _ . ' . manufacturer's prout
Were there any owners of herds who used only common land, and no pnvate land, for grazlng7
A compelling optical illUSion called the Illusion of
(D)
68
_
.IIaeJ.llYflO!dedarfllg
! -::::.:tum
70
Plantings of cotton b/oenglneered to produce Its oWn insectIcide against bollworms, a major cause of crop fatlure, sustained little bollworm damage until thiS year ThiS year the plantrngs are being serrously damaged by bollworms. Bollworms, however, are not necessarily developing resistance to the cotton's Insect/clde. Bollworms breed on corn, and last year more corn than usual was planted throughout cotton. growing regions. So It /s likely that the cotton IS Simply being overwhelmed by corn-bred bollworms.
TaxMSlOll
!==::fIx:rates which ....JBding taxpayers to encourages even more fixes by hidIng taxable
III
-:::flar:nbed •
above could not result
.... fDIIawjng were true?
. . . . .in fix rates tends to function as an =::~tIxpIye:. rs to try to Increase their
In evaluatrng the argument, which of the followrng would /t be most useful to establish)
"'PllillIllfDIIs for detechng tax evaders, and ~_llIV8ring some tax revenue lost through
(A)
Whether corn could be bloengineered to produce the InsectiCide
• III mmore than they cost, but their ftle vanes from year to year.
IB)
Whether plantings of cotton that does not produce the Insecticide are suffering unusually extensive damage from bollworms this year
_llIrIm.leers establish Income tax rates In "nl4ll!lnerate acertain level of revenue, _lIlow adequately for revenue that will _-llIUIh evasion
(C)
IiIIIRIfVIlt''I'outinely hides some taxable IIIIntclln be Induced by a lowering of tax
~:::P~h ding such Income unless fines for
I
ID)
Whether plantings of bloengineered cotton are frequently damaged by Insect pests other than bollworms
(E)
Whether there are rnsecticides that can be used against bollworms that have developed reSistance to the insecticide produced by the bloenglneered cotton
lied at the same tIme.
"8'irollot differ from each other with lIII:.'1II11'118 of taxation that will cause ~...1Ius
Whether other crops that have been bioeng/neered to produce their own insecticide successfully reSist the pests against which the insecticide was to protect them
71.
Because postage rates are rrslng, Home De orator magazine plans to maXimize ItS profits by redu mg by one-half the number of ISsues It PUblishes each year The quality of articles, the number of articles published per year, and the subSCriPtion perce 1'1111 not change. Market research shows that neither subscrrbers nor advertisers 1'1111 be lost If the magazine's plan IS Instituted. Which of the follOWing, If true, proVides the strongest eVidence that the magazine's profits are likely to decline If the plan IS instituted) IA)
(B)
With the new postage rates, a typical Issue under the proposed plan would cost about onethird more to mall than a typical current Issue would. The maJorrty of the magazine's subscerbers are less concerned about a possible reductron In the quantity of the magazine's articles than about a pOSSible loss of the current high quality of ItS articles.
(C)
Many of the magazine's long-time subSCrIbers would continue their subscriptions even If the subscription prrce were increased.
(D)
Most of the advertisers that purchase advertlsrng space In the magazine will continue to spend the same amount on advertising per issue as they have In the past.
IE)
Production costs for the magazine are expected to remarn stable.
7
at mand In re_.ltl.. nllllil~~:::: the re""""'" iI
111_
Which of the f l,uwII"•• the apparent PIJ' IA)
Over half !hie retaillr', of payroll elxPElncfjlures; percentag ........ management salin-.
IB)
The retll er' CIJIIlImIlr ........ prlmanly of pea the earning of othIlfl wlIo. . . . . wage
Ie)
The retailer' pe a Increased suDsta mlnrmum wage r When the In rnse
(01
went Into elf ~:::= wage rate for emp lUst above mlinimunn. . IE)
The majority ...~::= as cashiers and m mtrlllnum WIglt.
IM_IIII.SO pmcIucliVl! MIlIlIrb t1111t1t lIIeY
A roduct that represents a clear technologiCal
74. advance p over .competing products can generally .
technological comman d a high price. Because . advances tend to be qUickly surpassed and anres want to make large profits While they s " com P . h th " many companies c arge e maXimum can, h d B possible price for suc . a pro uct. ut large profit! on the new product will give compet/tors a strong IOcent,ve to Quickly match the new product's capabilities. Consequently, the strategy to maXimize overall profit from a new product IS to charge less th" the greatest possible price.
~"CIlftlln prICes fell 11Ie MI. cotton prICes by offenng . . . .1lIa!1IIt of IItfllr cotton acreage
IIMIJINtt IIIPPOrt payments up to a
IJIpalUll per farm Nl;)t1lft'5 program, if successful, will not be a lluclget WInch of the followIOg, If
hlitbnls for an explanation of how this otton pnces meant operating cotton farms. and the government Iff.e from taxes on farm profits.
In the argument above, the two portions In boldface play which of the following roles?
fl/llducllon In severa' countnes other decfined slightly the year that the liIIIlllIt~~ent program went into effect in Q.
(A)
!la111r1t yur thlt the support·payment program
(B)
...'" Iff8ct, cotton acreage 10 Qwas 5 percent "-u IN level/n the base year for the program.
!ifI"".
lIIIIClII.d maXImum per farm meant that for otton farms the support payments ......- I. per acre 10r those acres that were from production than they were for
farm
(C)
The first is a consideration raised to argue that a certarn strategy IS counterproductive; the second presents that strategy. The first is a consideration raised to supportthe strategy that the argument recommends; the second presents that strategy.
The first is an assumption, rejected by the argument, that has been used to justify acourse of action; the second presents that course of action.
(E)
The first /s a conSideration that has been used to justify adopting a certain strategy; the second presents the intended outcome of that strategy.
_ _Ib auld not use the colton acreage that • WIIIlcIrawn from production to grow any
primarily on revenues from paYing patients to offset losses from unrelmbursed care. Almost all paYing patients now rely on governmental or Private health insurance to pay hospital bills. Recently, InSurers h been strictly limiting what they pay hospitals for th:ve care of Insured patients to amounts at or below actual costs. Which of the follOWing conclUSions is best supported by the Information above? (A)
(B)
(C)
The first is a consideration raised to help explain the popularity of a certain strategy; the second presents that strategy.
(0)
who Wished to Qualify for SUPPort
75. United States hospitals have traditionally refied
(0)
(E)
Although the advance of technology has made expensive medical procedures available to the wealthy, such procedures are out of the reach of lOW-Income patients.
If hospitals do not find ways of raiSIOg additional income for unreimbursed care, they must either deny some of that care or suffer losses if they give It. Some patients have incomes too high for eligibility for governmental health insurance but are unable to afford private insurance for hospital care. If the hospitals reduce their costs in proViding care, Insurance companies will maintain the current level of reimbursement, thereby prOViding more funds for unrelmbursed care. Even though philanthropIC donations have traditionally provided some support for the hospitals, such donations are at present declining.
76 CO'I lUeS
when any
1I!ll1flll1-::::=
SClel1Ce to cone ude that III regarded as a true col
..
The exlllanation offenJd ' .. WhICh SCientific POPularizers f t l . _ SCientists assume that
(A)
senous scientific res_MI." activity butre a group of colleaaues
IB)
research scH!nlists teIId colleagues those scilllli envy
(e)
(0)
IE)
".
.
a sCIentist can bee II~:=== without havlllg OmplJlU1lll1O IIIJ research research sc enlists bel. .' well known as polpu/alrizers all motivated to do importallllll
.
..
no Important new ':::~::::= or accurately asses themselves scientis
)
WIIiCh 01 the lollOWlng most logically completes the
78
passage?
1marlier
r
The figures In portraits by the Spanish painter EI Greco 11541-1614) are systematically elongated In EI Greco's time, the Intentional dlstorlion of h~ figures was unprecedented In European painting. Consequently, some critiCS have suggested that EI Greco had an astigmatism, a type 01 visual Impairment, that resulted In people appearing to the distorted way that IS characterlslic of his pain However, this suggestion cannot be the explana because __ .
dropped (AI
Candy manufacturer: Our ma results In there being ess ca e e candy bar than III the unprocessed whICh the chocolate IS made.
tA)
(B)
some people do have elongated bodies somewhat like those depICted In EI Greco's portraits
(B)
ICI
If EI Greco had an asligmatlsm, then, relahve how people looked to him, the elongated figur In hi paintings would have appeared to him be dl lorted
(C)
does rot specify eat h the manufactunng process
IOj
ven II EI Greco had an astigmatism, there hav been no correction tor It available In the period In whICh he Iiv d
treats the consumer hea th adllIOC'!lI as though It ere abou eac than about the manufacture
(E)
mer I contrad t the c te' con ru WlUlOUI d to e that the rea:!iOIlII" un ound
IDI
I) th r w r non Europ an rli IS, even In I r 0' lim ,who In Iud d In their wcr hum n fi ur th t w r Int nlionally diStorted
ess
The candy manufacturer's respo se s refutation of the consumer hea a because It
several twentieth-century artists have consCiously adopted from EI Greco's paintings the systematIC elongation of the human form
Ifly Iron and
n I rklloday, I btu d
79
falls to address the Issue of e er e 0 caffeine III the cand bars so d b the manufacturerls enough to eep peep e a ed assumes Ithout warrant tha a unpr e sed cacao beans contain a urnf rm ilfT1(lUllI caffeine
. . . fur GMA'P verbal Review 2nd Edition
cal Advertisement: that Mayor Delmont's critics camp J. aln abau t the jobs h Yet were lost in the city under Delmont's leaders '~'th the fact is that not only were more jobs create an were eliminated, but each year since Delmont to:ak office the average pay for the new jobs created s been higher than that year's average pay for jobs citywide. So it stands to reason that throughout Delmont's tenure the average paycheck in thiS city has been getting steadily bigger. Which of the follOWing, if true, most seriously weakens the argument in the advertisement? (A)
(B) (e)
(0) (E)
The unemployment rate in the city is higher today than it was when Mayor Delmont took office. The average pay for jobs in the city was at a ten-year low when Mayor Delmont took office. Each year during Mayor Delmont's tenure, the average pay for jobs that were eliminated has been higher than the average pay for jobs citywide. Most of the jobs eliminated during Mayor Delmont's tenure were in declining industries. The average pay for jobs in the city is currently lower than It IS for Jobs in the suburbs surrounding the city.
. corp, a manufacturer, 2 Vita . wishes to make Its 8.. tl·on booth at an Industry convention '" Informa . '''are productive In terms of boosting sales. The booth offers Information introducing the company'S new products and services. To achieve the desired reSiJlt. Vita corp's marketing department Will attempt to attract more people to the booth. The markelrng director's first measure was to Instruct each salesperson to call his or her five best customers personally Invite them to Visit the booth. a Which of the following, If true, most strongly suPPorts the prediction that the marketing director's first measure will contribute to meeting the goal of boosting sales? (A)
(B)
(C)
Vitacorp's salespeople routinely inform each important customer about new products and services as soon as the decision to launch th!!1 has been made. Many of Vitacorp's competitors have madep~' for making their own information booths more productive in increasing sales. An information booth that is well attended te~ to attract visitors who would not otherwlseh?~ attended the booth.
(0)
Most of Vitacorp's best customers also have business dealings with Vitacorp's compelitors
(E)
Vitacorp has fewer new products and service available this year than it had in previous yeNS-
8. 3
An eyeglass manufacturer tned to summer quarter by offenng its dilltllllll discount If therr orders for that QIUIflIW those for last year's summer Quarter percent. Many distributors qualified for Even With much merchandise discount Increased enough to produce ahealttly profits. The manufacturer plans to repeat by offering the same sort of discount for quarter. Which of the follOWing, if true, most clearly pojntl flaw in the manufacturer's plan to repeat the successful performance of the summer quarte (A)
In general, a distributor's orders for the Sllll,,* quarter are no higher than those for the_. quarter.
(B)
Along with offering special discounts to qualifying distributors, the manufacturer increased newspaper and radiO advld_ir those distributors' sales areas.
(C)
The distributors most likely to qualify for manufacturer's special discount are whose orders were unusually low a
(0)
The distributors who qualified for manufacturer's special discount WIIrtfJ....~·, decide how much of that diSCount therr own customers. The distributors' ordenng summer quarter left them fall quarter.
. 5
(E)
lIIOI.
4.6 Answer Explanations 22. 23. 24.
A
25 26 27. 28. 29.
A
B B
C C
45. 46.
B B
47.
E
48.
B
0
A
The following discussion is intended to fam I ...._
65.
B
66.
C
67.
B
approac esto crfltlchal reasoning questions The PilrtlculMqluenIGNill1., representative 0 t e kinds of critical reasoning uestions Rem.ember that .it is the problem solving strateg: that '-_~1N. particular question.
68. 69.
D C
49.
E C C
70.
B
C
50.
0
71.
D
30.
0
51
0
72.
B
31. 32.
E C
52.
C
73.
A
53.
A
74.
C
33. 34.
A
54.
E
75.
B
C
55.
B
76.
D
77.
E
78.
C
B
o
4
E E C
43. 44.
A
64.
B D
35.
A
56.
36.
0
57.
E E
A
37.
A
58.
0
79.
C A
A
38.
B
59.
B
80.
A
39.
B
60.
A
81.
40.
E
61.
C
41.
82.
C
C C
62.
D
42
E
83.
E
63.
E
A
h
lIa.._you
..
IS 1=-1II......
I.
Which of the following, if true, most logiCally completes the ar&\Jll1ellt below? Manufacturers are now required to make all cigarette lighters chl1
(AI
the addition of the safety levers has made lighters more exlK!f\SlVe than they WIIleII..... '. . . . was Instituted
(B)
adults are more likely to leave child-resistant lighters than OOIH:tukke5l5tant liIt_s in . . . . . . accessible to children
(C)
many of the fires started by young children are qUIckly detected and extingulislllld tIWIIW • • unlike child-resistant lighters, lighters that are not chdd-reslSlant can be OIl\!faletl two years old
(D) (E)
approximately 5,000 fires per year have been attributed to children playiq wiItJ IiPlIrsi 1III1ln,. levers were required , "'_111'
Argument Construction Ianufacturers must equip all cigarette lighters with chiJd-'':~::::::: children can figure out how to circumVl:llt the safi:ty ~ aDd
Situation Reasoning
What pointwollld most logtil11{v compld~ tIN ~ _uIIll.~f'!!1I! the number of fires C1used by children playing WIth hgb,", ~ In order for children to tart tires using lighters equipped be given the opportunit)' to figure out ~~Slters.ImY a:~........ _,. them. Ther must. that I • have a to
are man:
expe=
If safety-lever-equipped lighters lighter' might be sold. Thi would m tht ~ . to tart fires WI u",D1. them Ie opportumty
B C
Correct. Thi statement properly identifies a ~ . likelye:ltiI'lgUisbllll'b•• to ha ... explains why children are ItI~~~, The peed with which fire are that are starred.
"'-'IIfIIl~~
D
This provide a rea on to beIi decrease. rather than ta the lewer fires are likely tID be IllUIla
E
Th is inlormation about does not ha nlyblCllUllt~~ no that th salery ...,.....
1h
than Iip~ .,.~!
t
1UlI1""'!".
..~*' tile
t rovide hlgh·speed ground transportation problem of airport congesbon IS fOI PmPlementalion of this plan would cost far 500 I apart The success u I lying 200 to ml e s · th mber of airplanes clogging both airports and existing airports and would also reduce e nu
Ull1loi1fng, if true, could proponents 0 f th e PIan above most appropriately cite as a piece of eVidence • • of tile" plan? Iffective high-speed ground·transportatlon system would require major repairs to many highways and . . . OIit-/lllllof all departmg flights In the nation's bUSiest airport head for a destination In a major city mllesrway
3.
People's teleVision-Viewing habits COUld be electromagnetic waves that are reflected back to tile II10Ililbrld determine how many persons are WIthin tile V1eWinR area a mOnltorrng system, but a proponent resllOllds"The af1"- . .1. ClIIIlM they watch, Viewers won't feel a thing: 8Wragedolltllfn• • • •
::-:~:::::
Which of the follOWing ISsues would It be most m rt I pO arrt to resol\le health effects of the proposed system'
IlIISI-IrInsil Improvements
(A)
Whether the Proposed method of monrtonng VleWeI'shIp can dis1li1a..IIll. . . . I...
(8)
Whether radar speed mOnitors also operate on the PfI/lCIPle of analyzillll nllllcllll•• electromagnetIC radlalion
(C) (D)
Whether the proposed system has been tned out to vanous areas of What uses are foreseen for the viewership data
systems
(E)
Whether the average dose that the proponent descnbes s a shorMe
AfIrp propOrtion of air travelers are vacationers who are taking long·distance flights.
Evaluation of a Plan
IlIlQOrity of travelers deparling from rural airports are flying to destinations In cities over 600 miles
lIWIY
lI""_lIInew Itrports are being bUill ,n areas that are presently served by hlgh·speed ground·transportat,on
"lUGln 011 Plln
Situation
Providing high-speed ground transportation bet\veen cities 200 to 500 miles apart is a mor cost-effective and efficient way to reduce airport congestion than expanding ex' ting airports,
Who/ roidmu Juppor/J the plan? The transportation plan will work only if people are !Jkely to .use the high-speed ground transportation. Ifhalf the flights leaving the bu Ie t a,r~ort Ry to destinations within range (200-500 miles) of ground transportatIOn, then many people might choo e high-speed ground transportation over aIr travel, Then, fewer Rights would be needed and airport congestion would
People's televi ion habits could be tracked usmg dlectronJag_ic ...... i plan are concerned that the electromagnetlc waves wo'uJd health, but proponent ay that viewers willli:d Mrtllo! ......,~.. radiation is Ie s than one che t x-ray,
Reasoning
decrea rep
I
are an ar~ment a!(ainst, not for, ground transportation.
• I I menl properly identifies evidence that suppOrts the plan, Im_," re not m luded in this proposal, which addresses travel bet ' .. h ., ween ma.Jor cItIes. rang I arne routes are a threat 10 the plan. th Ilh arpon are ongesled hel"lus ' of I d' . tin tlon wllhm SOD .( . '. c .nng- Istance travelers, rather than 10' s, oIrgues ag,lInst the propos'll.
A
B
D
This speaks to whether the data ramcd from thc:~:::::=::: . 0 b' · I .. habi not to wbcdacr to monitor peopIes te e\'1 100 health effects. . . .all . rel -- J to the dispute. lOut This 1.- tangeon ~ an:u IDOl_lIS radiation were in fact dangerous. radar ..... since no intl)rmation i i\ n aboutwbletb~IDOIIIII.1"'dlll"'llII r.ldar -peed monitors' Comparable;.;..!!D1_.....:..It.... tele\'iion -~ -tern. the in/OrmaIiOO It would he u -eful to investig-ated through UI out in \ ariou ho\ Iikd th The u to h.
_.111
prop h
"'IIIJndIlllllon
h d Irom mllrtary contractors IS determined by a • 1IlII_ JNIYS for standard weapons purc a~e s contractors to protect their profits by adding .....rd"fllstorica' costing· Hlstoncal c~sttmg at:;e previous year's contractual pnce. • _ . ' " based on the current rate 01 mIla lon,
Since the mayor's publiCity camPaIgn for er-
5.
.
bus~§§;§~a~~~~1
traffic Into nse the midtown area of city has eeiilttsId 7 ...... eqUivalent In the number of the Persons ~ campaign has convll1ced many PeoPle I buses a eaw cars at Which of the follOwing, If true, casts the most seoous doImt
fuIowmg statements, II true, IS th e best basIs lor a cntlcism 01 histoncal costing as an IIIIICIlIt IOlIId prlcmg method lor military contracts l auvemment might contmue to pay lor past Inelficlent use 01 lunds. rile at inflation has vaned considerably over the past twenty years. COubactual pnce Will be greatly attected by the cost 01 materials used for the products. MIny fIx~rs question the amount 01 money the government spends on military contracts. prierng method based on historical costing might not encourage the development of Innovative
(A)
Fares for all bus routes 111 Gree
(B)
The mayor of Greenville "des the bus......, a City rISen an average 5 IlII'tent ca.riIIr "'''1II11ll Road reconstrucbon has greatly reduced the mber lanes . to the midtown area dunng the oast numonths at.? leading • ,.
(e)
MI~lIII
(D)
The number of buses ente"ng the midtown area all bI'eenVlle Cbq same now as it was one year ago.
(E)
Surveys show that longtime bus tlders are no more satisflll'ied!ll "'11II were before the mayor's pubhcrty campaign began.
UIIIInt Evaluation
MJllfary contractors add a percentage increa e, based on the rate ofinflation, to prices peellied in previous contracts for standard weapons.
e h_._
~"" GreelMllr b. ,.....
Argument Evaluation
;~lI-lIfti,",
What couldDe wrong in limply adding apercentage increase each year? If the anginal COntra t price accommodated the contractor' inefficiencies, the government's overpayments for these inefficiencies arc simply perpetuated, and money continues to be wa ted. Additionally, historical costing assume cost only go up, never down.
Q1rrect.• Thl .oGIClUIC h
Situation
Reasoning
I lement propcrly identities a serious problem with historical co ting.
raeal co ling involve, makinK a pricc adjustment based on the most recem inflation t th I mllatlon rate, havc varicd is nor a weakness of the method.
ftIl1l11N1lIy rhe
0
I of maleriaJswas .'ilelored into the .previous year's contractual price, and the /Jowed by '11 l(lflt'alms'IOK will bkely reflect any increase' in the cost of
llOn th amOUnt ICOlIOIImic:aUy ound m Ihod,spcn, on milit"r)' l'Ontracts cvcn if the Contracts , ,'He priced ndrdwponol" . n y. III1JOV,lflve weapons arc not discusscd.
Traffic into midtown has d~ by JlCIctnt, and equivalent amount. The mayor' publicity C3IIIpaJgn What castr doubt on tbis ctmcllMimr1 Another RaliOllllblr: CIJ....... Ilf1ll7..,. the decrease in automobile traffic and the increase: conclusion su pect. Road construction impeding IIXCIIID l _ - .. . . . . . .
period of time i a rea onable altanatM aplao....
would likeh' hm~ di ounged people bum dm>iDg
ha\-e probably taken the bus.
A
An increa e in fare might be a reo inerea e.
B
The mayor' deci ion to ride the bus strengthen rather than we-alt n uppon
C
orrecr. Thi: .tatement properly iden conelu ion.
D
If more bu:e \\ re runn midtown. Rulin out tbi scenarJO P.l 'engees pc:n:ei\ b c,m:e of the incre:asl:d lridc:rsliup.
E
1h
nab]
1he:=~c::;:==
hIdren at patrick'S house In the morning iitolIl_rar~1iIC drilllr8ll Parents leave th~~t~ ~ay Patrick at an hourly rate for the child ...~_-' oteachworkWllek, the pa Ily deQuate but not always uniform, .. uno ..IV ves IS usua a d be unpredictably absent. 1lIII.<1IIII weeIdY Income Patnck recel hiId are likely to get sick an iinlllllllr,wheR c ren f making Patrick's weekly Income both h the best prospect 0 UI d'I p/lns, if put IOtO etteet, as IIIlI edeCIU8te? d Imllar arrangements, so that the two of #'-4,."".. des child care un er s Pool resources with a neighbor who provl as Patnck currently does. ld IIIem eooperate In canng for twice as many chi rend d th a fixed weekly fee based upon the number of
........ ce payment by actual hours of child ,care provi e WI to provi'de . nSJ'~ II b expected
181 lIours of child care that Patrick would typlca y e
, t k children 'I t f providing child care 0 SIC . (C) Hwe a ful~lime helper and Invest In facil les or even In a week when half of the (01 Increase the hourly rate to a level that would provide adequate Income children Patrick usually cares for d are absent. l bl for child care each day, so that parents can leave their lEI Increase the number of hours ma e aval a e I t children 10 Patnck's care for a longer period each day at the current hour y ra e.
7
A researcher discovered that peOPle who on tests of mental health than do lleOpte from thiS experiment that the Immune wiIIl P1l11Ec1s :::::
:::::~~
Sf'."
The researcher's conclUSion depend s on which of the followlna _ (A)
High Immune-system activity protect does. s against mentall\lness bettIr
(B)
Mental illness is similar to physIca I dIsease In Its effects on body syslllfl': People With , high immune-system actlVI"t" cannot develop mental Mental Illness does not cause pea pes I ' Immune-system activity to"""0:15, dec Psychological treatment of mental III ness IS not as effectNe as IS medical tr·.m.1l
(e)
(D) (E)
Argument Construction Situation
Finding that p ' how i 'Immune- v tern actIVIty eop l e Wit tests than people with normal or high I~els of immun,e-s",_ _ld~"'1II concludes that the immune sy tem protects mental as well
Reasoning
What assumption does the researcher make? The researcher asl4mJ activity .can inhibit mental illnes as it does phy ical illn un.:l_ mentall~ness might itself depres immune-sy tern aCtIVIty, the relll_ _ that thiS IS not the case. If mental illness caused a decline I then lower levels of immune- ystem activity would be expeelllCl higher levels would merely indicate the absence of illness rather illness.
Evaluation of a Plan Situation At the end of the workweek Patrick is paid a certain amount for each hour of child care he has prOVided. Patrick usually receives adequate weekly income under this arrangement, but in the winter Patrick's income fluctuates, because children are unpredictably absent due to illness. ....lOnlng
Which plan would be mos/like~v /0 meet/he two goals ofuniform weekly income and adequate wukly income? Patrick must find away to ensure that his weekly income is both adequate-that is, not reduced significantly from current levels-and uniform-that is, not subject to seasonal or other fluctuations. A successful plan would thus most likely be one that doe not increase Patrick's costs. Further, the plan need not increase Patrick's weekly income; it must merely ensure that that income is more reliable. It should therefore also provide some way to mitigate the unexpected loss of income from children's absences.
Thl P;~ might raise Patrick's income slightly, because he and the neighbor might payout less in ~ t e~tobl thm resources. But this plan would have no effect on the problem that uny.....lcta e a sences pose for Patrick's weekly income.
~ ~::~~mp:~ Pbrloperly.identifies a plan that would most likely keep Patrick's income the same amount 0 f money per child as he does a y receive and unifurm (he would _. approximately h WbiL! pJa h receive t e money regardless of whether a child was present or absent). n mIg t somewhat mitigate the un r d' bT . children' absences-because p t p e Icta I Ity 10 Patrick's income that results ld ......-it aren I to keep sick children at .. would mcrea e Patrick's costs Pa . s wouh IbesIeS l'k ley 111411dUcl trick' Incom and might thymg a Ie perhand investing in different facilities us resu t 10 t at in come b" th emg madequate me at parents did not balk t h . . •Patrick's incom would most~i~t mcrease in Patrick's .hourly rate and 1JlCOme unifurm Hi ' y be adequate, But thiS plan would not mcome would , s two goals wI'th r rd P . co nt'mue to fl uctuate when children Paitrk:k' Incom . h gah to atnck's ,mcome: adequacy and uniformity. be pal'd t:lOr more hours of child care , In t1._atP e might to " maau: atnck s weekl ' I f 'c y Income both adequate and o unbormity.
"'-"'If
.,uelre..
......
A B C D E
The researcher does not distinguish between high and normal levels ofimDIII_...,..... so this assumption is not needed. The researcher's inference is not related to the effects of mental illn Immune-system activity could protect mental health without offenng illness; this assumption is not needed. Correct. This statement properly identifies the re earcher' underlying _1UD...IIi. illness does not decrease immune s} tern activity. Since different treatment are not di cu ed. any a umption about
The correct answer is D.
h al and physiologicalIdvanables. I cal geograp IC , 15;~E§~""~~ldonof OII1111Sms I ofIneco ogJ w 'ays, and should, therefore, Yle a random different rs ,n a surpnslngly definite pallern. n ___HI shows that extlncbO occu
vanety
1
the tossil """" u
lIIIis/IiIIIll the Slme time.
faIoWilII if true. torms
d
I explanation of the palterne extinctIons
the best baSIS for at least a parlla
.-1lw1/ll1oSStl record?
mental disturbances that affect numerous
episodes of extinctiOn can result from widespread environ
diIfItIIII s p e c t t ! s ' t h articular sets of charactenstlcs unique to certJlII extinctiOn episodes selectively affect organisms WI p IIleir sptCH!S.
ltd
adual changes In their local environments.
•....,. es become exbnct because of accumu a e gr Some ....._1 . d human Intervention has changed the
(I))
In IIOlogICaJIy recent times. for which there IS no fossil recor ,
tEl
flIllem of extinctions. SpecItS that are WIdely dispersed are the least likely to become extinct.
In parts of South Amenca Vitamin Ad.... region. agncu It unsts are attempting 1m _ ....... '''-VI ' - toclIClency I a 0-'-........ potato called SPK004 that IS nch In b t prove nutritlOll by encour.... chances of success, Since sweet pot e a-carotene. whIch the body t currently grown contain lillie beta ,caro a atene IS a staple of the region' diet and "I 8,,!cUb.;
~:::::i=
Which of the follOWing, If true most st 1 . rang Ysupports the prediction that the plan (A) The growing conditions required b th conditions In which SPK004 flY e vanetles of sweet potato currently , ..IWa. can Ounsh (8) The flesh of SPK004 differs from th t f traditional foods would 10 k a a the currently cultivated sweet potatoes a somewhat different when prepared from SPKOO4 (C) There are no other vanetles of t swee potato that are Significantly IIcher In betacar (D) The vanetles of sweet potato currentl y cultIVated In the region cantlin some tl1\IO lacking In SPK004. (E)
"rgument Construction Sftllllltloin The fossil record reveals that species become extinct in a surprisingly definite pattern, with multiple species vanishing simultaneously. .,1IOII1n1l
9.
Which pointprovidts a basisfor txplaining thepal/ern? The passage states that the process ofextinction depends on so many variables-in the ecology and geography of the environment and in the physiology of the species-that the expected outcome would be a random pattern of extinctions. Yet a definite pattern is found instead. What could explain the disappearance of multiple species at the same time? If there were significant widespread changes in the environment, multiple species could be affected simultaneously, causing their extinction.
There are other vegetables currently grown In the region that contain more betacar currently cultivated vanetles of sweet potato do.
Evaluation of a Plan Situation
Agriculturists believe that if farmers in a particular South Amen an region new beta-carotene-rich variety of sweet potato, PK004, the vltamm dellicimc:y suffered in that region can be alleviated, Even though sweet pota the region and the body can convert a sweet potato's beta-carotene m varieties currently grown there contain little beta-carotene
Reasoning
What would most support the suum oJthe pliln to imprrrvt nulnhon to plant SPK004? What, that is, would make farmers respond ponm,.)y encouragement to plant PK004? Farmers III the region would probablyb-iDldill_ substitute SPK004 for the varietie of weet potato they currently be assured that SPK004 would grow as well a those other vanetJes turn most likely lead to PK004 being sub,tituted for current vanetICS in staple dishes, and thus to an improvement in nutrition in the regiion.
Correct. Thi tatement properly identifies a basis for explaining the pattern of many species becommg extinct simultaneously.
1hia explanation of selective extinction does not explain how many species become extinct at the 1liiie arne.
A
explanation addresses only some species, not many species.
.pull,. . 1& based on what the fossil record suggests; more recent times having no fossil
a
lie
outside the consideration of the passage.
tt:==::wh:
Ii
'
ICh pe.cies are least likely to become extinct does not explain a pattern of extinctIOn of many species.
B
C
D E
Correct. This statement properly identifies a factor that would upport a plrecllicfioD success. If dishes made with SPK004 look different than tr.Iditional weet potato elisha people might be less likely to eat tho e dishe '; in uch a situation, the 1IUI"'!l1iI likely, rather than more likely. It is SPK004's beta-carotene content relative to the be~-earotene COO currently grown in the region that is relevant here. It:~not of sweet potato that are richer in beta-,-aratene than P This suggests that switching fro~ current~ gro~ - nutntlon . , 'III t he rgl'on' aflect e . thl undennm ,fa er
m:.. ...'111"'''
to improve nutrition will su -reed. . th it beta~'OteJlII~:::~=~ d These other vegetables. e Plte cultivated varieties of weet potato. are :~;;::: the region. This information d plant SPK004 will hdp to meet
The correct answer i
...11'111......... 2nd Edition
following best completes the passage below?
. t participating countries favored umform ts to the North Sea, mas . d -"/J1Cel1lt onference on envrronmentaI th rea tal damage could be attribute to a .• h th or not speCific enVIronmen . . I CllrllJolson the quality of effluents, weer der to avoid excessively restrictive contro s, ...rtlieular source of effluent. What must, of course, be shown, In or thIt _ _· .......
11.
IndliDies.:::::::
Shelby Industries manufactures and sells the same gauges a Jane 40 percent. of thek. cost of manu.facturlng gauges at both Shelby Industri e and Industries IS see Ing a compelitlve advantage over Jones Industries Therefore Shelby Industries should lower employee wages. Which of the follOWing, If true, would most weaken the argument above'
(AJ
Because they make a small number of preCISion Instruments gauge ma uf8Cturetrs Ci. . . . ,• • discounts on raw materials.
(B)
Lowering wages would reduce the quality of employee work and thiS redu e lowered sales.
aD of any gIVen pollutant that is to be controlled actually reaches the North Sea at present
(e)
Jones Industries has taken away 20 percent of Shelby Industnes' buslnes over
enVIronmental damage already inflicted on the North Sea is reversible
(DJ
Shelby Industries pays ItS employees, on average, 10 percent more than doe
(E)
Many people who work for manufactUring plants live In areas In which the manufacturirl& I.~ for is the only industry.
any un~orm controls that are adopted are likely to be Implemented without delay any substance to be made subject to controls can actually cause environmental damage the countries faVOring umform controls are those generating the largest quantllies of effluents
um.nt COnstruction ttu on In the face of environmental threats to the North Sea, restrictions on effluents are considered. _lDlllng
HDW tan txcwively restrictive con/rols be avoided? To prevent pollutants from entering the North Sea, countries decide to control the quality of effluents. They need to control only those effluents that cause environmental damage. There is no need to re tnct harmless effluents.
Evaluation of a Plan Situation
Reasoning
Ie blU1lledtaey ofadopting controls does
not prevent the controls from being overly restrictive. tement properly identifies the fact that controls on harmless effluents would be tflctlve and 60 should be avoided.
~=:::::ary re trictions involves analyzing the quality of the effluents, not the ~
countne favoring the restrictions.
rw..IU),to:::;e that all of a pollutant reaches the
orth Sea. It is necessary to prove A
lI!1.dalDIIIC that has already been caused is outside the scope of the restrictions. illJ!JFiI revenlbl Will do nothing to prevent unnecessary restrictions. B C D
E
1h
Two companies manufacture the same product, and ClDlploYft\9llP companies account for the arne percentage of the man.ufaetutrinl seeks a competitive edge by lowering employee wage What point weakens the argument infavor ofltlWt1TllK flM!"aP1lllt GrI/l..,,:~"1IIIi lll p that the result of lowering wage will be a compebtlft edge...II"-,:through offering it products at lower prices as the n:suh: costs. If evidence can be hown that 10~1f"u:I~IIII1~.lIlIIl • • competitive advantage, the argument IS ; ; ; wages is lowered employee morale and . us declining quality could lead to a compebtlft dislIdviUlll.,.1rIl11l
Becau e the company i unable to get lowerre
help::::':
Theloss of bu ine would only edge; it indicates nothing about 41l Its relatively higher emploYIII= WltPI·..• supporr for the conc1usiOD weaken d. We do not kno t 1UlI1iVViia II..
deficit If they did countries with the largest budget ._IMlMllrltCIUII /8rII trade wh s'd ficit figu~es are adjusted so that different ...Ildlillqjest IrlIde deficitS In fact. en e , "...._lbIelD each other, there IS no such correlation. .llIIlbOlIIlft III true. whICh 01 the 10llowIOg can properly be IOferred on the basis of them?
13.
-:ttl. \ti1
Which of the lollowlng most IogicaIycom. . . . . . . . . . . IIl 11i The last members 01 a now-extinct SlleCles 16,000 years ago. Prehlstonc cave of a Eutira'tIllll........ I pa nbngs III France "-'Fossils 0 thiS animal, however. do no show - . •••• cave palnlings are therefore IOaccur Ie any Iunp NweI""ortatw.-a
....swlth large nabonal budget deficits tend to restnct foreign trade. IIlOIftINI"SOIIs of the deficit figures of one country with those of another are Impossible. ~:~. country's nabOnal budget deficit Will not necessanly result in a lowenng of any trade defiCit that ~
tllIYhIve WIlen c:ountries are ordered from largest to smallest In terms of population, the smallest countries IIIlIIII~hIv'I! the smallest budget and trade defiCits. CaunIrIe with the largest trade defiCits never have similarly large national budget defiCits.
~
1"1IIIII1It Construction o correlation is filund between large national budget deficits and large trade deficits. What Inference (an be drawn from this inftrmation? Since the passage states that national budget deficits do not correlate with trade deficits, it is logical to anticipate an mferen e about the independent nature of the relationship between the two kinds of deli il . One po..ible inference is that reducing one deficit need not result in a redu lIOn of Ihe olher.
(A) (6)
(C) (D)
(E)
some prehistOriC cave pa n ngs n Franee aso deotct fOSSils of the giant deer are much ore COtlVnOn animal humps are composed of fa ssue the cave paintings 01 the glan deer Nere patnted only one currently exis 109 spec es 0 deer hump
Argument Construction Situation
Representations found in prehis n ca~ pamtmgs m F deer peCle -the last members of which \ivai m 1rdaDd aIloillt • depict the deer a ha\'in ~ hump on i ba I" the feature a hump.
Reasoning
IVhat point would ml)jt logi
would
we orne supporr if rhere were information indicating that there was a correlation 1IIItWlleIIIarge budget d liuts ami small trade deficits, but no such information is given. Ie thaI reliable wmparisons have been developed. t tern nl properly idenlifies an mference that can be drawn from the given
s regard SIIlCe
A B
c D
E
COrl'e4:lt lUlI'WIt
.,lICIl8,_I_
i
th t are especially pernicious. First, It often
5~~:'_
pestiCides mfarmmg has two effects t at onally gives rise to Insect.lcide-resistant _ ieS II .... often Unln l t d " .. area Second'Itd II beenthe ones most resistant to I ,an t hey are tIlIt survtve a partiCular mseclJCI e WI
f d th t the effectiveness of the sustained massive use of 1tt.,1."'lIbo\Ie it can be properly In erre II a ssummg that each IS a realislic possibility? Illiellded by doing which of the fa oWing, a
15.
In an attempt to promote the WIdespread use 01 waste, the council of a small town plans 0 ban of paper eXls!. The council argues that SIflCe enVIronmentally preferable.
end" _r.yleldlng vanetles of crop plants
Although biodegradable plas c goods are now available member biodegradable paper goods to be safer or the r'llIVTlI'nt
(B)
The paper factory at which mas a biodegradable paper goods.
What 'tin bt dont to prolon/( th" ,jftrtivrnm ofpestiride use? It can be inferred that the OlIFmg u of a particular pesticide will not continue to be effective against the future aeration of pe t with an inherent resistance to that pesticide. What would be v gam tthese tuture Kener,ltions? If farmers periodically change the particular pelticide they u e, then pc Is resist.lOtto one kind of pesticide might be killed by IRclther lb. would continue. with pests beinE( killed off in cycles as the pesticides are n d It I al Il pllssible that this rot.ltion might allow some of the pest' natural mi to UrvlV • at least untilthc next cycle,
•_".,.lata ..
.1Il
(OJ
Since most townspeople prefer pias c goods 0 paper goods many IIISl:atll:es. purchase them III neighbOring owns where pas c goods are avatIabIe
IE)
Products other than those derived Irom wood pu p are often used are entirely biodegradable.
Situation
nOlhlll11 ... In
IemI I IIC ,'lIccti\,cncss ,1'
A town council con iders bannin the ale ofdis]pos;lblc plastic goolJds b:wIlidlalllllll preferable.
Reasoning
JVha! problem mightthert b~ In tlx ,-oll1",ilj amount of pia tic used by the :inzens of the howe,'er, onlY a: effective a-the upport't prefer dispos~ble plasti· _ - and the ro
The exi -tence of biodegradable pia n' d good un uitable to it" em·ironmenr.u
It
pd
the Rl8lUawre
are paper sub titute becau'e paper i bIodegradable
allinn Ih.ll wulJ extend the effectiveness of
part III the arJ.:UOl,·nl. SlIlce pestil-ides wuuld hc !l1l1I1 Irr lev.lIlt,
goods
Evaluation of a Plan
mount ofth pe !lcides helllJ.: usc'd will not hell' the situatiun since the nt
employed
After other towns enacted Similar bans on e sale of plas discernible for several years.
twn bout heml,ally st.lble in cctil-ides is E(iven to make a sound inference. t prop rly Id ntitie
e ownspeople are
(el
. . . . .Ill of. P1.n
nttnued high-level pesticide use often kills off the targeted pests' natural enemies. In addItion, the pe ts that survive the application of the pesticide may become reaiiJtant to It, and these pesticide-resistant pests will continue breeding.
ban the sale
(Al
IIllIIg the QuantitIes of pesticides used
....... few fields falloW every year
paper
Which of the followlIlg. If true, IIldlCa es tha the .1' plan the town counci s enVironmen a\ goa s'
rmcally lable Insecticides
,.IlIdiI=IIr switC/lmg the type of Insecticide used
;::.~~:=::=::::!
B
That the 1001 f.lCton will in "rea p,rodluctioo that .\ sufficient am~unt of u h produ from di:po. able pia tic
of thc usc of
thi
,!!;oaL. En\'ir<>nmental ben Ii .I< ,,'r,ll\ ,Ir: after similar
oroPCd)'
D
E Th
t aftliweJr 11111.
:;=~:=~:::
delays at lIIe nation's increasingly busy airports have Increased by 25 percent.
!:==~:~more:~aI:'lIIe
takeoff and landing slots at the bUSiest airports must be allocated to
the following if true. casts the most doubt on the effectiveness of the solution proposed above? ""P' au es of delays at the nation's bUSiest airports are bad weather and overtaxed air traffic
control eqUipment
17.
A major health Insurance comPillly
I ilIDII
procedure IS first approved as "rnedicatt .~;~~::::= to save the company the money IIltght OIheic1llSe
:~:::::::::
has recently announced that In order to reGJce
Which of the following, If true prOVIdes the Ss1Jlnllillllcestigelit j!lSIiIiali....r
(8)
Patients often register d ssabsfaclJon Wdh PhysIcians often prescnbe speaa DrDCelbes of the patient.
canlllllrcillllrf,ne
(e)
The review process IS expens e and Draetlcaly always res~
AItM. small Midwestern airport doubled ItS allocation of takeoff and landing slots, the number of delays
(0)
(A)
lIirIne deregulation began, the number of airplanes In operation has Increased by 25 percent.
60 percent of the takeoff and landing slots at the nation's busiest airports are reserved for
were reported decreased by 50 percent. SInce deregulation the average length of delay at the nation's bUSiest airports has doubled,
(E)
ult/on of • P1.n
Wl
=_
__
ift_~"""_"I_
The company's review process does onterfere than one effective procedure IS available to select
the -::::::::::::~~~ one I
The number of members of e comDaRY-aPllolnted ~ ............'. . . .1. . . on the cost of the procedure
o redu delay, more lakeoff and landinR slols at the busiesl airports should go to
Evaluation of a Plan
mmer lal airlines.
Situation
In order to cut co • a mao r beaIrh iDlunDa: aJllIIp., that it will pay for pecial promlum 001, if'dIll .._ ...ail....... nece sar)' by a miew panel.
Reasoning
What PUel ofinformatrtm flY)lJd ,_t 111.• •,jraf;J,..- . . . . . . . company to a\'e money, It would need
What pomt (Ill/I thr mal/ dOIl/l/ oll/h,' propoJed JO/lltioll? Evaluating the effectiveness of thl olutlon ~eans ex.arnining the relation helween the problem, an increase in delays, and the oJullolI, all lIll:rcol'l' III available takeoff and landing slot. Could the delays be u ed by other fa, lor, Ihat till' solutIOn fails to address? If the major causes for de'lays r r ltd 10 we.ather "lid air Ira conlrol equipment, not to the number of slots f~r If and landlll~, then the proposed solullon of changing the allocations of the slots und to be I etlclIiVl'.
tn,
~III'IICL 1111 n
wbat~~~:~::::5=
abandoning rule. Under the compan~'the money? The pand itsIihDilIlr lie Furrher. the co t sa .......... demo . ignitlcant numbers of]proaxillll'l5.
.
t t m nl proper) idelltifies ,I \waklll'Ss in the proposed solution.
~m
: of plane how wi1\' "irpor! .If,' hu ier. Since Ihere is no information rt ne are pm ,lie or 1011I111erll.II, tillS t.,temenl neIther casts doubt on the
:::~:':u;p ~t
~.~_t1IIUC~'h
ar addll,oll,lI lots Ih,lt l"nuld be allocable to coml11ercial t om upporl for till' PropOSl'" olutiol1. how lh t Ih olullon 111.1\ work, h w lh
0p 01 Ihe ,'rohletl1'' tlisi ,ll ltl l ",'S lIot
l'.\
A
Thi ,ugge'ts that patienrs might be pressw:ing_irP. . . . . . . procedures tor their Jilm n and dem'in" <- them. But if .lbando~ing the panel' mie';,.,,'OUi1d
B
This sugge"cs that :erta1ll prtKl:lbes necessan. \\ hich in tum suggests ~ them. It'this I th then ,1Il1l1unt of mon l.
I douht ,)n
C
~;~::~=5:
I ci i,'lI.
D
E
1.
c,'mpan~'", ts. .... om t. Thi tatemCllt DftllIICII1
ndEd
"
e of raw cotton has fallen considerably In the last year.
rrw wool the wholesa Ie Pri Clothing stores has not yet fallen, It WI'11 Inevi t abl y f all. e of cotton clothing at re ta IIC true mo t erlously weakens the argument above?
proceSSIng rrw colton for cloth has Increased dUring the last year e of rrw wool IS typICally higher than that of the same volume of raw cotton. of the average retail clothing store have remained constant during the last year. alway lag behind changes In wholesale prices. "UlIlVlstilna rrw colton has Increased In the last year.
h wh I ale pnce of raw colton has lidlen significantly, the retail price of ClOElOn clothing In tor will inevitahly fall. I p ""
akin! Ihu tlr/l,umrntr Consider carefully the difference between the two JllCJdu~tlfor whl h co I arc hein~ compared: cotton and cloth. This argument IUlimel th t lower whole ale pricC' rc>r ,I raw product must necessarily result in lower I pn for a proce cd prOlluLl. What other factors could have an impact on the II pn of((llton c10Ihin~? I f any of the costs of transforming the raw nto pro d prodm I inne,I", thc'll the retad prices of cotton clothing will nClOeI'I.lr.1 t II
.lalll:lllent properl) Idcnllhe
.1
I nd r t IIlpme I lhc nl"'roltlll~
10
t I
nl
Forger cannot dupli Ie all thecbllllXlaillill analyzes, includin the nn Computers eqUipped th on file.
Reasoning
Wh.l! condusr tate' that th accompli' hed ....'......·r enough to all sonlCOl'le
r.lw cotton and
tnf the ret.1I1 ste)re. If that ...'w1Id.1e n t III rath r th In rl III', II' I'n Ihl,' th,ll lhe rl·t.1I1 prius could 1,)110\\ :'" DnI:el. llU th ar 11111 nt I nnt W"lk"lIed II
Situation
we.lkncs ill the ,tr~lIlllellt.
I and raw oltoll.ue .rrekv.11l1 to price ('han!!:es
wh I
Argument Construction
en
A B
installed 10 m) t barlkSbe<:aUSIC cotl\ l r tho t th re mal propril:wy cc. .niI-
ha f .lIm IIllb 1"Ifl'nll .lI1d Ih.1l tholl h the ret.lil jllll 111 ar IIIIICIII h.••dll'.1l1 t.• kell till' la~ into .lln)Ullt
urn dan
. '0 IOt,)rmation about th .\Ithou h th it re would
OfT
ct. Thl tat m nt vropcrly iIllenitifil:s
D
\lth, u hit
l'
th r' a ,thlt 10 th \\Ith, n Iu
nf r,l\\' lI>lll>n .11'.1 do not .111,' t ntH 'nl
..
much lower failure rate than companies financed by . . .ftllmrd by wnture capitalists have a rtant causative factor In the success of a .....IllIIIlI~'Jf-~; therefore must be a more Impo I f • ,.......11I ' I h acterlslics of the entrepreneur, the qua Ity 0 ~=:::1IlIn ere such factors as the persona c ar II or the management structure of the company. following if true, most seriously weakens the argument above? ":::etPltaI~ sts tend to be more responsive
than other sources of financing to changes In a start-up
financial needs. _IC plannmg 0 f a start·uP co mpany IS a less Important factor in the long-term success of the c:ompany than are the personal characteristics of the entrepreneur. More tIIan half of all new compames fail within five years. management structures of start-up compames are generally less formal than the management structures of ongoing businesses. Venture capitalists base their decIsions to fund start·up companies on such factors as the characteristics of the entrepreneur and quality of strategic planning of the company.
21.
AphaSia, an Impairment of the callaClty to brain. Many people With stroke-relatecl":~;I:&4~:::::::= One proposed explanabon for such recovenes that language center, develops ItS latent language
Capabifjlies::::::::
Which of the follOWing. If true, most strongly 5UIlPorts (A)
C
(6)
(e) (0)
(E)
Argument EVlluation When venture capitalists fund start-up companies, the failure rate is much lower than when the companies arc funded by other means. The success of start-up companies, then, may be attributed more to their source offunding than to any other factor.
"'nJnll
In a study of local brain aclivrty In people pertDr!lllng a showed higher actlvrty levels 10 the nght ha of the ~:rg:,~all:e::-.::::::: Ablow to the head Injuring the left half of the brill can language IndlstlOgUishable from ha produced by a strolce. Among people with stroke-related aphaSIa rl!CO'll!nng IostCiIPilCilylDG.I.......... Impairment of those capacilies norma Iy controlled by the _ ........ Astroke that damages the left half 01 the bram often causes ~5icl.ir.IIIlI"'''''body that lessen over time. ,,.,.':.cItI"'~ Studies of numerous people With aphas a have Indicated that \tie fIIlt1iD1l511l11t. . . . . production and those that govern language comprehension are
Argument Evaluation Situation
capacity to u e language_ This impmmcnt IS ClII1m ~~=~~:::~== year of a stroke, which may be explained the b usually not the major language cente~ compensate for damage to the left Ide of the
Wha, poml wea/um Ihe ar/(ulnml? The argument concludes that the source of funding the Ingle most important f.1ctor in determining the success of a start-up company. But what if the source of that funding, venture capitalists, considers other factors hefor makinl( its investment? Venture capitalists may evaluate the characteristics of the ntr pre~eur a well as lhe company's strategic plan and management structure hefor deCiding to fund the start-up company. If this is the case then the most important causative f,lltor in thc success of the company canna; be said to be the our of the funding. I
of v nlure l.'pit.lli'ls i, ,I poinl in LlYor of the argument, not against it.
bout the r latlW IIllporl,lnre of Ir.llCl~ic planning and the personality of the nolit waken the ,lrgul1lell! h".\llSe it docs nnt address the importa~ce of these nan lal backing. n
Reasoning
A
Correct. This statement properh iden -
B
That a blow ro the head :an Ie ult in ph,-- ,-argument is concerned with finding an not with what other than ~ -aboutwhethcr This proc'ides no eVI'den ••
ful tart up . h' I . lomIJanll", '" Igl failure rates arc irrelevant. aJ with th Ull s rllC of t t . .lr up romp.lll1eS based on their sources of • f n not rt up lOmp.lIlic III "cnl' I . I . ,.. r.\ Will ongoing businesses Ius no bearing cvidl'lllC th.ll cw.lkens tl\l' argUl1lent.
Whal t"t,idmc( supports fix txpIaNIIi capabilities 10 compmsattftr_da~ ''III~9: bisk.PO.~"_'''~I. bl' evidence that when pertornung a " t~ the left half of their brain h more activity people who have not ,uttered u-h darnag~.
m only Ull
nt prop r1 ad nllfi
Strokes that damage the left half of the brain
D E
SUllpOniI.,rIII:~"'."IliII":llI
ft"
language 'apabilitie th t all VI ."L.llt ~~.... of c.lpacities concrolled.b th right patienllS completd) uninlol -cd m _\m efle.:t that a rroke m 11II ....
Th.ll language produ br.lin indkate nothl •lph.l '1.\ pall n I
Th co
tans'werl
bOIl
bout::::::.~ •
• •_
. . . . 2nd EdIIIon
pply IS strictly controlled: farms along the ~~~§~lhe~ Colorado Rnter, use of the rlV~: wa e;O~U,rngatlon But the trees that grow In narrow .'milld allocation that they are a,lIoW,e thO u:f~re If farmer; were to remove those trees, more rivl!r's banks also use ItS water. c ear y, er , be IVIiIIble for crop ,mgallon.
~_MGr""
t
following, it true, most seriously weakens the argument?
tree along the mer's banks shelter It from the sun and wind, thereby greatly reducing the amount of ....,lost through evaporation. of farms a'ong the river will probably not undertake the expense of cutting down trees along the banks unless they are granted a greater allocalion of water In return. ~ of the tree species currently found along the river's banks are specifically adapted to growing In places where tree roots remain constantly wet. 111I trip of land where trees grow along the river's banks would not be suitable for growing crops If the trIIes were removed.
.owners
distrIbution of water allocations for Irrigation IS Intended to prevent farms farther upstream from uSing Wiler needed by farms farther downstream.
um.nt Evaluation
Near Chicago a neWly bUilt hydrOPon spinach, produces on 1 acre of floor IC sPinach "fact-" _ . a ~:=~::::~ ' 't h' space What It take 1 I t for e eCriCI Y. are Igh, however, and the s In California field spinach, the sPinach co P tyach prod eel mmon sold throughout Which of the follOWing, If true. best supp rt profitable? 0 s a prOJection that the 5Pi1ild1tJl_lIf1 (AI
Once the operators of the facllrty are experienced the be able IJ~ all III. . . . . . 25 percent. ' yw
(B)
There IS virtually no scope for any further reduclJon In the cost per J)lIUndtar C........11
(Cl (D) (E)
Since spinach IS a crop that ships relatively well, the market for the ~=~::::~ more limited to the Chicago area than the market for CalifomlCllie II A second hydroponic faCility IS being built In Canada, taking advantage of IRUiPIll_ vegetable prices.
Situation
Spinach grown hydroponically in a controlled ellVllOmD:ent is four times as expensive a the popular washed field splllll;:b a-III
Reasoning
What point best JIIpports thi projrchon ojprojitalJi""J'JUf ICKbytI.~_rl/l_. hydroponic spinach cannot compere W1th the Califu be another wa)' for the hydroponic pinach to compete sw:cesmdlJ' ."'~I~ washed pinach? That i , what feature of the hydropo spinach unable to match? If the Califunua spmach I t3iJ~illIIrit:Ib~:==: herbicides, health-con ciou con ume~ could be wiJlling b) PI'I for hydroponic spinach that i grown in pristine COlldiltiolIll111i11. . . . .111 pe ticides.
What point weakem the argufIlmf? The reason given for removing the trees is to make more water available for crops. What if the trees actually conserve water rather than unply consume it? Jf the trees protect the river from sun and wind, their presence can greatly reduce the amount of fiver water lost to evaporation. Thus, the farmers would loa wat r, not gain It, by cutting down the trees.
Canect.1I1I tatement properly identifies a weakness in the argument.
m nt I about the gre~ter availability of water !/the tree are removed; it is not about
:;~.::::~:~~litioDi are nee ~ ugument
Unlike washed field spinach, the hydrOPOnically grown sp nach s untainte I ~;:~::: and thus Will sell at exceptionally high prices to such customers as II:
Argument Evaluation
Water is scarce and precious in the land along the Colorado River, and the amount allocated for irrigating farmland is strictly controlled. Farmers would have more water available for crop irrigation if the riverside trees, which also use the water, were removed. lllueHllng
23.
A
sary tor the farmers to agree to remove them. This statement does not
_lIment IUgg t that c rtain tree speri' . I h" ....Millo typ I f h h es a ong t e flver tend to remove more water from IC 0 trees, w IC strenl(thens rather than weakens the argument. lll:ern the greater availability f h o water, not t e greater availability of arable land. h I r aUo atlon doe not have 'lny h.. . r irrigatIon. ,carinI( on w et 1er removing the trees will
B
C
D
E
The 25 percent reduction in operating ro IS cannot mpe difference, so this statement doc not upport the pro
Knowing that the price of CaliiOmia pinach can go. :~;::::I::I; . pmac . h'lan po I'blv. rompen te tOr the b hydropOniC products. Correct. This tatement properly I~en~ spinach to compete uccessfull Ith . .I an advantage sh The ease ot. sh'Ippmg The,existence ot•a ",aCI'1'Ity 10 di anaadd:;;:=;'~;;! fi protitable, becau e the ana
The correct an wer i
25 IIIat the PZ 1000 has the fewe st injuries per accident of any car In reports p. 1000 IS one 0 f the safest cars available today. if true most senously weakens the argument In the advertisement?
TIIffic Safety Institute report listed many cars In a
, ,
ther classes that had more injuries per
did the PZ 1000. Id th n have any other kind of car in ItS class. e PZ 1000s have been so a " • •ntyellS many mor lst4..... till class to which the PZ 1000 belongs are more IkI eIy to be Involved In accidents than are other lit ClIS between the number of injuries per aCCI dent for the PZ 1000 and that for other cars In Its Alifflrence IS QUite pronounced. 1111 HilhWav Traffic Safety Institute Issues reports only once a year.
-.a....ItIEVIlultlon · An advertlsement cJ' alms th at the PZ 1000 i one of , the safest cars. available; I dit bases h this claim on the Highway Traffic Safety Institutes report that thiS mode ha t e kwest injuries per accident of any car in its class. ....",ng
Whatpoint wea/:enJ the advertisement's claim? Examine closely the difference between the report and the conclusion the advertisement draws from It. .Whlle the Highway Traffic Safety Institute compares the PZ 1000 to other cars In Its class, the advertisement compares the PZ 1000 to all can available today. What if the class of cars to which the PZ 1000 belongs is a more dangerous class of cars? In that case, while the PZ 1000 may the safest car of a dangerous class, it cannot be said to be one of the safest cars available.
Which 01 the lollowlng most IOlPcaIIy
Situation
Reasoning
e higher Incidence ofinjuries per accident in other classes of cars supports rather than weakens
tatement properly identifies a weaknes in the advertisement's argument. trengthens, rather than weakens, the argument.
. . . .~mqofthe reports is irrelevant to the adverti ement's claim.
In a population of animals, food scarcity cause later lqllRlClucilift_1I!I population is hunted excessively, earlier-maturing animaIII".IOI.. the population. What point would most logically romp/en the mgumnrt1 For the ialin_ _ of use in determining what cau ed mastodons' extmcttoD, l1li. . . . . . to indicate the age at which mastodons reached repmductiw what the argument suggests can indicate cause of ext1JIctIOR.l( I~~~ from before and after mastodon populations began to da:Ha1l, II.-Ilft.. those fossilized rna todon reached reproduetlve matunty will have a good idea of what caused their extinction: if maturity late, it wa probably food carcity, but if they _1tUI"11III"~1II! likely hunting.
--iIlJI,....
the adverruement's argument.
that the PZ 1000 is the best seUing car in its cia s might be explained by the fact that it t car in its class, but if this has any effect on the argument at all, it would be to _rr'."!J&l:hen rather than weaken it.
----...
Either load scarCity or excesSIVe huntitla individuals In the group will reach reprodu:ellllllllllll'~ individuals that reach reproductive matu l1lIIIIity.r.1III determine whether preh,stonc mastodo:~WI CClIlIe are fossilized mastodon remains Irom both bel eatinct ~:::~::= ore and after (Al there are more fossilized mastod dec I,Ine th an from after that penodonremalnslrornlhe _ D. .UI. . . (Bl the average age at which mastodons I ' fossilized remains rom a gIVen perjod reached ItIlRll_ _ esta bl IS' hed I rom their (el It can be accurately estimated Irom lossll d Ize remains when rnastocIonI __••• (Dl It IS not known When humans first began h t '''',....~J! un 109 mastodons '. (El climate changes may have gradually reduced th lood iIabIe e ava to IIII&t!J dt • Argument Construction
A B
C
D
This fact onl), helps indicate that there was a decline; it tells us DOIhiIIl a1~~~ decline. Correct. This statement properly identifie a point that 1Dg;ial1ly Olllll!pIIll~ explains how the fossilized rna rodon remain could be mal to rna todons' extinction. The point at which rna ta
::::'!!
which is concerned with hOnWustodloa ... scarcin' and hunting affect became extinct. E
This fact only sho about whether fossilillld.~ hunting that a tu
1h correct answer
. ._
e most calls received from them are MilI)'IlIrp/ans to deactivate the CIty'S fire alarm boxes, becau: since most people now have ry mayor claIms that the alarm boxes are no longer neee, sdSstrl'c't where there IS the greatest risk B th ty's commercII I , pub or private telephones. ut e CI b xes are still necessary ,dents and few public telephones, so some alarm a
27,
State spokesperson: Many busln s5People e road system. Those people are mIStaken as'~'U ~-"'''lIIIt has spent more money per mile on road
IfIJntarning the fire alarm boxes costs the city more than five ml'II'Ion do r. lIa s annually 'Oomrnelrcil81 bUildings have automatIC fire alarm systems that are Inked I ,. directly to the fire department. The fire department gets less Information from an alarm box th an It does from a telephone call,, ,
(A)
-'lIClftlng
l!."'~l.1 ,.IIII111C:n
The mayor considers fire alarm boxes unnecessary since most people have access to public or private phones. An editorial argues that some boxes should be kept III the ommercial district since the district has few residents and few public phones.
What point weakem thl' I'ditorialj argllll1t'nt? Even if there are few private or public tel phones available in the rommercial district, it may be that alarm boxe are not ne ary. What if alternative systems to notify the fire department are already in pia clOthe commercial distrirt? If the commercial bUildings are already using automatl fire alarm systems that arc directly linked to the fire department and provld automated notifications, then the alarm boxes are indeed unnecessary.
Adequacy of a state's road system s generally less III1JIlOftillt lID bUSiness there than IS the aVailability 01 qualified ~1lIO'1ees.
(e)
Over the past SIX years, numerous bus nesses haw! left the 5p(*e!IPII'SGIrs businesses have moved Into the state, ~..,..~
(D)
In general, the number of miles of road In a state s road system 1IePl!llds population of the state,
(E)
Situation
A state poke person Ie ponds to the cntlClSm observing that the tate has spent more DIOIIlqpa' any other tate has pent.
Reasoning
Whatpoint undmnines the rpoA".np.~r:r P1!_"'~"VIII~ imprm'ements not Ie ult in an ~ more monevon road unprovanen needed mo~ improvm than on road improvemen pro improvement and not th t It
rgument i abuut the nelessity of alarm boxes in the commercial district. This nly tell u huw ust!y those hoxes arc; it uues not indicate that they are not necessary. t m III properly identities a we"kness in the editorial's argument.
."_nerltiJl bt=lilde th pomt, smce the ar/(urnelll inuicates that it is not likely that the fire I
t I phon l,11I re/(olruin/( a tire in the wlllInereial district,
rial
....IIWIIJ..
A
A slower fate ofinccea in spcmdliD,!~dc~IIIII~~~~~!!! state ha pent more than oth and
B
The argument i about t h e :
• •4-'.:ethat the fir d partment musllr.lVelto lhe cummercial dislrict addresses the . ._lUI nlipOlU1C tim thl pumt due nut weaken lhe ar/(ument, eCC:IIlIl}'" nUll
Only states with seriously inadequate road systems need to !iIIl!INIliI. . . . . . . . . .,.~ improvements.
Argument Evaluation
nal
~"'IIII_wi1J
In the spokesperson's state, Spend ng on road 1llI:1IlIl/1!IIII1IIIl~"I"_ilI""'(iI past SIX years than rt has In several other state-
(B)
ily' fire department IS located much closer to the residential areas than to the commercial dlStllCt. average almost 25 percent of the public telephones In the city are out of order,
tlon
H!lIlI"::::':::::::=1
Which of the follOWing, If true, most seriously ullldeIminl!llllll! n_llitr_..~~!
1iI""fo11owing if true, most seriously weakens the editorial's argument?
Argulnlnt EvaluatIon
••
, II publi lelephunes were frequently out of order,
point doe not undenmne mcllll'lltilWlllllD.~:'I.I_~l ince the road tem I not point i irrelevant to the IIIWIIIIDL
D
This background WOniIlUi. . . . . . l~ adequa' of the
t. Thlis sta._I..
1h
t ansI,,",,' II
awarded the coupons by Bravo Airlines for flYing !~:~:~ upons frDIIl people wIIO pIe who pay less for the coupons than they would lilies The coupons are~suldto coupons re suits in lost revenue for Bravo.
."'1!1
Is from Bravo ThiS me
the buymg and seiling of tree-tr8l/fll coupons, It would be best for Bravo AIrlines to restnct the
nbe awarded In a particular year unber of coupons that a person CI nd members of their Immediate families of the coupons to those whO were awarded the coupons a that the coupons can be used to Monday through Fnday amount of time that the coupons can be used after they are Issued number of routes on viluch travelers can use the coupons evaluation of a Plan 'el coupons reduces the revenue of an airline. The company that buys an d se II s tral airline want to discourage the trade in coupons. lINIoIIlng
How can thc air/inc discourage its CIIstommfrom selling thei~ couponJ.?Limiting the use of the coupons to those who earned them and their immedIate famIlIes rewards. passengers for flying the airline and encourages them to contlOue to do so whIle at the same time making the sale of the coupons useless.
LImIting the number ofcoupons awarded a year does nothing to discourage their resale. orreet. This tatement properly identifies a limitation that makes the sale of coupons useless whIle maIntaining the coupons' value as a reward. LImItIng the time of use to weekdays does not discourage resale and makes the coupons less valuable to the airline's customers. Impo mg a date by which the coupons must be used does not discourage resale and diminishes th oupon' value as a reward. bIIg the routes available does not discourage resale but does reduce the coupons' value as a
companyhe~~~~;:5!1
Tect 29. ProInsurance speCial antitheft deVices or alarm Pro-Teet, as part of an effort to their cars have antitheft deVice reduce s or alarm sy terns Which of the follOWing, If true p , rOVIdes the strDngest (A) The decrease In the nsk of incicallllll "'. ...,.••• such alarms. car theft conlerred by a (B)
The number of policyhOlder h insurance companies. s W0 have filed a cia mIn the Ilil!ltVl_ic 1_"';'';;;''
(C)
In one or two years, the discount tha certain highly effective antlth It d t Pro-Tect IS offenng e eVlces. Currently, Pro-Tect cannot legall . Yraise the premiums It charges I a car theft.
(Dl (E)
The amount Pro-Tect has been paYing out than for others. .. oncar-theftclillmshaslleeo .....rll......
Evaluation of a Plan
Situation
An insurance company is paymg more moneyonClUM~~~::.===: reduce these ~ayments, the company is planning 0 cars have antItheft device or alarm systt:ms bec:allllulr*:1ll ~~~~ stolen. ' -
Reasoning
What piece ofinformation would indteaJ thtJuIwJ'" .ti to reduce its annual payouts, and one way fur tbat by Pro-Tect to be tolen. To help accomplish 1'ni"""ftIdtll policyholders whose car.; are so equipped, or alarm systems are Ie s likely to be stolen would interfere with the uee:ess ofPro-1i investing in antitheft device or a1ann SJ'II:IIIIS it~"'t,!,!" than the discount they will receIlIe if makes installing antitheft devi secmlilat.l.......i most likely ucceed.
iJ"'' 'i•
beCllllllEUl'."
reward.
B. A
B
om..,• ••t
Pro-Tect's plan is de igned to increase the more cars equipped with car alarm reclluall Pro-Tect's plan i unlikely to acbielle I Pro-Tect's claim in relation to tboscnl,..... 1111 Pro-Teet' plan to reduce its own car-thlt&.1II
"_It••
C
Correct. Thi statement ugges owner to install antitheft dc1/ic1~ indicates the plan i Ii
D
Because Pro-Te t' p1 to do 0 are irre1levaiDt tD
E
Pro-Te t' nothing abotlt
u<e of the present staffing shortage In public schools. have not been the primary ca ent years teachers have not experienced any b the tact that In rec pnmanly cau ed y I kept pace with salaries In other professions. worlung ondmon and their salaries have no lowmg It true would mosl supporlthe claims above? been hired under the new hiring standards. have th ofesslon would not Many t her alrea dYin e pr h hgher educalionallevel than In the past. I g the profession Wit a I 1J'I more tea her are en enn f h S a reason for the current staffing shortage me t a her have clled higher slandards for 1;ln~ :nal freedom as reasons for their leaving the M ny t a her have Cited low pay and lack 0 pro es I
Aproposed ordinance reQuIres the
(Al (Bl
Most IIldlviduals ha e no orma Ir..rww Since new homes are a a percentage extremely narrow In scope.
(el
The Installation of sma e de ec OfS sprinklers.
(0)
In the City where the ord nance was pr~"" vyv~ auer. respond to a fire was less an e na average The largest proportion of property damage when no household member IS present.
(El
Argum nt Ev.lu.tlon
explained by poor working conditions and poor salaries ' lh e r ae her hIIflage (;1 n he
on
Argument Evaluation
rdlh" Ihan hy trider hiring pfaClices. R tOnln
II'huh "IlUm, nl!U/'I'0r/, Ihi. /'01111
I adler 11IIIng ala II aim
Thl I I
ofvit'w? This argument about t~e cause of the
:E:~:~::;:::::=:!
presence of abyfire. However amember r"'illlleilrlllral extingUished a household caused by reslden a res ~. Which of the followlIlg, If true WOIJld
prate Ion he new hiring slandards as a reason for no t en terlng th e I d I M ny pro pe live leachers have CI e pr te Ion
Itu
31
.
hllflage di misses one possibility-obstacles created by higher standards 10 and mdllr es .II"'lher Ihe failure to improve working condition and A re plln e Ihal give's inli,lInallon suggesting that working conditions and Ire 10 t.1l1 wlw Ihere i "shortage of teachers would support the claims.
Situation
A home builder claim that mjU1l1IIg _ _lie III_ _ significantly decrease propmy darmj~ 90 percent of home tires are put
Reasoning
rn III Inlld Ih undellnllll' the lLums made
because it suggests that the toughened I lid r I 11111(1.1 III 1.1< I he Ihl prilll.lry I,IUSe of Ihe shortage-rather than supports them.
1111111 d 11\ undellllille lile d.llms, helausl' it is udd that more teachers With a hil~r c:u,..ullon II v I Ir CIIICIIII~ tile prole 11111 if working conditions for teachers have not od I II tllr I alher h.lV< 11111 kepI up Will. thuse ufothcr professions. t rn
ltalcnlent mod Ilv ulld IIllllle II. hr t .1.11111 1lI,ldl' ill the p'lssage. CCIlrnld.
I f more than 90 perc'Cnt i mid n
Ii\"( there. then no IOnlul ttainin 4lI_rs
B
m 01 poll' rl) 1<1 IIIlne cmll'1I1e th.lt pay .lOd workll1g conditll,"s have caused th pit ~ Ion IllI tllII lIppOfl Ihe Il.lil11 that these l:lCtors .m: '.luseS of the
I t
Ih hr I lanll IIMd 11111... 1'.1 .Ig\".
fn
hom
3~~ument.
The ,1fl?;ument I about prink!
Ihofilalll:.
mlO
The -m.IU pel'l.'Cnu<,
o F
o
The.u ument i, n It about conllp&risO :~:::==; orr ct. Thl tatement 1)I'll~::::ifie!I~. hl)\\ me: that th t daJ:nIl~
The correct n
ites led to a corresponding surge In ....... teleVISion sa tel I _"""'I mls/llPs WlU' e As a resuIt, Insurance premiums shat uP. making l/IIlIIIIIlIW;Illd""",_u", -.,._-" rwrIIing satellite Insuranc d dded to the pressure to squeeze more IWI vnue Th In tum ha a to launch and operate. IS. • rating satellites. tIy pe l.aIilllCllout of curren 0 b e best supports the conclusion that the th the informallO n a ov . IIle follO\llllll if true. taken together WI , television satellites will continue to rncrease. , few Units Insurance premiums are IatIve Iy , read over re . Since-the nsk to Insurers of satellites IS sp
_"lIIIlIlSiVll
33. Art restorers who have been study," th when subject to climatiC changes hagve feOund factor that thet the oil.~~:;::! II Th e restorers therefore h changes we. III paint, that causes the detenorabon. ypotheSlze that It a !lye Which of the follOWing. If true, most stron I g YsUPPorts the re torer (AI Re, nalssance 011 paintings with a thin I f ayer 0 gesso are les likely to h climatiC c anges than those with a thicker layer
necessanly very high. ' e s of failure are generally Impossible to pinpoint with WIlen satellites reach orbit and then fall, the caus
(8)
lII)
Renaissance 011 paintings are often painted on wood n cWllh",• • • contract when It declines. e panel whl Ii._
confidence. d t IIltes the more frequently those satellites break The greater the performance demands place on sa e ,
(e)
ICI
Oil paint expands and contracts readily In response to changes In temperatur and so IS little affected by changes in humidity.
(Dl
An espeCially hard and nonabsorbent type of gesso was the raw matenal Renaissance all paintings.
(E)
Gesso layers applied by Renaissance painters tYPically consisted of a coar e b several Increasingly fine-grained layers were applied.
down. 'II bers that no economies of scale can be realized. 8>1 Most satellites are produced In such sma num " lEI Since many satellites are built by unwieldy internatIOnal consortia, rnefficlencles are inevitable. Argument Evaluation
Itvltion
onlng
A rise in the number ofclaims after a series of accidents has forced insurance companies to raise prices for coverage of television satellites. Consequently, new satellite are more expensive, and existing satellites must perform more. The co t of television satellites will continue to increase. Why might the (~st rontinue t~ inmase?The passage says that the existing satellites are being asked to work harder than previously. If that increase in workload brings with it an increased number of breakdowns, growing satellite repair and replacement costs will add to the already increased co t of insurance premiums.
Argument Evaluation Situation
Renaissance paintings are subject to deterioration due to ch,anlRe1 actual paint is not a factor in this deterioration. Instead, rest ~pol-""t, III gesso, the material under the paint, that causes problem for the paimiJlp.
Reasoning
What would most strongly supp~rt the hypothesis that gaJo, 1Il1llrir/,l~:I=;::= indication that gesso is affected by climatic change would D supporting the hypothesis. What could how that ge so IS afti:ete. ill till.... extent of a painting's deterioration i directly related to the aDIlO111l11'.*_M1i. under that painting, then the ge 0 dearly plays soroe put
high rate of premiums is a given; nothing in this statement reflects why the costs would mCRa e funher. difticulty ofdiagnosing the causes ofsatellite failure shows one reason the costs are high' it not show why they are increasing. ' C....:t.1his tatement properly identifies f: h . a actor t at explalOs why costs will continue to
lG::~~:Of::te& II
haws one reason they are expensIVe; '. It does not explain why costs
A B C
D
• • GIl)' partially account for initial hi
h . '. . g costs, nothlOg 10 this statement explams
E
Correct. This statement properly identifie a point upporting the blJP~"~ This suggests that another factor-the w~ of the pane1s-bas deterioration. Thus it weaken the hypothe I that gesso tblllfl~" This merely reinforces given intormation, that the paiDl J deterioration. . . about an connechOD bd,",,~ Because this gives no mtormanon nonabsorbent tYpe ofges 0 and the type of of the properties ~nd usage of th~ ~~dltypeetcriolldlalt. gesso is respon ible for the pamnngs Because we are told nothing about whl• • ~III~ decreases the likelihood that the restorers' hypothesi •
The correct answer is •
ommercial airliners and only those Private d t0 C re restrlcte . 1lN1aUI_e:entrBHytoeated uports we Id b forced to use outlying air. fi eId s. Such a .,*,"""JICIa~ most of the pnvate-plane traffic w~u . keOf midair collision around the centrally MI"'lIlIOUnt at ~rivate-plane traffic would reduce t e [IS hch of the following assumptIOns? on W I II I cated airports for most pilots of private planes. 'OUtIJinI8irfields would be as convenient as centra y o . . ffi dt h die commerclal·alrline tra c. Most outlying airfields are not equippe 0 an . dth radar ntrall located airports are not eqUiPpe WI . Most prlv8te planes that use ce Yk f b omlng Involved In midair collisions than are private planes. mmerclal airliners are at greater [IS 0 ec . .. . Id ntually lead to increases In commercial airline traffic. Areduction In the [lsk of midair colliSion wou eve
[I Cl:lllClusiclllldrawn In the first sentence depends
umlnt COnstrudlon ltuatJon
~lSDnlnll
and private planes with radar would be allowed to use the . I . I' . ffi ld O nIy commercia air mers alnpace around centrally located airports; most private-plane tra c wou use outlying airfields instead. The result would be fewer midair collisions around centrally located airports. What assumption underlies the cOIlc!usioll that most private planes would beforced to use outlymg airfields' The argument assumes that if the proposed restriction were put in place, most private-plane traffic would be rerouted to outlying airports. Because the re triction allows private planes that have radar to land at the centrally located auports, It must be assumed that the planes involved in mo t private-plane traffic at tho airports are not equipped with radar.
35
.
Two de.cades after the Emerald R D lVer amwasb It d . was still repro uClng adequately In the river UI, none of the_ temperature In the river below the dam frombelow the dam Since the ~:::::= SOdeg [Ising water temperatures must be I I rees to 6 degrees nvo ved In Signaling the native spe K!S Which of the follOWing statements If t , rue, would most strengthen the scM!I1\iIIts'IlJpl"''' (A) The native fish species were stili abl t the annual temperature range rema e 0 reproduce only In Side stream of the 1M' iii•••• inS approximately 50 degrees (B) Be.fore the dam was bUilt the Emerald R lb· ' Iver annually overflowed its bank Crltlca reeding areas for the native species of fish. (C) The lowest recorded temperature of the Emerald R·Iver before the dam was built t he Iowest temperature of the river after the dam was bUIIt has been . recorded . (D) Nonnative species of fish, introduced Into the Emerald RIVer after the dam wa competing With the declining native fish species for food and space. (E) Five of the fish species native to the Emerald River are not native to any other I'M! Argument Evaluation Situation
The construction of a dam has significantly reduced the range of walEr tall1pl1~~t"~ the river below the dam. Scienti ts have implicated this change m fish species to reproduce adequately.
Reasoning
What evidence would strengthen the hypothesis? To te t the hypotneSlS,scilmti. . to study the same fish in the arne river, but with only one variable t:::~ temperature range of the water. If the arne peCle of6 h successfuJIIDy water that retains the same temperature range that the ~ bad W"'ttlill was built, then the cienti t have likely fuund the cause of the pnJbllm.
Co'ownsence I not at issue. C8J:l8city ofoutlying airfields to handle commercial airlines is outside the scope of the quution
A
~1IIl1Ct, Th
B
tat ment properly identifies the assumption that most of these private planes lack would no longer be allowed to land at the centrally located airports. ftVID-I1lI8I11C tn.fIi could be divert d to outlying airfields even if this statement were not true. mm mal aIrline traffic is beyond the scope of th e question. .
C D
E
Correct. This statement properly identi6e evidence that trengthens The overflow's creation of breeding area offers an alremative strengthens the hypothesis about temperature range These difterence in lowest recorded temperatureS are simply spt:cifit proposed cau e; they do nothing to upport the hypo
c"'top_••".
The introduction of nonnative pecies competing fur fuod thus offers an alternative hypothe i . The rareness of certain specie points to the sevmtt)' ~i t1••pIDi..........
The correct answer is A.
I::;::
air by telling the muscle cells encircling sfrom nOXlo us teeu/es fight damage to the ung An asthma attack occurs when the messenger contract This partially seals off the lungs. th ngs like pollen or household dust. e to harmless I ee:tiVated unnecessarily, ,n respons . lan to develop a medication that would t serrous flaw 0 f a P of the following, if true, pOints to the mos es sent by the messenger molecules referred to lnl8llt asthma attacks by blocking receipt of any messag I
I
Ibove d the messenger molecules that trrgger asthma !AI Researchers do not yet know how the body pro uces (B)
(el (D)
(E)
attacks on's messenger molecules more easily activated than Researchers do not yet know what makes one pers another's for several years, because of long Iea d times 10 both bl I Such a medication would not become aval a e development and manufacture. Such a medlcallon would be unable to distingUish between messages trrggered by pollen and household dust and messages trrggered by noxIous air. Such a medication would be a preventative only and would be unable to alleviate an asthma attack once It had started.
Evaluation of a Plan Situation
e Dnlng
37.
Which of the followmg most log Ily Ica completes the II'RUmenr? Although the pestiCide TOX has b since 1960 has prohibited sale Ofefen WIdely used by fruit &nMer b rUlt on which TO - .. . replace d y one that allows sale of frUit any XreSidue ~ ; : : :" • • the change Will not allow more TDX f on Which trace amounts TDX on rUit than was allowed In the 196 (A) pre-1970 techOlQues for detectlO g TO X than the trace amounts allowed b h reSidue could detect It only when yt e new regulabons (8) many more people today than In the 1960 clean reSidues off the frUit s habitually purchase and eat fruit wittllllll IIllIIliIIl.,1I
.roe'"
(e)
people today do not IndiVidually consume 1960s any more pieces of fruit, on average
(D)
at least a small fraction of the frUit sold e h TDX than the regulation allows ac year slOce the early 1960
(E)
the presence of TDX on frUit In greater than trace amounts has not been shown children who eat large amounts of frUit
Argument Construction Situation
The JunKS are partially sealed off when certain molecules signal the muscle cells of the ainvay to contract. While this process prevents damage to the lungs from noxious air, harmless substances can trigger the process in asthma patients. A medication to block thiS process is considered.
pesticide in use since the 1960 ,i soon to be adopted. lhl ~llIation regulation, in place since 1960, that prohibited the ale of fruit amounts ofTDX residue ha been detected. Reasoning
Wk.. i, Iht plllllio dtvelop the mediratiollflllwed? Consider the action that the medication .is intended to perform. How might that action be problematic? The process. 100tiated by the ~essenger molecules has the useful and necessary purpose of protectmK the JunKS agamst harmful agents in the air. A medication to block the 'IS unnecessary, .m Proce s completely would not d'lst'mguls. h between when t h e process the presence of "harmless substances ,wen and h that process IS " entirely necessary, 10 . t he pre n e of noxIous . I datr ' Thus the med"Icatlon wou Id Ieave asthma patients unprotected firom putentla amagc to their lungs.
Altlhaulgh It might be u eful to know how the bod d lIICellilltYforthe d v lupment ufth d Ypro uces these messenger molecules, it isn't e prupose medicatio . d" clcvllloJ~ In wh r the relevant h . I ' n, many me Icatlons have been f p )'SID ogy I not very well understood ItlVlry 0 some p ople's messen er I I . . t long leaclt d g mu ecu es IS not the issue 1m oe not xplain a II . h . •tallmlclnt properly Id 0116 the fa:~t~:tt t~ medicati~n itself. "m1lting ary pro s. e mediCation is flawed because it could n
no !aIm to alleViate asth
ma attacks once they have begun.
A regulation that will allow the sale of fruit with trace amoun
What point would most logitally complele Ik aTgu~ How " " " _ I l l new regulation will not allow more TDX on fruit now than a&_d iIIl d. The most important thing to note i that the 1960 reguIatton plroJiUbillldllll~ on which TDX residue could be J(ttcted. Detection teehni have been particularly sensitive, which would haw: alIowM SiOII. ·mlll. . . . though undetected-on fruit. If tho e teehrnqucs couId amounts ofTDX that are now allowed. then it couId be allowed on fruit will not be different under the new n:g~IioIL
A
Correct. This tatement properly identifie a pomt that IogicaDy I:OIIlIpII_
B
This statement indicates that {the new regulation d aIIo then people may end up con uming more m H regulation will in tact allow more TD on fruit tha I to th People' consumption at-t-rUlt..I 'rrelevant I
C D
E
sale of that fruit allow. The question i about allowance pfCSlsed statement onh- sugge ts that the older rqllla1i08 . ~.I·nOD end up being true at the new ''5-
r
This might show that the ne regulation were to alI~ m~ ....... irrelevant to the qu tlOD t • 10 the lWlOL fruit than wa alia
The correct n
_litCGmplit\BS tile passeg. be/OW?
will demand a high potential return on .. f ....tors- eIIoUt Iosmg their m , t ards This principle IS the undamental ,.,..tisk ust be offset by the chance 01 grea rew . ~s and it IS ,IIustrated by the lact that . • 1*'14I. .11t , k very risky IOvestments without worrying about IIilcltsSfUl mvestors are distingUished by an ablhty to ma e oney tloe more they
th non loans backed by collateral
iiiOIlIY
a . ..• rs receIVe higher Interest rates on unsecured loans t by banks can actually be below the rate of Inflation
,,···ilt'tllltlisof high Inflation, the ,nterest paid to deposl ors. . .. . . I rate of Interest that It will expect all of ItS individual lIlY one time, a commercIal bank will have a sing e topsy tit potential return on mvestment 10 a new company is tYPically lower than the paten la re urn on
39. Acertain mayor has prOpOsed ill fee 01 ~'cIaI_1_. fee will alleViate the city's traffic COl ; round-triP bus fare from many nearby c:::.~m:.~,)~::=:! Which of the follOWing statements if true • , prOVides the best. . . . . . (AI PrOjected Increases In the pnce 01 gasoline wlllinolCll:leilllil ealle alitlii (BI The cost of parking fees already k •• . ....... ve h·IC Ie Into the City than to take amabu s.esit~_ '--CIUlJ ..- ... (el Most of the people currently riding th bus e do not own ....mft.Il •• (0) Many commuters OPPosIOg the mayor's plan have indicated congestion than pay a five-dollar-per-day lee
.,ifIII_ I 1I 1 ".111_.
(El
tMlItnlllnt In a well-estabhshed company
During the average workday, private vehIcles owned and . for 20 percent of the city's traffic congestion. opewalal
..
Evaluation of a Plan IIUlIIellt Construction
The principle of determining interest rates i related to the risk involved in making the !nve tment of a loan. Potentially greater rewards will lead lenders (investors) to accept greater risks.
Situation
In order to alleviate traffic con~ the llmataylllrlll'lll_•• private vehicles entering the city. Since the ~in...u._• • • the mayor believes many drivm will switdllDoo.&
WhICh tlUlmplt illustrates the principle that greater risks shouldproduce greater rewards? The mpl mu t be about the relationship of risk to benefit. Lenders take a greater risk wh n loan are unsecured (not backed by collateral) because there is a chance they u1d 10 e their money entirely. The principle indicates that the lenders-who by mtlon are inve tors-would demand the reward of higher interest rates.
Reasoning
Whatflaw exists in the milJ6Ts ~Thc IDQOrlll_ money is the decisive i ue for drivm.1£ howcva; _ _ considerably more in parking fees than tbey-.Id .~. . . . . saving mane)' is not the primary rca.soo uc dlIllIlIi~"IilrI:'1 than take the bu . Thi suggests that drmD 10. . . . d..... ave money.
m rou ty njoyed by some investors is not relevant. While risky investments are tern nt doe not m ntion their return.
lI:I:.
A
ltarcnlCllt prop rly identifi an example that haws that riskier loans-those not .~••IllIU_ral--n:ccive th b n fit of higher interest rate . !n tIme ofinftation doc not mention potential ri k or potential
B C
matt r thel vel of risk, contradict the prin iple
This statement doe not indicate whether the iDl:JasediQII~~ private vehicle into the city, and thercfure oatilldalt....... flawed. Correct. This tatement properly idcn Current bus riders are not relevant to currently driving private hiel !n
D
lany driver may continu to~DI:U:~~I:;~ buse '. The mayor' plan d III
E
The 20 percent figure sh city; thi doe not pomt 011
lower rate of return for such riskier
The correct an
41
Dental researchers recently diSCOVer pneumonia and strep throat. Theyf~~tootIb&llIiSQIlI_"' • •" for lour weeks. For that reason MnnIA contat.:atiunUl:::::~:::= ' ~~""'" shWd r!lllac:e their 11 Which 01 the lollowlng, II true would ' most weaken the COOt above? (Al (Bl
The dental researchers could not d toothbrushes had been used for I ISCover why toothbrush cootll11im. '-IIIIIIJ Our weeks. The dental researchers lailed to n ..... pathogenic mlcroorgaOisms. I ves gate contam ".n",,, 01 t'lOlI1bru9hes.." " "I
n_
a_riIlli
,..tl
(el
The dental researchers found tha am people Who cause pneumoOia and strep throat. theongnCldence 01 thused toothbrushes: : : : : : : : : used uncontammated toothbrushes. ese diseases was
(D)
The dental researchers found that people wh 0 nnsed ell oothbrushes thor0l_1y . use were as likely to have contamlOa ed oothb rus hes as were oeoole Who on hurnedly 10 cold water after each use.
(El
The dental researchers found that, after SIX wee souse greater length of use correlate with a higher number of bactena being presen .
a tooIhbni. elld...
Argument Evaluation Situation
Researchers have found that, after four weeks of use, toothlbnlSbes contaminated with disea e bacteria. Therefore, toothbruslxs sh«J1IlId month.
Reasoning
What infannation 'lJ..'eakms th
condUSlf}n~
-laIJ_
The COIOOWes toothbrushe should be replaced monthl). Clearly, the assumptlOD conclusion is that such replacement i' ne.: . because the m . . . . . . . . .( 1, potentially harmful to the u:er of the thhrush. Do people becau e of their use of contaminared thbru ~ It mlpalrisoll of._ _,• • people u ing uncontaminated coothbru h and toothbrushe -ho\" no ditference be D the ~1UPs, potential harm is pro\'en wrong nd upport r COIolSiDD
aet1I.u,
B C
_.aI.
The recommendation to -han, e e\1~f) four siII.,blllll••• cannot explain why it take: that Ion!!; for ntanllrultMJO The failure to investigate other po ible n ofllOO1dl1i.....a. . . . . recommendation. orreet. Thl. statement pn)perlv id n ,ont,lluinated roothbru h i no and tim ,h.lllging toothbrush
D
Sin,e 'ven ',\teful mainren re,'ommend,ltion i trengthellitd.
E
II' the '\)I)t.lmination \)1)
1h co
the ba,t ria t answer
I
ti
em lallng additional measures to pressure the ~~~;~son striIle lpinst coalco Ire cont Pd ers are considering as their prinCipal new tactic union's contrect proposa.I The unIOn lea db Energy Incorporated, the same tpInst Gasca gls stlllons, which are owne y
I
own Coalco
II elevant to the Union leaders' consideration of
wIIich of the following questions IS LEAST drrec y;therr contract proposal? III1ft,pIBii I boycott of Gasco wlillead to acceptance 0
nental United States
Which of the follOWing conclUSions (AI
nue losses by Gasco seriously affect Energy Incorporated? current Gasco customers eaSily obtain gasoline elsewhere? . roposed by thiS union? lither miners' unions won contracts Similar to the one P other un ons that have employed a Similar tactic achieved their goals with It? other orporallons that own coal companies also own gas stations?
(6)
Ie) IDI
~~"'.1IOn of I P1ln IIu8tIon In an effort to pressure Coaleo to accept their contract proposal, union leaders consider org njzm~ a consumer boycott ofgas stations owned by Coaleo's parent company.
IOnlng
43. LawS requlnng the use of headhghts daylight vlslblhty 15 worse In countrj during ~ -;:~;.:::::~ In preventing colliSions In those cou:~rther from tile e ::::~~::=~ from the equator than IS the Conti s In fact, tile only
Whuh question is LEAST relevolltto the boycoll decision? Union leaders have a number of qu stions to consider in deciding what to do. This problem requires finding the one question that is NOT relevant to deciding if a boycott would be an effective tactic. xamine each in turn. Whether other corporations that own coal companies also own Ka tations is NOT directly relevant to whether a boycott of Gasco gas stations will ocr Coako to accept a proposal.
1\.n000n how eflou Iy the parent mrporation would be hurt by revenue losses from the boycott
IE)
Argument Construction Situation
Laws requiring the use o~ headlights during the daytime prevenung car colli Ion m countrie with lower daylighr visibillily, ....~ .. at greater distance from the equator. The onlv cooomes Im>iml located farther from the equator than i the connnenral Um
Reasoning
What conclusion can be drav.. nfrom thi; 'njOrmtlJiem1 Cowmies laws are all fanher from the equator than I the C'OIItinmcaI UIIIIlIed 511-.. location is significant because daytime lbility the continental United tate. Howetfeem at prevm be in the continental United tare with I greater pltlXillBit} rea onable to conclude that ouch Ia would be I there than they are in the countri farther from the eqt'-".
n:J1~anl
nnol ea ily gel gas elsewhere, the hoycott is likely to fail, so this que tion is
~ u:llon hav won similar conttarts, then the union's proposal is reasonable. This h BU (eeded (or faIled) with a simila I h' " r failu ) of Ihl plan. r p ,In, t at information qu II n I th
d rei
IS
relevant to the
I
COU d be most property drawn Drivers In the contlOental Un Ited States who sed become Involved 10 a colliSion u their headlights durill .....,..... . as Would dnvers wIlo did In many countnes that are farth fr . . . er om the equato than VISibility 15 the slOgle most Import t f r the COIl11n!llll.t1. .~. . . . an actor In automolMle c''''li!lic~lIlS. The proportion of automobile colliSions th t .~ States than In the countnes that h d a OCcur III the daytime IV r r ..,_. . . . . ave aybme head ight laws • Fewer automobile colliSions probabl .h YOCcur each year In countnes IhaI: hMI dII• • •" occur Wit 10 the continental United States. Daytime headlight laws would probably do Iess t0 prevent automob Ie ~1IIlisions in, lilt ...... States t han they do In the countries that have the laws ~,
A
nl on Ihal does nol dear! 'h Ito Ih bo all d .. } ear upon the plan; an amwer to it is eel Ion. B
C
Although daytime headlight u'e rna} be Ie etfeenve IB coumrienridl_.~ be concluded that U. . drivers using daYtime headhgb would be just as likely to ha\'e collhon a ~ who The passage offers no evidence fur the con lu 0 tha collision in the.e countries. l\lany factors be:ide- u e ofh adligh duringdIJ/lilhd........._ t'lCto~, may van' widely from on:oun to UIlltbl.. other t:lCtl~rs or' about their rei n I
VII_•••••
D
E
\Vithout specific data. no conclu occur. Correct. Thi' ratem ot J)Il:lJlClrly
1h correct answer I
1_"
b nk failure because the government against ace IS partly responsible for the high • ......41rnlllld alltllS.. II finIflCIIlIy protected .hot thiS Insuran •• «OfIOIIllS1 ,rgues financlal u.... lOcen tI."e to find out whether the bank that ........'.l1li cIllposiII IVI .. lIS from cIeIlO5IIors any I bve then banks would need to be '1fIIOI' IIors were more se ec , e '","51 fadure If depos compete for deP05llor
Often patients HI h ankle fractures are because their or hoped,s s are cOt1cemed number of follow·up x·rays ere rl!V1ewed about found to have healed eorree Y herefOfe Initially Judged stable,
45
•money
• • •llI/IIilfllrlumenl make whICh of the following assumpliOns?
'.res
WhIch of the folio Ing. I true
bo rowers default on loan repayments,
(A)
are caused when btg r Is at several different banks, .e,.ljplilcant ............. of deposllor malOla,n accoun b k " pr..".,. P.... f I h or she lends to be 10 selectmg a an. ha I depo511 the more care u e I , depo IIOrlIlIere o banks IS not a significant factor mbank lIIf"",,, In the f rafes. paid fa deposllors by dfferent I
(8)
(e)
PolartiaI depositor Ire able to determine which ban ks are secure against failure.
(D)
ArlIU/IIIeIIt Conltnlct'on
Situation
Wha, a u"'P'lon under/mIn' Mllom;,/1 tlrgllllJrlll? The economi t argue that bank would h v III he mure e, urc in a mmpetitive environment with more discriminating dcpo II lr The e'(IO"rn. I cll'III.r.I,I:'" potential depositors to be more selee/iv/' in It .n hallk IlU Iherefure J1IU t Ill,liew that many depositor have sufficiently und.d ahoul whal make .1 hallk 'e,ure ,I,l:ainst failure and can often apph' those 'n leI rill 1111 II wIll", h.lllk ,Ir,' <'lllCe,
r lIllllllll t' undl'lh in. ,I' ull1ptinll th.ll p'""lltl.d 'Hur '\ lire.
X·rays of patIents 0 many d ere The healing of ankle rac ures follow·up x·ray.
Orthopedists routmely order o 0 'UD -fays Argument Evaluation
"r dcra f 10 uran,c"(h an k d"I'I, "po. 'The imur'mce . removes any finanCial incentIve for depo 1I0r 10 cck Ih" e hanks thaI arc Ihe mosr secure against failure. In the absence more: lelllvc depo u"rs. Ihe hank, need not be secure ro compete for depo its.
III nllfi III III It h Ilk
Doctors who are general Drae ers r he stability of an an e rae ure eDlre Many ankle InJunes for C a of the ankle.
at
derec:t are emaled
stNeIai
surg
reoarec:t
(E)
n CCOIlom. I ",nlen d Ih aIlh •- hl"h ,., rare o{bank failures can partly "be blamed . on
RulOlllng
s
Often patient wirh ankle fra surgery are gi\'en tollo\\ -up _ examination of a
Reasoning
.\
B
\)
I
1h
tamwer
ac res
and waking cycles differ from those of the other
--::::::::-:~d1'IJIJepertner' ~ach other and have more violent arguments than
iii
tewet actJvibe sleeping and waking patterns. Thus. bOIh partner follOW the same e
....
an serIOUsly Jeopardize amarnag
sleeping and waking patterns also occasionally
5POU e follOW Ihe same ard ze the couple' marnage CYC;:lCiof IndMdual lend to vary from season 10 season, pmg and waking cYcles thaI differ slgOificantly from those of their spouses rr
a ue at work hav been found 10 express hOShhly by adopllng a different sleeping and
thaI f th r pOU e tudy mo I P ople'
In the past most alr\ ne compa!lJeS him:zed heavy. and alfhnes eQu PPed e Dlanes has been the safe one-a c ar UIOICa11JOn mlOlmlzlng fuel costs, Which of the follOWing. If true,
weaken the argument abOve?
haPI'Y
47,
(Al
ISl tel
Last year's bes -se
os' se,IOlJSlv
g a' '0- -03' a _ 3 No airline company as a t1 0d ~_
The pnce of fuel as gher' s sold poorly.
(0)
Because of Increases n e cos year than In any pre ous yea
(El
Because of technological mno a 0 most other airline seats on e 3'
leepmg and wakmg cycles can be controlled and modified
e sa e
e
Argument Evaluation
The afe-t airline _ea -ere h airlines had bought lew 0 ate r ones, the airline h minimizing fuel co' "
Situation
v lu on t 11\ m d cllIplc ren,.I th,ll POllSCS with different sleeping and wakmg ...,,',... I nd til h ve t wcr .1l!lvitie in ll>11111101l and more intense arguments than C
.IXJU"~. wllh 1I1111ar IccplII~
.111.1
w"kin~ l ydes, '!1lUS. different sleep-wake cycle lOa)'
lan' r a 111.1111,1 •
....,..,..1\ Ih
"'1t"""nl
ReaSOning
" " 1/""" "10 Ih 'ITK"III'"( prc'll,i,,'!. would IIlOJI undalll;rlt tbe '1'!' I fi r Iii 1/,//1 1011 Ihe ~Ivc'n prel11lSes In the passage provIde
nIh Ilia relit I~ I' w"k, l )dl' lelllito Ill" ;ll'l'Ompanied h\ m,\rit.11 1 Ih n I 1I111!h "' crt th.l! th, .llIlrrl'llt IccI' wake l'yd~s c.\lIse the mo,iI. I I But wha!II thc .l11f, rllll IeI'I' w,lk" ,rcb resliit from pree i ting ....",..,,1' Ih ar 11111 lit wOlll.l b, 1I",lkCIll'd II :1.11,1 shows th,ll lInh,lppil\' ' ho 11111\ h\ dill> r,Itch ,,,I0l'tlll~ c\'llc's dilkrc'nt ttomthose
The ne\\ inll.)fm.lrion d ,lird m
B l
D II
~ IIl,UO,"
•
,llId
till' 1
'1,lti"n 1\11' \\ Ilh
Ic .1.1 ,,' 1>1.111\111 clrlk"'lIl p,ll! 'III
III
111
III
It 11
do
• till
11\1
not pn l\ 1,1
F The
n
ri
:I
the officer has taken the suspect to the
•1Ir",,".~ until I. Such approval IS denied If the - .... thearres
~;;;;~EtIIe. ! ~~=::.:::; :::_:';A'W_
....I. ....-~ IS based is insufficient. A government
otlJiOIl8I BrreItJ meet the standards for adequacy of h fore recommended that, because the pert has I ere I Asn M'FoiCI The ex t h commander's approva no longer be Iu d the wa c . th "If .. IIrp., , however, since there IS no. assurance at the :-.:::: ...... 0., rous : 1dIlion shoukl be rejected as dangeonce, app rovalls no longer reqUired. rICOIlNlJlj to be observed 'I'll'!"''' stJndards will continue
'.rre
II "'........
:
two pMions In bokIfICi play which of the folloWing roles?
claim the accuracy of whICh IS disputed by the edtOrlal' I , the second IS a conclusion drawn in a. f the editorial ....- to SlJIIPOrt the main concluSion 0 . d IS a conclusion that was drawn from that an observation that the editorial disputes; the secon al that the editorial opposes; the second IS a . ............. _ ... findln, Introduced to suppor t the main conclusion of the editorial; the second IS that main
first a finding that was used In support ofta pro~~:Sed to support the proposal. Jo...._ that was based on Ihat finding and In urn w oIllilrVltion
l1Ie first a conclUSion, the eV!dence for whlch the editorial evaluates', the second IS part of the eVidence 4:OIICluIion c:bd In favor of thaI conclusion. n
Argument Evaluation
on
rre
by Midville police officers arc provisional until approved by the watch
mander, Hecau e almost all such arrests meet that standard, a government efIi en pert re ommends that this requirement be dropped. The editorial opposes mm nd tion, on the grounds that without the approval reqlllfement the rand rd for arrest might not (()ntinue to be satisfied by the arresting officer.
rea oning, arguing that the compan em hin: how to u e the ~licroron compurers.
Reasonll1g
IVhat point "iJ.',',I1,ms tht tTImU mlPUlK" 1Ul""".,1he ~::~:I: tor Vitech equipment i based on redumI tniinirJg d gene~ mana~r rejec this Both
~
do /ht /wo portlom In boldja(( pla,' in tb,' ,'r/itoril/I? The first boldf:lce portion "lII'II that pl'OVl lonal rr t almost always meel the standards for evidence that nfore Th wnd boldface portion slates that officers' time Iplll'ln'lli I rgely wa ted. This is what the efficiency expert lI1ter' from portIOn; nd from thl the fficiency expert, in tu rn, concludes that pproval hould no longer be required. But the editorial
to a\,OId the.e cO IS altogether_ the deci ion between the
~I IIlIlt l~iIlIil:e
Ion I rre t meet the standard tilr 'Idequacy nd baldfac portion, ,lJ1d rejl'C!s the condusinn
p nlon I tru , na1
I
fth reasoning gi\'en ,Iho\'e. n I' main condusion hut to th (ond holdt:ll'e portion i I n p rt' cundu ion. in t h p'IISSIIg.;, n I ti m th
t t m nt
IXPClt"B' lOti
:::::::::= ~
when u'ing the Mkroton COIl,IIf'U:qrslClial£alJ~~~ co. t' a' 'ociated with ernplo exclu ively trom this IlJId ~~IIIo\-erall (0 '[5 tOr the :ompan .-.~~ ,\
R
Thi: intl>rmatilln about ~ would till have to be tra gener .11 manager, rep1y~ IeN:~~:':;;~ ~l,lna rer Teters only to the The intllrmation th t COInp~.1'G pll. ihl to d h t ner.ll manlJl'C:J"sncOlll.o
a_."'.•
A division manager wants to repIacc MicroconCOlmpldll"'IfiII"'~.,_ order to reduce training costs tOr new stalI1he geJlald I!I'W!
Situ" Il)n
t. Th. statelDi-'
argum nt Ulcmflty
d If a poor corn harvest IS predicted, pnces ste ::::: ...... before they are harve . futures fall. This morning meteorologists "",. ted nees of corn . d ~.ft ct IS predic •p t morrow Therefore, since a equate com n n starting o ' d Mleec!l!d for the corn-gTOWIng reglo futures will fall sharply to ay. lll rem I pnces of corn _*,,11I the current aD ssunllVa
~
st weakens the argument above? . . true mo I pollinatIOn stage will not produce a not recelVl! adequate mOisture dunng Its cntlca lIy this season than last season. t tlng more drama tIca fl for corn have been uc ua ted to extend well beyond the corn-growing tomorrow IS expec f meteorologists predlcted or th t has devastated some of the corn crop Will spread
.riaUI~re experts announced today that adisease a .....1IIv IlN!fore the end of the growing season.
h d k h slcal possession of the corn t ey tra e. people who trade In corn futures rarely ta e P y
ArI._:lvl!ultlon . h harvest is expected to be small and drop when a harvest is rap fu tures nse w en a . d' ay's weather forecast for the corn-growmg area pre IctS peeted to be Iarge. 1i0 d . mu h needed rain. so corn futures Will fall today.
51.
A company plans to develop a 1lr0000000we......
microprocessors that distingUish weeds --. ~:~::::! machine claims that It Will reduce !abo from crop r costs by wtuaIIy Which of the follOWing IS a conslderalJ on In favor of the ton1pany' imIl.... (A) There IS a considerable degree I o variation In shade of (s) The shade of color of some plant t s ends to change..-~ a_tile. . .,. (e) When crops are weeded manually -rr'_Y plants from weeds. . overall sze anclleat !hape (D) (E)
Evaluation of a Plan Situation
A company plans to develop an automated wm!lIJg III8lcbh.....,_ weeds from crop plant by difference in shade of costs by eliminating the need for manual ~
Reasoning
Which oplioll describes a considmllion /Ixu 'WtJlIid supports the plan by claiming that the machine would red"'llIII_,. .. eliminating weeding by hand. The cornet opaon Labor costs will be reduced only if the machine works shade of color to distingui h between ~ and plallll.. to have distinctive color without sacrifiang other lt1. . . .11111 that the machine could be u cd effectively.
Whal mformalioll wtakt/l5 Ihe argumtllt Ihat comIlIlures willfit!l? The prediction that rn futures will drop sharply today is made solely on the baSIS of the forecast of ram, which would lead futures buyers to expect an abundant crop. However. if it becomes known that ome harmful circumstance such as a devastating disease will everely ffect th corn crop before the end of the growing season, this knowledge may lead bu r of futur s to expect a smaller harvest, causing prices of futures to rise rather than fill
_lIOIllnll
ItlIllOJIllent tell at what exact point in the ~rowin~ cycle rain is critical to a good harvest, but matlon about th. year's harvest. CIOIInparilOn of pa t pn flu tuatlons do s not affect what will happen to today's corn futures predIcted ram The argument is not weakened. I the ram that aRi cts the corn'~rowing region, not areas beyond it; nt to th predl tlon. .lItcment prop rI Id ntlli info . h ~J~~~ ormatIOn t at weakens the argument. m I Irrel ant to th price of corn futures.
Selection and genelic maOlpulatlon all I ants distinctive shade of color without alte ow Pth of WtuaIIy any !IIl!!CIeltllflli. . . . nng elr other characteristic Farm laborers who are responsible for the manual weeding times In the growing season when extens lVe weed ng IS not "'!Cessary. Oft
A
Greater variation among weed plants would make it more difti between weeds and crop plants. and thi would make It .-, • effective.
B
This option tend to di. favor the elfectiven of the machllJceli i or the plants to be distingui.hed. the more rompl the crop plant based on their color. This option tends to di favor the Iikelv bene6 of the . .dW• • _ _. .i11 weeding di tingui. he weed from crop plan into account. If the machine d not distinguish __ &aID c"ll reliabh' as manual weedin d then the unnece'sary. Correct. laking crop plan of the weeding machin . 1hi, does not favor the COII11PID)r'S illlIlIiUD.... still be t.\sk other th n weedilll costs even if th n
tm
c
!
D E
1h
with ,t, job applicants who have Ii~::~:::::::::~~use. i tlJIijlIb will be elIminated from consideration. jIepInds on whICh 0f tite
folfoW1I1g assumpliOns?
bi! Sl/CCessful if it ,ncludes Interv;~s. program than IS the development of a Job _Itla"/Illft fmpOrtant part of a successfu Iring ersonalities are unsUited to the requirements of CIIIICCurBl8IY Identify applicants whose P
wh ther Job applicants' personalities are SUited to the of an InteMew IS to evaluate e • _ _llIlnts of the lob. . . . . . .lIf;joblllPlrcants. personalitIes to the requirements of the Job was once the most Important factor ,..IrerIlNN)SI
53.
Useful protein drugs, such a In ulln
The statements above most strongly which of the following would be be fisuPPDrt a cla'm that aresearch pro.Ift' ne ctal to users of prOle,n drul51 (A)
Coating msulin with compounds that resistant to digestion are broken down by target
(8)
Converting into protem compou d b n s, Yprocedures that work ,n the IabcJrlt~ resist digestion
(Cl (D)
RemOVing permanently from the dlgestlv t e sys em any substances that dlest ..'lIIiIIlil·~" Determlnrng, In a systematic way hat and whether they tend to be brake~ d enzymthes and bactena are pre own WI 10 the body Determining the amount of time each nonpratem drug takes to reac
IIl'IllIlllInI hinng deCISions.
laftlMtConst",dlon 'The mterview is a necessary part of hiring because candidates with unsuitable pcnonalitie are eliminated from consideration.
(E)
Argument Construction I.J
Whalrs Imng assumed in this argument? The argument puts forth one reason that the intervieW is important: it eliminates candidates with unsuitable personalities. This presuppo es that interviewers can, with a fair degree of accuracy, rule out those aan,bdates whose personalities do not fit the needs of the job.
I
Rea oiling
ent does not go so far as to say that interviews guarantee a successful hiring program. 1lUI1II'1IIll_do not prioritize the parts of a hiring program. tement properly identifies the assumption underlying the argument. one reason that the interview is important, but it does not say it is the only in hiring, and is irrelevant to the argument.
Ire:::::::!!
the Skln · If proteins are taken orally, they drugs, however, contam chemical bo taken orally. ncI 1Ilat are IlOt
A
B
Since prot~in drugs taken orally are dlge ted and do not must be IllJected under the skin. Some nonprotem drugs hln..~..._ to digestion and may be taken orally.
n_a.........
What procedure might be beneficialfor usm oJpmtem for users if protein drugs could be administered orally . _ Digestion has been the obstacle to the oral useofl..eh.d......11W• • have chemical bonds that resi t digestion If prolIe similar chemical bond to resi t digestion, thus aIIowiDg cells, then the protein drugs could be taken onlIy, ...... 111.... drugs. ."'"1~~;;a
Correct. This statement properly identifie a proteduJee :=:~=::: protein drugs: coating a protein drug ( uch iDsulin) II reach its target cells would allow users to ondly....IItIi• • •
_It
No benefit for user of protein drugs i gsined by cmlfttlill~~ compounds becau e then they too would tollei. . . . .
C
The digestive system need the ubsitalllClllS this procedure would do more harm tIlIllllllll"
D
Determining whether normallypftll--..... not offer any specific benm tor The time required by DODpro1.-~~ to users of protein drugs
E
The correct an wer'
•
e animal hides. New environmental ll to preserv d a nIitieS of common s ter that results from this use, an ,In -:::::~;4lSIS Ia/I8 ~st of dlspoSIllB of sail wa lace of common salt. Research has • frOeesed the e potasSium chlonde In p to yield a crop fertilizer, leaVing a rm_is considenng a plan to us I otass lum chlonde use could reprocess the by-product a p - I I SlIJIII! w1urne of waste for disposal. .d n place of common salt, It would be otassium chlon eI P .tmlmilng the Impact on company profits 01 uSing EXCEPT research all 01 the following· It needed to preserve a given quantity fthe common sa f . What difference II any IS there between the cost added to preserve the same quantity a hides? " the cost of the po tas Slum chlon e nee I hdes using common salt SUI abl e for of animal hides and t 1BI To what extent IS the equipment InVOIve d Inpreserving anima I h preserving animal hides uSing po.tasSlum chlonde? .. I f the waste generated In reprocessing t e nstraln the d,sposa a What environmental regulatrons, If any, co by-product of potassium chlorrde? I. d to preserve hides resemble that which (0) How closely does leather that results when common sa t IS use
55. There IS a great deal of geograPhical 'Ili'MIIun variation per hundred thousand peO\lle bmweenn~::=:::~ prostatectomies, and tonsillectomres !II
«"".J/Iril
To support a conclUSion th.at much 01 the van allon I due to IIl1l1ecetl~;~=: . Important to establish which of the fOllOWing? (AI
A local board of review at each hospital "'t(lI'ds examine the r" surgical procedure was necessary
(B)
The vanatlon IS unrelated to factors (other than the surg pr'ocI.ftllllllllllll" InCidence of diseases for which surgery might be conSidered There are several categones of surgical procedure (other than hys'len!!etunills. lI.lIIII tonsillectomies) that are often performed unnecessanly
...wllllltfor Tanco to
IE)
results when potassium chlonde IS used? . hlorrde an effective means for preserving ammal hides Are the chemical properties that make potassium c . d. ? the same as those that make common salt an effective means for olng so.
(e)
(Dl (E)
For certain surgical procedures, It IS difficult to determine alter the DpIlr.iolI......... were necessary or whether alternative treatment would have succeeded With respect to how often they are performed unnecessanly hysterec:ton~ies, 1If~.~ tonSillectomies are representative of surgical procedures In general
Argument Construction
Sltulltlon
)llIIIeIllI"l
ew environmental regulations will increase the costs of disposing of the salt water that re ults from the use oflarge amounts of common salt in leather m~nufactun~g. The manufacturer is considering switching from common salt to potassIUm chlonde, because the by-product of the latter could be reprocessed to yield a crop fertilizer, wIth little waste left over to be disposed.
The frequency of certain surgical procedures. e g.. hytl1a:etlilllli., and tonsillectomies, varies dramatically by geographical rq:ioJJ.. II."~ conclude that the disparity is to a large extent the result unnecessary surgeries.
Reasoning
What additiollal inftrmatioll mUll b( I1Wfir t.'hisamdMI.
different factors in different region InJghtreasonablym_arlilr"'rW1 Diseases or medical condition for which these :~::::~:: might be more common in one geographicalua of such geographical variation in the mCldence of lpC1'tiIJI_ ruled out, it would be ri 'k)' to conclude that the YalriatiQD surgical procedure .
In orda 10 delamille whelha it would be profitable to swilehfrom using common salt to using pollUS/um chloride, which ojtheJive questiolls does the manuftcturer NOT need to answer? The
chemical properties making potassium chloride an effective means of pre erving animal hides mIght be quite different from those that make common salt effective, but there is no partIcular reason for thinking that this would impact the profitability of switching to potUlilum chloride. The relevant effects on the preserved hides might be the same even if properties that brought about those effects were quite different. Thus, without more rmatJo/l than ii provided in the passage, thi que tion is irrelevant.
. . . . . ~..... diIp al co t that would be gained by switching to potassium chloride CllllIlIIJad out If the co t of pota sium hloride needed far exceeded that for common salt. chloride would force the manufacturer to replace the equipment it uses it might be Ie profitable to switch. . " '••tle
my litt! Wi te as ociated with using potassium chloride in o al are ry high due to environmental regulation , it might
oflllO~ullum
hloride looks substantially different from ed, then th I ather might be less attractive to FJI1'"Q~mlicl of wit hing to potassium chloride.
h t pota ium chloride and salt are
,4&,_11
an i su as to whether :~lUiy. n I u of effi 'ti ne t
p_.,
Situation .Iu.tlan of • PI.n
A
B
c D
E
This statement undermine uch a condu Ion smce JI: cilII:$a JI"I-••'" reducing unnecessary procedure . ts the geographial YarilIIioa - --, Correct. This statement u~. . this POssiI;~~ of certain diseases-and b)' eliffiper£ormedmattng . JD unnece san' surgeries are bemg lystelllletrll." The argum~nt is concerned onl With h surgical procedure are irrelevant. . . 'Ind'I ate that there This intormatlon __ • • would m · surgeries-but t h1 intended conclusion. The argument in irrelevant to the condusialt.
meat However. as per capita Income In
!
___ consumplJOll 0 f meat has also risen toward thet world ~::=:::;a~~1IOlII ..... per .......-pn and und of meat. Therefore. Since per capi a income ::IJOl:J1:Ids: allraln to produce one po Gortland will soon have to Import either """ WIll not Increase. oduc ::~:: gram pr UV"
na
mplJOn on wInch the argument depends? o t1and will not decrease substantially. G . , tDltM 11Cf1!11"e • devoted 10 gra n production In r tant dunng the country's years of growing PGlNJlltion aI Gartland ha remained relatively cons h me acroSS all income levels. I t 1111. Dlr (illlll:aconsumption of meat In Gartland IS rough y e sa Gnrtlallll neither meat nor grain IS subject to government price controls. . ortIand who mcrea e their consumpliOn of meat will not radically decrease their consumption
57.
~~ \we Je-n1iegatun lIIIcIleI•
1Ii
The response of highly automated circumstances IS unpredictable systems COI1lr........... CGlIIIDII.OIIIII_II1_ WhICh of the following conc us 0 ns can mas automated nuclear·mlssile defense sys em controlled prOQefly be by .....""a
com_ ~::~:::~
Within a century after ItS con srucon a nuclear war.
(A)
(6)
(e)
e
;f'Illlill ys em
The system would be destro-ed 1 I a e(p OSlO It would be ImpOSSible for e sys em 0 ds of a nuclear weapon.
(0)
Whether the . system would respond nappr pna e the localion of the blast.
(E)
It IS not certam what the system's response 0
ge meteorite oa:uned
o a
ge meteonte
did not plan for such a contmgency
AIIf\IlIlentCon ruction
ounlry prevII>U Iy elf. 'URil ienl in ~min ~nd meal will soon have to import one or th other of holh hel.lu CiI' ",n,umpliol1 of mcat has nsen as per capita lficome has " It lake evel.11 round, of ~rail1 to produce ol1e pound of meat. Whal ndlilol/
Inlill bt lrut jor
UIIII'II(lII .lh"l1l tlu .lll<.lgl· del'otcd
10
d nllfi Ih I
Situ
lun
ReasonlOg
Ulllplll1Il II loll IIIllS" "Umpllon of ra III .
A meteorite e.xplosion equi 'a1ent to n century. Automated <) tern, on lled occurrence unpredictabl. What conclusion «In b dratrn oJ of the y tern to un pl(tt:! ~ru~'lmrn.,~ information is a\-ailable bout h ~ th circum tances, that i<, tQ n e fit h t
A
The fact that the ~ -tern', tepon'e uld .1 re ponse would actually be inappropna. It told whether any p.uri.:ular m t rit nt ","1111,1 produlx
B
The premises offer no e\;den, that th
D
The giyen infomution i, n t tell between .1 nuclear blat and a rneteoril[e eJlJl!clSion. '0 information is, iy n in th ... lI.llDa....
gram; it assumes only that the
I' r l.11'1I11ll1llI1U. 11111 l'''I'UI.1I1011 Iel'ds, "1 mIL m uml'lloll III ,.,"cllcr "II 110 I 'II llslH j' 'h' ullon I)\ 'Income level. f, rIOI'll olllmi • II l.mllllllIcl','1llI on.1ll .Issumption . 'Ihout ICII"CPlOIIlCl'lv
Argument Construction
Iht (ollr/miol/ 10 /It' Ir/lt'? Meat consumption is rising.
h I b"UI I(ralll lOll 111111'111'111 A ,harp reductIOn in the amount of grain directly n um d by IIlCdl edltl ",uld ""lIl'el1,ale fi,r incrcased meat consumption. making "II: CUln 1u 11m I I e. It I'e0l'k did r.u!ir.lily decrea,e their grain consumption. it might t b nary 10 Imporll(rdlll or Im.II, SIIKe the argument condudes that the mporr r n arv. 11.1 ullle Ihal direrl wnsllmrtion of grain by those who begin • t m I will nol plllnl«
'
.........
MeteOrite explOSions In the Eartw approximately the force of a ....,;I~::~
I bep;in to e;\{ me.H do
IV III
E
Ii_
the y lem. orr ct, lhi, statem nt pro e pillion" )"lJ be un;:ert.un it
The correct ,msw r i
WIll remam so. In the Unrted States this year, -::::::I"':O.~lJb/e::: IIICl projectiOns are thlt it d even though the number adults who i If by tobacco termers hIS Increase ,
01
e In tobacco sales and decrease mthe .IIlI'MllWlng, if true could explain the Simultaneous Inereas ••f.Ultswho smoke EXCEPT: t smoke IS greater than the number Duirfn/l tIIi year, the number of women who have begun 0
01 men who moke this year IS greater than the number 01 adults s
quit smokulg t number of teenage children who have begun 0 hive QUit smokmg dunng the same penod. o have begun to use chewmg tobacco or snuff IS greater Durin& thiS year, the number 0I nonsmo kers wh thin the number of people who have qUit smoking. e people who have continued to smoke consume more tobacco per person than they did In the past. c.M_ of the garettes made In the United States this year were exported to other countries than was the
61.
Amilepost on the towpath read -21reasoned that the next milepost f on the ide facinI h d h orward on the the pat an t e other. Howeve r, the milepOst one P8th . .fur1IlIr . mile Which the following, II true, Would explain the discrepancy descnbed (Al The numbers on the next mI Iepost had been reversed (8) The numbers on the mileposts d m Icate kilometer t (e) The faCing numbers Indicate mI s. no mite . I es to the end of the ath (Dl Amilepost was missing between th t p not mile . . e wo the hiker encountered (El The mileposts had originally been p t I u In pace for the use of mounta Argument Construction
01
~"
I,
Ie..
Reasoning
II t year m nt Construction The number of adult Americans who smoke has decreased, but the amount of tobacco old has Increased. Wh, hpomt does NOT htlp explain the apparent discrepancy between the decrease In adult _II and the inmast in tobacco sales? Many possible explanations exist: an increase in teeIIa~ mo~rs. tobacco's use in products that are not smoked, increased consumption by the remaining smokers, an increase in tobacco exports, and so on. To answer this IOn, u e the process of elimination to find the choice that does OT provide an Ian hon.
......
anformatlOn is compatible with an overall decrease in adult smokers but also with n adult smok N. How many new male smokers were there? How many women the "informal' • the . . . .linadulnot told-so ke . Ion prm'd I ed'In t h"IS chOICe cannot explain • t ;"lClreased mo r Butb In e It IS compat'bl .h I e Wit an overall decrease it is not a ." toaoale. ' uId 0 tthe de lin'In aut d I smokers and so could explain the
"'1I1llII::;Wt
moker auld be explained by an increase in the use of
tim wh n fi wer adult Americans smoke.
-:::::'.:::!
A B
.wI
A hike~ sees a milepost marked 21 on one Ide and next milepost to read 22 on both 'Ides. However, the a What explains the discrepancy? The hiker as ume that the nul~:::::~= distance she has traveled from her journey' begmmng and d the dIstance to her journey's end. That i , at the first mil t come 21 miles and has 23 mile left to go. In fact, the nlUDIlbc1rs of her reasoning. At the econd milepost she has 20 miles 24 miles.
Reversing the numbers would not make any difference; according reasoning, both numbers would be 22. What unit of measurement is used i irrelevant to the hilu:r
t.
C
Correct. This statement resolve. the di crepancv betlfteD the the hiker's expectation about them by howing how the bikt:r misiDllIIIJ. . . . .
D
A missing milepo t would not explain the di repan H 22 on each side, its di crepancy with the 2024 milepostwodl
E
The numbers are mea ure of di tance. not time
The correct answer is C.
the IpOdc.l'#
al produce antibodies that destroy the VIrUS by _"Illl'I"s:-"'III~11S ofmiCI tYPIC Iy ally develop keratitis, a degenerative ~,herpesWus gener I .........MIaI ••feClId with a of cells," this part of the eye close Y resemble 11II"411I1118. Sh:e protetns on the surface es of keratitis are caused by antibodies DItlVIrus surface SCientists hypOthesize that these cas
•
t to the SCientistS' hypothesIs? . . ble proteins found In variOUS organs of mice. telns that closely resem wus have su rface pro to herpes infections, and these mice rmce 1tI8t are unable to form antibodies In response • IICI........ t ghl the same rate as other mice. • .-.... a rau y d I keratitis produce as many antibodies as ... 11I11 are nfected with a herpeSVirus but do not eve op • • :IlId rmce that do develop keratitis. esponse to herpes Infections, and these mice rna that are unable to form antlbo dles In r • • these Infections without ever developing keratllis. 1tI8t have never been Infected with a herpeSVirus can sometimes develop keratitis.
,..,flIIlCIwq. If true gM!s the greatest additional suppor
63.
Tradllionally, deCISion making by 1I1llI1aI__ Intullive deCISion making How &211 thIl did most mlddle- or 10we·r'leve~r. a recent study .... effective than careful, This confirms a reasonmg
methOdI~nagers
The conclUSion above is based 0 ..... n.."Ich of the following iISSUn1IItionIl (A) MethodICal, step-by-step re asonlng IS Inilllllr~ for (8) Top managers have the ability to ~1lB"a~.I_. making deciSions. use either IntultNe re8SOnlng Of mellDdiel" tI"~ (C)
The decisions made by middle- an d Iower~evel rna reasoning as by uSing Intuitive rea sonrng. nagers can be made as
(Dl (El
Top managers use intuitive reasonrng In k ma Ing the ma)Ol'lty of their tltc:isiarll. Top managers are more effective at d eclSIOO making than I1lIddIe- Of lOIIIIer-lI!UIlllll!. . . . Argument Construction Situation
Intuition, med significantly more by top managers than by miildlemanagers, IS found to be more effective th srep-by-step lalDlliIJI decisions. an
Reasoning
Wh~t .assumption does the argument makoti The study shows
. . . . .~It.I.IIIItIDn cientt
hypothesize that the reason most mice infected with a herpesvirus ~evelop keratin, a degenerative eye disease, is because the antibodies from t~e mouse s Immune stem attack proteins on the eye's urface by mistake. 111lS IS because prot m in certain eye cells closely resemble the virus proteins that antibodies attack.
Wh h ophon, iflrue, wouldprovide the gr.·alest additional supportfOr the scientists' hypolh su attempting to explain wkl' keratiti.r o(mrs? When x is thought to cause y, it is Important to consider whether y occurs even when x is absent: if it does not, that gthen the notion that x is causingy in those cases where they occur together. If the lentl •hypothesis is correct, and the antibodies produced by the mice's immune ll:m to combat the herpesvirus arc causing the keratitis, we should not expect to d tin m herpe -infected mice whose immune systems are not producing those tlhodl Thu, th scientists' hypothesis is supported by the finding that herpesml that do not produce antihodies do not develop keratitis. n rn only the herpesvirus, so this choice is of little, if any, relevance.
"iId~1 hlu no be rmg on th hypothesis, lilr it concerns merely the connection between the pm nt of th h rpesvirus disease. whereas the hypothesis concerns the antibodl nd th d vclopment of keratitis. r..,rdWillL Wldcrmin the hypa theSIS, . for I'1' .antlhodles ' . produced .III respon e to ng th keratltJ one woul·'· •. h i ' - I u expect t at at cast III some of t lC beq.~iri" donotd v'lopkerauus . . .It.IS Ilecause they arc dchclent '. .III
intUItIOn more in deci ion making than the other ~ drawn that intuition i more effective. But the staIIld pam_I" inadequate support tor the conclu ion. so it is reasooahIre must be based on an un tated a umptlOn, such as the lISllUDlptiaal • •'. . . . when employing intuitive deci ion making, make _ eftid~ dm_1IIt1 middle- and lower-level managers. .thout some such 3SSum,... .,. . . . . . fails.
-"-*illll
A B
c D
E to develop antibodies aga inst herpes t mildly weaken it, lilr it suggests that rganl m's re ponse to the virus.
the::'::~=
\.yhile the argument is consi-tent with thi idea, the inapp·lOFm_ _ .I~~"" is not assumed. Top managers' ability to witch deci-ion methods than the other. The etfectivenes of deci ion-making methods. not the i. the subject of the argument. The argument would not necessaril f.ill if thing iPl;~P.!C~ assu med-Ior example. ifit were a umcd that decisions. Thus thi -tarement d not ba to to be added as an a umption to the fa 1Ift:mi_ inadequate unles om addinonal aSS1ll1l1jptKlIl'''' Correct. This is the be t choice ar~ument would fail.
The corr ct an
er i
roduce high Yields 01 the same crop year eded when f.rmers trY to p Iy their crops and to rotate their . . . ..,., estic/des, ne larmers to dlversl suPPlies Experts therefore urge I crop larmers mus pvernmental pnce-support benefits or a • IIVIA! ye.rs
t have produced that same crop lor the ?
h I lIowlng conclusions. "'rttlltllll'Hmts above If true best support which 0 I teo .. ., . . t efforts to reduce water pollution. I rm prices work agalns The rules for governmental support 0I a t·l· s and pesticides is to take larmland out 01 lIutlon Irom ler Ilzer The only solution to the problem 0 I wa ter po
Cl (01
production. .' s thus redUCing costs lor chemicals, but Farmers can continue to make a profit by rotallng diverse crop , not by planting the same crop each year. kt ssible for farmers to reduce the application of New farming techniques will be developed to ma e I po fertilizers and pesticides. I h t re not high enough to allow farmers Governmental price supports lor farm products are set at leve s t a a
65.
Which of the follOWing most 10RICaily COIIlIlII1ln Utranla was formerly a maJor petroleum
(e)
(D) (E)
most of the Investment In new 011 fields
Re;uonin,g
Farmers are urged to rotate crops annually because the chemicals they mus t use when ontinuing to produce the same crops pollute water supplies. On the other hand: farmers may receive federal price-support benefits only If they have been produclllg th arne crop for the past several year .
t
men cann?t m~ke a profit by producing the same crop year after year i rmltlon glv n. In the prem'lse s. 1he .IOIOrmatlon C • • gIVen suggests that t h'IS
d.Jletwillybe fa1&e,
In
Reason ing
Which ofthe optiom would mosllogically compltlt lhe aJ'I"""mtlr" the conditions that led to Utrania' no longer bemg a ma.JOr III__ investment in new oil field due to a stagnant economy lIand::~::':=~ passage then says that due to changed regulatory and ec:n1'*i!hK now be rapid development of new oil fields. one±c'r an increase in Utrania's oil exports. To logically com~ IbItIIlp." explain how oil export might not increase even wbm decrea ed oil export ha been removed. 'UJ'PC"IC tMIC1_• • • • • • oil consumption. A dramatic increase m tIH: IalIt al'car • reasonably be expected to ignUi~tlyiDCllellC_ _ could eat up the added oil producnon from
If;'''.
A
B
these farmers would benefit by price-support measures C
nclu ion about farming techniques other than crop ' farming : ' techniques . Ielrly i tlng and not new or yet to
A country that had been a major oil exporter has seen I decades due to economic stagnation, a failure to invest m new DCII11, depletion of its old fields. But looser regulations and an rapid development of new oil fields in the country
alii.
~lIputl cited in th pa sage believe that the rotation of crops is the solution not the removal '
ilI"
Argument Construction
tarement properly identifies the conclusion supported by the evidence.
liIDlADd from productIon.
IR
many of the new all fields In Utrama are likely to be as productive as those th..t_-.i period when Utrama was a major all exporter
Whal conc!uJion can be drawnIrom thiJ inftrmation? Farmers wish to receive the priceupport benefits offered by the government, so they grow the same crop for several y ar . In order to continue getting good yields, they use the high levels of chemicals ne e ary when the same crop is grown from year to year. The result is water pollution. lb gov rnment's rules for price-support benefits work against the efforts to reduce wat r pollution.
eo.....et,. lbl
•.
Ut rama IS expected to come new technology IS available to recover all from old 011 fields formerly ,'egardll,••
Argument Construction tlon
~
regulatIOns Inhibited Investment In new eXpOrter but ::r fields became depleted. Utranla's c 011 fields In con '11 urrently ImprlMn regulations. WI undoubtedly result In the ra I I conclude that the rapid development of pd development of new new fields WIll result 1\ hiaher (A) the price of oil is expected to rem I . aln re atrvely stable over the IlI!llt _ ... _ (B) the Improvement in the economic Situation I proportion of Utranians who own automobd~ Ultama IS expected
Situation
to get out of debt.
.......::!!
D
n~ information in the two statements about the E
This choice is incorrect. There i no reason why exports from increa ing. Correct. An increase in car ownership wo:,,~!d~::; supports the claim that oil exports might If anything, thi ugge ts that oil exports• • completion of the argument. The advent of new rechn.oIoIn' allow.... depleted would mean that increase. Thi doe not help the new field
1h c
wu.--
t ........ :11
ser) would always be used less carefully • Hh11t ......... land held In common (that's, open to any uammon land because the benefits would • •• _..... ted to overuse c Id b • •,ruing land Each rancher would be temp th t results from overuse wou . e spread indMdual while the costs of reduced land Quality a grazing land with 433 million acres of , res of common users But a study comparing 217 million ac beller condlliO n. pihg land showed that the common 'and was ,n ost useful In evaluating the significance, In relation to answer to which of the follOWing Questrons would be m rdm cla,m , of the study described above? ? d both common and private land. In common land over uSing private land for W Old any of the ranchers whose land was studle use (81 Old the ranchers whose land was studied tend to prefer us g grazing' . . I It to the common land before either was used Ie) Was the private land that was studied of comparab e Qua Iy for grazing' . Were the users of the common land that was studied at least as prosperous as the users of the pnvate (0) land? Were there any owners of herds who used only common land, and no pnvate land, for grazing? IE)
67.
Acompelling optical IllUSion called slowly the larger the objects are. T~e illUSion of ve ily :::I=~ With a small car approaching I b erefore a mOlon r . sound to be lower than It wou The conclUSion above would be mare properly drawn If It were made (A) truck's speed IS aSSumed to b I e ower than th (8) truck's speed IS assumed to be th e Car s k'. e same as the car' (e) truc s speed IS aSsumed to b h , e Igher than the car's (D) motonst s estimate of time available IS trucks approaching aSsumed to be more accurate with (E)
motonst's estimate of time avail bl cars approaching a e IS aSsumed to be more accurate With
Argument Construction Situation
Argument Evaluation Situation
Hardin claims rhar common grazing land is used less carefully than private grazing land because each rancher tries to get the most benefit from the common land, thus overusing ir. Comrary to this claim, a study comparing common grazing land with private grazing land found the common land ro be in better condition.
R.asonlng
What other mjormatiol1 might be IIseflll il1 evaluatillg the stlldy? The study finds that common grazing land was in better condition than private grazing land, which seems to undermine Hardin's argument. To decide whether the study finding really undermll1es the argumem, we need to ask whether the comparison between the common a~d the private grazing land was sound. In determining this, we need to know, not Just how the lands compared after grazing use, but also how the lands compared before that usc. ~or example, the private land may have been greatly inferior to th~ pubhc land even before grazing began. If this were the case, the comparison proVI ~n.g thedstudr findll1g would have been invalid, and the study finding would not ucce 111 un ermll1l11g Hardin's argument.
not matter whether ranchers used one or both fI ........ oflandi matters. types 0 ands; only the relative condition of
1"'lDCber.·~ferencesare not being questioned. n roses the i sue of wheth
• 14IltWireWlth like.
h d er t e stu y finding was based on a valid
1I11Televant to the condition f h IInPIftIll"ftnchers u d l O t e two types of grazing lands. e on y common lands b . . ecause It IS the relative condition of
Reasoning
A
B C
D
E
~~ optical illusion makes objecr
appear to be mov1I1g more 51 t .IS concluded that a driver's estimate of rhe time available to With a small car approaching than wirh a large truck approa hm What underlying assumption is made in th( reasoning? From the glv opucallll.uslOn, we can lOfer that, to a mororisr crossing a hlghwa approachmg at 60 mtIes an hour seems significantly faster rhan a I approaching at the same speed. The motorisr's estimate of time avail thus be larger for the truck than for rhe car. The assumpllon t ted in the passage, is that the car and truck are approaching al t
If the truck is moving more slowly rhan the car, then the driver' perception I affected by the optical illusion. Correct. This statement properly identifie' an a' umption impliCit in the rasollirJi~ If the truck is moving faster than the car, rhe moron t might judge that vdlid. moving at the same speed-or even that the truck WI movmg t'astu It does not matter for which type of vehicle rhe dnver' estimate oflr: this assumption is nor helpful in explaining the.condu Ion that the iIlusiaQ appear to be moving more slowly than mall obJects. This contradicts information in the pa sage indicatmg that the Ial)DII distortion in the perceived speed.
The correct answer is B.
til."
lor a promotion period when the prOduct oduct to retal Iers . I d '"IIIftlr5 5OmetiII'II!s discount the pnce 0I apr d matlc increase mamount 0 pro uct sold by .. 1IYl",1ed to consumers Such promotions 0 Iten result In a racould often make more pro fi t by not holding the nuflcturers to retailers Nevertheless. the manulacturers I
otiOnS
above about the manufacturers' profit?
h of the following. II true, most strongly supports the calm I cturers to retailers is carefully calculated to represent The amount of discount generally offered by manu a d t the minimum needed to draw consumers' attentiontlsing to the pro uc . prrces to consumers IS about a week, discounted d or many consumer products the perro d 0 f aver not sufficiently long for consumers to become used to the sale prrce. r d I . t d ced the purpose of such promotions IS to keep the products In the ror pro ucts that are not new y In ro u , . om etlng roducts minds of consumers and to attract consumers who are currently uSing c P p . WI Dunng such a promotion retailers tend to accumulate rn their warehouses inventory bought at discount; they then sell much 01 It later at therr regular prrce. If a manufacturer falls to offer such promotrons but its competitor offers them, that competitor will tend to attract consumers away from the manufacturer's product.
69. When people evade mcome tax b lawmakers to raise Income tax ~:te~ not declanng taUb e income.iR~:: This, mturn, encourages even m . which causes the tax burden 0 . ore taxpayers to evade IRcome
The VICIOUS cycle described above couId not result unless wht h of IIle foIlMillll (AI An increase In tax rates tends t f • • • • ""'. mcomes. 0 unction as an IRcentive lor taxpayer
(Bl
Some .methods for detectrng tax evaders and thus brmg In more than they cost , but telr hsuccess ' me raterecovenng vanes from yea
(el
When lawmakers establish Income tax rates IR order to generate a r ....bin aII ow adequately for revenue that will be lost th raugh evasion No one who routmely hides some taxable Income can be Induced by a Iownll such mcome unless fines for evaders are raised at the same time una, Taxpayers do not differ fram each other With respect to the rate of taxation thsIl ....r...,. . taxes.
(Dl
(El
Argument Construction Situation
Argument Construction
atJon
During promotion periods. manufacturers discount prices and dramatically increase the amount of product sold to retailers. However, manufacturers might make more profit without the promotions. How rouldpromotion periodl cut profitl? It is stated that promotion periods result in mcreas d product sales to retailers. How could such sales decrease the manufacturers' potential profits? If retaile~s buy more than they can sell during the promotion period, th Will tore the surplus 10 warehouse and sell it later at the regular price. anufacturers lose their normal profits on these sales; moreover, the manufacturer Will . Th e resu1tlOg . losses may be greater thannot b filling orders . while . the surplu eXists. any gam from mcreasmg sales or winning new customers during the brief promouon penod.
amountthofdi count Isho uId Iead to greater profit for manufacturers, _eat lnlmum IIDt explam e pot ntlB loss of profit. lDOt1lm penod favor manufacturers because consumers do not become • 'lwier
~ contnbute to higher, not lower, profit.
tifie a factor that strengthens the argument. lII'IllIItIiDDi
In 10 Bof ustomers to competitors, this statement
Reasoning
A B
C
D E
When some people evade income taxe by hiding taxab must be raised, placing a heavier tax burden on honest tupq_ turn, encourages more people to conceal taxable income. What mUlt be true in urderfor this 'Y'ltto~ConsUler drive this cycle. It i said that tax evasionJon 1egisla: the loss of tax revenue . This would be trUe only ifllWlllllllka....1lII•• income tax rates to e tablish, had failed to slIfficic:ntIJ revenue that would inevitably be 10 t to evasIOn.
Any incentive to increase pretax incomes would counter tIM: nIIlllIllllt. and thus break the cycle. Success in detecting tax evaders, no matter h
variabIa arl.HiiJlct..
breaks the cycle. Correct. This statement properly identifi the UlUIIlIrMk-fail to consider the revenue 10 t to eVBSlOII when thIlycll• • tax rate that drive the cycle. Higher fines deter evaders and th Thi is not an a umption of tile 1I1l[U8:. . ..
t.ltra.~
Either everyone would e The correct answer i
. . . . . . . ._"" Jnd Edition
major . cause I dof crop t wn msectici de agalnst bollworms, a. otton bloenglneered to produce I s a the plantings are being senous y amaged by ..•....;rned little bollworm damage until this year. This year ·stance to the cotton's Insecticide. Bollworms Iy developing resl . . . s Bollworms, however, are not necessan t d throughout cotton·growlng regions. So It IS likely I.,nled on corn and last year more corn than usual was plan e I I , IS simply being overwhelme d byco rn .bred bollworms. that the cotton
Because postage rates are nSlng H ' orne one·hal f the num ber of ISSues It publ h Decorator mqatine ..... )l "'laIity per year, an d th e subscnptlon pnce IS IIes each year The ... advertisers Will be lost If the magazl:' noIt change Market res_ell espan IS Instituted Which of the follOWing, If true, prOVides th t the plan is instituted? e s rongest evidence that the malraDllI's
71
In evaluatrng the argument, which of the followmg wou Id It be most useful to establish?
IAI 181
Whether corn could be bloenglneered to produce the Insecticide Whether plantings of cation that does not pro duce the msectlcide are suffenng unusually extensive damage from bollworms this year . Ie) Whether other crops that have been bloeng I·neered to produce their own insecticide successfully reSist the pests against which the Insecticide was to protect them IOJ Whether plantings of bloengineered cotton are frequently damaged by Insect pests other than bollworms IE) Whether there are insecticides that can be used against bollworms that have developed resistance to the insecticide produced by the bioengJneered cation
(A)
(B) (e)
(D)
Argument Evaluation SItuation
(E)
Although plantings of cotton bioengineered to produce an insecticide to combat bollworms were little damaged by the pests in previous years, they are being severely damaged this year. Since the bollworm breed on corn, and there has been more corn planted this year in cotton-growing areas, the cotton is probably being overwhelmed by the corn-bred bollworms.
Reasoning
[,/ r1'oluoting thr mgu1lle71t, which question would it be most useful to have answered? The argument states that the bioengineered Cotton crop failures this year (1) have likely been du~ to the increased co.rn plantings and (2) not due to the pests having developed a res!'tance factor.to the insecticide. This also implies (3) that the failures are not due to some third
It would be useful to know how the bioenO"ineered Cotton is fari g' . h f h' If h · · . . n m campanson to t e rest a t IS • _11'1 tton crop t e bloen~lneered cotton is faring better again t the bollworms that fact would PPOrt th argument belause It would sUCTg' nh l ' ". , •uu:r L __ han d,th b"lOengmeered cotton is .. being es at t Ie IIlsectlClde IS still combating bollworms. If.' on I .aDa, that h h' . more severe Yravaged by bollworms than is other ugg t at t ere IS some thud cause that is primarily at fault. >
would probably b u eful information to t h o s e " , . DiOllllpnelClrec! otton But wheth h tf) Ing to alleViate the bollworm problem In . er sue corn could be dev I d h b ~llon·__rtllla blDenglne n:d cotton to be d db e ope as no earing on what is
boUwo
d
amage
y bollworms this year
nn amage on non blo ngineered c o '
. :::::~an~general i Imply worse than Usual tton IS worse than usual this year. then s . 'd d the bollworm atta ks on h . • a pestlCI e resistance doe not nee rqllWll thu hi b t e blOengmeered Cotton. n bl ngm n:d to resist h th th lam I. true ofth'pests ave not successfull}. resisted ·'-lIaillll~:ereld tton ha . d IS Cotton. Furthermore. the t;lCts r 81 te bollworms. Clllrllge blo "gin r d h IIII Qf'COI:ll:ln /JII) h Cotton as no bearing on why t 1 ar than in the past.
;
VY:U11
allevl the bollworm problem in U ting th argum nt. Ev n if th tan to th in t1 being U d thi
With the new postage rates a tYPical ISS d .I h . , ue un er the propO ed pia ""',....__ .. "'lout .. mal t an a tYPical Current ISSue would. The majonty of the magazine's subscnbers are less concerned about a pO!ISiIlIUt_ _ of the magazine's articles than about a POSSIble loss of the current hg Many of the magazine's long-time subscnbers would conbnue their sublSCriPtions price were increased. Most of the advertisers that purchase advertiSing space In the magaZIne w amount on advertiSing per issue as they have In the past Production costs for the magazine are expected to remain stable
Evaluation of a Plan Situation
In the face of rising postage co ts. a magazine decide to cu issues it publishes a year, though the quality and quantIty of subscription price will remain the arne. Market resc:an:h iDdan dllt1l11, not cost the magazine any ubscribers or advertisers
Reasoning
How might thr plall causeprofits to Jedine?The magazme p reducing costs. What could lead to lower profits desp1te decline if revenue i 10 t-and revenue will be lost ifadYertillall ...... l~1 amount on advertising per i ue when the number Therefore, the adl'ertising rel'l:nue will be cut m balfllllia . . . .1Ij profits is likely to result.
A
B
C D
E
If the number of issues is cut in half and the postage-ntelllB~peritd"I""JII"" . go down. 1h'I taternent d not .. mailing cost sull profits. This tatement is irrelevant to the magaziM and quality of articles will not c~ange. willI. . . . . The passage a) that the ubscnpnoo pnee provides no u eful evidence. Correct. This tatement properly lden. ...._ . IVill not be t hi shows that d,h'erhseTS aclvatising will be I t. Stable production woWd
The correct ans"....'rill U
of people and pays most o.f them at the ds loys thousan ht r of baSIC household necessities em P f the minimum wage rate t a Increased the rate Yet followmg a federally man~ated ~t~~::r~~sed markedly. rating costs considerably, the retailer s pro h apparent paradox? t tIIil!tt of the followmg, if true, most helps to resolve e a roll expenditureS; yet only a small percentage of Over half of the retailer's operating costs consist of p Y those expenditures go to pay management salaries. r who depend on the earnings f The retailer's customer base IS made up primarily of people who earn, 0 a (81
(el (01
(EI
others who earn, the minimum wage. d ubstantlally after the Increase In the minimum The retailer's operating costs, other than wages, Increase s wage rate went Into effect. When the Increase In the minimum wage rate went Into effect, the retailer also raised the wage rate for employees who had been earning Just above minimum wage. The majority of the retailer's employees work as cashiers, and most cashiers are paid the minimum wage.
The colton f,arms of Country Qbecame 10 Consequent y, colton Prices fell. Th 25 percent of their colton acre e llDVernment triId:~:::::! farm. age out of production direct
produetNl.~'_'~==
The government's program ,ccessful If su Will at b is the best baSls for an explanatl f . n e a net burden on the "1111.... on a how thiS could be so' .. (Al
Depressed colton prices mea t n operating losses for cotton farms and taxes on farm profits.
(8)
Colton production In several co t program went Into effect In Q. un rles other than Qdechned slightly the ~ bU'........
(Cl
The first year that the support-paym t its 'evelln the base year for the en program was In ellect. cotton acre. program. The speCified maximum per farm meant th t f per acre for those acres that were wthd aor f very large cotton tarms the 5IJPporl PIlIllllIIIIiII . . . . I rawn rom production than they were to _ _ Farmers who Wished to quahty for support p t .. from production to grow any other crop. aymen s could not use the cotton atleap .
(D)
(El
Argument Evaluation Situation A discount retailer of household necessities pays the minimum wage to most of its employees. When the minimum wage rate went up, the retailer's operating costs rose. However, it profits also rose. Reasonmg
B
]3.
What information helps explain the paradoxical situation that the l-etailer's profits rOse even though its costs rose? Consider the nature of the cost increase: wages have gone up. If the retailer's customer base includes many people who earn minimum wage, their buying power has risen with the minimum wage and they can spend more. This would explain the rise in profits.
Evaluation of a Plan Overproduction of cotton had led to falling price. To boost prices offered direct support payment to farmers who took 25 percent of out of production. Surprisingly, thi plan. if ucce sful. will not be budget.
Reasonmg
What would explain why the plan wi/! not a net burdm t It will not if as much or more will be added to the budget revl:nue as the amlllUl~ support payments. What government revenues would account receipts? If, when cotton price- were falling. the cotton farmers were paying lower ta.xes to the government. When cotton pnees iDe. ._ (I. . . . cotton is offered in the market) and cotton farmers make gJelter pnJ6e.. government is able to tax tho e profits. In the program. compensate for the added expenditure.
This tatement helps explain the impact of the wage-rate increase on costs but does not explain how r1Sm~ costs could lead to profit.. orrefict. This statement properly explains the surprising impact of the wage-rate increase on pro ts. Hth r tailer's other costs also rose, then the paradox of the reta'l' fi' m tenous, I er s pro ts I even more ing other wa~es contributes to even hi her o ' . In explam how higher costs could lead to profits~ peratmg costSj there IS no information to
fbl._._
Situation
A B
Correct. This tatement properly explain - how no net budget The decline in cotton production in other countrie does not Clll.iIl
C
a net burden on the budget. The difference in cotton acreage doe not explain how tbI:
detai1 bout minimum-wage jobs does not e I' h . ""_, ""'.'.are ri ing. xp am ow the retailer could be gaining profits
B. D
E
the direct-payment plan. This statement spells out an implication of would not be a net burden on the budget The speCified maximum per farm. which
&'l-~
tM:p~I":O:~:~::=
farms. This does not help to explain ho farmers cannot use th idled unlikely to be a re nUll so~lJ!;C: JlI!~9
The correct an
.l.
ducts can generally command a , .dv8/lCe over competing :~:ed and companies want to make ~ 10 be fIlllckly ,UfP m possible price for such a u g incentive to quickly tile etitOrs a stron d .match the new ""f1~1Is ~'" roduct Will gIVl! comp fit from a new pro UCtlS to charge .. lIIe new P t maximize overall pro Consequently, the strategy a iii t posSIble pnce h f the following roles) boldt.ce play whlc a 1M'••lIIIt!nt above. the two portions In s counterproductive; the second t a certain strategy I • conSIderatiOn raised to argue tha pruents that strategy th t the argument recommends; the second first IS a consIderation r"sed to support the strategy a
i~===::::--:;0111'" "...-
"::::i
m.x1m
pruents th.t strategy. I t of a certain strategy; the second presents first IS a corIS,deration r"sed to help explain the popu an Y lIlat strategy th t has been used to Jusllfy a course of action; the 11Ie first IS an assumption, rejected by the argument. a 8ICOnd presents that course of action. th d f adopting a certain strategy; e secan presents t d The first 15 a conSideratIOn that has been use to JUs I y the ,ntended outcome of that strategy.
75.
United States hospitals have tr''''IIIionIIh ""IIl1MIl unrelmburse d care. Almost al .hospital bills. Recently, InSurer have patients to amounts at or below actual Which of the follOWing conclUSions IS best (A) (B)
(e)
(D) (E)
SU\lIIUIIld Although the advance of technology has
such procedures are out of the reach of ~ e:::::~"1i lUll' IICOI•• If hospitals do not find ways of raiSing additional some of that care or suHer losses rt they gJVe ilinc:clllll!! far - • . Some patients have Incomes too high for eligibility for "111111ll........ afford private Insurance for hOSPital care .. If t.he hospitals reduce their costs In prnuU4..... v·~·6 care lllSUialCt ClIIllIII. . . .1I reimbursement. thereby ProViding more funds for unreimlMned Even though philanthropic donations have traditionaly PnJVided SlIl,u..ilIifI donations are at present declining.
Argument Construction Situation
Argument Construction
l
I;'
Oite n, when a company co mes out with an innovative product, .it will price the . product as high as it can to maximize profits before the competitors qUickly catch up. But thiS is not a good strategy because the very high price of the new product only n ourages competitors to match the technological advance more qUickly.
ltuirtilon
,.IeI_nlng
Wh/(h oplion Iml dmrilJ(j Ihe rolfJ Ihallhe boldftce portions play in the argument? This type of Item concerns only the argument's structure-the way it is intended to work, not the qualtty of the argument or what might trengthen or weaken the argument. So even if a boldfi portion could be used by the argument in a certain way, all that matters is its acruallOtended role. The fact that technological advances tel1d to be quickly SIIrpassed serves pardy plain why many companies charge the maximum possible pricefor SIIch aproduct. In other words, the first boldface portion helps explain the popularity of the strategy p n d 10 the s cond boldface portion. The conclusion of the argument. however, is that the trat gyexemplified in this latter boldface portion is unwise. so the argument whole oppo that strat gy.
t boldfi portIOn c~uld. be used as part of an argument that the trategy nd boldfa portaon IS counterproductive, that is not how it is used here. ImIlll1lCiiattly U the word ImauSf and serves to explain the occurrence of what is nd bold portIOn. . . .-1111 th econd boldfa e por t'Ion presents t he strategy that the argument n I nt with the analysis of the reasoning presented above. umptlon r j t d by the argument; rather. it is affirmed in
PO,ll1buhe Ii
t b Idfa portion has been used to justify the ugh It lmpli s that this is part of the I I arly, th econd boldface KY, but rath r the mans of bringing DlIaina high pri ).
wS°
1IIII11IIIII'IbiMliD'vVO'
Hospitals have historically used the mama tw........hi the costs of unreimbursed care, and most paymg~~:::= hospital bills. However, health lDsuren han: 11 paid to the hospital to amounts at or below tilt IIlIIpM
Reasoning
A
B C
D
E
What conclusion can bt drawnfrom tIN i1r}nrdlmlDr._J_ with the recent change, paying pattents wiD 110 lautpi'li unreimbursed care. What will happen to UDlreiDma_M figure out a new way to cover the costs, orthq is to uffer the 10 e incurred by urm:imbuned
The passage doe not examine specific medical prou>\"a. 10 l"')dl available only to orne i not warranted. Correct. This tatement properly identifi aconch'5'll!l The passage does not di cu differen betMcn them can be drawn. The pa sage states that in uranc«: comparn any reduction in the co ts of health cue delMry1. a information i provided about the role of I~Ui. them may be drawn.
The correct answuis B.
- __
;mportant new research and accept as their field willi tile 10" of doi/Ii nbst WinS renown as an expounder of scienc • II1II,......... ..when any SCie I b e '-111l1li MIll sinIIar motiv8tiO/l Therefore, pulaflZer should no anger e regarded as a IlIdIInce most other scientists conclude that thiS po t fic popularizers are held by research scienlist nwhich sClen I s IIIIIInItion offered above lor the low esleem ,
. . . . .lUI.
. . . . . . tfIIt IOU
lBl (0)
sclenbfic research 's not a sohtary act,vlty.
but relies on active cooperation among a group of
Re earch scientists desire to do important new research and treat as colleagues just tho e who have a similar desire, When a scientist becomes popular among a general audience fur explaininl{ priOl:iples ofscience, other scientists have less esteem for this popularizer, no lon/:er re/:ardinK such a scientist a a serious colleague.
,
Wh,ch of the foliowlOg, rt true lA) (8)
(D)
lEI
Hit'lllir/' make about scientists who become popularizers? The wmmunlty 0/ scientists shares a common goal: to do important new research. What would cause this wnulJunit)' to disapprove of a popularizer and to cease to regard the populaflZer a a (OlIc.,/:uc' It must bc because man)' scientists believe that becoming a populanur IS inlOmpatihlc with desiring to do important new resean:h
t mak thl a sum~tion, of murse-hut it IS not an assumption on whIch the n tlOn I' {Ifl,all\ d,pcnd , !hc explanation wncerns the scienti ts' motivation. not their I dUlIll( r ar h
dJlCllloa -r
I
m nl proprrl Id nllti an Jn d prnd
,I
sum
t·
pion I1n
I ' .1
IV l1l
".
, prOVIdes the stFontlest
Scrap Iron IS now used In th . e proouc no predict that scrap Iron Will b rOIl hall e used e en more en_...... Furnaces that Process scrap -'-'~""'J the ......". Iron can be bu a h exc ange than can furnaces ha operated process rare
••
A country could mine it n. t deposl of exchange. :'\e"ertheles it u'lIOn re with minillnal CIIJ~."' 1 arce oreiem-vr"'ft-n:scr_.~M. fior recycling into steel· 'thi ~, teel :0" .~-'6" , earn; more l reIgn ~ IVhich statement supports the ""!Uy a'-buy; forelgn-ex i' ch ange re erve toro.:! ng ....._ bu• scrap 1IOD • WlKII It
lton ?re with ~nimal expenditure ot tore cxclIange. But if Iron Invokeslat I~ . expenditure oftore ~ the policy 1- Justitied.
A B C
D
, ,
I the explan.ltlon tor ,nentlslS
ned wuh Whelh r non ,il,1It i , · I f s Lin 1lllll'rst,lnd new researdl, hut r.lther o I nil I who rC)'e '1 I I p"pu anzers as colleagues.
can be milled
The pnce of scrap Iron On SUlI.ttar c• .,.-.,••• ernattona ma"~Country Y's lorelgn-excha '''''~ rose Sl(pllificclllllw ge reserves dr There IS Virtually no differ 00Iled SlIJIIitic.1I1ll1y Iron are. ence In Q a between steel pr'oQJlCecIl from
Reasoning
lentl
I I m nllgldv another r a on ~h.II slit'lltis's may reject a popularizer, but because it i not n Imp I 10 Ih pa aK, III not .lSsumed, f Ih. I lru ,II d nOl addres tIll" (Of<' iss ' ,. I ...._ Ul 0 t Ie argument: what 'nentists believe nof pupu Iam
C
Evaluation of a Plan
WhlJI IJHUmplio11 do r<'JfiJr(h
R sonlnll
for~~;;~::::::~ Ign l!XdIanae UliIU 1I:0Il""
thus produced earns more vast depOSits of Iron are wh
Situation
Argument Construction
n
Country Yuses Its scarce for
(e)
colleague those SCientists whose renown they envy research SCientists tend nolto regard as colleagues I ted any Important research a Clentl t can become a famous populanzer without haVing camp e . known as populanzers of sCience are not motivated to research Clenbsts beheve that those who are we II do Important new research no Important new research can be accessl ble to or accurately assessed by those who are not themselves sclent,st
Sltu.uon
77.
E
J--"r"lnI
\ ignificant increa e in the international pri IIOIJ W1JuId bea Ill........ significant drop in toreign-ex.:hang.: ~n means thatCOl_mgtlllc .._ _ important, 0 thi. statement pro"ide n ruppon Thi - statement helps to preempt a po' 'ible bi 11 sLrap iron i' interior to ~teel from iron reo But It equ~' chlmging the present poli.:y-namel th I red from therefore provide- no e,iden,e that uu1d distiDl:t1y polic~', .IS opposed to .:han~ing if. The incre,Ling use of s,fap Iron . r Ii i' an a .:eptable .tlrernall\ to red hom uon \\'h~ther O\lntry Y': poli<:\ III Correct. lhi, swemenr pro rll ideo
The orrecl an wer i E.
~_II1-""""s the pasSlJll? VI''''I''V 4) e systematically elongated. In EI Greco's 161 w",tInl-~ painter EI Greco /1541- d ~r European palntmg. Consequently, sorne l1li
,.. -".... I distortion of human figures was unprecedente e of visual impairment, th.at resu It ed In people Greco had an astlgmatlsm, a typ ntlngs. However, this suggestion cannot be "that tenstlc of hiS pal him nthe distorted way that IS charac
'111.""'" EI
(0)
IE)
IInlltioln because· a ted from EI Greco's paintings the systematic ra' twentleth-century artists have consCiOusly ad p ngation of the human form. . h depicted mEI Greco's portraits d somewhat like t ose some people do have elongated b0 les Ie looked to him, the elongated figures In hiS if EI Greco had an astigmatism, then, relative to hotw:eop d to him to be dlstor e . . . palntlngs would have appeare . b en no correction for It available In the period In even if EI Greco had an astigmatism, there would have e which he lived G 'tme who Included In their works human figures that there were non.European arbsts, even In EI reco s I , were Intentionally distorted
Argument Construction . . . b th Spanish painter EI Greco are elongated. Some critics infer Situation FIgures In portraits y e . . h d · b EI Greco suffered from an astigmatism t at rna e people appear that thIS was ecause elongated to him. But this explanation cannot be correct. Reasoning
79.
Consumer health adVocate Your delivers a specified arnount of C.ff~ctft.lJtI . . your customers addicted. In Candy maonufacturer: Our rnanuf t bar than In the unprocessed cacao b ac unnllH'oc eans from whIch tilt The candy manufacturer's response IS fl because It awed as a refutatlon of tile
:::~~::=~
btequcnt arti depicted human lorms r andtsheintentionaUy d' as more elongated than human "".uauy IRt t Y Id so to mimic EI Greco 's pamte . d figures, that does not mean that ...- ... Mre mlentlonaUy elongated, option provides another possible e I . f, . ll/t,llVilience that the people EI Greco ~p anatlOn or El Greco's elongated figures, It would hi ' pamted had such elongated figures . .....IOPl:lon lie pereelVed the images of I' . P'" ofthc people themselves Th' peop e In hiS paintings as too long, ~_ would not provid . IS means that even if El Greco did have rtId? e an anSWer to the question: Why did El Greco paint _ _ ._11
4:_._
. .
~1ICt lIl_ti m in EI G ' ~.lB.lltllled EI G reco s ~ay does not undermine the reco to pamt elongated figures. I.~t\oll • fOr the d'lstortion-namely, that EI Greco d'd I it
)"-ink that this is the correct explanation (and
'...
(M
fails to address the issue of wh th enough to keep people addlcte~ er the level of caffe ne In the C8ll!ft 1"1 ii
(8)
assumes Without warrant that all unprocessed cacao beans nta does not specify exactly how caffeine I co na IS ost In the manufactunna treats the consumer health advocate' prace about the manufacturer's candy In ge~::~ument as though It were about
(C) (D)
(E)
merely contradicts the consumer health advocate' I the advocate's reasoning IS unsound scone uSlon without III
Argument Evaluation Situation
A candy manufacturer is accused of adding caffeine, an add chocolate candy bars with the intent of keeping its custom manufacturer responds to this accusation by saying that the chocolate candy bar than in the unprocessed cacao bean fro made,
Which option would most logically complete the argument?We need somethin.g that
provides the best reason for thinking that the explanatIOn suggested by CntICSastigmatism-cannot be right. The critics' explanation might seem to work because ordinarily an artist would try to paint an image of a person so that the image would have the same proportions as the perceived person. So if people seemed to El Greco to have longer arms and legs than they actually had, the arms and legs of the painted figures hould appear to others to be longer than people's arms and legs normally are. This is how the explanation seems to make sense. But if astigmatism were the explanation, then the elongated images in his pictures should have appeared to EI Greco to be too long: he would have perceived the images as longer than they a~alIy ~re-and therefore as.inaccurate representations ofwhat he perceived. So asttgmatl m cannot be a suffiCient explanation for the elongated figures in his paintings.
.
Reasoning
What is theflaw in the candy manufacturer's resprmse' Fint COIlIidilr, 1r1 indeed refutes the advocate's charge. In actuality, IDSteldoflila_ .. the accusation-adding caffeine to its chocolate bars to IM_• • •
uep
manufacturer substitutes an entirely different subject, the 1DI111l1.lrl•• beans. The manufacturer' re pon e i a divenwn, not
A
B C
D E
Correct. This statement properly identifie the flaw in the response not answer the question whether adding caffeine to candy bus ...,.• .addictive. Even if the manufacturer did make thi assumptIOn, the irdiJn~IDi"" accusation, which is not concerned with naturally ODCUllriIllJCl..1II The precise amount ofcaffeine 10 t in the m The manufacturer does not treat the health aetiOC.SII The manufacturer doe not contradict the aooJSlli"~
The correct answer is A.
b having its employees work at home.
"lIIml.'_I8~on office-space expenditures ~ent for SIX months. DUring this period.
"1'_1Ied volunteers from Its staff to try the arrange
.lIJtyoftllese employees was as high as or higher than before. alnst deciding, on the basIs of the trial results, to ....114'18 following, if true, would argue most strongIYag .....melll tile company's plan?
The employees who agreed to participate In the tes t 0 f th e plan were among the company's most self-
81
polrtlcal Advertisement: Mayor Delmont's CritiCS camilla about~-~~a:&;= fact is that not only were more In : f 10bs creatld average payor the new Jobs created has stands to reason that throughout Delm been hither then _ . " bigger onfs tenure the . . I. _ _•• Which of the follOWing If true most 1~1ltlII1iII , • serrously wealcen the ar'&un1l!llt (A) The unemployment rate In the City h h IS Ig er today than (8) The average pay for Jobs In the cty I was at a ten-yea low wlNlB (C) Each year dUring Mayor Delmont's te lh nure, eaverage pay for than the average pay for jobs crtywlde.
motivated and Independent workers. (8) The savings that would accrue from reduced office-space expenditures alone would be suffiCient to Justify tile arrangement for the company. apart from any productiVity Increases. (C) Other companies that have achieved successful results from work-at-home plans have workforces that are SUbstantially larger than that of XYZ. (0) The volunteers who worked at home were able to communicate With other employees as necessary for performing the work.
(D)
Most of the Jobs elrmlnated durrng Mayor Delmont's tenure were
IE)
(E)
The average pay for Jobs In the City is currently lower titan it farjabtllllltl• •
MInor changes In the way office work is organized at XYZ would yield increases in employee productivity Similar to those achieved in the trial.
Evaluation of a Plan SituatIon
To save money on office space expenditures, a company considers having employees work at home. A six-month trial with employees who have volunteered to test the plan shows their productivity to be as high as or higher than before.
R sonlng
Why wou/~ the trial results NOTprovide agood reason to implement the plan? GeneraliZIng from a small sample to the group depends on having a sample that is representative. In thiS case, the employees who participated in the trial are not representative of all employees. The employees who volunteered for the trial may be the type of em~loyees who would be most likely to work successfully at home. It would not be wise to base a generalization about all employees on this sample.
II",..Ill
Argument Evaluation Situation
Every year since Mayor Delmont took ollice, avenge _r In• • average pay for jobs citywide. So, the avenge JII.J.lIIlII,iAJ since Delmont took office.
Reasoning
Which option, if true, would most ""OW jobs continually exceed that for jobs generally: IIetlt old jobs that still exi t. But uppose the ti:IIIowiu&~ ..... highest paying jobs are eliminated and repI~d.lIl_lI'I'" still pay more than the average job The rauIt. .IMJr.! for a new job would be greater than that JDr_~ jobs would nonetheless decrease. 1hus, if C'1lIIJ'''~''. that were eliminated paid better on ~. . weaken the argument: the conlCchibll5islD"iDIlaa....:.j~M.. it is based is true.
orree:t. Thi lalement pro~erly identifies a Raw in the trial that is the basis for the plan.
1h
tate7t
~tDC UPPOtrts the Imhplemcn~ation of the plan. Moreover, it is not based on the trial no answer t e question.
age gIVe no Information about how com a . . h ~r.-- r the rehability of the trial results. p ny sIze mIg t affect the implementation of would nd to Upport the plan. ave money on office space n t t . l~realC produ tIVI'ty" :-: I IrreIevant.,0 0 Increase productivity, so an
A
The percentage of people in th~ city who bPCJJP"lllllll pay for jobs citywide i increasmg or dec.....
B
\oVhether the average pay 10 years is immaterial to the compansoo
C
Correct. Thi information eliminated had higher aYelrqe turn would mean that the a,,~~
D
This, too, ha no for job in those dlcclJinJiil ll
E
Th i i al 0 irrdlevllilt. city and jobs in dIa lUll
The correct 1D11_1~
_"1
_
. . ..., IIIIV1l1w 2nd Edltloll
. th t an Industry convention more productive In ....... manufacturer. WIshes to make its information boo . a the company's new products and services To III.. " of boosting sales. The booth 0 ffers Informatlon tntroduclng nt will attempt to attract more peop Ie to t he booth.. ~lld111lM the desired result. Vitacorp's marketing departme I person to call his or her five best customers lhe marketing director's first measure was to instruct each sa es Illd personally Invite them to VIsit the booth. diction that the marketing director's first measure Which of the followtng, If true, most strongly supports the pre WIll contribute to meettng the goal of boosting sales? stamer about new products and services as (AI Vrtacorp's salespeople routtnely tnform each Importan t cu soon as the decIsion to launch them has been made. k ng their own Information booths more productive (8) Many of Vltacorp's competitors have made plans for ma I
83.
An eyeglass manufacturer tned to discount If their orders for that boost sales lor 1Ile Il8l 1IiII . . quarter exceeded .. Many dlstnbutors qualified for this dthosefor ~:~=:! produce a healthy gain In net profits~ Ewn With 11lICIl" discount for the fall quarter. . e manufacturer IIIans to "II1II Which of the follOWing, if true, most clea performance of the summer quarter? fly pOints to a flaw 1l11le ..... btllll ... eti'l'. ..... (Al
(8l
(el
In general, a dlstnbutor's orders f th .. or esummer QUarter are nohilllerlllln'_'''I''~ Along with offenng special d.IScounls t ,.... radio advertising In those dlstributors'o qlua "J,ng cistributors the sa es areas. The distributors most likely to qualify for the manufacturer's unusually Iowa year earlier. special discount .
1l8IIIIactlnr_._.
(el
tn Increasing sales. . An tnformatlon booth that IS well attended tends to attract visitors who would not otherwise have attended
(0)
(D)
the booth. Most of Vltacorp's best customers also have business dealings with Vitacorp's competitors.
The distributors who qualified for the manufacturer's speclCll discount were free that discount to pass on to their own customers.
(EI
Vltacorp has fewer new products and services available this year than it had in previous years.
(El
The distributors' ordering more goods In the summer quarter left them overstlIcIIIdblllal.
Evaluation of a Plan
Evaluation of a Plan
Situation
Reasoning
A manufacturer wants increased sales from its information booth at an industry convention. To boost sales, the marketing department seeks to attract more people to the booth, and the marketing director tells the salespeople to invite their best customers to visit the booth.
Situation
Which point best 5Upports the marketing director's plan? First ask what would be a valid
Reasoning
reason for inviting faithful customers to visit the booth. Such invitations should assure that the booth will generally be busy with visitors. If people are more attracted to a well.-~ttended booth than to an empty one, then more potential customers are likely ~ VISit t~e busy ~ooth, and more vi itors should produce more sales. The marketing dlfector IS operatmg on the principle that success breeds success. Making sure that the bo?th is well attended by Vitacorp' current customers is likely to attract more potential customers and thus boost sales.
"R~_I..
tell tomden alrea~y ~av~ all available new product and service information, they are - - , t o re pon to the invitation to visit the booth, this point is a k . h I '~~petltoirt" _1:Il'... , w e a ness 10 t e pan. .• cnort toward the same goal may hurt Vit 'IIi plan. I acorps e arts, so this point does not ment properly id ntifie a point supporting the marketing director's plan. t more vi iton to Vila 0 'b h VltliCOl h ' rps oot to encourage more sales and does rp re I ustom n with its competitors.
,would
ugge t that the plan would be less successful.
A manufacturer successfu1ly boosted sales and gained oct pmfiII ....1. . . . quarter by giving distributors a discount if their onIas ex........ orders by 20 percent. The manuf.u:turer plans til repeal tIM: 1III:l~1".". discount again in the fall quarter.
What is theflaw in the manujamnrr'r plim t 1he plan assuDMS"1 . ucceeded once will work a second time. Why might tIM: pIm _ _I .. If the distributors increased their orders during tIM: S'JIDmer were eager to take advantage of the discount. tIM: muir DIal,..,........ overstocked for the fall quarter. H so, they will DOt
"",".M
good , and the plan of continuing tIM: discount will
A
This i irrelevant to the plan inee rclMJIt qlWttlS frJI
B
Increa ed adverti ing should continue to COIl
C
Even if the qualifying di tributon teaebcdbciCloo..-ly::,r ...., . . alify . the fall distributors who will qu 10
D
The di tributon' freedom to decide how true in both ummer and fall quartaS 1 Correct. Thi tatel1\cot propedy idIIl8··
E
The correct answer is E.
""'~!II!I
5.0 Sentence Correctio Sentence correction quest' Ions app . h uses multiple-choice quest' ear In t e Verbal sectIOn'" ~.._ IOns to mea to reason and evaluate argu sure YOUr ability to ~ h B ments, and to · EngIIS. ecause the Verbal . . correct written matenal c sectIon Includ be genera II y Lamiliar with som f h es pa age from "'="ma1 e 0 t e mat . I h ., assume detailed knowledge of th . .ena; OWCver, neIther with critical rea oning and read' e tOpiCS diSCUS ed. Sentence correett Ing comprehe . . I nSlOn que hons throughout test. You will have 75 minutes to question. comp ete the Verbal section, or about 1 Sentence correction questions present '. a tatement In which word are ulllilallineet. you to eIect from the answer options th b . f . · d . e est expres Ion 0 the Idea or relatiOluhiip d . . . . un d erI me sectlOn. The first answer choice alw h a1.~ [, .d I . ay repeats t e ongm phmsi our P~OVI e a ternatlves. In some cases, the original phrasing i the best cluilice. underlIne~ section has obvious or subtle error that require correction. Th e qwcsti. . . . to be famIlIar WIth the stylistic convention and grammatical rule of tandard wrllftII.. to demonstrate your ability to improve incorrect or ineffective expression You should begin these questions by reading the sentence carefully. ote whether t ....~ obvious grammatical errors as you read the underlined ection. Then read the carefully. If there was a subtle error you did not recognize the first time you may become apparent after you have read the answer choice. If the error' you can eliminate some of the answers as being incorrect. Remember that In selection may be the best answer.
5.1 Basic English Grammar Rules
entence Correction
_.l..'._
. k to re 'ognize and potentially torIrrd Sentence correction que tlOns a you ~ . If II. . . . H the e rule are not exhausb1le following grammar ru Ies. owever, c h G tAT t · h a way to prepare ,or t e more about EngI IS grammar as . available on the Web.
Agreement 'thin
Standard English requires elements WI agreement: noun-verb and pronoun.
NOlin-verb agreemm.t · I.ngular ubj E.\·amples: Correct: "1 walk to the Correct: "We go to Scbi::~_IID1.~! Correct: "The number Carre t:"Th m
·th the noun or pron
pronoun must agree WI ..
enyou~II,
oun it refers to in person, number ,
Grammatical Construction Good grammar reqUires ClItIIpIIIa. . .~
. "When one dreams, you are usually you are usually asleep." Incorrect.
Fragments: Parts ofa &entau:e that Example: - ..... Correct: "We saw the doctor and hit And his nurse." lIIIne.l. .
,.!lL"J....
....".,Ia""':t: "When the kids went to sleep, they slept like logs.: ....._1lID'1IlCC:
"When the kids went to sleep, he slept like a log.
Run-on sentences: A run-on &ental· iJlIl~IepIlllliIlll._'''. . ce 11 two 'ne • •k • • punctuatIOn. t4 Examples:
d ffi ' Iy the appropriate part of speech. There are Ords should be chosen to reflect co~ctly an Ie ~~:~ answering sentence correction questions, - . I words that are commonly used IOcorrect y. attention to the following conventions.
Correct: "Jose Canseco is still a feared L.......~ , ......... IIIIIIt I ncorrect: "J ose Canseco is still a fearedUU1a; L...~_ "'~._"",.I&i ITlIlUC( 1DOIt ph ..
/:: to reI' involving more than two objects. Between is AwlOtIfIktwttn:Among is used to re,er atlons h'ps .1 uaed to efer to elationships involving only two objects.
Constructions: Avoid wordy, redundant COIII1rUI:tiona Example: Correct: "We could not come to the meetingbl'ranR . _...~. not come to the meeting is because ofa confIkt..
Ewmpla:"Wi d' 'ded "We d'IVI'ded our wInnIngs " .. rrect: e IVI our wmnlOgs among the three of us". Incorrect: between the three of us." Correct: "She and I divided the cake between us." Incorrect: "She and I divided the cake among us,"
Idiom As/liAt: As can be a preposition meaning "in the capacity of," but more often is a conjunction of manner and is followed by a verb. Lilie is generally used as a preposition, and therefore is followed by a noun, an object pronoun, or a verb ending in ing,
EMlmpla: "I work as a librarian." Incorrect: "I work like a librarian," Correct: "Do as I say, not as I do," Incorrect: "Do like I say, not like I do." Correct: "It felt like a dream." Incorrect: "It felt as a dream." Correct: "People like you inspire me," Incorrect: "People as you inspire me," OilmllCt:. "Thee's nothing like biking on a warm, autumn day." Incorrect: "There's nothing as biking WInD autumn day." orrect:
• ""tf)lInl 'WOrds: Mass words are nouns quantified by an amount rather than by a number.
e...t DOUne n be quantified by a number.
bought a loafof bread." Incorrect: "We bought one bread." me mu~ h~ppi~ess." Incorrect: "He wished me many happinesses." aed many buildings. Incorrect: "We passed much buildings."
lhouJd not be uaed a a ub tiMe for lor me. re,· I.ncorrect: "Mom and myself had to go to the store." me, Incorre t: "He gave the present to Dad and myse1f."
It is important to avoid nonstandard exprusions, a1though ._~ conventional grammatical rules. Be cardiJ1 to IIIIe the Cllllllllrllllll". and parts of speech. Prepositions: Specific prepositions have specific pUlplml Examples: Correct: "She likes to jog in the morning.' Inwiil'rt: -sIll: Correct: "They ranged in age from 10 to 15.' Ir.:onect: ...."'1Il111. .tlltl
Ib._,.
Correlatives: Word combinations such as"ootooly the same grammatical type. Examples: t1wIk her_,"-"Correct: "I have called not only to Incorrect: "I have called notonlytot1wlkher~.~~. Forms ofcomparison: Many forms mDow ClUlIIIII whereas 1m than refers to a wntinUOUl choice. Farther refers to eli tanIle .-J~ Examples: Correct: "There wen: fewer my elas thi year.' Correct: "There was told. om: t: choco! te or D
Incon:ec:t:
PreJent perftct: Used WIth an indefinite time in the
_"'p1IJ_1I
ExampleJ.
that detract from the logical argument. . . d so it is clear what word or words they are ....IiMf1"1Nrms Modifiers hould be ~ Itldo~e I' the\' can cause illogical references or . . . .It to modIfy. If modifiers are not po luone c ear), . ......... ~...,..,lioln ,or distort the meaning of the statement. ",pIes n I t . "I put the cake bv the door that I Olttet. "I put the cake that I baked by the door, ncorrec, '
bUrIllCI.'· "
. h d I' , ookie nIncorrect: "Reading my mind the . .' orrect: Reading my mmd, she gave me tee !Claus c cookie he gave me was delicious." ,," orre t: "In the Middle Ages, the world was believed to be flat. Incorrect: In the 1iddle Ages,
~:~~~:~=:!
Correct: "I have traveled all Correct: "He ha gone to hOYer, thewodd; Ie 00
mce he. . . . . . . .
PaJt perftct: This verb form'IS used WIth L~
past.
..... 10
d~, th~";~=~
Example' Correct: "By the time I left fa r sch00, I t he caR bad been ........
Future perftct: Used with will have, this verb form .IUlaibClul.~ii
~
another event,
Example: the world was flat."
Correct: "By the end of the day, I will have tudied fur all
Present progressive: Used with am, is, or are, thi
rellelism on tructing a sentence that is parallel in structure depends on making sure that the different elements in the sentence balance each other: this is a little bit like making sure that the two sides of a mathematical equation are balanced, To make sure that a sentence is grammatically correct, check to ee that phrases, clauses, verbs, and other sentence elements parallel each other.
happening now.
EKIlmplts·
when another action occurred.
Olttet" "I took a bath, went to sleep, and woke up refreshed." Incorrect: "I took a bath, sleeping,
and wakmg up refreshed." orreet: "The only way to know is to take the plunge." Incorrect: "The only way to know IS taking the plunge."
Example:
Correct: "I am studying for exams." "The rudent is tudyiDg iJr _ . .~.
Past progressive: Used with was or were, this vern form describeelililiii~~~ Example: Correct: "The student was tudying when the lire alarm
~."
".1/'4.-
broke out,"
Future progressive: Used with will be or sbaJJ k this
_bt....AI• •
continue into the future.
• ltOlrtcal Construction n
vern form describeI .. ,II1II......
tructure avoids CO?structiom that are awkward, wordy, redundant, imprecise, or when they ar free 01 grammatical error.
left on vacation, we watered the pi ants, c-Ilee ked to see that the stove was off, •I n" ro b correct:d Before ' L.J II we left t0 go on our vacation, we watered checked to IJIW n turne 0 ,and set it." '
Example:
Correct: "The student will be tudying for exams
Present perfect progressive: Used With haw began in the past, continue into the Example: Correct: "The tudent has been tudyinI
Hut perpa progrtssivt: Used was co.:upleted befi>re another
Example:
led y." oJ will go to school tomorrow." walkeld, ran), a progres i
p~lI:ct P
I
rro ~
IIIlI".
ei...
Correct: "BefOre th
th
tbJlIlIlI~Itd··-
3.
uggestions There are two basic ways you can study for sentence co
rrection questions:
• Read mlterla' that reflects standard usage. . ' . . h the bas',c conventions ofstandard wntten English IS simply . f: '1" ami Janty Wit . fi . O ne way to. gam bl . I '11 II be fcound in good magazme and non ctlon books to rea. db' d ulta e matena WI usua y editorials in out tanding newspapers, and the collections of essays use Ymany college and university writing courses. • Review basic rules of grammar and practice with writing exercises. Begin by reviewing the grammar rules laid out in this chapter. Then, if you have school assignments (such as essays and research papers) that have been carefully evaluated for grammatical errors, it may be helpful to review the comments and corrections.
5.3 What Is Measured • Correct expr..slon A correct se~tence is grammatically and structurally sound. It conforms to all the rules of standard wntten English, including noun-verb agreement, noun-pronoun agreement, pronoun co.nslstency, p~onoun case, and verb tense sequence. A correct sentence will not have dangling or formed modifiers; unidiomatic or inconsistent expressions' or faults , In para e constructlon. '
~Isplacleldl' Impro~erly
• Effective expreulon
~:;~~::;:I;;n~~~~:;;~e~:::nidthea orhrelahti~nship
clearly and concisely as well as an at t e c olce with the fe t d' I ne e sarily th hest answer It means th h wes an simp est words is at t ere are no superfl d dl . . compIIcated xpressions in the best choice. uous wor s or nee essly • Proper diction An efti tive entenc also uses proper diction (D' ctlOn . meanings of words and the appropriatenes f' 'd refers to the standard dictionary n b s 0 wor s 10 Conte t) I I' aiaib1c • you mu t e able to recognize wh h h x. neva uatmg the diction of for the context. et er t e word are well chosen, accurate, and Il
carefully. or relationship that th
~lIIdp"II.llIfD
r rrorl nd pOlllbl
mongth
8
entence should express.
e correction. before re.dlne the
wer h' Olces. Remember. in
=::::::~
5.
Make sure that you evaluate the Pay attention to general cl' sentence IIlCI the ccll l ml l c=11. language d . amy, grammatical and :.. :_-~,an appropnateness ofdietlan. . JUIUIIIaQC ~ - - . ,
6.
Read the whole sentence substitutl A choice may be wrong b~c . d"1 the choice that JlIU 1ft. . . . . . . . ause It oes not fit tmImm.......U.. t h e sentence. Remember that O---~l or ItnICIlIMIin some sentences will1'l'rn11_ allRl. . . sentence requires no correction,c hoose the fim answu --,-- 110
5.5 The Directions
entence correction questions test three broad aspects oflanguage proficiency:
::'::::IMtlIftDt
4.
Read each anSWer choice The first answer ehOlee . •'--.CIrefIIIr. hi t S a~s:ver if you thinkd;;" rqIeats the qncle.!j exammmg all the h t the sentena: t.__• I at er choices oar .'''.~i Try to determine how to . Some of the answer ch . correct what yau oew_IIII?I... "_ rnaYchange .1.:uS 1IR 11III ch ange everything th. that atOlces IS Wrong. W~'lS"
orne case
These are the directions that you will see for sentence corra:non qUlllli_,wI.'JIll_ GM~T test. If you read them carefully and understand them c1I:uly Wi_I. . . . . you not need to spend too much time reviewing them ooa: JOU ue.tt_1-'test IS under way.
:nil!
Sentence correction questions pre ent a sentence. part or all of which sentence, you will find five ways of phrasing the underlined p'mgr. . . tiIIt....iia the original underlined passage; the other four are dilfcmJt.1fJOU tbiaIr. best, choo e the first answer; otherwi e choose one of the odIas.
!WI.""
This type of que tion te ts your ability to recognize the CDIlm_a..... standard written Engli h. In choosing your answer. toIlow the English; that i ,pay attention to grammar. choice ofwaids, answer that produce the mo t effective sentenee: this UII_~.III awkwardne s, ambiguity, redundancy. or gnm!Nral
_"GMAte v....., RevI.w 2nd Edition
5
pie Questions rt of or all of which is underlined. ts a sentence, pa f' f h . of the sentence correction questions presen . h nderlined part. The Irst 0 t ese repeats . f phraslOg t e u . E I' h . ments of standard written ng IS to chOOse Beneath the sentence you will find five ways 0 . F II w the reqUire . 5 I 0 a . d entence construction. e ect t h e anSWer the onglOal' the other four are different. • . word chOICe, an s I your .nswer. paying attention to grammar, h uld make the sentence c ear, exact, and free that produces the most effective sentence; your answer s a mbiguity, and redundancy. . . . awkwardness, a of grammatical error. It should also mlOlmlze
1
Like ants. termites have an elaborate social structure In which a few individuals reproduce and the rest Meserving the cQIQny by tending JUveniles. gathering fQQd. buildmg the nest. Qr they battle Intruders.
IA) (8) (e)
3.
are serving the colony by tending Juveniles, gathenng food, bUilding the nest, or they battle
HQwever much Untted States VQters may agree thgt there is waste in gQvernment and that the gQvernment as a whole spends beyond its means, it IS difficult to find broad support for a movement toward a minimal state. (A)
However much United States voters may agree that
are serving the cQlony /n that they tend Juveniles, gather fQQd, bUild the nest, or battle
(B)
serve the cQIQny, tending Juveniles, gathering fQQd. bUilding the nest, or by battling
(C)
Although United States voters agree
(D)
Even though United States voters may agree
(E)
There is agreement among United States voters that
(D)
erve the cQIQny by tending Juventles, gathering fQQd, by bUilding the nest, Qr by battling
(E)
erve the cQIQny by tending Juventles, gathering fQQd, bUilding the nest, or battling 4.
me bat ave • !Ike honeybee hives, have reSidents that take Qn different duties such as defending the n e acting as sentmels and to sQund a warning pproa h of danger, and sCQutlng Qutslde the r ew food and rQQstlng sites. ntinel and tQ sound ntmel and Qundlng n I and ound n I and to ound 1J1l11tit~el oundlng
Hundreds of species of fish generate and discharge electriC currents. In bursts Qr as steady electric fields around their bQdles, uSing thelt POWer eilhertnJinll.. ~ attack prey. tQ defend themselves or aJs;;;: ~mmuntcatlng and navlgatmg. (A)
either tQ find and attack prey. to defend themselves, Qr alsQ fQr cQmmuntcatmg and navigating
(B)
either fQr finding and attacking prey, defend themselves, or fQr cQmmuntcation and navigation
(Cl
to find and attack prey. for defense. or communication and naVigatIOn
(D)
fQr finding and attacking prey. to defend themselves, or also for communtcatlon and naVigation
(El
to find and attack prey, to defend themselves, or to communicate and navigate
Despite the agreement among United States voters to the fact
The voluminous personal papers of Thomas Alva Edison reveal that his Inventions typically sprang to hIe not in a flash of Inspiration but evolved slowly from previous works. (A)
sprang to life not In a flash of inspiration but evolved slowly
(B)
sprang to life not in a flash of msplration but were slowly evolved
(C)
did not spring to life in a flash of inspiration but evolved slowly
(D)
did not spring to life In a flash of inspiration but had slowly evolved
(E)
did not spring to life In a flash of Inspiration but they were slowly evolved
6.
ALabor Department study states that the numbers of women m 10 ed outside the home rew m re than a thirty-five percent Increase In the past decade and accounted for more than slxty·two percent of the total growth In the civilian workforce. (A)
numbers of women employed outSide the home grew by more than a thltty-five percent Increase
(B)
numbers of women employed outSide the home grew more than thlrty·five percent
(C)
numbers of women employed Qutslde the hQme were raised by more than thirty-five percent
(D)
number of women employed outSide the home Increased by more than thirty-five percent
(E)
number of women employed outSide the home was raised by more than a thirty-five percent Increase
(()
lEI
8
Asthea:~:=:=
cwerthe--
IIlCreasingly caml'lllllt, ....,.. theycan
11.
tIJ !C) (0)
IE)
o
It was decided by the company manager to give her staff a raise the deciSion of the company manager was to 1M! her staff a raise the company manager decided to give her staff a raise the staff was given a raise by the company manager a raise was given to the staff by the company manager
The nSln!! of costs of data-processing operations at many financial Institutions has created a growing opportUnity for Independent companies to provide these services more efficiently and at lower cost. (A)
The nSlng of costs
IBI ICI IDI IE)
RISing costs The nSlng cost Because the nSlng cost Because of nSlng costs
NatJve to South Amenca, when oeanuts were IOtr~ ~Afnca by portuguese explorers early 10 the Sl~ ~ntu;they were QUickly adopted Into Afnca's.. ~;n;u;ure, probably because of being so similar to the Bambarra groundnut, a popular Indigenous plant.
(AI
(Bl
(Cl
(0)
(E)
when peanuts were Introduced to Africa by Portuguese explorers early In the sixteenth century they were QUickly adopted Into Africa's agriculture, probably because of being peanuts haVing been Introduced to Africa by portuguese explorers early In the sixteenth century and QUickly adopted Into Africa's agnculture, probably because of beIOg peanuts were introduced to Africa by Portuguese explorers early in the sixteenth century and were Quickly adopted into Africa's agriculture, probably because they were peanuts, introduced to Africa by Portuguese explorers early in the sixteenth century and Quickly adopted into Africa's agriculture, probably because they were peanuts, introduced to Africa by Portuguese explorers early in the sixteenth century and having been Quickly adopted Into Africa's agriculture, probably because they were
12. William H. Johnson's artistic debt to Scandinavia is eVident in paintings that range from sensitive portraits of citizens in his wife's Danish home, Kerteminde, ~ awe-inspiring views of fjords and mountain peaks in the western and northern regions of Norway. (A)
and
(B)
to
(C) (D)
and to With
(E)
in addition to
Growing competitive pressures may be ellCOlra 13. auditors to bend the rules in favor of clients IlII1I ---- N!!fWs lNY for Instance, II ow a Questionable loan to remalll on the books In or er to malOtalO abank's profits on paper.
d
(AI (B) (C)
clients; auditors may, for IOstance, allow cltents, as an IOstance, to allow clients, like to allow
(0)
clients, such as to be aliowlOg
(E)
clients; which might, as an IOstance, be the allOWing of
14. The themes that Rita Dove explores in her poetry ~ universal. encompassing much of the human condition while occasionally she deals with racial issues.
1Ile.(A)
lBI lc)
they . .
IDI they . . (EI being 17. Travelers to Mars gravrty for long peitidl ralftation,
contall1lllatJon IIlI Mal lilii
(A)
is universal, encompassing much of the human condition while occasionally she deals
!AI contend willi the
(B)
is universal, encompassing much of the human condition, also occasionally it deals
(BI
(C)
are universal, they encompass much of the human condition and occasionally deals
(D)
are universal, encompassing much of the human condition while occasionally dealing
(E)
are universal, they encompass much of the human condition, also occasionally are dealing
15. It IS well known in the supermarket Industry that how items are placed on shelves and the frequency of inyentory turnovers can be crucial to profits. (A)
the frequency of inventory turnovers can be
(8)
the frequency of Inventory turnovers is often
(C)
the frequency with which the inventorY turnS
(D)
over IS often how frequently is the inventory turned !Nfl are
(E)
often be how frequently the inventory turns OW' can
soa and perhaps MIl
contend wiIIJ 1IJe soli. and perhaps MIl (CI contend willi the so~, and peliaps (01 cotltetldill with
soa andpe....
(E)
COliletldic soi and pel J
18.
1
ICP' ,
lieenu years
t
..._.,..,.,..",,,,,,. 2nd Edition
...... mo en ient of lMl1g writing systems, ~~:::'ten of thousand of IdeographIC ~
each character a m'OIature calhgraphlC GlIIDIIIlSilimlMldI..J~ own square frame.
lB
D
IE)
2
(8l
(A)
e h haracter a m'nlature call,graphlc ompo rI,on ,nslde ,ts a I the haracters a m'nlature ca'hgraphlc ompo ruon ,nslde their a"!he characters a mIniature ca'hgraphlc compo ,tlon mSlde Its every characler aminiature callIgraphic compo ,hon Inside their each character a miniature calhgraphlc compo IliOn Inside their
(B) (C)
Dechnlng values for farm equipment and land, ~ cpllatera' uatO t wblc.hJarl!J.er bQUQ\'l.JQ get through tIlt..har.Yfil ea on. I gomg to force many lenders to Irghten or deny redlt this spnng. (A)
researchers are producm.g a senes of Japanese f I 22 . robots that can ,dentlfy human aCIa _ex~p~re~s",SI",,011.S.JQ. hlch Ihev will then respond: t herr goal.s prrmauJy, w a robot that Will empathize with us. !:.reallng
23.
LJIIe Auden, the language of James Merr.i!t 25. arch, and conversatlOnal-gNen to eOl1llllex 5yntac1tic flights as well as to proSaiC free-verse stroH
expreSSions, to which they will then resPond; therr goal IS primarrly creating expressions, then responding to them; pnmanly to create expreSSions and then respond to them; the researchers' pnmary goal IS to create
(0)
expressions as well as giving a response to them; their pnmary goal IS creation of
(E)
expreSSions and responding to them; pnmanly, the researchers' goal IS creatmg
IAl
like Auden, the language of James Merr
(B)
like Auden, James Mernll's language
IC)
like Auden's. James Mernll's language
(Dl
As with Auden, James Mernll's language
IE)
As is Auden's the language of James Mern
beSltP*
26. The Baldllck ManufactUring Company has for se era
years followed a policy aimed at decreaSing operab g ~osts and imprOVing the efficiency of Its dlstnbutlon system.
Consumers may not think of household cleaning products to be hazardous substances, but many of them can be harmful to health, especially If they are used improperly.
E
(A)
armed at decreasing operatmg costs and Improving
(B)
aimed at the decreaSing of operating costs and to improve
(C)
arming at the decreaSing of operating costs and Improving
the collateral against whIch farmers borrow to get through the harvest season, IS whICh farmer u e as collateral to borrow agarn t 10 get th,ough the harvest season, IS
(AI
Consumers may not thmk of household cleaning products to be
(0)
(B)
Consumers may not thmk of household cleaning products being
the arm of which is the decreasing of operating costs and Improving
IE)
the ollateral which I borrowed agamst by , rm r to g Ith,ough the harvest season,
(C)
Aconsumer may not think of therr household cleaning products being
with the arm to decrease operating costs and to Improve
whl h 'arme, u "COllilterill to borrow n t 10 t throu h the harvest eason, are
(0)
Aconsumer may not think of household cleaning products as
Iia ,al ag 'n t whllh farme, borrow to l!h, u h th harv sl a on, ,
(E)
Household c1eanmg products may not be thought of, by consumers, as (A)
Obtarning an investment-grade rating eep the county's future borrOWing costs low pr tee
(8)
To obtarn an mvestment-grade raMg the county's future borroWing costs 10
IS
Cdn hu,t ome la,ge many tlni ca efo' :::~:~:~ : 11I5--can plung or ev n 10
24.
In recent years cattle breeders have mcreasingly used crossbreedmg, In part that therr steers should acgulle. c flln bilrae rI I and partly because crossbreeding IS Said to provide hybrrd vigor
27.
tC) I 1m lor hom owner , ~.ll) ditvlllltll hom own , In
(A)
In part that their steers should acqurre certain characteristics
(B)
In part for the acquiSition of certain characteristiCS In therr steers
(C)
partly because of therr steers acquIring cert'ln charaetellstle s
(D)
partly because c rtam chilr,lctell tiCS shouldbt a qUired by therr teers
(I
partly to acqurr c rtam chara tell tiC ,n thell' t er
(D)
(E)
eep
and
protect HaVing obtarned an Investment-grade rail In keeping the county's future barr OIl c t low. protect To obtain an Investment-grade ra\lng the county's future borroW OIl IS protecting Obtalnrng an Inve tmen!g ade the count • borro lOll prate tlng
D
29
8
..
........"...,.llIevIew 2nd fdltlon
-===:::1°
1nuclear-reactor fuel have already II rbit around the Earth, and the chances ;'......'.....;llIOn Involving such material Increase greatly SIb .mount of both space debriS and satellites cpn!jnue to nse
(81
(el (Dl (EI
32
34
(AI (81
Like Rousseau, Toistol rebelled like Rousseau, Toistoi's rebellion was
IC) IDI IEl
As Rousseau, TolstOi rebelled As did Rousseau, TolstOl's rebellion was Tolstol's rebellion, as Rousseau's, was
Polio, although It IS eradicated in the UOiled States, It contlOues elseWhere and IS abl e to be
(81
Polio, although eradicated in the UOiled St t It still continues elsewhere and can be aes,
A
equipment, such as lights, that are viSible and must be turned on and off and underestimate It
(C)
Although slill continUing elseWhere, polio has been eradicated In the United States and could be
(B)
when equipment, such as lights, that is Visible and must be turned on and off and undereslimate it when
(0)
Although havlOg been eradicated In the UOiled States, polio still continues elsewhere and IS capable 01 belOg
ICI
(0)
Visible equipment, such as lights, that must be turned on and off and underestimate that
IE)
(E)
Visible equipment, such as lights, that must be turned on and off and underestimate it when
(8)
(C)
35.
urnng when a child urr ng when children It
urred when achild
Ie
IOl
Although eradicated in the UOited States, polio contlOues elsewhere and could be
IE) 37.
Some b 'Idin ha were destro ed and heavil damaged In the earthquake last year were constructed in violation of the city's building code. (A)
Some buildings that were destroyed and heavily damaged 10 the earthquake last year were
(8)
Some buildlOgs that were destroyed or heaVily damaged in the earthquake last year had been
(C)
Some buildings that the earthquake destroyed and heavily damaged last year have been
suggest that the economy should continue ItS eKpanSlon IOto the comlOg months, but that
(0)
Last year the earthquake destroyed or heaVily damaged some buildings that have been
(8)
suggest that the economy IS to continue expansion in the coming months, but
IE)
Last year some 01 the buildings that were destroyed or heavily damaged In the earthquake
(C)
suggests that the economy will continue its expanding in the comlOg months, but that
(0)
suggests that the economy IS continuing to expand Into the comlOg months, but that
(E)
suggests that the economy will conliOue to expand In the comlOg months, but
urred when they were children urred as each was achild
equipment, such as lights, that are visible and must be turned on and off and underestimate that
The rise in the Commerce Department's index 01 leading economic indicators suggest that the economy should contlOue ItS expanSion IOtO the coming months. but that the mixed performance 01 the index's indiVidual components IOdicates that economic growth will proceed at a more moderate pace than in the first quarter 01 thiS year. (A)
The WalierstelO study IOdlcates that even after a decade young men and women stili eKperienCe some 01 the eHe ts of a divorce occyrring when achild.
36. EQlio. although It IS eradicated 10 the Unrted States ~nliOues elsewhere and IS able to be brou h . t1. country by VISitors. g t Into the (A)
(AI
as the amount of both space debriS and satellites continue to rise as the nse conllnues In both the amount of satellites and space debriS as the amount of space debriS and the number of satellites continue to rise With the conllnually Increasing amount of space debriS and the number of satellites With the amount of space debriS continuing to Increase along With the number of satellites
like Rousseay. 1olstOi rebelled against the unnatural COmpleKlty of human relations 10 modern society.
LacklOg Information about energy use, people tend to overestimate the amount of energy used by ~ ~ch as lights that are VISible and must be turned..QQ ;d off and underestimate that used by unobtrusive equipment, such as water heaters.
had been
39
assoc ate w less med a QUickly than d associate with medlcabon and
Arecent study has lou years, many datto 5 h rather than lace the th e costs 01 malpractice (Al
had elected ea
IBI IC) (0)
had ele ted have eected e havee ted
(El
havee
ted
42
10 e to their u tamer how long
access to fund from depo Ited
Twenty-two leet long and 10 leet 10 diameter, the A"'-I IS one 01 the many new satellites that IS a part QL 15 years elfort of sublecllOg the mteract,ons pI Earut. atmosphere oceans. and land surfaces to detailed scrullOy Irom space. IA)
isclose to the r cu tamer how long e to lund from a deposited
(B)
10 e to It cu tamer how long access 10 fund tram deposited
(C)
/0 e to It customers
e to fund from a (0)
(E)
43. pproa hi Ih 10undJllon 01 uflly I
d
Jl V
satellites that IS a part 01 15 years effort of subjecting the mteractlons 01 Earth's atmosphere, oceans, and land surfaces
(A)
satellites, which IS a part 01 a 15-year effort to subject how Earth's atmosphere, oceans, and land surfaces Interact
processmg steel scrap In 0 a SPfCla zed gr 01 products has enabled sma 0 capltalmto new technology and re a
IB)
satellites, part of 15 years effort of subjeChng how Earth's atmosphere, oceans, and land surfaces are Interactmg
processmg steel scrap In 0 a SPfcaa ed gr p 01 products has enabled sma m s p capitalmto new technology. rema g
(C)
satellites that are part of an elfort for 15 years that has subjected the Interacltons of Earth's atmosphere, oceans, and land surfaces
the processing of steel scrap in 0 a specaa zed group of products has enabled sma s capital Into new technology, rema nng
10)
satellites that are part of a 15-year effort to subject the interactions of Earth's atmosphere, oceans, and land surfaces
small mills, by processmg steel scrap 10 0 a speCialized group 01 products, ha e been a e to put capital into new technology and remalO
IE)
small mills, by processmg steel scrap IOto a speCialized group of products, have been ab e 0 put capital into new technology and rema ned
Though the term 'graphiC deSign" may suggest laymg QuLC.QrPJl.[~ r n nnu I re rts the hav CQlDe N slgOify Widely rangmg work, from package deSigns and company logotypes to signs, book lackets, computer graphiCS, and film tilies. IA)
(8)
IC)
(01
(I
suggest laymg out corporate brochures and annual reports, they have come to Signify Widely rangmg uggest laYing out corpor ate brochures and unnual reports, it has come to slgOify a Wide I ange of ugge t corporate brochure and annual report layout, il ha Ignlhed Widely ranging h v u g t d corporate brochure and annual r porllayoul. II ha Igl1lhed] Wide r nge 01 hav u t d laYIII oul corpor t brochure nd nnu I r port , they have corne to I nit Wid Iy r ngmg
45. Under high pressure and mtense heal. graphite e most stable form of pure carbon, changes nto e substance commonly referred to as d amond a remalOlOg thiS way whether or not the heat and pressure are removed. IA)
remalOlng thrs way whether or n t
(B)
remalOing like that e en as
IC)
remalOlng as such \ het er or not
(0)
remalOS 10 thiS a although
IE)
remains thus e en hen
"rflalllIIes for the incineratiOn of solrd : : : d the danger of shifting 8IlIS from landfills oolluting the :
Recently discovered fossil remains strongly suggest 52. that the Australian egg·laying mammals of today are a branch of the main stem of mammalian evolution [lither than develOPIng Independently from a common ancestor of mammals more than 220 million years ago.
apart, but rather
(B)
(C)
apart, but rather that of
(0)
apart, but that Qf
(E)
apart; it is that Qf
(0)
rather than a type that developed Independently from rather than a type whose development was Independent of Instead of developing Independently from
(E)
instead of a development that was Independent of
(A)
Ma.·;not expect payments when It IS does not expect payments to be made when
conditiQns, and they use the set Qf QbJective cntena derived from these analyses In deciding
(C)
cQnditlQns and, In uSing the set Qf Qbjective critena derived frQm these analyses, deciding
(0)
cQnditlons and, uSing the set Qf Qbjective crltena derived from these analyses, decide
(E)
conditions and, In thllr use Qf the set Qf objective cntena they derive frQm these analyses, they decide
peyment are not expected to be paid when they
decision, the Supreme Court ruled that two r .,ds In the erghteenth century.
:i:::
two uPstate New York countlls owed
II
thr e tribes of Onerda Indians for zure of
" '_ _tItINew York countlls owed tbr tI: bes of Onerda Indians unlawful serzure of
"1111".,
ork Counties to owe tribe of Oneida Indians for __nof rk COUntias that owed '-Hof Oneida Indians
I~=NIw=YO'k Indian
54,
cQnditions, and in using the set Qf Qbjective criteria they derive frQm these analyses, can decide
(B)
'lYIIIents are not to be expected to be paid
lbe
apart; rather
(B)
suggests that executives examine a firm's external enVIronment and Internal conditions and in u In th set of objective criteria they derive frQm these analyses, can decide Qn a strategy.
whtn
(A)
rather than developing Independently from
53. The normative model of strategic decIsion-making
they. .
The energy source on Voyager 215 not h h t anUClear reactor, In w IC a oms are actively brOken [lI1!ler a kind Qf nuclear battery that us Ibrt. es natural radioactive decay tQ prQduce power.
(Al
(C)
tillY do not expect payments when
55
A patient accusing a dQctQr Qf malpractice will find It difficult to prove damage if there is a lack Qf SQme other doctQr tQ testify abQut prQper medical prQcedures. (A) (B)
(C) (0)
(E)
should there be no testlmQny from SQme other doctor Iac kIng another doctor tQ testify
!AI
to IIlI
56 ArchaeQIQglsts In Ireland believe that arecentl discovered chalice, which dates from the etgh:h century, was prQbably buned tQ keep from being stQlen by Invaders, (A)
tQ keep frQm
(B)
tQ keep It frQm
(C)
tQ aVQld
(0)
in Qrder that it WQuid aVQid
(E)
in Qrder tQ keep frQm
(A)
(B)
(e)
IC! (01
IE)
57. AccQrding tQ its prQpQnents, a prQPQsed new style of aircraft CQuld. by skimming alQng the tQP of the atmQsphere, fly between mQst PQlnts Qn Earth In under tWQ hQurs,
If there IS a lack Qf SQme Qther dQctQr to testify unless there will be anQther doctQr to testify withQut anQther dQctQr's testlmQny
mitial
AccQrding tQ ItS prQpQnents, a prQPosed new style Qf aircraft CQuld, by skimming along the tQP Qf the atmQsphere, fly between mQst POints Qn Earth in under tWQ hQurs. By skimming alQng the tQP Qf the atmosphere• prQpQnents Qf a proPQsed new style of aircraft say it CQuid fly between mQst pPlnts on Earth In under tWQ hQurs. A prQPQsed new style Qf aircraft could fly between mQst pQints Qn Earth in under two hQurs, accQrding to its prQPonents, with It skimming alQng the tQP Qf the atmosphere.
(D)
A prQPQsed new style of aircraft, say Its prQpQnents, could fly between most pOints on Earth in under tWQ hours because of Its skimming alQng the tQP of the atmosPhere
(E)
According tQ its proponents. skimllUl18 the top of the atmQsphere makes it (lOSSIble _ proposed new sty'e of aircraft cQllld 1Iy lllh most points on Earth In under
a:
k publishing bUSiness, the second 60 /nthe.xtboo
rter IS histOrically weak, because revenues are ~ : : marlsetlng exoenses are high as companle~ 'e for the coming school year. prepa,~
(AJ
(B)
: : : : : an become ,neffectlve If
,::::by :
III
the same place, one reason the finding that there are much ons of pestlcfde-degradlng rn 0115 with a relatively long history of use than ,n salls that are free of such
repeatedly In the same place, one reason certaIn pesticides can become ineffective IS lI!!IPSted by the finding that there are much r populatIons of pesticide-degrading rmerobes In Salls With a relatively long history of pe tic/de use than In Salls that are free of such hemlcals
If u ed repeatedly In the same place, one reason ertaln pesticides can become Ineffective IS uue ted by the finding that much larger populatIons of pestlclde·degradlng microbes are found in oils With a relatively long history of stic,de use than those that are free of such mi I finding that there are much larger populations pe ticide-degradlng microbes In Salls With a N111t1W1Iy long history of pesticide use than In thIt are free of such chemicals IS lIIIJllliIIve 01 one reason, if used repeatedly In lIIIce, certain pesticides can become mu h laraer populations 01 II"II..-ldina ml robes In saris With a history 01 pe liclde use than In U h h.micals Suggests ptflllClde can become ~'....Itellly n the ame place
10'1'
(C)
61.
parliament did not accord full refullee~~ ......... of the recent Immigrants because It believed IIIIl dJeward s them for entering the country ;h~
62
low and their marketing expenses are high as they prepare low WIth higher marketing expenses In preparation low, whrle marketing expenses are higher to prepare
(C)
(0)
(B)
dOing It rewards
(e)
to do thiS would reward
10)
dOing so would reward
(EI
to do it would reward
(E)
low, whIle their marketing expenses are higher In preparation
Many policy experts say that shifting a Porlion of 63. health-benefit costs back to the workers helps lQ.. control the employer's costs, but also helps to limit medical spending by making patients more careful consumers.
Almost a decade after New York State passed laws to protect patients by redUCing the grueling hours worked by medical reSidents, twelve hospitals have been Investigated by state medical officials, finding, that all twelve consistently break the laws, many reSidents work longer than 24 hours straight and that m r h n half he sur ical re id n s w rk more than 95 hours a week.
(B)
to do It rewards
low and marketing expenses are high as companies prepare
(0)
(A)
(A)
twelve hospitals have been investigated by state medical officials, finding that all twelve consistently break the laws, many reSidents work longer than 24 hours straight, and that more than half the surgical residents work an Investigation by state medical offiCials of twelve hospitals have found all twelve consistently breaking the laws, that many reSidents work longer than 24 hours straight, with more than half the surgical residents working an Investigation of twelve hospitals by state medical officials has found that all twelve consistently break the laws, that many reSidents work longer than 24 hours straight, and that more than half the surgical reSidents work twelve hosPItals were Investigated by state medical officials who found all twelve breaking the laws, With many residents working longer than 24 hours straight, and more than hall the Surgical reSidents work an Investrgatlon by state medical officials has fOUnd that, of twelve hospitals, all twelve on lalently break the laws, that many reSidents WOrk lonaer than 24 hours straight, With more halt th uralcal reSidents worklna
(A)
(C)
helps to control the employer's costs, but also helps
(B)
helps the control of the employer's costs, and also
(Cl
not only helps to control the employer's costs, but also helps
(0)
helps to control not only the employer's costs, but
(E)
not only helps to control the employer's costs, and also helps
WI (EI
67
(B)
It rises for loss
(C)
those of losses rise
(0)
the rate of loss rises
IE)
there are rises for the rate of loss
65. The market for recycled commodities like alumInum and other metals remain strong despite economic changes in the recycling industry.
IA)
commodities like aluminum and other metals remain
(B)
commodities like those of aluminum and oIIler metals are remaining
(e)
commodities such as aluminum and metals remains
(0)
commodities, such a alU/l1llll.lft metals, remam
(EI
commodities like the CORP and other metlls ItlIlIIllI
Unlike wiIh IIIe ...... prell1lUlllS for . .SOIlI'II_1Ifi1llllll alfected bytl1e _ _.....
rnavor
that the rate of addition to arable lands will drop While those of loss rise. those of loss rise
:::::::= Unlkellle.~~=:= :::.. 1*_
Faced With an est_d. . . . crty's In
64. Ms. Chambers IS among the forecasters who predict
(A)
lilIre
lBl
The gyrfalcon, an Arctic bird of prey, has surVived a fi close bush r with extinction: Its numbers are now~ l!fI!es greater than when the use of DDT was Sharply restricted in the early 1970s.
70
exonctlOn; Its numbers are now five times greater than (B) extinction; Its numbers are now five times more than (C) extinction, their numbers now fivefold What they were (OJ extinction, now With fivefold the numbers they had (E) extinction, now wllh numbers five times greater than
Atter suffering $2 billion In lasses and 25
13 the nation's semiconductor Industry, Wh
ChiPS that run everything from computers ill!!! ~tellites to dishwashers, appears to hjl\(f II1ade ~ lOng-awaited recovery_
(A)
raptured customers 'rom other pame but also forced them Iy captured customers from other e comp,me but It also forced them only captured customers from other omp,me but also forced these pame nly ha aptured customers from other omp,me but also these companies been forced nly ptured customers from other phone mpanle , bUilt ,Iso has forced them ...." ....Iu
ZL
Analysts blamed May's sluggish retail sales on unexciting merchandise as well as the weather, ~ and weller than was usual in some regions, which ~ sales of barbecue grills and lawn furniture.
ettlement at Chaco Canyon were bUilt IA)
:
:::~U~la~r~s~ca~le~,~w~'t:h~m;o~re~than 75 care.!lJ1/Y to 600 WID eam were omplex regional system of roads.
IB)
with mor than 75 carefully engineered IIrIJCtlJr of up to 600 room each, were h m r than 75 carefully engineered 1ln~lrel, of up 10 600 rooms each, 0 room, ach thaI had r fully engine red r om nd with each r fully nglneer d r m hhdben
72
(Al
computers and spy satellites to dshw ashers appears to have
(B)
computers, spy satel1ltes , and d,shwashers appears haVing .
(C)
computers, spy satel1ltes, and dishwashers appears that It has '
(D)
computers and spy satellites to dlshwash ers, appears that It has
IE)
computers and spy satellites as well as dishwashers, appears to have
should be Integrated into every business Course , others say that students will take ethiCS seriously !l!lli.. if it would be taught as a separately required course
colder and wetter than was usual in some regions, which slowed
(A)
which was colder and wetter than usual In some regions, slowing
only If it would be taught as a separately reqUired course
(B)
only If It IS taught as a separate, reqUired course
since It was colder and wetter than usually In some regions, which slowed
(C)
If it IS taught only as a course reqUired separately
(DJ
being colder and wetter than usually in some regIOns, slOWing
(0)
If it was taught only as a separate and reqUired course
(E)
haVing been colder and wetter than was usual 10 some regions and slowed
(E)
If It would only be taught as a reqUIred course separately
Slale offiCials report that soaring rates of liability IllliIrance have nsen to farce cutbacks In the operalions of everything from local governments and school d,slrlcts to day-care centers and recrealional faCilities. (A) (B)
IC) (D)
IE)
75. Scientists have observed large concentratiOns of heavy-metal depOSits In the upper twenty cenbmeter of Baltl a 1m n whl h r nI WI growth of Industrial actiVity there. (A)
BaltiC Sea sediments, which are COIISlStent WIth the growth of Industrial aeb ,t there
(B)
BaltiC Sea sediments, where the growth 0 Industrial actiVit IS consistent WIth these IIndiIWS
(C)
BaltiC Sea sediments, finding growth at mdu trial actiVIt
ID)
edlment from the Baltic ConSistent With the growth In the r
IE)
ed,ment from tile lh growth f inCkIstrll. . .
rates of liability Insurance have risen to force rales of liability Insurance are a force for rales for liability In sur ance are forCing rises In liability Insurance rales are forCing
C
74. While some academicians believe that bUSiness ethiCS
(CJ
r Ih n 7 arefully engineered up I
makes
Ii bllity Insurance rales have rI en to force
lD
zz
81 pracIucinJ .1IIf1 center ~
9B6
when the Department of Labor began to SInCe 1 shnent officers' tees to be based on alloW IIJlII! anage perform. seyeral corporatlo~ fundS they m -d
h~
The computer company's Pl'esentlrli......
B"
andsprud
) (A
III VlIOI:
or spreadlnl I8SIlIt of los of Vllor, or spreading -- ~._ 11110 of Vllor, or pread
resultine 10
I
of Vllor, and spread
[ 'n., CClmputer company has announced that It wIll PlllcIIIlle Ihe olor-Prlntmg dM Ion of a rival company mI!jon whICh IS oart of a dea/that WJ1~e Ie t manufaclurer In the office color prlntlng
I8J (OJ
rnlIion. whICh I parI of a deallhat WIll make million a part of a deallhat makes miff n a part of a deal makIng
m Ion a milion a
82.
Oownzonlng, zoning that tYPically results In the reduction of hOUSing density, allows for more open space in areas where ~1'@1eI..Q[ rVI s XI .
pre ent and indicate that
ndlllon I pr
(B)
so that several attemp s a rE!VJsmg CllrDorall! strategies have failed to corree
IC)
In that several attempts a rE!VJs g cO'POrate strategies have failed to Correc em
(0)
that several attempts to rev se corpOrate strategies have failed to Correc
IE)
that several attempts at reVlS ng c rpOra e strategies have farled to Correc e
or
the Advancement of Teaching, compan es e United States are providing Job training a d genera education for nearly eight million people. about gguivalent to the enrollment of the na on s four-year colleges and universities.
0'
IS
so that several attempts to rE!VlSe COrIIOI: strategies have failed to correct all!
85 According to a stUdy by the Carnegie Founda
a parI a deal to make parI of a deal Ihat will make
COlldltion wh nil' pr .........'nl
stag,~nati~on~~;;;::::::
oltechnotOglca/ managerial blunders ~rporate strategies have fa led to C!lrrect il
nt and Indicate Itldllt
IA)
equivalent to the enrollment of
(B)
the eqUivalent of those enrolled
IC)
equal to those who are enrolled In
In
(A)
little water or services eXist
(0)
as many as the enrollment of
(B)
little water or services eXists
IE)
as many as are enrolled In
(C)
few serVices and little watel eXists
(0)
there IS little water or servICes available
(E)
there are few servICes and little avarlable water
IA)
now It draws on the wor s both of cont:emtOOOr} H,spalllc authors who II e abroad and a
(B)
now draws on the war s of COl1!efTl1lorarY Hispalllc authors, both those who and those who live
IC)
It dra vs on the ark of contemlponry Hi$plllll: authors now. both tho e IMIlg DOlO live
(0)
dr aw no
(E)
HI paniC author hVlng draw on the rk now HI palll uthor h Ibn"'l~
86.
nt lid Indicate It 83
ndillon wh n III th r dlld
In theory, International CIVil servants at the Unrted Nations are prohibited from continuing to draw alarles 'rom their own governments, In practice, however, some governments merely substitute liVing allow nces !QrJl!flr .emD/Qy Wy k I by lh :m to the Untied Nations. (A)
lor their employee 'paychecks. J signed by them
IB)
for Ihe paycheck of thell employees who b n a Igned
(e)
lor lh p y h k of Ihelr rnployee, haVing ben Ignd
(0)
In plac ollh Ir mploye tho ofth m Ign d
ha~
'p y he k , for
in pI ofth P h k olth h b n Indbythm
II
mplo
to
on the
war both
I11III'''.
......M'reVeItNIIIlevl_ 2nd Edition
-::~:::pr e sed foods and excelling at 11 olncldental Wade wa nght any apparent connection the eating of Id Dr Wade be nght, any apparent n bon of eating Wade I nght any connection that IS pparent between eating of It Wade I nght, any apparent connection betwe n eating uld Dr Wade have been nght, any connection pparent between eating
o
Seismologists studying the earthquake that struck 91. northern California In October 1989 are stili IOvesltgatlng some of its mystenes: the unexpected power of the seismiC waves, the upward thrust thqL threw one man straight Into the air, and the strange electromagnetic signals detected hours before the temblor. (A)
the upward thrust that threw one man straight IOta the air, and the strange electromagnetic signals detected hours before the temblor
(B)
the upward thrust that threw one man straight Into the air, and strange electromagnetic signals were detected hours before the temblor
(C)
(D)
t funding for the park's development, which """IV 1111 upen to the public early next year, IS f r development of the park, which p nto the public early next year, be fun4~inl f r th development of the park, nto th public early next year, to be r th p rk' d velopment, perhaps open rly n xt y ar, be dMlIIotllflltlnt funding lor th park, which could ubll rly next year, I to be
(E)
the upward thrust threw one man straight into the air, and hours before the temblor strange electromagnetic signals were detected one man was thrown straight into the air by the upward thrust, and hours before the temblor strange electromagnetic signals were detected one man who was thrown straight Into the air by the upward thrust, and strange electromagneltc signals that were detected hours before the temblor
Two new studies Indicate that many people become 93. obese more due to the fact that their bodies bufJ!. ~Iories too slowly than overeating. (A)
due to the fact that their bodies burn calones too slowly than overeating
(B)
due to their bodies burning calones too slowly than to eatmg too much
(C)
because their bodies burn calories too slowly than that they are overeaters
(D)
because their bodies burn calories 100 slowly than because they eat too much
(E)
because of their bodies burning calones too slowly than because of their eating too much
th ur to a home at the end of each da IOstea to confine them to a hotel.
(A) (B)
IC) (D)
I)
the freshwater snails that are the parasite's hosts for part of ItS hfe cycle the freshwater snails that are the parasite's hosts In part 01 their hfe cycle fr shwater snails which become the parasite's ho ts lor part 01 ItS hfe cycles Ir shwater snail which become the hosts of the p raslle dunng til para it's IIIe cycle para It's ho t ,fr hwal r snails which be om therr ho t dunng lh rr hI ycl
f
(A)
to allow members of the Jury to go home at the end of each day instead of to confine them to
(B)
that would have allowed members of the jury to go home at the end of each day Instead of confined to
(C)
under which members of the JUry are allowed to go home at the end of each day Instead 01 confinmg them m
(D)
that would allow members of the JUry to go home at the end of each day rather than confinement m to allow members of the Jury ta go home at the end of each day rather than be coofined to
92. SchistosomiasIs, a disease caused by a parasitic worm, IS prevalent In hot. humid climates, and It has become more widespread as Irrigation projects have enlarged the habitat of ~ freshwater snails that are ~illJ1e's hosts f.QLDart of ItS life cycle.
a
94. Judge Bonham denied a motion to allow members of
(E)
96
are concentrated mthe monsoon June to September, and the skies are .,rerally clear for the rest of the year.
181
(D)
IE)
98
(A)
WhiCh, causlOg plant and animal extinctions, marks
(B)
which caused the plant and aOimai extinctions and marks
(C)
and causlOg plant and aOimal extlOctlons that mark
(0)
an event that caused plant and animal extinctions, which marks
(E)
an event that caused the plant and aOimal extinctions that mark
Unlike the UOiled States, where farmers can u ually depend on rain or snow all year long, the ra n m most parts of Sn Lanka Unlike the United States farmers who can usually depend on rain or snow all year long, the rains m 010 t parts of Sn Lanka Unlike those of the United States, where farmers can usually depend on raIn or snow all year long, most parts of Sn Lanka's rains In companson with the United States, whose farmers can usually depend on rain or snow all year long, the rains In most parts of Sn Lanka In the United Stales, farmers can usually depend on ram or snow all year long, but In most parts of Sn Lanka the rains
Although Napoleon' army entered RUSSia with far more upplle than theY had mtheJ! PLe.VllW.s. camoall!ns. It had prOVISions for only twenty·four days. (AI BI
100 Sixty-five million years ago, according to some . SCientists, an asteroid bigger than Mount Everest slammed IOtO North Amenca, which, causing Pla~ ~nlmal ex,,"ctlons, marks the end of the geologiC e P .d ra known as the Cretaceous eno .
they had mtheir prevIous campaigns, their prevlOu campaigns had had,
States Constitution and Important as a body of work In political science as well, represents the handiwork of three different authors. (A)
and important as a body of work In political science as well, represents
(B)
as well as an important body of work In political SCience, represent
(C)
and also a body of work of Importance in politIcal science IS representing
(0)
an Important body of work In political sCience and has been representative of
(E)
and as political sCience an Important body of work too, represent
of Harriet
:::~:~~~~:e:~~ all of th qualllie 10 th tac of dang r, an l!Y, and n bihty to plan In
of
0;;; C!!ntJct. the
(A)
In isolatIOn from contact With Europeans Ion than ger
(B)
Isolated from contact with Europeans longer than
(C)
in Isolation from contact With Europeans Ion er than were g
10)
isolated from contact With Europeans longer than were
(E)
In isolation and Without contacts With Europeans longer than
(A)
Although the first pulsar, or rapidly spinning collapsed star, to be sighted was In the summer of 1967 by graduate student Jocelyn Bell,lt had not been announced until February 1968.
(B)
Although not announced unlil February 1968, In the summer of 1967 graduate student Joce n Bell observed the first pulsar, or rapidly sp nrung collapsed star, to be Sighted.
(C)
Although observed by graduate student Jocelyn Bell in the summer of 1967, the discovery of the first Sighted pulsar, or rapidly splnn ng col apsed star, had not been announced before Februar
101. The Federalist papers, a strong defense of the United
th y had for any pr VIOU campaign, nth r prevlou campaign , mpalgn, f r ny pr VIOU
InUits of the Bering Sea were 10 ISQla!!lm It 103. With Europeans longer than Aleuts or Inuits North PaCific and northern Alaska.
102. As buslOess grows more complex, students mafO(lng, lQ P CJ9md areas like those of finance and mark: tlng1la.\'e. bttrLllecomlng increasingly uccessfulln the Job market. (A) (B)
(C)
(0)
maJonng In specialized areas like those of finance and marketing have been becoming IncreaSingly who malor In uch specialized areas as finance nd marketll1g are becoming more and more who m Jared In peclJlized areas such as those of financ and m rk tlng are being lOerea mgly who majOr In pe laliz d ar a like tho of fin n e nd marketing h v b n be omlOg mor nd more mlJor din u h p
1968. (0)
The first pulsar, or rapidly splnn ng collapsed tar to be Sighted was observed In the summer of 1967 by graduate studentJocelyn Bel but the discovery was not announced unt! FebnJarY I
(E)
The first Sighted pul ar, or rapidly 5PfIVllTlR collapsed star, was not announced un Febfllllry 1968, while It wa observed III the StJlImII' 1967 by graduate student JoceI)'n
h wof Pliny the Elder wrote the only 109 =e:s account of the great eruption of Vesuvl~ ~ ;;Ietters to the hlstonan TaCitus.
ntral on generalJOns of bluegrass .......... wa also an ,nsplralJOn to many lan thatmcluded Elv,s Presley and Jerry ...''0". who e mu IC differed Significantly from infIue d generallons of bluegrass artIsts. also r d many mu IClan • Includmg ElvIs Presley a d J rry GarCIa whose musIc d,ffered gnrficantly from wa mnuenllalto generations of bluegrass rll t . wa aloin prratlonal to many mUSICians. !hal 10 luded [Iv, Presley and Jerry Garcia. who e mu IC wa different significantly In ompan on to wa Innuenllal to generations of bluegrass rll I • al 0 lrl plred many muslc,ans. who n luded [ivi Pre ley and Jerry GarCIa. the mu I of whom d,ff r d ,gn,f,cantly when mp r d to n ,nnuen on em',dllons of bluegrass w loan In Plr tlon to many mU!;ICl
(A!
The nephew of Pliny the Elder wrote the only eyewitness account of the great eruption of VesuVIUS In two letters to the hlstonan TaCitus.
IBI
To the hlstonan TaCitus, the nephew of Pliny the Elder wrote two letters, being the only eyewitness accounts of the great eruption of
VesuvIus. IC) The only eyewitness account IS In two letters by the nephew of Pliny the Elder writmg to the histonan TaCitus an account of the great eruphon of VesuvIus. (DJ Writing the only eyewitness account, Pliny the Elder's nephew accounted for the great eruption of Vesuvius in two letters to the historian Tacitus. lEI In two letters to the histOrian TaCitus, the nephew of Pliny the Elder wrote the only eyewitness account of the great eruption of Vesuvius.
110.
wr
WI"..
d
_IY
p tin II
au II
(Al
Being a United States Citizen since 1988 and born In Calcutta In 1940, author Bharati Mukherjee has
(B)
HaVing been a Umted States citizen since 1988, she was born In Calcutta In 1940; author Bharatl Mukherjee
(C)
Born ,n Calcutta In 1940, author Sharatl Mukherjee became a Umted States citizen In 198B; she has
(D)
Being born In Calcutta In 1940 and haVing been a Umled State cllizen since 1988, author Bharall Mukherjee
(E)
HaVing been born In Calcutta In 1940 and being a Ulllt d State Cit, zen slnc 1988, author Bh rali Mukherjee
nergy
prlMl1lted from dl Ip lin by
."llWIIfllllJlCllr mb In
~d five centunes after Eurooeans arrrted 10 III N wworld on C I mb Da 1992 Pr the ~ed a 5100 million Investment In the sear~;; f ~terrestnalmtelligence. IlL (A)
Initiated five centunes after Europeans arrIVed the New World on Columbus Day 1992. Pro ec SETI pledged a $100 million Investmen 'n the search for extraterrestnal Intelligence.
(8)
Initiated on Columbus Day 1992. five cen unes after Europeans . . arnved 10 the New World .a $100 million Investment In the search for extraterrestrial intelligence was pledged by Project SET I.
(C)
Initiated on Columbus Day 1992, five centunes after Europeans arnved in the New World. Project SETI pledged a $100 million Investment In the search for extraterrestrial Intelligence
(Dl
pledging a $100 million Investment in the search for extraterrestnallntelligence, the initiation of Project SETI five centuries after Europeans arrived In the New World on Columbus Day 1992.
(E)
Pledgmg a $100 million Investment In the search for extraterrestrial intelligence five centunes after Europeans arnved In the New World. on Columbus Day 1992, the initiation of Project SETI took place.
B
gene a G ee epICS and 00ze1lS
B
' lhe 1lOII!lrY ,afPllllr.
IOOst oI
ZIId Edition
S.8 Answer Explanation
Key
D
8 A D
A
30 B 31. e 32 A 33. 0 34. 0 35. [ 36. [ 37 B 38. [ 39. [ 40. e 41. E 42. E 43. B 44. 0 45. E 46. e 47. e 48 A 49. e 50 C 51 A 52 8 53 0 54 C 55 8 56 B 57 A
8
E
A
59. 60. 61 62. 63. 64. 65. 66. 67. 68. 69. 70.
A
71.
8
72.
e
73.
A
A
e 0
e 0
e 0 A
e 8
74. 8 75. D 76. E 77. 0 78. D 79. E 80. C 81. 0 82. E 83. 8 84. D 85. E 86. 8 87 D
88. 89. 90. 91. 92. 93. 94. 95. 96. 97. 98. 99. 100. 10l. 102. 103. 104. 105. 106. 107. 108. 109. 110. 11l. 112. 113.
The following discussion of sent ence cOrrection effective approaches to these kindS f representative of the kinds of se t 0 questions n ence correction
e D 8 A A
0 [
e
I
Like ants, termites have an elaborate so I f d d cia structure in which a ew In IVI uals reproduce and th e rest ire.. ~rvlng the colony by tending luvenlles . gath enn&. food. building the nest. or they battle Intruders. (A)
A
(8)
E E
Ie)
0
E 8 8
D 0
A 0
a e E
e C E D
ID) IE)
are serving the colony by tending Juveniles gathering food, building the nest, or they batUe are serving the .colony In that they tend juvenlIes gather food, bUild the nest, or battle serve the colony, tending Juveniles, gathenng food, building the nest, or by battling serve the colony by tending Juveniles, gathenng food, by building the nest, or by battling serve the colony by tending juveniles, gathermg food, bUilding the nest, or battling
Parallelism; Rhetorical construction The sentence most effectively uses parallel structure to contrast two type of termite in the social structure of termite colonie: tho e who reproduce, and tho e who serve the colony in a number of ways. The progre sive verb form art serving should be changed to simple pre ent ten e serve to parallel reproduce. In the final Ii t of responsibilities, paralleJi m demand that all assume the gerund form as objects of the preposition: by tmding ... gllthmng . .. bui/Jin rn battling.
A
B
The progressive verb torrn art stroin I inappropriate for this general claim about termite behavior. It hould parallel th previous verb reproducc. It i unn introduce a new clau e or thty in!Tllda,. hecause p
kward
2.
The volumIRoUs personal papers of Thomas Alva Ed,son reveal that hiS ,nventlons typ,cally Sprang t2.Jl!t nol,n a flash __ of IRSPlrat,on bul evolved SlOWly from prev,ous works,
4
""are not parallel to h"
I produces an agn:~m~nt not parallel to ,(oulmg,
smh"
t
(AI
sprang to life not ,n a flash of ,nspiration but evolved slowly (BI sprang to life not IR a flash of II1splratlon but were slowly evolved IC) dId not spnng to life IR a flash of II1Splrat,on but evolved slowly (0) did not spnng to life 111 a flash of InSpiration but had slowly evolved (E) did not spnng to life In a flash of InSpiration but they were slowly evolved
lie IIlIIIKI _>er i B. ffMyN RHlCh Umted Slales volers may agr~J te In governmenl and Ihallhe government
pend beyond ,Is means, ,t 's d'fficult to broad supporl for a movement toward a m'Rlmal
01 )
However much URiled Slates voters may aaree that De prte the agreement among Un,led States voter to the fact Although UnIted States voters agree Even though URited States volers may agree There I agreement among Un'ted States voter that
Parallelism; Idiom The construction nol . , , bill shows a contrast. The words following not must be parallel in construction to the words following bllt, In the original sentence not is followed by a prepositional phrase (i,z aflash of inspiration), while bllt is followed by a verb (evo/ved), To make the two contrasting elements parallel, not should be followed by a verb rather than a phrase.
,.,.11 II m; Grammatica' construction
I
wnllw mIl'''''', paralld rom t lau arc fi,lIolV"d by ,I mili" dall,e, ....... r II I lIhordlllal" I LIlIses ,Ill' bllth int1rodu d hv Ibal Ihu' 'b, r, IJ ... "lid 'bu' 'h,' thl
rr
II}
A
~,""mml
B C
The construction followll1g 1101 is not parallel to the constructlon following but. The construction lollow1l1g not IS not parallel to the construction lollowing but, Correct, In this sentence, 110t is followed b) the verb sprm!!; just as hilt is lollowed by the verb ,·vo/Vi'd.
\)
Hud, , evoh,,'d introduces an incorrectl'crb tmse.
E
'[he ronstructlon lilllowing 110t is not par,dld til the constructlon following hut.
Ion 01 111,11'
The correct answer is C.
5
then
Iv • [
Q1Qr
cornrnYOIcatlO!! and navlgatH1g.
lAl
Ilh r to hnd and It k pr y, 10 d 1 nd th m elv • or I 0 for ommuOi llIl!! nd
n VI tina
IC) (OJ
(El
either for finding and attackll1g pr~ del themselves, or for communication and navigation to find and attack prey, lor defense or commuOlcatlon and navigabon for finding and attacking prey, to defend themselves, or also lor commun cabon and navigation to find and attack !hemseIves . prey. to defend . or to commuOlcate and navigate
Idiom; Verb form The sentence explains that fish di charge electnc currents for several purpose •which afe most efficiently and effectively described in a parallel structure: tofind and attack, to defend, or to communicate and navigate, The use of either i inappropriate in this sentence becau e more than IIVO uses of electric currents are listed; idiomatic usage requires either to be followed by or to identify alternatives, not by aIJo,
A
Either inappropriately introduce a Ii t of more than two alternatives, and it should not be followed by or a/so; parallelism requires that for communicating ,md navigating be changed to to commlm",lIe and navigate.
B
Defend is not parallel with the li t of gerunds. leaving the reader to wonder how to make sense of d1~nd thmlse/·'es.
C
lhe lack of paralleli m obscures the relationships among the items in the :en • it is especially confusing to Ii t,an inhrun~-e phrase (Iofillt!, ..), an object 01 a preposmon (lor t!1~nse). and noun· with no gramman:a1 connection to the \'erb phea e (fOmmllnimtlon and n,H I ~h n.
D
Hundred of species of fish generate and discharge leclnc current, 111 bursts or as steady electnc fields around Ihelr bodle ,USll1g til Ir power Illi' [ tQ..fillil ~1lW:JlJH:eoHlelend
(B)
E
This ,m.wer choi,ce.•11 0 'iolate ,parall~ by mi.ing ,m intinltlve WIth ob of p~epositi~n for. Or also i an urudioma COntradictory expre Slon orrect. The different v.triou sp d 'oflish_Xlupd."j••~. pllwer ar correctly e par,lll I infiniti correct Il8S'WU
B
......1IId right and obllgahOns Involved In the l. . . .lbuIlionand consumpllOn of those resources III,: : : : : and obllgallOns Involved In their and consumption dIlined right and obllgaltons as they were tMlIvIdm its dlstflbulton and consumption whllH right and obligations were defined In stribubon and consumption distribution and consumption of them dIfined by fights and obligations " 111m, logical predication; Grammatical ClDII!drulctlon ClOII'I'Il4~1
wntt n entenLe usc, a series of n tru tions to descrihe how the ·I(lJ'IWIIOl:iel:Y orgam d ,., providt·d . ..• and th v rb match each 01 her. an t parallel I m.
Ca,,":ct The three verh, arc p,llallc! in this ~finljrrKI
not parallel to orgalli t'd .lIlll
AllIJI.y Wtn'tl word and InlpreL i c, ,11I.1 r f, renl
Delighted by the reported earnings for of the fiscal year, It was deCided bv - __ t,Ipnt ~nager to give her staff a ralSI:
!he-"I..ilI
9
II
th parallel nd C
D
E
(B)
the deCISion of the company manager give her staff a raise was to
(C)
the company manager deCided to gIVe her staff a raise
(0)
the staff was given a raise by the company manager a raise was given to the staff by the company manager
Who wa delighted? The company manager was delighted. The long modifying phra e that introduces the sentence describe a person, not it, so the delighted person must be the ubject of the sentence. Correcting the modification error also changes the construction from the wordy passive voice, il was decided by x, to the more concise active voice, x decided; the active voice is generally preferred.
By is needed between phones and wing to clarify that this sentence explains how people avoid charges. AlII lime which should he replaced by when.
A
The modifier illogically describe it. not the company manager. The passi\'e voice i wordy.
B
The modifier illogically de cribe tlx decision. The con rruction decision ~"tlx ... 'was ... i-wordy. Correct. The modifying phra e correctly modifies the complmy "hl11ilger; using the active voice create a more concl tence The modifier describe- Ihtstilffrath r thaD Ihe (ompany mtllhl. er; the pa 'w wordy. The modifier iUogicalh' descri passive Yoi,e i. word).
C
[lIJleat! nonsensically contrasts avoiding lOll cll;lr~es With the method for ,Iccomplishing thIS. Dllrillg which is awkward and unclear, espCl:iall)' as dllrillg appears tWice in the same phrase.
D
Correct. 'Ille sentence is dear, concise, and ~r.\Illmatirally rone,!.
E
Ill' i~ .needed hetween phoneJ and wing to clant)' the explanation of how petlple .Iwid churKe>· Wh II inappropriately t.lkes the plolL'e Llf bl'. ob unnK th t;"t th.1t IIsin~ mobile phone t m ht and on w ckcnds is .1 mcthoLI tor Idmg toll harK s.
It was deCided by the company mana her staff a raise ger
Logical predication; Verb form
This sentence explains how people avoid toll charges on their home phones, and this purpose is best clarified by using the preposition by to connect the phrase call avoid . .. with the explanatory usillg Iheir mobile pholles. The phrase al a lime which is confusing because it seems to sugge,t a time other than al nighl or 011 weekends. Which is the wrong relative pronoun for referring to time; substituting when for the entire phrase streamlines the sentence and makes its meaning clear.
./1
(A)
(E)
Rhetorical construction; Grammatical construction
A
rrd btl al,on IlIoglL.111I rdt'r
'--"."-1
phones. uSing their mobile phones for making (AI long-distance calls at night or on weekends, at a time which phones. Instead uSing mobile phones to make (B) long-distance calls dUring the night or weekends, dUring which phones by USing their mobile phones to make (C) long-distance calls at night or on weekends, When phones uSing mobile phones for making long(0) distance calls dUring the night or weekends, When phones when uSing their mobile phones to make (E) long.distance calls at night or on weekends, a time which
The correct answer i
10
Ihl: !Ising of costs of data-proce many financial institutions haS oPPortumty lor Independen these service more fllc,illlltttlil
IAl 8)
D
11
C D
_ . .~tbe . . .II.d.me noun It becau e It~:::=~~swhen.
..-and
indiJect.
"t.1l1lDiIll:Jlhas no main
'ntrDtiMud and 'luirkly .~ adjectival phrases.
lIll1-ce IS properly structured
correct e entence has no main ;HruClll and adoptedboth
tives. . . .~.tlIll~IeIDtence has no main ..... ilrltedMt:Iti and hofJing bem ... adjectives.
E
-,ullltcomplete the idIOmatic _llD1lcorrectl . CIftlIlCt.ln thl sentence, to correctly COJIIplell:s the idiomatic construction range ", K toy. And to does not complete the idiomatic . . expression correctly. With does not complete the .dlomatic expression correctly. In addition to does not complete the idiomatic expression correctly.
The correct answer is B. Growing competitive pressures may be encouraging 13. auditors to bend the rules In favor of cltents: aUditors. may. for instance, allow a questionable I?an to remain on the books in order to maintain a bank s profits on paper. (A)
clients; auditors may, for Instance, allow
(B)
clients, as an instance, to allow
(e)
clients, like to allow clients, such as to be allowing
(D) (E)
clients; which might, as an instance, be the allowing of
Grammatical construction; Rhetorical construction "A~nson
artistic debt to Scandinavia IS that range from sensitive portraits wife Danish home. Kerteminde, allll IWS of ~ords and mountain peaks In M1~11 hIIr1hem reglons of Norway. ~n,s
This sentence correctly joins two independent dau e with a semicolon. The first clause makes a generalization; the second clause gives a particular example that supports the generalization.
A
Correct. This sentence correctly has twO independent clauses with linked ideas joined with a semicolon.
B
In trying to condense two main clauses into one, this construction produces an ungrammatical sequence of words with no clear meaning.
C
The preposition like should not be used to introduce the infinitive phrase to Ill/OW . .. ; the COmparative preposition like is properl}' used to draw a comparison between twO nouns.
D
urh aJ to be aIlUWi"\f1l~lQI". idiomatic expression The semicolon is followed by inco rred construction rather -01 • • ~~ independent clause.
The correct answer is
14
.
The themes that Rita Dove explores In her POetry uDlversal. encompassing much of the human cClI"'Itillll. Wile occasionally she deals with raCial ISsues (A)
IS universal, encompassing much of the human condition while occasionally she deals
(B)
is universal, encompassing much of the human condition, also occasionally it deals
(e)
are universal, they encompass much of the human condition and occasionally deals
(D)
are universal, encompassing much of the human condition while occasionally dealing
IE)
are universal, they encompass much of the human condition, also occasionally are dealing
Agreement; Grammatical construction The plural subject of the sentence, themes, requires the plural verb are in place of is. Because the themes of Dove's poetry encompaH the human condition and deal with racial issues, there is no need to make this a compound sentence by introduce a new grammatical subject, she, in a final clause. A single subject with two parallel verbs is the clearest and most efficient form for this sentence.
A
B
C
The plural subject disagrees with the ingular verb. The sentence hould retain the focus on the single subject themes rather than introduce a new subject and clau e at the end. . elisagrees with the The plural sublect Is ...singular :.J. . ''Iy it _ 'fINII verb. Because also occasional' introduce a new main clau e, til comma between condition and also i an UllI connector (creating a comma p\i The comma between univmM insufficient connector, splice; the ingular verb -~".".,. with the plunu ubJ t...-,_ "'.'.'
B
1lIf~"'1III1 Drllilld a being a ,arne ammal
senrence muoduces all these am travelers wouldhaw to do, repeating 10 construction in the final item of the list redundanr.
eel a a game ammal
..,..""nt; GRlmm.tlcat construction Th pronoun and nouns that refer to the plural n un rguanar mu t be plural, as should the verb fOllowmg rhe (correcred) pronoun in rhe second tau e Thus, the eotencc should read; Iguanas . .. Ihry ar. sIIII/,nut! ,IS game allimal<.
A
A
II if and ag"me "lIimal do not agree with
B
The verb form warding is not parallel with the other items in the list.
C
Correct, The sentence uses proper grammar and parallel construction.
'KUtll1t1J.
B
II" docs nol agrce with igllallas or game Illlimak
C
Correct. In thi' 'cnrcncc, Ihey art' and game '/fIllllill< propcrly agrce with igllalll1J.
D
.rI g,/m,' illIlIIWI doc, not agrce with igllanas;
D
h'lllg" unnele"ar)' and awkward.
1 !;tlllie '1II111111! doc, nnt agree with 'gllallas. 'Ihe "wnd inuepcndeot clau,e rcquires a uhjet t ,Illd .1 verb, not thc participle b,'mg. The corn:ct an.wcr i. C.
7
Traveler to Mars would have to endure low levels of graVity for long periods of lime, aVOid large doses of radiatIon, nILwl h .ct!Mll.c.~11 r IV Mwt.an ~~ .......",r V n ltilvlngJQ ward off ontamlllahon by Marllanlife forms, (A)
51
ontend wIth the chemically reacllve Martian 011, and perhap even haVing 10 ward
I I
E
I I
I
The phrase havillg to is not parallel with the other items in the list, and it unnecessarily repeats the sense of the introductory phrase, which identifies all items in the list as things travelers would have to do.
The participial form contending violates the parallel structure of the list of infinitive phrases. The reader is misled into thinking that contending with chemilally reactive Martian soil describes what travelers would have to do to avoid radiation doses. The participial phrases eontmding with and warding riffviolate the parallel structure established by the list of infinitive phrases.
18.
Except for a concert oerformance that the composer himself staged in 1911, Scott Joplin's ragtime opera Treemonisha was not produced until 1972, sixty·one years after ItS completion. (A)
n nd WIth the hemiCally reactive Martian I nd p rhap v n ward
Except for a concert performance that the composer himself staged
(8)
Except for a concert performance With the composer himself staging it
Ie)
Besides a concert performance belllg staged by the composer himself
(0)
Excepting a concert performance that the composer himself staged
with th
heml lIy reaclive Martian V n wardlnll
completed actio
D
Excepting usually app constructions; it is not th this sentence.
E
This sentence is awkward ant
The correct answer is A.
19. Chillese, the most ancient of living Wrltlllg systems, consists of tens of thousands of IdeographiC characters, each ch racter a mllliature calli ra hlc composition inside Its own square frame. ([)
IA)
each character a miniature calligraphiC compOSition IIlslde Its
IB)
all the characters a miniature calligraphic compOSition inside their
(C)
all the characters a mllliature calligraphiC composition IIlside ItS
(0)
every character a miniature calligraphiC compOSition Inside their each character a miniature calligraphiC compOSition inside their
lite correct answer is C.
ntend With th ch rmcally reaclive Marlian II, and perhap ven warding
4l11f1nd III With th hemically r active Martian d p rh p ven to ward
Belngsta~d
(E)
Agreement The underlined part of the original -enlen.:e am as a modifier, or a phrase in apposition, de:.:nbmg Chinese ideographic characters. The moJlhe~ Correctly uses the singular tor all three term. chtllilet..;; compoJitioll,',md its all agree.
::.::::::: _iIII
Agreement ..........
B
_ .......~ V.rlMl R_w
d Ed t "
(El .41.wlI!ld property values can hurt some large • •brI. !bey are ootenbally devastaliO& for eqUity ,n many cases sentmg a hfe s savings-can plunge or even ppear
lAl
they are potentoally devastating for homeowners, who e BI they can potentially devastate homeowners In thatthelt for homeowners they are potentially devastaliOg, becau e thelt (01 for homeowners. IllS polentrally devastallng In Ihalthelt EI II can potenllally devastate homeowners, whose
1111
cnlcm< I "''''., I
,lila wnrise.
r t<
10 trot rly 1'1/11/1'1,
and
111
who I"
Logical predication; Rhetorical construction This sentence uses twO complete clauses to presen tWO main topics-the capabilities of robots t designed by Japanese researchers and the goal that motivates thiS deSign. The first clau e mOSt effectively uses a succession of parallel verbs to describe what the robots can do: identifj expressions and respond to them. Beginning the second clause with the possessive pronoun their creates ambiguity, because it is not clear whether the pronoun refers to the robots or the researchers.
A
Rh torlCllI construction; Agreement
'!hey clearly refers to
B
"'
orre r.
lht r,",,'IIIJIl,hip helween nouns
nd pronoun pr
"p
B W
o
I
lflHu
t In
Ion I l WI( I l .1IIe1 l
Ill/mIll"
rd! wkwar
this sl'nh.'ncc; the
kar.
r"luIIII.IIlI; 111 IhrllIht'Jr IS I•.lIld .lIl1hi/o:UIIIIS,
The pronouns they and their in this version of the sentence are ambiguous, possibly referring to both researchers and robots. The phrase then responding to the", should be converted to a main verb to parallel identify and to make clear that the robots can do these two things. The semicolon should be followed by a complete clause, but in this version of the sentence it i tollowed by an adverbial phrase.
Idiorn; Agreement an idiom that ;0 Th e sentence uses . ~ correctly
expressed as thmk ofx aJ y. The Use of to ht I incorrect.
A
To be is incorrect in the idiom I th L ~ In"ajx asy.
B
L Being is incorrect in the idiom to 110'm~ofx asy.
c
Being is incorrect in the idiom to think ojx as y. 7helr does not agree with a rOTlJUtrler. Correct. This sentence uses the idiom correctly: think ofhouseholdproduC/J aJ.
D
E
14. In recent years cattle breeders have increasingly used crossbreedrog, In art that their steers shoul a Ult certain characteristics and partly because crossbreedrog IS said to provide hybrid vigor. (A)
In part that Iheir steers should acquire certa n characteristics In part for the acqUisition ot certain characteristics In their steers partly because ot their steers aCQulnng certa n characterrstlcs partly because certaro charactenstlcs should be acquired by their steers partly to acqUIre certain charactensllcs In tile r sleers
C
Correct. The wording is ~oncise and unambiguous.
(B)
wllh tfO/' fll' t'rlll/"'; IIIlhrll wonl, md Iwkw..,d
1)
The phrase (/5 well (II givl/Ig ,I respollSe to them IS wordy; the pronoun th,,;r is ambiguous.
(C)
E
R"'/,olldillg is the wrong verb torm-it should be an infinitive to parallel identifv.
ID)
wnh /'1 t flV f'"ll/eI; Mlrml'ial. I r dund III n I I r r
(jill
1lle correct answer is C. r produ Ing a ell IderltHy human f I I
of
pando
lhIm p m Illy
23. Consumers may not think of household cleaning pr!lll hazardous substances. but many of them can be harmful to heallh, espeCially If they are u ed Improperly. (A)
IB) ( )
101
The passive-voice construction is awl-ward and wordy.
(E)
Parallelism; Rhetorical construction
Consumers may nol think of household cleaning produc ts 10 be
The sentence gi\'e· twO reasons that cattle breeders use ~rossbreeding; the'e rea n should be intr duced in par,llle! wa~ . with the p.m/\,. The inhnitive to ,lffjur" cl adv and Conciseh convey· the purpos of th
Consumer may not think 01 hou ehold cleaning product berog
A
Aconsum r may not think of Ih u household I nlng product bemg A on um r m y not think of hou ehold lean
pr du I
H
Id I nln produ I m
D
Eo
"lheCOii«l
25
The correct answer is D.
1h PI' ar 10 refu 111110, I'ltlO/,t'IIV 'VI/II/t'J) rh r Ih n III 10 ", OWI/m. n I. r
l
expressions and responding to Ihem; prrmanly Ihe researchers' goal IS crealll1g ,
crossbreedin~.
B
/1/ t.m ,hlluld be partlv. U d.lUs fh
11/ /'
...
~
81 C 10 (E)
logical pr.... ilia; " . " " "
MI._...
lllwlew nd d on
",tunng COmpany has for several : : : : : : lImed at decreaslOg operallOg the efficiency of rts dlstnbulion : med at decreaslOg operating costs and Improving rmed at the decreaslOg of operallng costs and to mprove mng at the decreasing of operating costs and Improvmg the aim of which IS the decreasing of operating a IS and Improving with the aim to decrease operating costs and to mprove
(Dl
(E)
To obtain an Investment-grade ratmg would keep the county's luture borrOWing costs low, protecllOg Obtaining an investment-grade rating, keeping the county's borrOWing costs low, would be protecting
Verb form; Parallelism
hnlliell.m; Rhetorical construction
This sentence lists three benefits the county will experience when it obtains an investment-grade rating. The auxiliary verb WIll appltes to all of these future benefits, so all of the verbs listing those benefits must take infinitive verb forms to parallel (wi!!) keep. Moreover, the possessive COIIIIIV! in the first verb phrase must be paralleled in th~ successive phrases with the possessive pronoun ils.
nu
A
Correct. The sentence u es correct verb forms and correct parallel structure.
B
The infinitive subject To oblain suggests that obtaining an investment grade rating is an objective in and of itself, not a means to achieve the desired benefits; the repetition of and between the phrases in the list is unnecessarily wordy. The correct verb form to anticipate future consequences is the present-tense gerund oblaining rather than the presem-perfect form having obtained. This version of the sentence is not onl), confusing, it is also word),. lhe faulty parallelism is confusing; prolectillg introduces an adjective phrase describing the subject of the sentence, but this makes illcmlSe into a verb with no subject. 11,;s sentence implies that obtaining an inveslmcnt-~rade ratin~ and keepinl( com low are the same thin~; this version of the sentence makes the rete rent of the posse"i\,c pronoun ils uncertain beclusc the posse<si\'l~ pronoun no \unl\er identifies with the po~ essive noun in a I\r;lmmatic;llly par,lllel n
(0)
neet ent nee uses the I(rammatically paraUel element decreOling and improvillg to d nbe the two aims of the company's policy. orrect. Decrrasl11K and improvillg arc grammatically parallel; aim"d al is a correct nd on ICC prc sl<m. a II/g and 10 improve arc not parallel. In th hefore d((rttlling LreatC' a /(erund, whl h I not parallel to the participle 11/
if
huh I awkward 'lOd wordy; Ih,' • ,"d';1I/ not parallel to ,mproviIlK. WllmlfJed m to 1 not the
Eating saltwater fish may slgmficantly reduce !be 28 ~art attacks and also aid for sufferer of oak. and asthma ' aCCord rheumatold arthntls . I n g to three research studies published In the New England .lou of MediCine. mal
D
E
(A)
slgnrficantly reduce the risk of heart att ks also aid for at and
16)
be signrficant," redUCing the risk of heart attacks and aid for
Ie)
slgnrficantly reduce the risk of heart attacks and aid cause a signrficant reduction In the risk of heart attacks and aid to
(D)
IE)
Slgnrficantly reduce the risk of heart attacks as well as aiding
B
Diction; Parallelism The word aid can be a noun or a verb; here it should be a verb that is parallel to the verb rduce. If aid were a noun, it would parallel risk and so would mean Illogically that eating fish reduces aid for slIfferers as well as the risk ofhearl at/acks.
C
Aidfol seems to be a noun, parallel to the noun rISk, indicating that ealing sa/tu:aterfish reduces aidfor sufferers. Aidjor seems to be a noun, parallel to the noun rISk, indicating that eating sa/ruaterfish reduces aidfor sufferers. Correct. In thi sentence. aidi u cd a a verb, parallel to the verb rduct. ufferro i the direct object of aid; no prepo irion i needed. fitd 10 i incorrectlv used a a noun, uggesting that e'liillg s'l/ruater.fish mlu
E
A
B
C
D
aid to sl/jJ;·n~T5.
E
\ Vhile this semence conve the correct meaning, it lacks the paralld tr\lCtIIft ftlund in the correct an wer
The correct answer i 29
n w ri
o that It that It
D
30
1be
UDlIJ1IbJgUOIK and
~ conect and use amount and
",l\IIlrsteIR stuIIJ
deCade young men and of the effects of a divorce
rrrmdMr conectly. D
E
lIy mdlcatc that Eakin-' lOla thc prc cnt day.
Iau_e:tan
B attaching the adjective I1IcreaJing only to y t o"J'haU omoun ,"r debriI, the sentence. fails to indicate that the number of sa~elhtes is also growl.ng. This leaves the funcllon . d of and. the number ojlatelliteI uncertain an confuSing. This version of the sentence is indirect, wordy, and confusing.
The correct answer is C. 32.
like Rousseau, Toistoi rebelled againdst the unnatural complexity of human relations In mo ern society.
ton of nuclear reactor fuel have already orbit around the Earth, and the chances coIlllsion Involving uch matenallncrease greatly IS lb'l/DOunl of both space debns .aolUalelli1e~
(A) (B) (e)
(D)
the amount of both space debris and 8telhte continue to nse til n e continue In both the amount of IIlte and pac d brlS til mount of pac debns and the number I Iht onllnue 10 me th ontlnually Increa Ing amount 01 space nd Ih number 01 at llite m unl of pac d bn continUing to I n with Ih number 01 alellites
8
~""'II:
.....
orlltal con tructlon
• •1Ke operll
with a III III) dille l"till~ ,. nu I r n:ador fud orh.t III~
(E)
like Rousseau, Toistoi rebelled like Rousseau, TolstOl's rebellion was As Rousseau, TolstOi rebelled As did Rousseau, Toistoi's rebellion was Toistoi's rebellion, as Rousseau's, was
logical predication; Rhetorical construction The preposition like correctly compares two equal nouns, in this case, two writers. The comparison must be between two equal elements, it cannot be hetween a person and an event. The original sentence is direct, clear, and conCIse.
A B
Correct. 1he two writers arc compared clearly and succinctly in thiS sentence. JolItoi:r r,'btllioll rather than Tohtoi is wmpared to ROIIH,'CI/I.
C
\oVhen used as a conjunction, cH should intmdlKe clauses, not phrases or nouns.
()
l'o/rloi" r,'btl//oll is compared to ROIIH,·all. To hI' l"llrrert, this construction would have to he,ll did ROII'h'CI/I, Fo/rlo/ rd,d/,'d, but this is ,. wordy ,.hernativc.
I:
1'alIto/'r rdld/ioll ... 11'c/I agai'lJl is 'IWk·W,. rd .lIld wnrdy; '!b/llor ,d'd/c'd ,'glf/wl is nlllre direr!.
(A)
(81
leI (0)
lEI
occurring when a child occurring when children that occurred when a child that occurred when they were children that has occurred as each was a chid
Logical predication; Agreement The original sentence has two problems. (t The phrasing implies t~at the young men and women had divorced In childhood. (2) The word cbiJd does not agree with the plural antecedent men and women.
A
Child does not agree with men and u;omen, and the phrase occurring when a child is illogical.
B
The phrase occurring when children i illogical.
C D
Child does not agree with men and v:omm.
E
D
Correct. This sentence correct both problems by making it clear that the divorce took place when the men and women were children and by u ing they and ..hildren to agree with men and 'women.
E
Each does not agree with men and i;.'0mnr; hal occurred i not the correct verb ten e.
The correct answer is D.
35. 34. Lackmg Information about energy use. people tend to overestimate the amount of energy used by eawprnent u h a Ii h tha ar VI Ibl an m and off and underestimate that used by unobtnJSM! eqUipment, such as water heaters
IA)
(B)
(e)
(0)
IE)
eqUipment, such as lights, thaUre stimandlllllllll must be turned on and off and unctere vi equipment. such as hghts, thatUlare . .t5~1iIllIllI"'~ I must be turned on and off and when equipment, uch as hgh be turned on and off and Uftd_ VI Ible eqUipment turned on nd 0 11II:1 lIB. VI Ible equ pment. turn d n
n, Verb form
buildings cannot be both ."'JAhi rgtd at the same time; they moat lfther. The ideas of this sentence are ressed using two verb tenses: the IiJnpIe ~. for the earthquake occurring last YQr; and the past perfect, had been, for the time pnor to that when the building were constructed.
1IIOIt."
a11til1h clauns about a
JMllltiit IlOntmues elsewhere, • " t back into the United polio has been eradicated lU 18 a condition of the other _expressed in a subordinate
A
The buildings are illogically said to be both destroyed and damaged.
B
Correct. This entence properly states that the buildings were either destroyed or damaged and clarifies that they had hun constructed before the earthquake struck.
C
Buildings cannot be both destroyed and damaged. The verb tense make it seem that they were constructed after the earthquake.
B
D
The verb tense illogically indicates that the buildings have been constructed since the earthquake.
c
E
This structure indicates that construction of the buildings, rather than the earthquake, occurred last year.
D
lllIIcOItd appearance of it as the subject of and IS leaves polio as a subject with 'DIe pronoun it is ungrammatical because vlio already occupies the subject position for the verbs continues and can be brought. The sequence of events in this version of the sentence is confusing. The possibility of polio being reintroduced in the United States is a consequence of its continuing elsewhere, but this version of the sentence obscures that connection. D
Although not grammatically incorrect, this version of the sentence is word} and indirect. Correct. The sentence is clear, conci e, and grammatically correct.
The correct answer is E.
37
SQme bUildlOgs that were destroyed and heavily damaged 10 the earthquake last year were CQnstructed In vlQlatlQn Qf the city's buildlOg cQde.
IAI IBI leI 10)
SQme bUildlOgs that were destroyed and heaVily damaged 10 the earthquake last year were 5Qme bUildlOgs that were destroyed or heaVily damalled 10 the earthquake last year had been orne bUildlOllS that the earthquake destroyed nd h vily damaged last year have been t y ar the earthQuak d troy d or heaVily m ed orne bUildlO1l that have been t y r orne of the bUildlO1l Ih t w r 4MtrOYld r he vlly d m g d In th rthqu ke
E The correct answer is B.
38. Astudy commiSSioned by the Department of Agriculture showed that if calves exercise and assQciated with other calves, they will require less medication and gam weight Quicker than dQ those raised In confinement. (A)
associated With other calves, they will reqUIre less medication and gain weight QUICker than do
IB)
associated With other calves. they requ re Ie medication and galO weight QUicker than
(e)
associate With other calves. they required Ie do medication and Will galO weight QUicker than
ID)
associate With other calves the hi less medication and will gain wet&ht more qUickly than do associate With other calveS tit medication and gain wellht more
IE)
V Afunded pemion system I B put and oontinue .lnlllftlllCt to u the present
:
:::::~Wben a choIce is
C
'INr tha" con tnIction-lhe
•••It"'''" thtmy-the x and the
y
':===~ In thi
case, the doctors have rather than (to understood) Pl(. To lIi.tllllllld to be repeated in order to maintain . . . . . . .lIIIbec:awlelt I understood.
D
•
'II hows an action complctcd in thc
E
rIy IYhlYmml is nor parallcl to jim. H~"I ~,r._'11
hows an action complctcd in thc t; IYItlYmml and pcmg arc not parallcl.
II.
D
.
Fang and 10 relire early arc not parallel. Olrect. In this scntcncc, have elecled shows a tJOn ontmuing into thc prcscnt; 10 relire and (to understood) jilC<' arc parallcl.
41.
Unlike a funded pension system, In which contributions are invested to pay future benefiCiaries, a pay-asyou-go approach is the foundation of Social Security. (A)
COITeC:tan
er i E.
e Gorton Dodd bill reqUires that a bank disclose to !bel[ customers how long they will delay access to funds from deooslted checks. thaI a bank disclose to Ihell customers how long th y will delay access to funds from deposited
heck nk to dl close 10 thell customers how long Hdelay access to funds from a deposited
_:II. ~
......,.IV
do not share one check. Correct. In this sentence, the pronouns ils and il agree with II bank, and deposiled checks is a logical fit with cuslomers. Requires a bank Ihal il should is not a grammatical construction. The singular check is illogical since the customers do not share one check. The passive voice is 10 be delayed conceals the bank's role in delaying access. The singular check with the plurallheir is illogical.
10 e 10 II customers how long 1l:elIIS to funds from deposlled
t:
the foundation of Social Security is a pay-asyou-go approach
(e)
the approach of Social Security IS pay-as-you-go Social Security's approach is pay-as-you-go
(0) (E)
12
Twenty-two feet long and 10 feel in dlamel h er, the AM-l IS one of I e many new satellites that IS apart of I e rs elfort f ub eclln he Interaction f ~mosphere. oceans, and land surfaces 10 detailed scrutiny from space. (A)
satellites that is a part of IS years effort of subjecting the interactions of Earth's atmosphere, oceans, and land surfaces
D
(B)
satellites, which is, a part of a IS-year effort to subject how Earth s atmosphere, oceans, and land surfaces interact
E
Ie)
satellites, part of 15 years ellort of subjecting how Earth's atmosphere, oceans, and land surfaces are Interacting
(D)
satellites that are part of an ellort for 15 years that has subjected the interactions of Earth's atmosphere, oceans, and land surfaces
(E)
satellites that are part of a IS-year ellortto subject the Interactions of Earth's atmosphere, oceans, and land surfaces
Social Security is founded on a pay-as-you-go approach
Thissentence contrasts two systems, afil1lded /,t'mloll system and Social Security. The sentence must be structured so that thc contrast is logical and grammatical. After the first (flllldedpel/sioll) 'ystem IS mtroduced and de cribed the second (Soc~l/I Semrily) system must be int;oduced. The onglllal sentence makes the mistake of Contrastin" tjt-111m, 1, { . . " l Pt'llSIOII sysNm With a pa)'-"s-
.
-
Afimded pensioll .rysl,·m is contrasted with an approach rather than with SOfial S,'(//I'i/l"
B
43. Though tile
Rhetorical construction; Logical predication
Logical predication
yOIl-go approach,
&enltna
'Jhe correct answer is E.
a pay-as-you-go approach IS the foundation of Social Security
(B)
Correct. The two systems are a logical, grammatical way in t~ IS
The correct answer is C.
" rtc'll must be followcd by an
InfinitiVe (10 retire).
Social Semri'y' approach rather than Social Seml'lly.
1hrir and they do not agree with a bank. 1hrir and they do not agree with a bank, and the singular check is illogical since CUstomers
Afundedpension system is contrasted with the foundatioll rather than with Sodal Se(//ritl'. Afundedpemioll system is contr.lsted wirl~ the appTrJarh rath r than with So, ial Se, urilY·
This sentence describes one atellite and identilie it as part of a larger space project de igned to scrutinize Earth's ocean, land, and atmo pheric interactions. The relative pronoun th'lt refers to atellites, so it hould be tollowed by a plural verb. The idiomatic expression i tffirt to rather than 1[ort of The correct adjectival term i 15-~ rather than 15 years. A
The relative pronoun that refers to tdli so it should be followed b the plural al'e; effol't to is the correct idioma expression; as an adjective. 1
15-year,
lBl
!Inlt""_
...... ""'"
Logical predication
lhis senrence conrrasts largl Ilulplanl' wirh 'mall
":~::~the Ingular noun If by the Ingular II
milk Since rhe lir t half of the sentence begins with in (onlrml 10 large ,fedplanlI, the second half should begin with Imall mills to make the Contrast
_,... pJunJ " Wid. Iy !~!~~~ nve ahon that moves
.
.:t1l__
D
I I
A
Large 'lee!planlI should be contrasted with Imall mills, not with pro((wng Ilul Ilrap,
B
Large 'lee!planlI appear to be contrasted with pro(ming lleel I(rap rather than With Imall mills; remaining and 10 pUI arc not paralleL
C
D
ugg , d and Ih'l' hm't do not awee with t rm, widllv ranging work" impre,'i,e. 45.
Large IlulplanlI appear to be contrasted with the promsing '!Islee! Itrap; remaining and to put are not paralleL
Correct. In this sentence, large Itee!plants are clearly contrasted with small millI, and to put is parallel with (to understood) remain.
alii". but this lIII_ the
always or definitely removed. little sense in relation to the re t of entence.
E Correct. The sentence coherently refen rhe possibility of heat and pre SUre bei to removed. The ,entence is clear and ~ and properly uses parallel "erb form . The correct answer is E.
lb The psychOlOgiSt William James believed that facial expressions not only proVide a viSible sign of an @lotion, actually contributing to the feeling rtse~.
IB)
emotion but also actually contributing to the feeling itself
The correct answer is D.
Ie)
emotion but also actually contnbute to the feeling itself
Under high pressure and Intense heat, graphite, the most stable form of pure carbon, changes Into the substance commonly referred to as diamond and remalOing this way whether or not the heat and pressure are removed.
IDl
emotion; they also actually contribute to the feeling of it
(E)
emotion; the feeling Itself IS also actually contributed to by them
E
H.
tee I plant thai take I,on arc
Inlhis way is unnecessarilyWlJldr, indicates that the heat and ~
emotion, actually contributing to the feeling Itself
ugg "Id doc' not awee with In/fl. '/he verb t n e u~est a lompleted art ion rather than an onw.;n/( one.
B.
v
(AI
HIl
_reel milNer i
8,1",aining should be para1IeJ1IIII1h1 wh(lher or nol i unnecetearily.....
immediately obvious and thus easy to understand.
anoth r; In rhl ontext. it i th work doe not move from another .If widl rangl '!Iwork show tlaltd. work I h of many different kind of not agree with Ilrm; widd., ImprecISe Corre.~ . In th,. entenle, If ha' a/(rees with , and th phra e a wid.. "mgt oj work I1I8ph a varrety of project,. Hm Ign!fitd ul(ge t a ,ompleted 'ktion nd thu dJ tort the mean in/(; u'iddl' 'lin Ing work IS impred,e.
c
Remained violates the parallelism with to pUI, and its past tense is not compatible with the present perfect verb have bem able.
Idiom; Grammatical construction (A)
remalOing this way whether or not
(B)
remalOing like that even as
(el (D)
remaining as such whether or not remains In thiS way although
(E)
remains thus even when
Parallelism; Rhetorical construction 'I!ti, 'entenu' lell, of two thlll~' th,It happen to Kral'hite undl'r inten'e he.It .md pressure. and th s arc h 5t pre'enl"d .1' p.lr.llI<1 prl'dic.lte'dJll/Igt and rmll,i,ll. 110/11 i, the mo't "c<1n0l11ic.11 way to ay Ihi< 'WIIV, likt 110111, .,. ",,10. llf in Ihi' mIl'.
This sentence should depend on the correlati\'l: construction 1I0t 011/1' x , . ' bill tllso y. where and .v are paralleL How~ver, the fault)' con tru tion m the original sentence does not properl) mclud the second element. blll.llso, and 0 produ a entence fragment. James oars that til 1al expression' have two ettect ': the\ pro of emotion and the" contribute to eaaotiioD. in thi .entence. 1I0t 0111\, should be fOIJo prot'id,' " ,'/JIbI.. si II O(tllI ffllOtio -,•• should be t"Howe:l b" (y) a"'uaJ, firling
R mtllning shuulJ he .1 1\I.lIn wrh. p.lf.lllei with (hang .
R mllmmg hould b p.lr.llld with th oth r b, han
n In
•
II
f hil. hi h
j/j
I
U
not th
t th In
fit of
8
i"<'If
(OJ lEI
Iltillrollllellns had been the recent l'mlllVprobIems has been the recently
problems 's the recently IllII1Y problems IS the recent tIIeir IllII1Y problems had been the recent _ _ _Int; Verb form; Diction
'the COIftCt u of pronoun rclerence, verh tense, modifier make the cnren'e dear and ea,y to undenrand The mgular po ,e"ive pronoun il; to the mgular noun rtloiler. The pa't peltQ:1 verb had /wn indi,ate' action completed bc6m! the tlon in the ,mple past ten'e ;old. The adJCCtlI/l:", nl modi tics eXlmded 101" ;Iump.
I
I
ParaIIe II'sm', Rhetorical construction Wh the construction is from x 10 y, x and y mUSt en t'cally parallel. In this case, be gramma I . hx and yare . . onmenral problems: x IS t I' pollutIon the rwo envtr '. or b I nd!;lI; andy is thepollutIon ofaIr by f!J water y 10 =t" . . tor~. Starting the parallel phrases With the lnanera pollution emphasizes the similarity of the problem; each of the other elements, ofwater/ofOIr and by landfill;/by incinerators, emphasizes difference. '0
A B
orred.
B
The dverh mmlll' modi ties onJy lhe ad) t,v Imded, 'lIggl'sling illogically lhal the I lump ha hel'll rm lit/V rXlmd,·d. 1 how pr cur, r.Hher Ihan mmpletl'd, I n, and the adverh r,/< lit/V modi tics only rhe ad) live r Imded,
/101
agrn' w H h I h"
D
_.1Id d NII.11
InCln r tlon of olld npr of hitting from IIMfill1 Pollytlnil the
Correct. The correct use of parallel structure clarifies the meaning of the sentence. Pollution of. .. is not parallel to incinerators
E
Water that is polluted incinerators that pollute
.
50. The bank holds $3 billion In loans that are seriously delinquent or In such trouble that they do not expect payments when due. they do not expect payments when It does not expect payments when it is It does not expect payments to be made when they are
(0)
payments are not to be expected to be paid when
IE)
payments are not expected to be paid when they Will be
tlilt
c
p"yments is a plural noun, so the singular II
is i< incorrect. Correct, In thi~ correct sentence, pronouns nd their referents agree, as do subjects and :heir verbs. The addition of the modifylllg phrase to be made clanfies the meaning of the sentence.
The active voice is preferable here, since the passive voice leaves it unclear who doe not expect the payments to be made. Payment> ... to be paid is redundant. Are not to be incorrectly suggests that the writer i prescribing that the payments not be expected. E The active voice is preferable here, since the passive voice leaves it unclear who does not expect the payments to be made. Payments . .. to be paid is redundant. Wil! be is not the correct verb form.
D
is not parallel to
The correct answer is C.
IA) IB)
8
B
c D
E
The correct answer is C.
thaI. ...
Ie)
11I1- flClIUIlS for th
Lack of parallelism makes these two phrases difficult to understand. Landfills polluting . .. is not parallel to the air being. ...
C II; agree, with rdai!erJ; the past perfe thad /wn indifates aClion prior to the Imple pa t /0/,1; and rt(ml modi fie, exlmd,d at. lump
pollution of the water by landfills to inCinerators that pollute the alt water that IS polluted by landfills to inCinerators that pollute the alt
Bank require~ the singular pronoun It not the plural pronoun they. ~e, truetu~ of when due creates ambIgUIty 10 meanIng.
;1 In a 5·to-4 deCISion, the Supreme Court ruled that two
upstate New York cQuntles owed reslltutlon to three tribes of Oneida Indians for the unlawful seizure of their ancestral lands In the eighteenth century. (A)
IBI
Ie)
(0)
Agr ement; logical predication; Verb form
Th plural pronoun tht'l' ,'annlll be used tll refer to th singular noun b,mk. '[he slrurlure llf thn''/o "01 t1tptcl Paynltllt wh "d,u is awkw.1tL1 and un r
IE)
that two upstate New York counties owed restitution to three tribes of Oneida Indians for the unlawful seizure of that two upstate New York counties owed reslltutlon to three tribes of Oneida Indians because of their unlawful seizure of two upstate New York counlles to owe restitution to three tribes of Oneida Indians for their unlawful seizure of on two upstate New York counties that owed restitution to three tribes of Oneida Indians because they unlawfully seized on the restitution that two upstate New ~1Ddi Counlies owed to three tribes of Onetda for the unlawful seizure of
52. Recen1Iy ci5aMIId that the AJSfJIII'.I." branch of the ....
(AI (8)
lC! IDI
_ . . . .'ihhv""''''Znd Edition
Rhetorical construction; Idiom condlllOns and, uSlllg the set of objective Criteria denved from these analyses, deCide conditions and, In their use of the set of objective cntena they denve from these analyses, they deCide
(Ol
thar the mammals . . . .,IR thought to be an offshoot of the ",,,,,,111'" 'l'oilltion and not a .. ~jIil_ilentof II ",mon IInwtor 0/ [all] mammals. ~n make a contrast usinR thc llOIlIItructJon x roth" than .I' or x instead 0/.1'; x and v'nRJIt be parallel m ellhcr casc. Thc mammals ImJrlth rather rhan (I'); hcrc V should I ofan article and a noun to match a branth. 1he nd halfof thc contrast may bc rcwrittcn a thIIt tJ. loptd tndepmdmtlyftom to complete the parallel construction. The idiom indepmdmtly ", I dlffi rent m meaninR from the idiom NlepnuJmt 0/. thc 10Kic of this sentence requires the u e of tndepmdmlll'from.
eEl
Grammatical construction; Verb form
I I
Thc noun clause introduced by that has one subject (executivrJ) and two main verbs (examine and deride). These verbs need to be in parallel form. The information about using objective criteria describes the executives and is therefore most efficiently presented as a participial phrase (using ) rather than a prepositional phrase (;n using
A
Dewloptng indr/,mdmtlvfroll1 is not parallel to abranth.
8
o
B
orner. l1,is idiomatically correct sentence properly uses a ~V/,'· in parallel to n branch. The vcrb dewlo/,"d i, preferable to the awkward and wordy relative clause using the noun dewlopm"'l; indl'/,mdmlo/distorts the orlKmal mean mg.
C
D loptng tndeplltdmllyfrom is not parallel to a branrh. While il tJ. 'rlo/,Il1' nt may appear to par;lllel n h, ad .dopmml thtlt w'" indt'/,Clld,'nt pr e a me,millK CCllllr.try 10 that GJIII'e.aed In th orl!!;n," scntcllce. The verb pr f, rahle to lh,' /loU/l tk11N!/~Ob,m nt
D
E
IIIll11I11Ct llIUWCT I. B.
).
and kw Th e underlined clause is wordy"aw ardl.
.J' .fsome other "octor IS not a COllect d 0; . Th' I I IOrnatlC
expressIOn: IS cause must be replaced ore conCIse construction. lrth" I. by a rn ~ elIa a,"ofc h be replaced by t e preposition witho t: an . b d u • rO"'e olher Jar/or IS etter expresse as another .J (II • UIJ(/Qr, and testimonycan be substituted for to teuifj. The result, WIthout another doctor; teui", I . c any, c earl) expresses III rOur words what the original statement poorly conveyed in eleven.
A
The clause is wordy and awkward' some other doctor is not idiomatic.
B
The construction y will happen unless x happens first requires the present tense following the unless clause, rather than the future tense used here.
(e)
(Dl
1heCOiltet
This alternative is awkward and wordy.
IAltokeep
to executives.
E
Lacking illogically and incorrectly modille" damage. "
(8
to keep Il "~m
Ie
to
ID
III order lIIIlitwlldftl!lll
By using the coordinating conjunction and, this version of the sentence creates the need for a second subject and mam verb; this second subject is absent. The participle deriding cannot function as a main verb. Correct. The sentence is grammatically correct and uses proper verb forms to express a clear and logically coherent mes age. This version of the sentence is wordy and indirect, largely because of the repetition of the pronoun they.
A patient accusing a doctor of malpractice Will find It difficult to prove damage If there IS a lack of some other doctor to testl~ about proper medICal procedures.
(8)
L,('
D
c
there IS a lack of some other doctor to testify unless there Will be another doctor to testify Without another doctor's testimony hould there be no testimony from some other do tor I'
IuJdnl nother doctor to testify
Correct. This sentence uses a phrase that is clear and concise.
The correct answer is C.
56. Archaeologists
d1::~==
centu discDVeled was
stolenbyiINadl!rS.
IE)
order III lIeep........:'~
Granuiialkal _ _tiI
55. The energy source on Voyager 2 is not a nuclear reactor, in whICh atoms are actively broken apart rather a kind of nuclear ballery that uses natural radioactive decay to produce power.
lA)
apart; rather
(8)
apart, but rather
Ie)
apart, but rather that of apart, but that of apart; it IS that of
(D) (E)
CAl
I
laC", 'J.J
This version is unnecessarily wordy and indirect. There is no need to repeat the subject, executives, with the pronoun they. This version is unnecessarily wordy because it creates a compound sentence by repeating the subject, using the pronoun they to refcr
The correct answer is D. 54.
1
D E
Grammatical construction; Logical predication The correct version of thi entence fucu on Contrast by using the con truction n I rather 1'. :\ com rna, not a emicolon should separ;te the two parallel pans ofth U ing a semicolon re ults in en unless a subject and v rb PIl)viciecl~~ n truction that follo th
Correct. Parallelism is mamtained
E
jHoponent • a proposed new ••If .n:nlfl could by slumming along the iatWl..... llllIlllIlOSPhere fly between most pornts under two hour
skilllll1illlalong the top of the atmosphere. tIRl",lIfl1lsof a propo ed new style of allcraft
could fly between mo t pOint on Earth In two hour Ii d new tyle of allcraft could fly betWlH~n mo t point on Earth munder two rd ng to It proponents. with It IkIrrwrlllll alona the top of the atmosphere Ii d new yle of allu aft, ay It propon nt, ould fly between most pOints on rth n und r two hour beCau e of Its IrJlmln alonllthe top of the atmosphere Accord,n to It proponent • klmmln/l along the of th atmo phere make It pos Ihle' that a pr d new tyl of alfcr It ClJuld fly between t POmt on arth Iflunder two hours. • •rIcIl con tructlon; Logic I predlc lion
The correct answer is E.
The correct answer is A.
58
~aln pesticides can become rneffect!Vl! rt used .- re ea dl In th same lace' one reason I b h findln that there are much lar r ill pestiCide-degrading microbes rn Salls Wllh i!.. ~Iatlvely long history of pesticide use than In so S l!!ill are free of such chemicals.
Lawmakers are examining measures that would reQUile banks to disclose all fees and account reqUirements In wlltrng. prOVide free cashing of oyernment checks and to create basIc savmgs g-d . accounts to carry minimal fees an reQuile minimal rnltlal deposits.
:J
(A)
prOVide free cashing of government checks. and to create basIc savings accounts to carry
(B)
prOVide free cashing of government checks. and creating basic savings accounts carrying
IC)
to provide free cashing of government checks. and creating basIc savings accounts that carry
(0)
to prOVide free cashing of government checks. creating basic savings accounts to carry
(E)
to prOVide Iree cashing of government checks. and to create basIc savings accounts that carry
IAJ
111~:
,0111\1
\
11 Ill'
If used repeatedly in the same place. one reason that certain pestiCides can become rneffective IS suggested by the finding that there are much larger populations of pesticide-degrading microbes In soils With a relatively long history 0 pesticide use than rn Salls that are free of such chemicals.
Ie)
If used repeatedly In the same place one reason certain pesticides can become Ineffective IS suggested by the finding that much larger populations of pesticide-degrading mICrobes are found In soils With a relalivel long h torY of pesticide use than those that are free of such chemicals
(D)
The finding that there are much larger popliillllO of pesticide-degrading microbes III relatively long hlstor of pe hClde use Salls that are free of such chenuc suggestive of one reason If used rep. ..., the same place. certain pe ltC rnelfeclive.
lEI
The finding of much Iar pesticide-degrading rrnc rei tlvel long hi of tho e th tare fr one re on rt UI DtSitid. melf tl If US
,h.II (1111 \' IS .1 ,,'Il',lrl"f cunstruction.
/'ro ,j.!, .\IId 10 ,mIl, .Il,' not p,ll,d ld. 10 ",rrr ul1d,',\( ,lIld '.111 h, c 'n ,I 1I1.1kll1g the ,lIoglt,11 1.\ll1Ilh.lt thl'plllpll l'l'( -~e,l!lng th~ ,I '"llll! I til, ,In\ 1111111111," kc ,ll1d r llllir I1lll1ill1,ll dl'llll It . I
8
"i alld I "",,,, I
"'I/I~ .ar ' lIllI p,Il,III<1
nUlI'I ,llId \
do nd "V I
n th tl
t
"l
1',1 mil I 11\ lin I Ir ,1l1r1',11I b II a
1110 I Ih
Ith,
I//I/.tr' IIllt
1I
111\ Ih I Ih purl
OUIII I
til
I minIm I d pu
Uri 1I\111l1n It
1
Certain peslicldes can become rneffec e rt used repeatedly In the same place; one reason IS suggested by the finding that there are much larger populations of pestiCide-degrading microbes In soils With a relatively long history of pesticide use than in soils that are free of such chemicals.
(B)
Parallelism; Verb form '111c corrcct vcrsion of thc sentence uses parallel structure to describe \\'I1rIll. In thiS C,lse, the other twO 1t"IUIll'll1r'l1ts '';111 he ~Ivcn ,I either lopro",d, .. _10 ,1",I,'Of /'10 ·,d,· ... ,ml/, , \\'Ith the I? understood. 111 ,Iddltlllll, \I Il1g th,' s,\lnc II1tinitlH l,,,,n lor ,I dltlrr 'I1t plilpO ,. in 10 ""II' is potenti,dh
min
sentence by following 10 diItlosnrith pro'1.'jde and 10 c~eale. In this Sftting, the form Ihal carry IS more readily undentoocl than 10 carry_
This version i wordy and repetitive (poJJiblr and ,ould repeat the ame meaning); because the antecedent for its is 0 far from the opening phrase. the reference is unclear.
lIS
c D
E
.....dIlIon
(A) (e)
. . . .' ...IdIaltlDn
irtItCUrml:t!y wn~n It u e parallel reatlOllS why textbook nd quarter : rro~nuf. Ilrt IJ The on truction III /h mmK "~hoof )'lllr i Liear, h awkward lOn truclIon 1I ing 1II1b1j~11US plural pronoun th Y and th,,,.
nlenle u e Ihe p",.dlel Ilr hIgh and employs rhe o"'/,,,nu a rhe ubjell of
7I.",,""m IlIogllallv 10 rder 10 r,'Vel/U". bJe I of /'r '/',Jr I Ihe .Imhigllo,,, ,h,y. not parallel 10 lou', .11,,1 it gIves no and. tlon of whal Ihe 1I1mparison is ppo d to he (Ilighcr than what;). 'Ihis n tru lion make It appear, illogirally, th t th low revenm h.lV<' higher marketing p n
H
(0)
(E)
Parallelism; logical predication This sentence is primarily concerned with presenting the findings of an investigation, so the main subject should be investigation, and the main verb should be hasfound. The findings are most efficiently presented in a parallel series of relative clauses beginning with that.
A I par,llIello/ow .1I1l1 "illllgil.d mpan 011 I hung 1Il.ldl'; Iht'll has r r f, r ot n<
_rect IIIlI1WCr
B
I I
an InvesllgaliOn of Iwelve hospItals by stale medical offiCials has found that all twelve cons,stently break the laws. thaI many reSidents work longer Ihan 24 hours straight, and thaI more than half Ihe surgical reSidents work twelve hospitals were Invesllgated by slate medical offiCials who found all twelve breaking the laws, With many reSidents working longer Ihan 24 hours slralght, and more than half Ihe surgical reSidents work an InvestlgaliOn by state medical offiCials has found that, of Iwelve hospitals. alilweive conslslently break Ihe laws. thaI many reSidents work longer Ihan 24 hours slraight, wllh more Ihan half the surgical residents working
11,e phrase finding that all.. has nothing to att.lch to in this sentence, bec",,,e hospItals has been made the subject of the sentence; the ilems in the list of finding arc not presenled in parallel form. 'Ille smgular subjccI requires a SIngular verb; the items In the list of findin,~, .lfC nOI presl'nlcd in p",allelltlrm.
(
Correct. 'Ihe sentcncc's structure m"kes it de'" and loglc,d.
()
'Ihe preposilion WIth gOI'crns hoth ex"mples ol ho\\' thl' hosp;I.lls hrcak rhl' I.n\', sO p,lr.lllcI, sll1 rcqUlrcs rh.1l bolh \'crbs .1 'sume 11ll' InK cnding.
E
1111' Cnlenl'l' is rqll'riri\'c .Ind indirerr; thl' h , of hndings " not p!l'scntl'd inl',lf"lIc1 form.
18) Ie)
IO! tEl
10 do II rewards dOing II rewards to do thiS would reward doing so would reward 10 do It would reward
Diction; Verb form The problem in the underlined section i how to refer back to the verb acc()fd. The cleare t and roost standard way is to u e the adl'erb J" rather than a pronoun such as it or this. The verb rtUartb is incorrectly in the indicative mood, the mood used to state a fact; in the conre.xt of a hypothetical action, the conditional would m::ard is more appropriate.
B
c
A
The pronoun it does not have a clear antecedent; the adverb so is preferable. Rewards should be would re1l'ard.
D E
B
ll1e pronoun it does not have a clear antecedent; the adverb so is preferable. Rewards should be would reward.
1IIctwitd
C
11,e pronoun this does not hal'e a clear antecedent; the adverb so is preferable.
D
Correct. The adverb so is correctly u ed to refer back to the verb accord; the conditional would r,"lL'ard is appropriate in relerring to omething contrary to fact,
E
The pronoun it doe not hal'e a clear antecedent; the adverb so is prelerable.
The correct answer is D.
6J Many policy e perls say Ihal shtfltng a portton of health·benefil cosls back 10 the workers 1le!PS to control the employer's costs, but also helps to medical spendtng by maktng palients more care consumers. (A) (B)
Ie)
helps 10 conlrol the employer helps helps the conlrol of the lIlIlI.....'. . . .~ nol onl help to ontroI bUI also help helps to control not nnIII.iM: not onl helps
Illd I
64
~=~===
......,. ."".1I1ld EcIItJon
.....l:)'tIed comrnod;ties like alumlOum
(A)
strong despite economic 109 Industry (8)
ClimnlOdiities Irke those of aluminum and other metals are remalOlng modll.es such as aluminum and other meta remains modll,e , such as aluminum and other remalO mmodltles, Irke the commodilies of aluminum and other metals, remains
(e)
(0)
met.,
(E)
Agreement; Rhetorical construction
logical predication; Agreement
Th mgular ubje
The sentence has been written so that aula il1Jurance is contrasted with Ihefrequency 0/claims. The correct contrast is between the premiumsfOr auto insurance and the premiumsfOr personal properly coverage.
Th plural verb docs not agree with the mgular ubject. B L th o/Ifldllates that aluminum and th r metal possess commodities rather th n emphfl' them; the plural verb IIr,' mm doc nor agre<' with rhe SIngular bJ t mark I. CcilfnlCl. The v rb agree wirh the subject, u h a prop rlr e prc",cs the million hlp b tw n r I vdrd (Ommodil;" and
A
Auto il1Jurance is illogically contrasted with thefrequency ofclaims.
B
Unlike with is an incorrect idiom; tll/to insurance is contrasted with thefrequency of clllims; the singular subjectfrequency does not agree with the plural verb do.
C
nd th r m lak
D wordy ,md hi phra mg ould """lUtfor rytl. d r eqUlV I nt to th ",,,, d'ht
Unlike auto Insurance, the frequency of claims does not affect the premiums for personal property coverage, Unlike With auto Insurance, the frequency of claims do not affect the premiums for personal property coverage, Unlike the frequency of claims for auto IOsurance, the premiums for personal property coverage are not affected by the frequency of claims, Unlike the premiums for auto insurance, the premiums for personal property coverage are not affected by the frequency of claims, Unlike With the premiums for auto IOsurance, the premiums for personal property coverage is not affected by the frequency of claims,
I
E
rf/.WIIIJinlll",alld oth , m lak
thefrequen~v ofclamlS is contra ted with the premil/msfOr personalproperty coverage.
Correct. The contrast between the premiums fOr aI/to illSl/ranu and the premiumsfOr I'aso/llllproperty coverage is clearly and correctly stated in this sentence.
" 1.I11~ike with is an incorrect idiom; the plural SUbjeCll'rt'nlil/l1Is docs not agree with the smgular verb is 1101 '11;'(/et!.
The correct answer is D.
67
Faced With an eslimated $2 billion budget gap the city'S ' mayor proppsed a nearlv 17 oercenlreductlon
~:h~t:~:~I~~I~~~: ::1=%=:~
hundr d of loc Iarts groups.
(A)
proposed a nearly 17 percent reducl!on In the amount allocated the preVIOUS year t ., I omamtam the city s major cu turallnslttutlons and to subSidize
18)
proposed a reduclion from the prevlou . Syear of nearly 17 percent In the amount It was II . t th 't' a Oealing to main aln . e CI yS major CulturallnstltulJons and for SUbSidiZing
(e)
proposed to reduce, by nearly 17 percent th amount from the prevIOUS year that was . e allocated for the maintenance of the Clty's m . . . ~or cultural institutions and to SUbSidize
ID)
has proposed a reduction from the prevIous year of nearly 17 percent of the amount It was allocating for maintaining the Clty·S major cultural institutions, and to SUbSidize was proposing that the amount they were allocating be reduced by nearly 17 percent from the prevIous year for maintaining the city's major cultural institutions and for the subsidization
IE)
Rhetorical construction; Parallelism The original sentence contains no errors. It use the parallel construction to maintaill and to subsidize to show clearly the two areas where the 17percent reduction in funds will be applied. In addition, the 17 percent reductioll is closely followed by Ihe amount allocated the previollSyear, making it clear what is being reduced by 17 percent.
A B
c
Correct. The sentence uses parallel construction and a well-placed modifier. To maintain and for subsidizing are not parallel. The sentence is impreci e, and It does not have a clear antecedent. For the maintenance and 10 Jubsid/ are not parallel, and the entence i wordy.
D For maintaining and to subsidi are not parallel, it does not have a clear anreeedent and the sentence structure m It U just what the writer i claimm
E JIIt/int,/ining and the sUMd pilralld, thn'doe not ha a antecedent: and the n n strllicllue milkes it unclear ju t h laiming. CQne(:t answer
Ie
(D
IE ParalleflSIlI VI" The senltDa:
companu:s
_rA1
antettdcnrofdlll .lDl1IIIf
..
'_.I_iII
new CIlIIIIpII., compc:ntne )lIiIl"
_."",!lII"~
B
70.
with more than 75 carefully engineered stnlClures. of up to 600 rooms each, were more than 75 carefully engineered s1hilCtu,re of up to 600 rooms each, more than 75 carefully engineered "stnft:tUlre of up to 600 rooms, each that had
The gyrfalcon, an Arclic bird of prey, has survlve~ a close brush with extinction: Its numbers are now ve times greater than when the use of DDT was sharply restricted In the early 1970s, (A) (B) (C) (D)
e f more than 75 carefully engineered cture of up to 600 rooms and with each
I of more than 75 carefully engineered ttrorctur,e of up to 600 rooms, each had been
I precllCitlon; Grammatical construction _l\llll~ce
make a claim about the scale (size, IDlllllt> th na aZI elliemenls and then h t lalm with a description of the 1It"lIelllll' eN ,Th cond part of the du d by the preposition with, ture fir t In terms of their n term of the roads that th r To d nbc the noun
-:::::=:~:ticiIPIlI form connfclfd should he
,
IOto
an adjective,
(E)
extlnclion; its numbers are now five times greater than exlinction; Its numbers are now five times more than extinction, their numbers now fivefold what they were extinction, now with fivefold the numbers they had extinction, now with numbers five times greater than
71.
72 slowe sa es of barbecue gnlls and I . awn furniture. (A)
cold,er and wetter than was I , usua In some regions, which slowed
fac rtH!
(B)
which was colder and wetter th regions, slOWing an usual In some
(A
ales
IB Ie
rate
(C)
since it was colder and wetter than usually in some regions, which slowed
(D)
bell1g colder and wetter than usually In some regions, slowing
(E)
havll1g been colder and wetter than was usual In some regions and slowed
Agreement; Diction; Logical predication
Logical predication; Diction
The original sentence contains no errors, 1he semicolon correctly connects the closely related ideas in the fWO independent clauses. The gy,falcon is the antecedent for its in the second phrase.
The sentence must clearly indicate that the inclement weather had slowed retail sale. Relative pronouns, sum as which, should follow a closely as possible the nouns to whirnthev refer. The adjective lIS1/al, rather than the adverb usually, is required when modifying a noun. The phra e wetter than usl/al is correct and conci e.
A
B
C 10 leally coherent £iQJIIICCI
D
Correct. The original sentence correctly uses a singular pronoun, its, to refer to the singular antecedent gyrftlcon, and it properly uses the construction its nllmbers are . .. grea/a than. 111e use of more instead of greater Inappropriately implies that there are now more numbers, rather than more gyrfalcons. Th e pronoun their is plural, and thus incorrect, since the antecedent gylfitlcon i sin~ular. Fivefold what thel' wae is awkward and nonstandard and implies that there are now more numbers, rather than more gyrfalcons. The pronoun thtJ' is plural, and thus incorrect, since the antccedent gy,fitlcon is mgular. The comma introduces 'I confusing phra e cming to modily extinction. F1vifoIJ th 1111mb rs thtv had is awkw,ml and n n t ndard and implies that there are now m numb • mlh r than mar g rt.llcons. In
du
nfu mg phra
n
A
B
c
D
The insertion of was i unnecessary and misleading. The referent of whi,h i unclear. because regions, not weather, i the neare t noun. Correct. This sentence i conci e, correct. and idiomatic, and 'uJhi,h has a lear referent, the'lJ.'eather. 'Vith the linking verb was. the adjective /lSl/al is needed in place of the adverb -flly. The relerent of'lJ.'hi,h i unclear use rt'uiollS not weather, i the n t noun
'"
'
This con ·truction i uncl and as unintentionall indi bJII analv'ts were colder
adje~tive uJual should adverb uJually
E
Th i con t:rucUOIIl.llP a unmtentl anal 10 tl ml
::::'l~".
10
lEI
rate f rsesl
ab
Rhetorical constnKtian;
VIII.".
Thi enten point It should redundanq cialrify aa_ _hII..... and usc the w rd
expressed m the contamcd m the adjawt. _ . , . should be mimd. til ~=:::: hows clearly When melDS a 1'-.~"':JIIt the ptqlOS1tIDD
A B
fi sh,OWIlIJIC UII~"fiiii
74
me academ a be teVe tha bus ness e t, shclUfd be I egrated," 0 every bUsll1ess course ay 1M udents ta p e h cs ser ously gh a separate {regu red course
IAl dtstrwasher
IBI Ie) d hw h r ,
(Dl
IEl
w" .111"11; C1in1l1nmltll:ll1 con
pnly If rt (OUld be taugh as a separately requ red course pnly ,f It taught as a separate, reqUired Course
If I
taught only as a course reqUifed ep.rately If ,I was taught only as a separate and reqUired course If ,t would only be taught as a reqUifed course, ,ep.rately I
Sc en ill a e hea 'Y·meta depOs of
(AI
Ba c Sea se
(B)
BallC Sea sed ~ s tndustna ae •J s COl'.ss~en1 Balbe Sea sed en - ".A CllI'tSlStl!!1t growth of tndus a as
Ie) (D)
sediments rom consistent w '" the area
lEI
sediments from e 8a the growth of d s a
Rh tori cal construction; Verb form, Diction
t flnd,t'flna I ",,,,trUel ion requlfe peLi hl 'erb tructlon. V rb form
75,
Rhetorical construction; Logical predlOtiOn
tm t • "or a pre,ent condltton, i1ke thi Jebate I" tween ,'",I
II" I II I 11 III the '1< I IU till lfr II" "II d, t n If /1 hnuld h th Id l
th l
1\
l1u 1Illih un
l1(tll 1I
a I)
tI\
lu II
lh 1'1
"
'ornewhat 11Io~ealh t h the noun Ii".li" ma e th \ 'here ha the ~ro\I'th -lIIdlHrnJ .acti,;rv rl.k ~ 'lhn-t i t , !!1J the indu In I takin~ p1.l Repl.lun Ibm- \I ith tb oh e thi pr blem
\
II 'bl h ha , 'u
II
h no
B
u" ...
I II
A the 'emenee is wrinen, t;;
(
'/' I I'
II
"' I • Illl'r I
11\
un 1
I
"'
xiii
l ()
l,'rr
1
Ih III
Inll •
It
II
3C.lMtv 1tI"e
h
• .....inI SIgIIS for a ma,or
E
1IIII_ldInfluclultions In local .n~ otlIIr deformabons of the
Rhetorical construction; Vertt ......
' tenee says iJlog icall) that changel ;n Th 'ssen. " hid mrOluremtnI are a warning .sign; It 5 OU . ay t hat ehanups ,,- in the stram are a warning sign.
The relative pronoun w!Jith antecedent, but none appea ~ a dear version of the entence 1h I'! In the OrIginal . eCOrnno_J announcement is entirely geare
The correct answer is D.
_
~IJJ'
The root systems of most flowenng pe,rennialseither become too crowded, which results In ass In vigor, and spread too far outward, prodUCing a bare center
78,
ana VIfYIIli
-'lInJ measurements of the stra,n across a
III 1liiie, and varying
Ie)
which results In loss In vigor, and spread resulting ,n loss In vigor, or spreading With the result of loss of vigor, or spreadIOg
(0)
resulting
IE)
with a resulting loss of vigor. and spread
dlIlIlIIilllllhe train as measured across a fault
(A)
and vanatlons of ....".15 lithe measured strain across a fault and vanatlons In dIInp In measurements of the strain across a fault zone. and varlallons among
(8)
10
B
Like a relative pronoun the a ., Pp0SltJve phrase (a part . ..) musr'h ave a noun or nou h p rase as a clear antecedent. th rb n hould be future tense. , I ' \'e
loss of vigor, or spread
C
The appositive phrase requires a clear fu antecedent; making does not' d' In /Cate ture tense.
D
This sentence is a little awkward (th 'I ' . I' artlc e a 10 a part 1 unnecessary) and says somethlOg rather different; as a parI of0 d(ol to make suggests that the deal it elf include making the company the largeII manufacturer rather than its being the outcome of the deal.
Th is sentence uses the construction etther .Y or y; .Y and y must be grammatically parallel. In this case, and Ipread must be or Ipread. The antecedent of which is unclear; replacing which remltI with mllltillg clarifies the meaning.
four phra e 10 Jeserihc Ihc for an arrhquake, 'Illesc phm'cs parallel The lir t .ign is IIIddm /*'JdIj'lllirnu", lo(ai I I m/( aCli1Jilv; the serond is tiltingtl,"" t r JtjOrmallOIlJ ojlh.. 1,'",lh', 1//11/. nfti.,. m lhl I u cd as ., noun, ;USI as MJI&nWIN'iIJ" ndjluclu'"IOII.lre '''"'''', '1111' first p rail I. 111<' Ihird .lnd founh I n re emhle ItlllII); In the IIIg ti",,,, re n t p mild hr' .UI ,. rlll'y.lre us"d as mher th n a noun. Chllll~lIIg " IIJI' ,Ir"ill; -~Il p '/' II flo make Ih, 1.1ll1·r two tillIII,.ralIlcJ,n un mu I pIa" wrh : ch""K'" wrMlhoN "
A B C D
Eilher is incorrectly followed byalld; which has no clear referent.
E
Or Ipreading is not parallel to (tther become. Wilh Ihe remIt 0/ is wordy and awkward. Or Ipmulillg is not parallel to ..ilha b....om'. orrect. 111e phrase n'sl/ltlllg ill 10J.! (!/vigor wnCIScly modifies the first clause; the either! or (Onstrl/ction is correct and parallel in this
Correct. The future tense is used throughout and the sentence structure i clear.
r, u "I ,I w,h .111.1 I J tn the noun }III
E
hilhl'/' Is IOwrrectl" folloll'ed b, '111,1; wilh tl /f'flilllllg Ion is word". -
'"Ie correct answer is D.
7
The computer company has announced that It Will purchase the color printing diVISion of a rival company lor 5950 lIllI1Jm whlml wrt Of.a lJ, I it WII m k It the I rgest manufacturer 10 the office color printing mark t (A)
million, whl h IS part of
(8)
million, p rt of a deal that make million, p rt of dim king
lei 101
IlUllion
(I
million
p rt 01 pari 01
deal that Will make
d I to m k d I th tWill m k
B
1heUbia:t _ _
81.
The correct answer is E.
80.
a contract 11JI'DIlI" IlIiIIIilgl!IS perbl"
"'l'ntcncc.
'Y'"
nd (J doe nol 1I)~j'.fi/llll" Ii/IIII ,and
There is no antecedent [, h pronoun Which. Or t e relative
11I4w
Idiom; Parallelism
rallellsm; logical predication
A
(A)
a condllion when It is present and mdlcate that there IS one
(8)
when a condition is present and indicate that there is one
(C)
a condition when It IS present and indicate that It IS present
(0)
when a condilion IS presenl and indicate Its presence
(E)
the presence of a conditiOn when Indicate lis presence
. _...u....... _
Edltlan
_._tndJution ....Iia_lICtJ n begun on the pa t and ....r-IR,""' rhe prr ent, and the frJrm of the 'lii'Il1i.~'III. .r IJIOW rhat contonu,ty, a the pre em ,.6lltlllmlCd once 1986, corporation hllV~ ..:om. In rh ngma! sentence, the pronoun Ih~y r referent Wh,) Ihry hould refer unlamlbij~ou Iy to", Imml ojJillrJ. /(rammatieally Of I somewhar undcar hnau c mVt'Jlnunl po< ,ve moditYinKfi'~J, and the plural onterven hetween ojJirtrl and the noun b un ho"ld replace bcgan; I at r fercnl.
H.
B
D
Ih~y
ha, no
Ha b 'K/II/ )muld repla,e h,'!."n; Ihcy has no I ar retC renl Allow Ih"l ,,/It hlJlul Oil is not a mrrect ,dwm; It hould he IIIlow ... 10 h~ h{/fcd on. rre
Suh litlllinl(Jt'tr ?/im"'IllJIt'fIl '.!Ji ' In 1111 '·IlI,'n,,· allows rhe prnnoun IhtV to refer 10 the oflm'n; hllve I"'K"II I.
C
D E
I
I
S~rvius hould be modified
A
byftw, not/iltl~.
The singular ,'erb (xulJ does not agree with the plural subject urviltJ. VVhen a compound subject is joine~ by or, the verb agrees with the closer subject. \Vhen a compound subject consists of two distinct units joined by the conjunction and. the verb must be plural. Lill/~ cannot modif)' urvim.
Correct. In this sentence,ftw correctly modifies urv;CeI; and correctly joins JerviceI and wa/~r.
modi fie paycblClu
B
Correct. In this senten.c dearer, because paycb¢Iu ~mploym. The relative clause~ modifie employefI, cIeady
C
Having been aHignedill governmentJ. llgIcaIIy IlIod
D
The correct construct'I . on, uh> not JUbSlilutfI x "'place 0' 1h ell . ~~ e~~~ 10 owmg paycheclu is Wordy and ~ The correct construction' .. L . IS Jugur/uUf x not subJtlfuteI x "'place
E
.
The correct answer is E.
83.
A<Jlgn~d by them ill~-""I!IlII.'Bit.u,
~ .Y.
e COnstlU<'hn"
followmg elllploy", is word\' and •..t.. __ • . a".. WiUQ. The correct answer is B.
In theory, international Civil servants at the United Nations are prohibited from contmulng to draw salaries from their own governments; In practice, however, some governments merely substitute living allowances for theIr employees' paychecks, aSSigned by them to the Ul1Ited Nations.
85 84,
The computer company's present troubles are ares of technological stagnation, markebng missteps and managerial blunders so lhat several attemots to reY!SP r ora e ra e les have failed t carr
prnpc:rl) mdH .Ift .1 uJnflllump; 'iituation.
(A)
for their employees' paychecks, aSSigned by them
A/~
(A)
(B)
for the paychecks of their employees who have been assigned
so that several attempts to revise corpora e strategies have failed to correc It
(B)
(C)
for the paychecks of their employees. haVing been aSSigned
so that several attempts at reVIsing COfporate strategies have failed to correct
(C)
(D)
In place of their employees' paychecks, for those of them assigned
in that several attempts at reVls ng corpora e strategies have failed to correct them
(D)
In place of the paychecks of their employees to have been aSSigned by them
that several attempts to revise corporate strategies have failed to correct
(E)
that several attempts at re sng corpora e strategies have failed to correct them
Ih"l
. l,d'"Wloll is
aII IIl[("reel
1.110"' Ih)' h.1 "" II, ,Ir rderellr. H''Kfll/ dlles n JI uad. atc rhe lOllllllUif\ lIl'lCSS,lf}' .lfter 19/11, C
A B
D.
(E)
logical predication; Rhetorical construction
XI
I
It is dillieult to tell wlHeh p.ut· of this sentence go lngelher heeause of ermrs .Ind cnnfusion in the undt..'dtllrd porhon. I i~li"~J: ,tI'OV.'tI11u'\ is the l tHII1Ct'rp.lI r offlll',h, cI~(, 'On It is hetter to ';1\" ~o 't I 11111(111, {lIb,'i' /llf Ii 'i"g ilI/O U'cUIl(fjor Ih, /'''1'' It" k, oj Ihol', 111/'10 I' r, [w, .Iuse It nukes the IIh t'llllinn d ·.It,'r. Ih, dung" ,II I) nuke' it c.1 I,'r In wrr,,,, I til<' Ill"d,h,.llion etrM Ih.1t ,11'1' ar in the I'hr.lse '/II/,~I1(.1 /1\' Ih' mp~ \' I .lIId ht inhl th mRlllIlIK p.lftnt thc III 'n, ,I fh l lI;t J l
wI r
1
Agreement; Rhetorical construction This sentence lists three cau trouble: .lnd .l,-Sew that 'm C.luses of the problem' ha\ fail most effiCient W.l\' to plain !hI ,',luses With the relati\ pronoun it .IS .111 object ,)t th \ rb M1wt.
Inc IIlgular pronoun .11lt 'c'
In B,are th r fhis i notrh,c 1:U"
" 'X"
1111 "
·JC:Ilt. the COEliwilCtil.J"
B
lh nu
ha
. Ii0 IIOWl ng Mlh i . not The con truetlon para II cI to t hc con truc tlon following and. Now m,J
D ",lIny a
h
people. the und
PMPI. ar too
(.on rruuj{JO
J
violated.
(AI
Clouds ,are formed from the l!VaIlorabon oceans water a S 'NarlTle(l nses high on 0 e a 050 ere iIIld tony droplets on m e PartIcles 01
(B)
Clouds form by the Sun s ar evil\lOr the water 10 the oceans atIlg r SfS atmosphere. condens g dr minute particles of dus
(C)
Warmed by the sun, ocean a er evaoorales rises high onto the almosp ere a COlIdense in tony droplets on monute pa es 01 s form clouds.
(0)
The water on the oceans evaPOla es warmea the sun, nses high IOtO e almo5Ohere ilI'Id condenses In tony droplets on u e oa es dust. which forms clouds.
(E)
Ocean water, warmed by the sun. e aoorates and rises high Into the atmosphere. h h e condenses In tinY droplets on mInute oa es 0 dust to form as clouds.
The correct a",wer i B. Last year, land values In most parts of the plnelands rose almost so fast and In some parts even faster lI1aJLwhatthey did outSide the plnelands,
87 HilI*IIC theater company ,n NCI' . .'" tt.llllJIIlCllWIY from the pan h cia Ie and
~:~:F~~:~:~:~p(llitry
~
10
so fast, and In some parts even faster than what they did so fast, and 10 some parts even faster than, those as fast, and 10 some parts even faster than, those as fast as, and In some parts even faster than, those as fast as, and 10 some parts even faster than what they did
(A)
the (B)
dtM on the work both of contemporary IIII*IIiC author who bv abroad and of Iho e nth work of contempotdry uthor bolh Iho who hve abrOad
who IN th work of contempordry HispanIC IUllIOf'I now, bolh Iho hVlng abrodd dnd who
(C) (0)
(E)
Idiom; Parallelism
'Ihi !'Il'ntclll'C
J I
whilh is
(orrcu iuiomaul" constrUltlon; [0 ,limos! 1II/t11It1'Y" tlll',,)rreel iJiOlll t~>r lhl lIlrnp,lti IlII 'n", tWIl c1elllent helll/: lllllp,lred, x ,lIul", 11I1I I he p"",IIc1, hlll Ihe 1I1l1l1l/"nd .,1/1<"1 ( ) I 1I1l1 p""dlcllll 'U.'b,illh, I' did \,) l'lIrd "It", 1I1 Ihe' pllld,'lId (x) 1I111 the elllllp",ed \I ith lhou (till' 1"lllllllllllllrrl'l tIl" tl'pl."-'II/: 1,11I1 11/1 \) IlIII II" p'lId,llId (\,). nnt
. ._ _1tA1 conttnlctlDn. Idiom; P r II IIsm
say~ \. ru"'c a/mosl IO}/J1 VI
II
"I,
\
II
l
Rhetorical construction; logical predication
Thi' senllnce describe:; a multi51ep pro.:
h ocem \I .lter is rransfunned into cloud>, 1h Me IllO I dearly pre;ented in chro ,-a! wnh ()(. OJ" "- It r a the main ,u~' -I the sallnlCC.
A
So 1.1111 II rd III I ,1.1 III ,I' Ifl
,\
1I1lllh,' "rtl I uhlllll lor \ tlll1p.H 1 lin
"1,"/1ll11 I Ill' 1,,11,,\\ ,I hI" '" till u 't. I /., I '" I II., nnJulh. lion u d III rill
II
II lIn I.~I/CI II"
III I ,I 101ll1'.H 1111
1'1 nu.• I
lhe
,1111< cedent
Ii. r Ih n:lall' p,ronoun
,. breb i .lJUhlgll)U. J aln ),LlIl ri • h, h.
\ omp.1I1 nil
I
Thisenrence prOlides no n iUogiealh su l> Ihallh an' e\ .'ror.ll~ .,fler il ri, high 111 the
.ltlllo,phe......
B
1,,'/,
I
I
I
B
b'o "
l orreel, lh
qu n
tllflll.ltllll1 I ,]
ar
D
III 1l1)IKhnllloiIDgI...... onkrollllllt Alp \I
h, ·h ,I,)ud Ih I
I I / v " II nnt p Ira II I III I.", I II. •
I .
\
r, <'UJldell.-.
,tOn.c.....
D r
air by the •~:::::::::1he IJlIIlboW' IJefDtt the temblor .l'IIItll'elec:tromagnetic sflllais were detected one man who was thrown straight mto the air by the upward thrust. and strange electromagnetic signals that were detected hours before the temblor
Witbwhuh . . . ."", rather than Iht "18~1I c:omctM by substituting
. .""", whU:h follows parll) for """,""CwhU:h follows dttltlopment). lIiI"a_ih ch a recommend, request, or the main clause, the verb .lIIIIllItllMtin the subordinate clause is ~ rather than indicative (is). In
If'llitA modifie dttltlopment instead of park. reqUIIM, not is.
Q!mlCt.ln thl • ntence, which clearly GKldiJliesparA; th subjunctive b,' correctly fuIIows propoJtd that.
Dt'wID'Pm nlfundmK distorts rhc mcaninji;. ulred, not
IJ
to bt.
lEI
parasite's hosts,lYfllllil" become their hosts
.
This sentence explains the inc · as a COnseque ~ ~ sc h" IstosomlaSI ofDte habitat of the kind of freshwa: enIaJpt the parasitic worm responsible fi the that host definite article is necessary befo:frn disease The because the sentence identifies a pam~tr snails snail, namely, those that host the cu type of . . parasite. The corre~t prepOSition to express duration in combllla.t1on With host is for, not in. As the parasite IS referred to as singular th . . , e posses tve pronoun III the final phrase must aI 0 be singular.
mar::
A
Correct. The sentence is clear with all pronouns and verbs in agreement.
A
Correct. This sentence correctly provides a parallel series of three mysteries.
B
B
1he is omitted before strange. The verb were delected makes the last element not parallel to the previous two.
The preposition in is inappropriate for expressing duration; the plural po sessive pronoun their does not agree with the singular antecedent parasite.
C
Because they use complete independent clauses, the last two elements are not parallel to the first, and the sentence is ungrammatical.
o
The constructions beginning one man and hOlll'S befort' are not parallel to the construction beginning Ihe unexpectedpower.
E
1l1e grammatical constructions describing the mysteries are not parallel.
Nl'IIIICC IUII1WU i B.
C
SchistosomiasIS, a disease caused by a paraSitic worm, IS prevalent In hot, humid climates, and It has become more Widespread as Irrigation projects have enlarged the habitat of the freshwater snails that are the parasite's hosts for part of ItS life cycle. (A)
the freshwater snails that are the parasite's hosts for part of ItS Iile cycle
IB)
the freshwater snails that are the parasile's hosts In part of their life cycle
(e)
fr shw ter snails which become the p raslte's ho t for part of Its hf cycles
A definite article should precede fteshwaltr mails to identifY a particular type of snails; the plural cycles is inappropriate because its refers to a singular parasite, which only has one life cycle.
D
A definite article is needed belore fteJbu:.utr snails; repetirion of the word purasilt makes the final phra e unnecessarily word)~ ..y da hould be singular.
E
The repetition of hosts make the final phrase unnecessarily word\~ q'des should be singular; Iheir hosls' h~uld be its hosts; the referent for the erond appearan of unclear-doe' it refi:r to snlli/s or the paTtlslte?
The correct answer is A.
92
..._..
Rhetorical construction' .", aft
Some of the earthquake's mpteries are described in a series of three correctly parallel elements: (1) the 'inexpectedpower . .. , (2) Ihe upward thrust . .. , and (3) the strange electromagnetic signals. ... Each of the three elements begin with an article (the), a modifier, and a noun. This parallelism is crucial, but each mystery is allowed the further modification most appropriate to it, whether a prepositional phrase (1), a clause (2), or a participial phrase (3).
requlIM, not the infinitive to bt.
II omitted, makinji; the construction awllMard and unclear. The phrase modifies dlt.fJIIc'!Jm nl, not park.
freshwater ~ parasite dunnI1lll...11
duI"'.liiiiil __,. ~
Parallelism; Grammatical construction
~1ftl~dr:. It; In this
101
The correct answeri
93.
(A)
due to the fact thlt too slowly thin
(B)
due to their llClGII.J than to ..till
::~"d~""'~!JI§g~~3~ t«, till ... _
A
B
c D E
....
-::::4Ile liII
96. (A!
JUry to go
nlther than
......,tIle JUfY to go home at the
~'dlvrather
than be confined to
(8)
I-'til pndlclltion
nee ha two po sible options 'tJJeJIJry: they can go home or be • hotel The first option is expre sed
:
!=~~iDiimt1
(C)
togohom~; the second option
the parallel /Orm (10 understood) be m the m~mb~rj oflh~jury are not ,4l1inR the ClOnlining themselves, the pas ive form be used. The can truction x inslead ofy, W111Cll," andy are infinitives, is clumsy; the Uiomlll:l ClOllIitruction x ralher Ihan V is better here Both can tructions require x a~d)' to be
(0)
(El
parallel. Th pas ive form 10 be confined is required. 10 allow m~mbm oflhejury . . , 10 confine th~m Illogically indicates that the jurors are domg the confining. B Th mfinitive form 10 be conjined is required, rather than the past participle. The sentence I awkward and wordy. M mbm oflh~ju,..,' is the illogical object in ".fining Ih~m. Co,!fining is not parallel to 10 home. 'The noun (onjinemml is not parallel to 10 go me UlnllCt. B confineJ 10 uses the infinitive form JU t logo home does; the 10 before be 1ud lund rstood and docs not need to ted. 'The ralher Ihan Vconstruction lI9PJpnllJlliialtely u ed m this s~ntence.
'~=:I~lntellilence say they will be
•
In understand English • lIt>lacol'llze objects, and
as an expert does-computers that WIll be used to diagnose eqUipment breakdowns. decidmg
whether to authOrize a loan. or other purposes such as these as an expert does. which may be used for purposes such as diagnosing eqUipment breakdowns or deCiding whether to authOrize a loan like an expert-computers that will be used for such purposes as dlagnosmg eqUipment breakdowns or decldmg whether to authome a loan like an expert. the use of which would be for purposes like the diagnosis of eqUipment breakdowns or the deciSion whether or not a loan should be authorized like an expert. to be used to diagnose equipment breakdowns. deCiding whether to authOrize a loan or not. or the like
(A)
(8)
(C) (0)
(E)
The underlined portion of th e sentence' .. appOSItiVe, a terminal no h I an kind of data being collec~e: p rdase re~t~ting the " I' r an provldm add !tlona IOlOrmation abo ut .It. Th·· IS IS gI . economIcal way to provide th . a c ear and e extra mformation. A Correct. The sentence'IS grammatically . correct and logically coherent.
The sentence presents three functions of intelligent computers, but these functions (10 diagnose . .. , deciding . .. , or olher purposes) are not written in parallel ways. Moreover, the final function is vague. Turning this final function into an introductory statement and using parallel forms for the two elements diagnosing and deciding creates a stronger sentence. Either the clau e, as an experl does, or the prepositional phrase, like an experl, is correct and idiomatic in this sentence. The series 10 diagnose . .. , deciding . .. , or olherpurposes should be expressed in parallel ways.
B
Which has no clear referent.
C
Correct. l\lovingfor such purposes as to an lOtroductory position strengthens the sentence; diagnosing and deciding are parallel.
D
7ht' use ofwhich would beforpurposes like is wordy and awkward. Which has no clear referent.
E
To be used, deciding, and or Iht' like are not parallel.
1h correct answer is C.
atmosphere Interact _\aft..... . Ions that-.... atmosphere. With Inte ~''''''1 ractlOns aft atmosphere that affects ectina atmosphere that IS affeCbng atmosphere as affects
Grammatical constructi on,. Agreement
Parallelism; Rhetorical construction
A
Floallng In the waters of array of buoys collects ancl~trl~;::~::~~. term mteractlons between U; atmosphere InteraC!lonslhat;'
B
The prepositional phrase wilh h ... asno I ' c ear noun or noun phrase to attach to and is therefore ungrammatlcal.
C
Using the restrictive Ihal after almosph illogically suggests that there are man:; atmospheres to differentiate from and'the one 10 question in this sentence is the one affecting global climate.
D
The restrictive Ihal also follow atmosphere as in answer C.
E
The phrase as affecls global climale functions as an adverb, but there is no verb for it to modify.
The correct answer is A.
97.
Unlike the United States, where farmers can usually depend on rain or snow all year long, the rainS In most parts of Sri Lanka are concentrated In the monsoon months. June to September. and the skies are generally clear for the rest of the year. (Al
Unlike the United States, where fIm\IIS till usually depend on rain or rains In most parts of Sn LanQ
(8)
Unlike the United StatlUlII depend on rlln or 5n1QW.I1IJ most parts of SrIIAll~.
lhis_lDiI", SltItDto
B
Complpl1'riI ... U. . . . . m ....'JtIIj.fl/l
c D
E
1his" ". . tllllllllli
".1III's
Irmy entered Russ,a with far
iIIl1IIll tIIen they had In their preVIQUS
(El
.1III1.1[1lI d prDVIs,Qns fQr Qnly twenty-fQur days.
bid n their prevIQus campargns, e r preVlQus campargns had had, they had for any prevIQus campargn, n the r prevlPus campargns, for any preVlQus campargn, " ,...ment; Verb fonn
The entence incorrectly switches between the plural pronouns tlx)' and their and the singular it to n:fer to the armv. The past perfect verb tense hadJ preferab'le to the simple past had, . becaus th. action occurred before the acnon 10 the maIO lause. In the context of Jupp/iesjora "flmpa'gn, the prepositionjor is preferable to the prepo .tlOn in. In cases such as thIS: where the tence ha multiple errors, it is often helpful to look among the answer choices fQr an alternate on tlU tion. Pronouns refcrring to the noun army shQuld be If and ifJ, not the)' and their; thc verb hould be had had.
B
7h,rdoe not agree with the siogular pronoun ,t, which treats (/rmy as a singular n un
do nnt agree wuh the singular noun nd Ih other pronoun. The verb should be hlld Th p po ltIon u cd hnulJ be jor, not ;11. 71Yi••llOuld b It
OIImlCt,. Thl Imple construct inn avoids the f pronoun agreement and verb ar,
rr t, and ("oncist'.
Logical predication; A9reement
Tubman maintained that she had Tubman had maintained that she has
(0)
This sentence describes a ",o-pan events, the secQnd Qf which h I ~uence of lo a panJcular categorization ofgeQIQgical Iias that it is not the first event (as:;~,; order lo claril) produced the time divisiQn (end of ~~.ke) thaI Period), but the first event's co Cretaceous . I . .) nsequence<; (b 'IQIoglca extinctIOns, the sent .. c enee needs an apposItIve lOrm to restate the c f ontent a the 10 . clause (an event), follQwed by a t\ h . ato . va-pan C ato of relatIve clauses (that caused . .. that mark . ..). h ..
Idiom; Verb form . a possesslv . e noun such as The apostrophe 10 Tubman sindicates that the word 0/ h~s been
1
omitted. It is correct to write Tubman s or Tubman; it is incorrect to wnte ofTlIb~lan$. The verbs maintained and had describe acttons that
et
were campIeted I'n the past and occurred at about the same time. Since Tubman died long ago, she ha d not has, the qualities of a great. leader. Her contemporaries were people who hved at the same time as Tubman; the simple past should be used to describe their actions.
A
A
OfHarriet Tubman's is an incorrect possessive. Has should be the simple past tense had.
B
B
OfHarriet Tubman's is an incorr~ct possessive. Maintain should be sImple past tense maintained.
I
C
o
E
OfHarriet Tubman's is an incorrect possessive. Have maintained should be simple past tense maintained.
CQrrect. In this sentence, the possessive is properly expressed with the phrase 0/ Ilibman; maintained and had both use the simple past tense. Simple past tense should be used in place of bQth verbs.
The CQrrect answer is D.
100. Slxty·five milliQn years agQ, accQrdlng tQ some scientists, an asteroid bigger than Mount Everest slammed into North Amenca, which, causing plant and aOimal extinctions. marks the end of the geologiC era known as the Cretaceous Penod.
C
D
The antecedent for the relative '. pronQun whIch IS ambIguous' it is there c I ' lore une ear what marks the end Qf the CretaceQus Period. The antecedent of which is unclear; the compound verbs cauJed and marks fail tQ indicate that the extinctions, not the asteroid strike, are significant markers Qf geological time. Following the CQnjunction and with a participial rather than a main verb is grammatically incorrect because it viQlates parallelism and produces a fragment at the end of the sentence.
B
C
wronglyYSUSUI~;;:==
D
Qfche parr Omi IQn
E
~troduces=~~=:=
CQrrect. The sentence is unambiguou , and the verbs agree with their subjects.
The correct answer is E.
The present progmlSift A bodyofv. is wordy and m_r1 11• •_
Which, referring to extinctions, shQuld be
followed by a plural verb.
E
A
The cwrect _ _wil..
101. The Federahst papers, a strong defense of the United States Constitution and Important as a body of work ,n pohtlcal sCience as well. represents the handiwork Qf three different authors.
(A)
WhiCh, causing plant and aOimal extinctions, marks
(Al
(B)
which caused the plant and aOimal extinctions and marks
(B)
(el
and caustng plant and animal extinctions that mark
(C)
(D)
an event that caused plant and aOimal exttncbons, which matks
(0)
(E)
an event that caused the plant and al1lmal exttnctlons that mark
(E)
and important as a body of work 111 pohtlcal sCience as well, represents as well as an Important body of work ,n polilleal science, represent and also a body of work of Importance 111 pohtlcal sCience IS representllli an Important body of work In poIiIicII and has been representative and as pohtlcal SCII!\C . . . . . 11• • work too. represent
Idiom: Logical predication
- : : : : : IIelbose of ill ..... 1JIen betlommg
The construction in iIololionfrom is awkward; the idiomatic way to express this idea is iIololedfrom. The comparison is ambiguous; it could mean the Bering Sea Inuits were isolated from Europeans longer than they were isolated from Aleut and other Inuits or that they were isolated from Europeans longer than Aleuts and other Inuits were isolated from Europeans. AddlOg were after Ihon will solve this problem.
sucb specialIZed areas as !JlI..1inI are being Increasingly
!
~Ea~~i pnaent dlUIC tlCn 6unnm verb growImOT( to lIS
grOWI
'lUalion. The main c1au,e eI&ct of thi growing complexity; ",,-,,"'11II11I the main clause hould also usc ..._IIt1ImICwm.. The pteient perfect progressive M"'~"'" _mmg I incorrect. The preferred way InIlU1fuce ClWIIples is with the phrase wch OJ, all. dian with the word lillt, which suggests a CJIWlUIg
Lih hould be replaced hy wch OI. Have been mmg I an meorrect verb tense. CcImICt. In this sentence, major and are Hamling are present-tense verbs; IlIch ... OJ is the preferred form for introducing examples. MJi 'fI i a past tense verb; IhoI( of is mlleCuauy and awkward. Beroming is pre(erabJe 10 luing for describing an wUOIdmg partern of events.
ahouJd be repla ed hy wch OI. 'ho.!e ofi, UIIlIllO~uryand
awkward. !foVt P"(II an In orrect verh ten'e. MIflhW lllilJOTt'fl. an awkward pas, -:::~ B mmg i preferahle to ImllK f{Jr II an unfoldmg pattern of e\'ents.
.._.Ulftnll
b B.
....,1111_
ISQlallpn frprn cpnlae I ,~: Al.ul or InUIls of Ihe Al. III
A B C D
E
".iIICI
The comparison is ambiguous. In iIololionfrom is not the correct idiom. Correct. The idiom iIolaledfrom is correctly used in this sentence. The comparison is clear and unambiguous. In isolalion ... wilhoul is incorrect and confusing. The comparison is ambiguous.
104. Although Ihe firsl pulsar. Qr rapidly sPinning cQllapsed star, IQ be sighled was in the summer Qf 1967 by graduate student JQcelyn Bell. It had nQt been annQunced until February 1968. (A)
(B)
A
AlthQugh the first pulsar, Qr rapidly spinning cQllapsed star, tQ be sighted was In the summer Qf 1967 by graduate student JQcelyn Bell, it had nQt been annQunced until February 1968. AlthQugh nQt annQunced until February 1968, in the summer Qf 1967 graduate student JQcelyn Bell Qbserved the first pulsar, Qr rapidly spinDlng cQllapsed star, tQ be sighted.
B
C
The opening participial phrase function - as an adjective, but it has nQ logical noun or noun phrase to attach to; Bell herself wa not annQunced in 1968. Grammatically, the opening participial phrase de cribe the first noun that lollo,,"s, but it makes nQ ense to say that Ihe dis,atom of the pulsar was obseT'iwl; 'dis,O'i'''J' ofthe . first sightedpllisilr is alSQ impreci e; one doenot discover a first ighting.
D
Correct. The sentence pre ents th~ equen:e of event clearly and in the pa t ten e. This sentence presents e,'ents in a wa~ that i confusing; as a conjunction, -hilt indica simultaneous events, but thi enten, • about events that occurred in a uenee.
AlthQugh Qbserved by graduate student JQcelyn Beilin Ihe summer Qf 1967, the dlscQvery Qf Ihe first Sighted pulsar, Qr rapidly spinning cQllapsed star, had nQt been annQunced befQre February 1968.
E
(D)
The first pulsar, Qr rapidly spinning cQllapsed star, tQ be Sighted was Qbserved In the summer Qf 1967 by graduate studenl JQcelyn Bell, bullhe diSCovery was nQt annQunced unhl February 1968. The firsl sighted pulsar, Qr rapidly sptnDlng collapsed slar. was nol annQunced unlll February 1968, while II was observed In the summer Qf 1967 by gradu te sludenl JQcelyn Bell.
The correct answer i D.
.....lIIIIlona.r
IE)
--11II...._ .
The subject of the Qpening dependent clau e IS pulsar, and the verb is Was . The cl ause needs to indicate not just that the pulsar eXIsted but that it was observed by Bell; the past perfect verb tense is inappropriate in the concluding clause.
(el
With Europeans longer p n lona.r Ihan lIJlQn 10na r
In iIololionfrom IS not the correct idiom. The comparison is ambiguous.
preen__
This sentence presents co d n 'lion tho followed by an unexpected OUtco t~ announcement of the d'ls m~ a delayed Covery of a Is A compound sentence us', po ar. .. ng a coord' conjunctIon bUI is an effect' lOatmg Ive way to conditions of the /irst puis 'gh' present th~ information about the sub arl tlOg andh t en sequent ann ouncement o£the discovery. The sente nce must cl ifv h i not about Ihefirslpulsar. b t h fi ar . t at it , u / e wp I be slghled. The verbs in the se t U Iar.. 10 . n ence must all be . past tense; usmg a past-perfect ' b 10 . c . ver to present lO1Ormatlon abQut the announ . '. cement of the dlscovelJ' mdlcates that this ann . . ouncement IIlQgICally tOQk place before the I . h te d . pu sar wa /irst slg
The correct answer is D.
lQ
'I
Verb for~: Logical construction
rellSlEied
:r:ar:ed~:~
C
the~
bet:a_,n_ radiation ~
lD
~to,.
~ inlrlIIltdn.........
IE
01
IEB~:===
nilli..
re~lhusm
__
Logiq/~
Th. SCDlI:IlI:e
and apjHopi.'" refers to I
B
D
I
andra __
and light. the sun IS the source ora com nuous stream It and light. also the sun IS the onbnuous stream BeIIidtIS heat and light, the sun IS also the 01 a continuous streaming
107. Bluegrass musIcian Bill Monroe, whose repertory, vIews on mUSical coliaboraltOn, and vocal style ~ Influential on generations of bluegrass artists, was also an Inspiration to many musiCians that Included EIYls Presley and Jerry GarcIa whose musIc differed Significantly from hiS own. (A)
_1m!
n the source not only of heat and light, o of a conbnuous stream The un I the source of not only heat and light but. a well, of a continuous streaming
Idiom LogICl' predlCltlon; Rhetorical
(B)
(C)
"lIltrudlon
(D)
(E)
C
The SUU)cct and verb d' o not agr . repertory, view" and II I. ee 111 subject with singular?e e I ~a, (compound and different in r: lar Influential to . ...
comparuon to are
unnecessarily wordy.
D
Influenced generations of bluegrass artists, also inspired many musicians, Including Elvis Presley and Jerry Garcia, whose musIc differed significantly from was Influential to generations of bluegrass artists, was also Inspirational to many musicians, that Included Elvis Presley and Jerry Garcta, whose music was different significantly In comparison to
were an Influence on generations of bluegrass artists, was also an InSpiration to many mUSIcians, Including ElVIS Presley and Jerry Garcia, whose mUSIC was significantly different from that of
Correct. The use of h of influenced and In,~ e eonel ~ form 'rlred' I Ii clarifies the senten 1h Imp 1 es and ceo t co . meI udmg a, aids th nelse u of e pronou unnece"ary wordine n error and
b)·
were Influential on generations of bluegrass artists, was also an Inspiration to many musicians, that included Elvis Presley and Jerry Garcia, whose musIc differed significantly from
was influential to generations of bluegrass artists, also inspired many mUSICians, who included Elvis Presley and Jerry Garcia, the music of whom differed significantly when compared to
B
E
108,
There is incorrect subieer J -"erb agree . repertory, viewr, and rt I. ment 111 b' ye ... 'wor (compo d su ~ect with singular verb) IV' .un and in when eomhared t ' Or mJluenttal to d r a are unnecessarih war y. 7he mwic ofwhom is c b . and stilted. urn ersome The.phrase were an inJluence on is wordl' and not The phrases war aman l . . IdIOmatiC. . . InSpiratlOII to and was rign!ficantly diffirmt are unnecessarily wordy. The ph~ase flam that of IS unclear and confUSing,
The correct answer is B.
Sound can travel through water for enormous distances, prevented from diSSipating Its acousltc energy as a result of boundaries In the ocean created by water layers of different temperatures and densities.
Agreement; Rhetorical construction; Grammatical construction
(A)
prevented from dlsslpatll1g ItS acousltc energ as a result of
'Ille original ,entence logllally intend, to explain [hat Ionroe's work influenced generations of ,mi,[, in hIS own mu,ical field and that he in'pifl'd many musician, in other musical fields. Who or what influenced or inspired whom must be more dearly 't,lted. Additionally, the original 'entence lack, preci'ion, being overlr wordr ,wd llSUlK phr",e, that ,Ire not idiomatic: Concise ,\l1d mnsi,t nt verb l,)fIllS, ,IS well as thl' usc of uburdinat phr,lSe, r,lIher than l'!auses, il1ll'r,lVe th pr ci.iun 01 the Selllt'nce.
(8)
prevented from havll1g ItS acoustic energ diSSipated by
(C)
Its acoustic energy prevented from dsSlpaltng by
(D)
ItS acoustic energy prevented from bell1g diSSipated as a result of
(E)
preventing ItS acoustic energy from dlSSlPll~ by
1h phr.ls w ~ il!!lu, 11tilll Oil is w,)fdl' ,lI1d I n t Idlumatl ; the usc 01 v rb lorms ,'r (th pr dl at of 'P r/ ry. 'lor, ,lI1d /I'l) (th pr d. t of M I ) I IlIlnfutln
logical predication; Rhetorical constNetIon
D
1h
E
The COilEd lIIIl_rrill C.
109==== B
BeIng born In Calcutta In 1940 and havlOg been a UOiled Slales c,tlzen slOce 1988, author Bharao Mukherjee HaVing been born In Calcutta In 1940 and being a UOited States cItizen since 1988, aulhor Bharall Mukherjee
(0)
nten I,e 10 rhe .orred pnlpotitional phra e In rhe :=;;:~ pi emenr of", fWO Irlla, to ppea... ro ugge r rhar
IE)
:
J~E~~~~: ph
:
10arthe rhcm ofI\<s. rhclerrer hcgmmng rhe
Imtlated on Columbus Day 1992 alter Europeans arr Cd the five centunes $100 million Investment: til New World a extraterresilialinlel gence w:~arch lor Project SETI. Pledged by
IC)
Imitated on Columbus Day 1992 five alter Europeans amved III the r ~ ~~ res Project SETI pledged a $100 m on les en III the search lor extraterrestrial n e gence Pledging a $100 million Investrnen . e seare for extraterreslnalinteiligence the I . • III aono Project SETI five centunes after Europeans arrived III the New World on Columbus 0 1992. ay
Verb form; Rhetorical construction
th problem
D
(B)
. 1988 and born in Calculta . lInce . in 1940 is an aw kw ard , ,vord)' construction, whICh preseors . an unclear and potentially confusing chronologIcal order. ince in the correct version of the sentence the onginal phrase (bring . ..) has been made IOto a main clause, a semicolon should separate. ,t from the second main clause beginning she has Itved. Bezng
ten uAAC r rhal rhe eruprion of VelAJViu took pial< 10 rhe lellef them e1ve . nrun th ore",c wllh 10 Ih,' h/lMrian TlUiJw I lum) and undear. 'Ihe verh " cern ,1I111(1."lIv 10 modi(Y " h ,.rcalml( Ihe .,wkw.",1 ,u~l(e'lion th t fh "'J'h wa Ihl rl','fJ)ilnrll "olm/l. Th nr OL: • meaning: I unde.l' due to an I nd d equ ",e of pr'l1ll ililln.,lph""e'. An a IIn/l a nolfr.Hi\'l' rt.. t ord; /0 (/((011111 flr m n 10 he Ihc (all (. of. Iising hoI h in I he m or OL r I UHlfu JIIg ,lJld hen.' ,uRgell Ihallhe neph, w ,.,"".<1 lht n The WIW( c ,II II t1,L:ge I' I hal rhe n of~ lJ\'1I1 Illok pl.ll C 10 rill' k'ller, t J
A
B
I)
The phrases arc expressed in an illogical and potentially confusing sequence. Ifaving been suggests that the citizenship came chronologically before the blfth. The pronoun she is the subject of the first clause; since the author's name is mentioned only aftcr the semicolon, she has no elear referent. Correct. In this sentence, the sequence of event, is expressed logically, p;tamm,lti,aUy, and concisely in each independent clause. 'Ihe pro!!:ressive verb fotms /Je111g /Jom and hl1V111g /Jem illop;ically stlp;p;est continuous action and /:,il to est"blish a logi<:.l1 time e(l"elKe. 'Il,e sentence is wordy and ,lwkw.ltll. 'Illl' prugr""lve verb l,mlls h'I'l'illg /0" 11 /001'11 ,111.1 /I'lIIg dloj!;lcalh sUj!;j!;est nllltll1UOllS .Il tllm ,lI1d l~lil t" est.lblish a logil.1! tUlle "'quelH", 11", s,'ntelln i word, ,lIld ,lwkw.Jrd
'Ihe correct lin weI' i, ('. nd
(01
([I
The original sentence becomes illogical when phrases do not modify what they arc intended to modify This sentence mistakenly a). that Europeans arrived illlhe Nev.' World on ColumbllJ Dav . 1992. It also ay. that Project ETT was iniliafedjit'e fellluries after ... Columbw Duy 199_. To make the modifiers grammarica1l) and logically correct, the sentence may be revised: 111iliald on ColumbllS Day 19Q1,ji" mJiun aji"r Europeans arrii"'d illlhe X,-~ H'rld, Pmje,l ETI..,.
A
Project ETI cannot haw been 'nih,,",," In,< ,miuTl"" 4i.., ... 1992, nor did Euroran fiLt .mi,·~ in 1992.
B
111;1;,,/,,/ ., modifies 100 171' III'\.
.
• n....
(·J/'~IOllllLte-.\d ot c,~ d
'ETI.
C
Correcl. llle nllldifie : are grammatl al" .1I1d logi,·.,11I "Mre<:l in thl senten<:
D
m. IIi il'" ... illl'l"lrn nten tll1lji:mc:nt IIl1/i~'/,on. 1hi is a
F Imll I d fiv c nlorle all r (lirapean lllived III th N w World an Calumbll 0 y t9 ,Proled TI pi dB d 100 militan IllV tm nl In lh , h lor lr I rr III IIllI lit n
D
Logical predication; Grammatical construction
111
(A)
Pledging a $100 militon IOvestment n he search lor extraterrestnallOlelligence five cen unes alter Europeans arrived in the New Wortd. on Columbus Day 1992, the Initiation 0/ PrOI& SETllaak place.
B
PIC4~
t"
IIv modifi ap
hi constructioll
E
D
E
The first example, presented as a present participial (offering) is not parallel with the second example, presented in a relative clause.
A
phrase is confusing and ambiguou .
B
Correct. The sentence is grammatically correct and the examples are parallel.
(AJ
resulting from the destruction of the largest library of the ancient world at Alexandna,
(8)
the destroYing of the largest library of the ancient world at Alexandna resulted and
(e)
because of the result of the destruction of the library at Alexandria, the largest of the ancient world,
(D)
as a result of the destruction of the library at Alexandria, the largest of the ancient world,
(E)
Alexandria's largest library of the ancient world was destroyed, and the result was
logical predication; Rhetorical construction; Grammatical construction Because it is introduced by a participle, the phrase that begins reJIIltingji"Olll illogically modifies later gt'll<'rations. ubstituting the idiom as {/ result if for r"JIIltingji"Ol1l corrects this error. 1he largest li/mllY if the allcimt world at Ale:>:andria is both cumbersome and ambiguous because it suggests that the ancient world was located at (and only at) /l1".\·andria. 'n1is problem is best corrected by breaking the series of phrases into two distinct parts: tht' lib/l/lY lit Alexandria, the IlIIgt'st ~ltht' ul/tit'llt world. I lere, the seronu phrase clearly modifies the first.
1he destroying ifis wordy and a vkward A"
creates a second main clau e, whICh would need to be appropriately punctuated with a comma before and.
The correct answer is E. 113 In A.D. 391, resulting from the destruction of the largest library of the anCient world at Alexandria. later generations lost all but the Iliad and Odyssey among Greek epics, most of the poetry of P,ndar and Sappho, and dozens of plays by Aeschylus and Eunpides.
Resulting/rom illogically modlfi lot generations. The erie of prepo ItlOnal
C
Because if the result if is redundant.
D
Correct. As a result ifbegins the phra e clearly and correctly in thi sentence; the library rather than the ancient world i properly located at Alexandria; the largest of the ancient world correctly modifie library.
E
Alexandria's largest library if the ancient world is an illogical reference. 1he result was mu t be followed by that.
The correct answer is D.
Percentllg
of
mil.......
Decem'"
Who Scored Verbal Scaled SCore
46-60 45 43 42 41
40 39 38
37 36
Percentap Below
99
98 96 95
92 89 87 83 81
78
35
74
34 33 32
69 66 64
31
58
30 29
56
28
53 48
27
43 Number of Candidates
Percentile Ranking Tables
Standard
Mean
.:::.~_ ..
Table 2
Percentages of E
xamln........
. . . .I8Ige1 of Examinees Tested from January 200S through
December 2007 (including Repeaters) Who Scored Below Specified Quantitative Scores
Quantitative Scaled Score
51-60 50 49 48 47 46 45 44 43 42 41 40 39 38 37 36 35 34 33
32 31
Percentage Below 99 94
Who Scored h10w
Quantitative Scaled Score 30
Percentage Below
29
25
28 27 26 25
24
88 84 79 77 75 70 68 63 61 59 55 53 51 46 42
14
24
29
20 19 16 15 13
23 22
11
21
10
20 19 18 17 16 15
9
40 13 38 11-12 34 7-10 30 0-6 Number of Candidates = 650,395 Mean = 35.6 Standard deviation = 10.7
December
8
7
5 4
4 4 3 2 1
o
Total Scaled 760-800 750 740 730 720 710
700 690 680 670 660 650 640 630 620 610 600 590 580 570 560 550 540 530 520 510
p 99 98 97 96 95 92
90 88 86 85 83 80 76 75 72
69 66 63 60 58 54 51 48 44 42 39
4
47 460 450 440 430 420 410 400
390 380
370 360 350
340 330
320 31
Reading Comprehen 1
27
2 3.
28.
4.
30. 31. 32. 33.
5.
6. 7 8. 9. 10. 11. 12. 13.
B Answer Sheets
29
34.
35. 36. 37. 38.
53 54 55
56 57 58 59. 60. 61. 62.
63. 64. 65. 66. 67. 68.
14.
39. 40.
15.
41.
16. 17.
42.
18.
44.
69 70
19.
45.
71
20.
72
21.
46. 47.
22.
48.
23.
49.
24.
50.
25.
51.
26.
52.
43.
73 74 7
. . . .:P Verblll Review 2nd EditIon
Sentence Correction Answe
I Reasoning Answer Sheet
I.
43.
64.
2.
44.
65.
3.
24
45.
66.
25 26. 27. 28. 29. 30.
46.
67.
47. 48.
68.
69.
49.
70.
50.
7l.
5!.
3!.
52.
53. 54. 55.
11.
13
32. 33. 34.
72. 73. 74. 75. 76.
14
35.
56.
15 16
36. 37. 38. 39. 40. 41
57. 58. 59. 60.
1 2 3 4
5 6 7
8 9 10 11 12
17 18 19
22. 23.
42
6!. 62. 63.
4.
5. 6.
30. 31. 32. 33. 34.
59 60 61 62 63 64
7.
35. 36.
8.
37.
9.
38.
66 67.
10.
68 69
13.
39. 40. 41. 42.
77.
14.
43.
78.
15.
79.
16.
80.
17.
44. 45. 46.
72 73 74 75
81. 82.
18.
47.
19.
48.
83.
20.
49.
21.
50.
22.
51.
23.
52.
24.
53.
25. 26. 27.
54.
55. 56
28.
57
29.
5
12.
65
70
71
76 77
7879